MRCS preparation emrcs questions Pathology

388
A 38 year old lady presents with a recent episode of renal colic. As part of her investigations the following results are obtained: Corrected Calcium 3.84 mmol/l PTH 88pg/ml (increased) Her serum urea and electrolytes are normal. What is the most likely diagnosis? A. Carcinoma of the bronchus B. Secondary hyperparathyroidism C. Primary hyperparathyroidism D. Tertiary hyperparathyroidism E. Carcinoma of the breast Theme from September 2012 exam Theme from September 2011 exam In this situation the most likely diagnosis is primary hyperparathyroidism. The question mentions that serum urea and electrolytes are normal, which makes tertiary hyperparathyroidism unlikely. Primary hyperparathyroidism In exams primary hyperparathyroidism is stereotypically seen in elderly females with an unquenchable thirst and an inappropriately normal or raised parathyroid hormone level. It is most commonly due to a solitary adenoma Causes of primary hyperparathyroidism 80%: solitary adenoma 15%: hyperplasia 4%: multiple adenoma 1%: carcinoma Features - 'bones, stones, abdominal groans and psychic moans' Polydipsia, polyuria Peptic ulceration/constipation/pancreatitis Bone pain/fracture Renal stones Depression Hypertension

Transcript of MRCS preparation emrcs questions Pathology

A 38 year old lady presents with a recent episode of renal colic. As part of her

investigations the following results are obtained:

Corrected Calcium 3.84 mmol/l

PTH 88pg/ml (increased)

Her serum urea and electrolytes are normal.

What is the most likely diagnosis?

A. Carcinoma of the bronchus

B. Secondary hyperparathyroidism

C. Primary hyperparathyroidism

D. Tertiary hyperparathyroidism

E. Carcinoma of the breast

Theme from September 2012 exam

Theme from September 2011 exam

In this situation the most likely diagnosis is primary hyperparathyroidism. The

question mentions that serum urea and electrolytes are normal, which makes tertiary

hyperparathyroidism unlikely.

Primary hyperparathyroidism

In exams primary hyperparathyroidism is stereotypically seen in elderly females with

an unquenchable thirst and an inappropriately normal or raised parathyroid hormone

level. It is most commonly due to a solitary adenoma

Causes of primary hyperparathyroidism

80%: solitary adenoma

15%: hyperplasia

4%: multiple adenoma

1%: carcinoma

Features - 'bones, stones, abdominal groans and psychic moans'

Polydipsia, polyuria

Peptic ulceration/constipation/pancreatitis

Bone pain/fracture

Renal stones

Depression

Hypertension

Associations

Hypertension

Multiple endocrine neoplasia: MEN I and II

Investigations

Raised calcium, low phosphate

PTH may be raised or normal

Technetium-MIBI subtraction scan

Treatment

Parathyroidectomy, if imaging suggests target gland then a focused approach

may be used

Theme: Head and neck lumps

A. Branchial cyst

B. Cystic hygroma

C. Carotid body tumour

D. Lymphadenopathy

E. Adenolymphoma of the parotid

F. Pleomorphic adenoma of the parotid

G. Submandibular tumour

H. Thyroglossal cyst

I. Thoracic outlet syndrome

J. Submandibular gland calculus

Please select the most likely lesion to account for the clinical scenario given.

Each option may be used once, more than once or not at all.

2. A 60 year old Tibetan immigrant is referred to the surgical clinic with a painless

neck swelling. On examination it is located on the left side immediately anterior

to the sternocleidomastoid muscle. There are no other abnormalities to find on

examination.

You answered Branchial cyst

The correct answer is Carotid body tumour

Carotid body tumours typically present as painless masses. They may compress

the vagus or hypoglossal nerves with symptoms attributable to these structures.

Over 90% occur spontaneously and are more common in people living at high

altitude. In familial cases up to 30% may be bilateral. Treatment is with

excision.

3. A 40 year old women presents as an emergency with a painful mass underneath

her right mandible. The mass has appeared over the previous week with the pain

worsening as the lump has increased in size. On examination there is a 4cm mass

underneath her mandible, there is no associated lymphadenopathy.

Submandibular gland calculus

The sub mandibular gland is the most common site for salivary calculi. Patients

will usually complain of pain, which is worse on eating. When the lesion is

located distally the duct may be laid open and the stone excised. Otherwise the

gland will require removal.

4. A 73 year old male smoker is referred to the clinic by his GP. On examination he

has a 3cm soft mass immediately anterior to his ear. It has been present for the

past five years and is otherwise associated with no symptoms.

You answered Pleomorphic adenoma of the parotid

The correct answer is Adenolymphoma of the parotid

Warthins tumours (a.k.a. adenolymphoma) are commoner in older men

(especially smokers). They are the second commonest benign tumour of the

parotid gland, they may be bilateral. They are soft and slow growing and

relatively easy to excise. Pleomorphic adenomas typically present in females

aged between 40 - 60 years.

Neck lumps

The table below gives characteristic exam question features for conditions

causing neck lumps:

Reactive

lymphadenopathy

By far the most common cause of neck swellings. There may

be a history of local infection or a generalised viral illness

Lymphoma Rubbery, painless lymphadenopathy

The phenomenon of pain whilst drinking alcohol is very

uncommon

There may be associated night sweats and splenomegaly

Thyroid swelling May be hypo-, eu- or hyperthyroid symptomatically

Moves upwards on swallowing

Thyroglossal cyst More common in patients < 20 years old

Usually midline, between the isthmus of the thyroid and the

hyoid bone

Moves upwards with protrusion of the tongue

May be painful if infected

Pharyngeal pouch More common in older men

Represents a posteromedial herniation between

thyropharyngeus and cricopharyngeus muscles

Usually not seen, but if large then a midline lump in the neck

that gurgles on palpation

Typical symptoms are dysphagia, regurgitation, aspiration

and chronic cough

Cystic hygroma A congenital lymphatic lesion (lymphangioma) typically

found in the neck, classically on the left side

Most are evident at birth, around 90% present before 2 years

of age

Branchial cyst An oval, mobile cystic mass that develops between the

sternocleidomastoid muscle and the pharynx

Develop due to failure of obliteration of the second branchial

cleft in embryonic development

Usually present in early adulthood

Cervical rib More common in adult females

Around 10% develop thoracic outlet syndrome

Carotid aneurysm Pulsatile lateral neck mass which doesn't move on swallowing

A 12 year old child is admitted with a 12 hour history of colicky right upper quadrant

pain. On examination the child is afebrile and is jaundiced. The abdomen is soft and

non tender at the time of examination. What is the most likely cause?

A. Infectious hepatitis

B. Acute cholecystitis

C. Cholangitis

D. Hereditary spherocytosis

E. Gilberts syndrome

Theme from September 2012 Exam

The child is most likely to have hereditary spherocytosis. In these individuals there

may be disease flares precipitated by acute illness. They form small pigment stones.

These may cause biliary colic and some may require cholecystectomy.

Hereditary Spherocytosis

Most common disorder of the red cell membrane, it has an incidence of 1 in 5000.

The abnormally shaped erythrocytes are prone to splenic sequestration and

destruction. This can result in hyperbilirubinaemia, jaundice and splenomegaly. In

older patients an intercurrent illness may increase the rate of red cell destruction

resulting in more acute symptoms.

Severe cases may benefit from splenectomy.

A 2 day old baby is noted to have voiding difficulties and on closer inspection is

noted to have hypospadias. Which of the following abnormalities is most commonly

associated with the condition?

A. Cryptorchidism

B. Diaphragmatic hernia

C. Ventricular - septal defect

D. Bronchogenic cyst

E. Atrial septal defect

Theme from January 2012 Exam

Hypospadias most commonly occurs as an isolated disorder. Associated urological

abnormalities may be seen in up to 40% of infants, of these cryptorchidism is the most

frequent (10%).

Hypospadias

The urethral meatus opens on the ventral surface of the penis. There is also a ventral

deficiency of the foreskin. The uretral meatus may open more proximally in the more

severe variants. However, 75% of the openings are distally located. The incidence is 1

in 300 male births.

Features include:

Absent frenular artery

Ventrally opened glans

Skin tethering to hypoplastic urethra

Splayed columns of spongiosum tissue distal to the meatus

Deficiency of the foreskin ventrally

Management:

No routine cultural circumcisions

Urethroplasty

Penile reconstruction

The foreskin is often utilised in the reconstructive process. In boys with very distal

disease no treatment may be needed.

Theme: Liver lesions

A. Cystadenoma

B. Hyatid cyst

C. Amoebic abscess

D. Mesenchymal hamartoma

E. Liver cell adenoma

F. Cavernous haemangioma

Please select the most likely lesion for the scenario given. Each option may be used

once, more than once or not at all.

7. A 38 year old lady presents with right upper quadrant pain and nausea. She is

otherwise well and her only medical therapy is the oral contraceptive pill which

she has taken for many years with no ill effects. Her liver function tests are

normal. An ultrasound examination demonstrates a hyperechoic well defined

lesion in the left lobe of the liver which measures 14 cm in diameter.

Cavernous haemangioma

Cavernous haemangioma often presents with vague symptoms and signs. They

may grow to considerable size. Liver function tests are usually normal. The

lesions are typically well defined and hyperechoic on ultrasound. A causative

link between OCP use and haemangiomata has yet to be established, but is

possible.

8. A 37 year old lady presents with right upper quadrant pain and nausea. She is

otherwise well and her only medical therapy is the oral contraceptive pill which

she has taken for many years with no ill effects. Her liver function tests and

serum alpha feto protein are normal. An ultrasound examination demonstrates a

4cm non encapsulated lesion in the right lobe of the liver which has a mixed

echoity and heterogeneous texture.

Liver cell adenoma

Liver cell adenomas are linked to OCP use and 90% of patients with liver cell

adenomas have used the OCP. Liver function tests are often normal. The lesions

will typically have a mixed echoity and heterogeneous texture.

9. A 38 year old shepherd presents to the clinic with a 3 month history of malaise

and right upper quadrant pain. On examination he is mildly jaundiced. His liver

function tests demonstrate a mild elevation in bilirubin and transaminases, his

full blood count shows an elevated eosinophil level. An abdominal x-ray is

performed by the senior house officer and demonstrates a calcified lesion in the

right upper quadrant of the abdomen.

Hyatid cyst

Similar theme in September 2011 Exam

Hyatid disease is more common in those who work with sheep or dogs. Liver

function tests may be abnormal and an eosinophilia is often present. Plain

radiographs may reveal a calcified cyst wall.

Benign liver lesions

Benign liver lesions

Haemangioma Most common benign tumours of mesenchymal origin

Incidence in autopsy series is 8%

Cavernous haemangiomas may be enormous

Clinically they are reddish purple hypervascular lesions

Lesions are normally separated from normal liver by ring of

fibrous tissue

On ultrasound they are typically hyperechoic

Liver cell

adenoma

90% develop in women in their third to fifth decade

Linked to use of oral contraceptive pill

Lesions are usually solitary

They are usually sharply demarcated from normal liver

although they usually lack a fibrous capsule

On ultrasound the appearances are of mixed echoity and

heterogeneous texture. On CT most lesions are hypodense

when imaged prior to administration of IV contrast agents

In patients with haemorrhage or symptoms removal of the

adenoma may be required

Mesenchymal

hamartomas

Congential and benign, usually present in infants. May compress

normal liver

Liver abscess Biliary sepsis is a major predisposing factor

Structures drained by the portal venous system form the

second largest source

Common symptoms include fever, right upper quadrant

pain. Jaundice may be seen in 50%

Ultrasound will usually show a fluid filled cavity,

hyperechoic walls may be seen in chronic abscesses

Amoebic abscess Liver abscess is the most common extra intestinal

manifestation of amoebiasis

Between 75 and 90% lesions occur in the right lobe

Presenting complaints typically include fever and right

upper quadrant pain

Ultrasonography will usually show a fluid filled structure

with poorly defined boundaries

Aspiration yield sterile odourless fluid which has an

anchovy paste consistency

Treatment is with metronidazole

Hyatid cysts Seen in cases of Echinococcus infection

Typically an intense fibrotic reaction occurs around sites of

infection

The cyst has no epithelial lining

Cysts are commonly unilocular and may grow to 20cm in

size. The cyst wall is thick and has an external laminated

hilar membrane and an internal enucleated germinal layer

Typically presents with malaise and right upper quadrant

pain. Secondary bacterial infection occurs in 10%.

Liver function tests are usually abnormal and eosinophilia is

present in 33% cases

Ultrasound may show septa and hyatid sand or daughter

cysts.

Percutaneous aspiration is contra indicated

Treatment is by sterilisation of the cyst with mebendazole

and may be followed by surgical resection. Hypertonic

swabs are packed around the cysts during surgery

Polycystic liver

disease

Usually occurs in association with polycystic kidney disease

Autosomal dominant disorder

Symptoms may occur as a result of capsular stretch

Cystadenoma Rare lesions with malignant potential

Usually solitary multiloculated lesions

Liver function tests usually normal

Ultrasonography typically shows a large anechoic, fluid

filled area with irregular margins. Internal echos may result

from septa

Surgical resection is indicated in all cases

A 72 year old man presents with symptoms and signs of benign prostatic hyperplasia.

Which of the following structures is most likely to be enlarged on digital rectal

examination?

A. Posterior lobe of the prostate

B. Median lobe of the prostate

C. Right lateral lobe of the prostate

D. Left lateral lobe of the prostate

E. Anterior lobe of the prostate

Carcinoma of the prostate typically occurs in the posterior lobe. The median lobe is

usually enlarged in BPH. The anterior lobe has little in the way of glandular tissue and

is seldom enlarged.

Benign Prostatic Hyperplasia

Prostatic enlargement occurs in many elderly men

>90% of men aged over 80 will have at least microscopic evidence of benign

prostatic hyperplasia

Pathology As part of the hyperplastic process increase in both stromal and glandular components

are seen. The changes are most notable in the central and periurethral region of the

gland.

Image showing enlarged prostate removed by transvesical prostatectomy with

massive enlargement of the median lobe

Image sourced from Wikipedia

Presentation The vast majority of men will present with lower urinary tract symptoms. These will

typically be:

Poor flow

Nocturia

Hesitancy

Incomplete and double voiding

Terminal dribbling

Urgency

Incontinence

Investigation

Digital rectal examination to assess prostatic size and morphology.

Urine dipstick for infections and haematuria.

Uroflowmetry (a flow rate of >15ml/second helps to exclude BOO)

Bladder pressure studies may help identify detrusor failure and whilst may not

form part of first line investigations should be included in those with atypical

symptoms and prior to redo surgery.

Bladder scanning to demonstrate residual volumes. USS if high pressure

chronic retention.

Management

Lifestyle changes such as stopping smoking and altering fluid intake may help

those with mild symptoms.

Medical therapy includes alpha blockers and 5 alpha reductase inhibitors. The

former work quickly on receptor zones located at the bladder neck.

Cardiovascular side effects are well documented. The latter work on

testosterone metabolising enzymes. Although they have a slower onset of

action, the 5 alpha reductase inhibitors may prevent acute urinary retention.

Surgical therapy includes transurethral resection of the prostate and is the

treatment of choice in those with severe symptoms and those who fail to

respond to medical therapy. More tailored bladder neck incision procedures

may be considered in those with small prostates. Retrograde ejaculation may

occur following surgery. The change in the type of irrigation solutions used

has helped to minimise the TURP syndrome of electrolyte disturbances.

A 58 year old man has been suffering from mechanical back pain for several years.

One morning he awakes from sleep and feels a sudden onset of pain in his back

radiating down his left leg. Which of the following events is most likely to account for

his symptoms?

A. Prolapse of inner annulus fibrosus

B. Prolapse of outer annulus fibrosus

C. Prolapse of nucleus pulposus

D. Rupture of the ligamentum flavum

E. None of the above

Theme from 2009 Exam

Theme from September 2012 Exam

The symptoms would be most likely the result of intervertebral disk prolapse. In disk

prolapse the nucleus pulposus is the structure which usually herniates.

Intervertebral discs

Consist of an outer annulus fibrosus and an inner nucleus pulposus.

The anulus fibrosus consists of several layers of fibrocartilage.

The nucleus pulposus contains loose fibres suspended in a mucoprotein gel

with the consistency of jelly. The nucleus of the disc acts as a shock absorber.

Pressure on the disc causes posterior protrusion of the nucleus pulposus. Most

commonly in the lumbrosacral and lower cervical areas.

The discs are separated by hyaline cartilage.

There is one disc between each pair of vertebrae, except for C1/2 and the

sacrococcygeal vertebrae.

heme: Paediatric neck masses

A. Cystic hygroma

B. Thyroglossal cyst

C. Rhabdomyosarcoma

D. Branchial cyst

E. Dermoid cyst

Please select the most likely underlying diagnosis for the situation that is described.

Each option may be used once, more than once, or not at all.

12. A 2 year old boy is brought to the clinic by his mother who has noticed that he

has developed a small mass. On examination a small smooth cyst is identified

which is located above the hyoid bone. On ultrasound the lesion appears to be a

heterogenous and multiloculated mass.

You answered Thyroglossal cyst

The correct answer is Dermoid cyst

Dermoid cysts are usually multiloculated and heterogeneous. Most are located

above the hyoid and their appearances on imaging differentiate them from

thyroglossal cysts.

13. A 22 month old baby is brought to the clinic by her mother who is concerned

that she has developed a swelling in her neck. On examination she has a soft,

lesion located in the posterior triangle that transilluminates.

Cystic hygroma

Cystic hygromas are soft and transilluminate. Most are located in the posterior

triangle.

14. A 3 year old boy is brought to the clinic by his mother who has noticed a mass

in his neck. On examination he has a smooth mass located on the lateral aspect

of his anterior triangle, near to the angle of the mandible. On ultrasound it has a

fluid filled, anechoic, appearance.

You answered Dermoid cyst

The correct answer is Branchial cyst

Branchial cysts are usually located laterally and derived from the second

branchial cleft. Unless infection has occurred they will usually have an

anechoic appearance on ultrasound.

Neck Masses in Children

Thyroglossal cyst Located in the anterior triangle, usually in the midline

and below the hyoid (65% cases)

Derived from remnants of the thyroglossal duct

Thin walled and anechoic on USS (echogenicity

suggests infection of cyst)

Branchial cyst Six branchial arches separated by branchial clefts

Incomplete obliteration of the branchial apparatus may

result in cysts, sinuses or fistulae

75% of branchial cysts originate from the second

branchial cleft

Usually located anterior to the sternocleidomastoid

near the angle of the mandible

Unless infected the fluid of the cyst has a similar

consistency to water and is anechoic on USS

Dermoids Derived from pleuripotent stem cells and are located

in the midline

Most commonly in a suprahyoid location

They have heterogeneous appearances on imaging and

contain variable amounts of calcium and fat

Thyroid gland True thyroid lesions are rare in children and usually

represent thyroglossal cysts or tumours like lymphoma

Lymphatic

malformations

Usually located posterior to the sternocleidomastoid

Cystic hygroma result from occlusion of lymphatic

channels

The painless, fluid filled, lesions usually present prior

to the age of 2

They are often closely linked to surrounding structures

and surgical removal is difficult

They are typically hypoechoic on USS

Infantile

haemangioma

May present in either triangle of the neck

Grow rapidly initially and then will often

spontaneously regress

Plain x-rays will show a mass lesion, usually

containing calcified phleboliths

As involution occurs the fat content of the lesions

increases

Lymphadenopathy Located in either triangle of the neck

May be reactive or neoplastic

Generalised lymphadenopathy usually secondary to

infection in children (very common)

An unusually tall 43 year old lady presents to the surgical clinic with bilateral inguinal

hernias. She develops chest pain and collapses. As part of her investigations a chest x-

ray shows evidence of mediastinal widening. What is the most likely underlying

diagnosis?

A. Pulmonary embolus

B. Aortic dissection

C. Tietze syndrome

D. Boerhaaves syndrome

E. Myocardial infarct

Marfans syndrome may present with a variety of connective tissue disorders such as

bilateral inguinal hernia. They are at high risk of aortic dissection, as in this case.

Aortic dissection

More common than rupture of the abdominal aorta

33% of patients die within the first 24 hours, and 50% die within 48 hours if

no treatment received

Associated with hypertension

Features of aortic dissection: tear in the intimal layer, followed by formation

and propagation of a subintimal hematoma. Cystic medial necrosis (Marfan's)

Most common site of dissection: 90% occurring within 10 centimetres of the

aortic valve

Stanford Classification

Type Location Treatment

A Ascending aorta/ aortic root Surgery- aortic root replacement

B Descending aorta Medical therapy with antihypertensives

DeBakey classification

Type Site affected

I Ascending aorta, aortic arch, descending aorta

II Ascending aorta only

III Descending aorta distal to left subclavian artery

Clinical features

Tearing, sudden onset chest pain (painless 10%)

Hypertension or Hypotension

A blood pressure difference greater than 20 mm Hg

Neurologic deficits (20%)

Investigations

CXR: widened mediastinum, abnormal aortic knob, ring sign, deviation

trachea/oesophagus

CT (spiral)

MRI

Angiography (95% of patients diagnosed)

Management

Beta-blockers: aim HR 60-80 bpm and systolic BP 100-120 mm Hg.

Urgent surgical intervention: type A dissections. This will usually involve

aortic root replacement.

An unusually tall 43 year old lady presents to the surgical clinic with bilateral inguinal

hernias. She develops chest pain and collapses. As part of her investigations a chest x-

ray shows evidence of mediastinal widening. What is the most likely underlying

diagnosis?

A. Pulmonary embolus

B. Aortic dissection

C. Tietze syndrome

D. Boerhaaves syndrome

E. Myocardial infarct

Marfans syndrome may present with a variety of connective tissue disorders such as

bilateral inguinal hernia. They are at high risk of aortic dissection, as in this case.

Aortic dissection

More common than rupture of the abdominal aorta

33% of patients die within the first 24 hours, and 50% die within 48 hours if

no treatment received

Associated with hypertension

Features of aortic dissection: tear in the intimal layer, followed by formation

and propagation of a subintimal hematoma. Cystic medial necrosis (Marfan's)

Most common site of dissection: 90% occurring within 10 centimetres of the

aortic valve

Stanford Classification

Type Location Treatment

A Ascending aorta/ aortic root Surgery- aortic root replacement

B Descending aorta Medical therapy with antihypertensives

DeBakey classification

Type Site affected

I Ascending aorta, aortic arch, descending aorta

II Ascending aorta only

III Descending aorta distal to left subclavian artery

Clinical features

Tearing, sudden onset chest pain (painless 10%)

Hypertension or Hypotension

A blood pressure difference greater than 20 mm Hg

Neurologic deficits (20%)

Investigations

CXR: widened mediastinum, abnormal aortic knob, ring sign, deviation

trachea/oesophagus

CT (spiral)

MRI

Angiography (95% of patients diagnosed)

Management

Beta-blockers: aim HR 60-80 bpm and systolic BP 100-120 mm Hg.

Urgent surgical intervention: type A dissections. This will usually involve

aortic root replacement.

A 72 year old man has just undergone an emergency repair for a ruptured

abdominal aortic aneurysm. Pre operatively he was taking aspirin, clopidogrel

and warfarin. Intra operatively he received 5000 units of unfractionated

heparin prior to application of the aortic cross clamp. His blood results on

admission to the critical care unit are as follows:

Full blood count

Hb 8 g/dl

Platelets 40 * 109/l

WBC 7.1 * 109/l

His fibrin degradation products are measured and found to be markedly

elevated. Which of the following accounts for these results?

A. Anastomotic leak

B. Disseminated intravascular coagulation

C. Heparin induced thrombocytopenia

D. Adverse effect of warfarin

E. Adverse effects of antiplatelet agents

Theme from April 2012 Exam

The combination of low platelet counts and raised FDP in this setting maked

DIC the most likely diagnosis.

Disseminated intravascular coagulation - Diagnosis

Under homeostatic conditions, coagulation and fibrinolysis are coupled. The

activation of the coagulation cascade yields thrombin that converts fibrinogen

to fibrin; the stable fibrin clot being the final product of hemostasis. The

fibrinolytic system breaks down fibrinogen and fibrin. Activation of the

fibrinolytic system generates plasmin (in the presence of thrombin), which is

responsible for the lysis of fibrin clots. The breakdown of fibrinogen and

fibrin results in polypeptides (fibrin degradation products). In a state of

homeostasis, the presence of plasmin is critical, as it is the central proteolytic

enzyme of coagulation and is also necessary for fibrinolysis.

In DIC, the processes of coagulation and fibrinolysis are dysregulated, and the

result is widespread clotting with resultant bleeding. Regardless of the

triggering event of DIC, once initiated, the pathophysiology of DIC is similar

in all conditions. One critical mediator of DIC is the release of a

transmembrane glycoprotein (tissue factor =TF). TF is present on the surface

of many cell types (including endothelial cells, macrophages, and monocytes)

and is not normally in contact with the general circulation, but is exposed to

the circulation after vascular damage. For example, TF is released in response

to exposure to cytokines (particularly interleukin 1), tumor necrosis factor, and

endotoxin. This plays a major role in the development of DIC in septic

conditions. TF is also abundant in tissues of the lungs, brain, and placenta.

This helps to explain why DIC readily develops in patients with extensive

trauma. Upon activation, TF binds with coagulation factors that then triggers

the extrinsic pathway (via Factor VII) which subsequently triggers the intrinsic

pathway (XII to XI to IX) of coagulation.

Diagnosis Fibrin degradation products are often raised.

Disorder Prothrombin time APTT Bleeding

time

Platelet

count

Warfarin

administration

Prolonged Normal Normal Normal

Aspirin

administration

Normal Normal Prolonged Normal

Heparin Often normal (may be

prolonged)

Prolonged Normal Normal

DIC Prolonged Prolonged Prolonged Low

A 53 year old man from Hong Kong presents with symptoms of fatigue, weight loss

and recurrent epistaxis. Clinical examination reveals left sided cervical

lymphadenopathy and oropharyngeal examination reveals an ulcerated mass in the

naso pharynx. Which of the following viral agents is most commonly implicated in

the development of this condition?

A. Cytomegalovirus

B. Epstein Barr virus

C. Coxsackie virus

D. Herpes simplex virus

E. None of the above

The clinical scenario is most typical for nasopharyngeal carcinoma. An association

with previous Epstein Barr Virus is well established. Infection with the other viruses

listed is not a recognised risk factor for the development of the condition.

Nasopharyngeal carcinoma

Squamous cell carcinoma of the nasopharynx

Rare in most parts of the world, apart from individuals from Southern China

Associated with Epstein Barr virus infection

Presenting features

Systemic Local

Cervical lymphadenopathy Otalgia

Unilateral serous otitis media

Nasal obstruction, discharge and/ or epistaxis

Cranial nerve palsies e.g. III-VI

Imaging Combined CT and MRI.

Treatment Radiotherapy is first line therapy.

An 18 year old male presents with lethargy, night sweats and on examination is found

to have left supraclavicular lymphadenopathy. A surgical registrar performs a left

supraclavicular lymph node biopsy. The pathologist identifies Reed- Sternberg cells

on the subsequent histology sections, what is the most likely diagnosis?

A. Metastatic gastric cancer

B. Hodgkins lymphoma

C. Non Hodgkins lymphoma

D. Tuberculosis

E. None of the above

Reed-Sternberg cells are characteristic histological cell type found in Hodgkins

disease.

Lymphadenopathy

Lymphadenopathy in the neck, axillae, groins and abdomen

Need to note: solitary/multiple, defined/indistinct, hard/rubbery/soft,

tender/painless

Causes of lymphadenopathy

Mnemonic: Hodgkins disease

H aematological: Hodgkins lymphoma, NHL, Leukaemia

O ncological: metastases

D ermatopathic lympadenitis

G aucher's disease

K awasaki disease

I nfections: TB, glandular fever, Syphilis

N iemann Pick disease

S erum sickness

D rug reaction (phenytoin)

I mmunological (SLE)

S arcoidosis

E ndocrinological (Hyperthyroidism)

A ngioimmunoplastic lymphadenopathy

S LE

E osinophilic granulomatosis

Which of the following lesions is least likely to occur in the presence of severe

atrophic gastritis?

A. Duodenal ulcer

B. Gastric cancer

C. Gastric polyp

D. Iron deficiency anaemia

E. Pernicious anaemia

Due the absence of acid a duodenal ulcer is unlikely to occur.

Gastric cancer

Overview There are 700,000 new cases of gastric cancer worldwide each year. It is most

common in Japan and less common in western countries. It is more common in men

and incidence rises with increasing age. The exact cause of many sporadic cancer is

not known, however, familial cases do occur in HNPCC families. In addition,

smoking and smoked or preserved foods increase the risk. Japanese migrants retain

their increased risk (decreased in subsequent generations). The distribution of the

disease in western countries is changing towards a more proximal location (perhaps

due to rising obesity).

Pathology There is some evidence of support a stepwise progression of the disease through

intestinal metaplasia progressing to atrophic gastritis and subsequent dysplasia,

through to cancer. The favoured staging system is TNM. The risk of lymph node

involvement is related to size and depth of invasion; early cancers confined to

submucosa have a 20% incidence of lymph node metastasis. Tumours of the gastro-

oesophageal junction are classified as below:

Type

1

True oesophageal cancers and may be associated with Barrett's oesophagus.

Type

2

Carcinoma of the cardia, arising from cardiac type epithelium

or short segments with intestinal metaplasia at the oesophagogastric junction.

Type

3

Sub cardial cancers that spread across the junction. Involve similar nodal

stations to gastric cancer.

Groups for close endoscopic monitoring

Intestinal metaplasia of columnar type

Atrophic gastritis

Low to medium grade dysplasia

Patients who have previously undergone resections for benign peptic ulcer

disease (except highly selective vagotomy).

Referral to endoscopy

Patients of any age with

dyspepsia and any of the

following

Patients without

dyspepsia

Worsening dyspepsia

Chronic gastrointestinal

bleeding

Dysphagia Barretts oesophagus

Dysphagia Unexplained

abdominal pain or

weight loss

Intestinal metaplasia

Weight loss Vomiting Dysplasia

Iron deficiency anaemia Upper abdominal mass Atrophic gastritis

Upper abdominal mass Jaundice Patient aged over 55 years

with unexplained or persistent

dyspepsia

Upper GI endoscopy performed for dyspepsia. The addition of dye spraying (as

shown in the bottom right) may facilitate identification of smaller tumours

Image sourced from Wikipedia

Staging

CT scanning of the chest abdomen and pelvis is the routine first line staging

investigation in most centres.

Laparoscopy to identify occult peritoneal disease

PET CT (particularly for junctional tumours)

Treatment

Proximally sited disease greater than 5-10cm from the OG junction may be

treated by sub total gastrectomy

Total gastrectomy if tumour is <5cm from OG junction

For type 2 junctional tumours (extending into oesophagus)

oesophagogastrectomy is usual

Endoscopic sub mucosal resection may play a role in early gastric cancer

confined to the mucosa and perhaps the sub mucosa (this is debated)

Lymphadenectomy should be performed. A D2 lymphadenectomy is widely

advocated by the Japanese, the survival advantages of extended

lymphadenectomy have been debated. However, the overall recommendation

is that a D2 nodal dissection be undertaken.

Most patients will receive chemotherapy either pre or post operatively.

Prognosis

UK Data

Disease extent Percentage 5 year survival

All RO resections 54%

Early gastric cancer 91%

Stage 1 87%

Stage 2 65%

Stage 3 18%

Operative procedure

Total Gastrectomy , lymphadenectomy and Roux en Y anastomosis

General anaesthesia

Prophylactic intravenous antibiotics

Incision: Rooftop.

Perform a thorough laparotomy to identify any occult disease.

Mobilise the left lobe of the liver off the diaphragm and place a large pack over it.

Insert a large self retaining retractor e.g. omnitract or Balfour (take time with this, the

set up should be perfect). Pack the small bowel away.

Begin by mobilising the omentum off the transverse colon.

Proceed to detach the short gastric vessels.

Mobilise the pylorus and divide it at least 2cm distally using a linear cutter stapling

device.

Continue the dissection into the lesser sac taking the lesser omentum and left gastric

artery flush at its origin.

The lymph nodes should be removed en bloc with the specimen where possible.

Place 2 stay sutures either side of the distal oesophagus. Ask the anaesthetist to pull

back on the nasogastric tube. Divide the distal oesophagus and remove the stomach.

The oesphago jejunal anastomosis should be constructed. Identify the DJ flexure and

bring a loop of jejunum up to the oesophagus (to check it will reach). Divide the

jejunum at this point. Bring the divided jejunum either retrocolic or antecolic to the

oesophagus. Anastamose the oesophagus to the jejunum, using either interrupted 3/0

vicryl or a stapling device. Then create the remainder of the Roux en Y reconstruction

distally.

Place a jejunostomy feeding tube.

Wash out the abdomen and insert drains (usually the anastomosis and duodenal

stump). Help the anaesthetist insert the nasogastric tube (carefully!)

Close the abdomen and skin.

Enteral feeding may commence on the first post-operative day. However, most

surgeons will leave patients on free NG drainage for several days and keep them nil

by mouth.

A 28 year old man develops an acute paronychia and subsequent spreading sepsis.

The tissue exudate has a higher protein content than normal tissue because?

A. Breakdown of tissue cells release protein

B. Capillary walls are more permeable

C. Increased blood flow transports more protein into the area

D. Intracapillary pressure is raised

E. Plasma cells release gamma globulin

The increased permeability allows the exudation of plasma proteins.

Acute inflammation

Inflammation is the reaction of the tissue elements to injury. Vascular changes occur,

resulting in the generation of a protein rich exudate. So long as the injury does not

totally destroy the existing tissue architecture, the episode may resolve with

restoration of original tissue architecture.

Vascular changes

Vasodilation occurs and persists throughout the inflammatory phase.

Inflammatory cells exit the circulation at the site of injury.

The equilibrium that balances Starlings forces within capillary beds is

disrupted and a protein rich exudate will form as the vessel walls also become

more permeable to proteins.

The high fibrinogen content of the fluid may form a fibrin clot. This has

several important immunomodulatory functions.

Sequelae

Resolution Typically occurs with minimal initial injury

Stimulus removed and normal tissue architecture

results

Organisation Delayed removed of exudate

Tissues undergo organisation and usually fibrosis

Suppuration Typically formation of an abscess or an empyema

Sequestration of large quantities of dead

neutrophils

Progression to chronic

inflammation

Coupled inflammatory and reparative activities

Usually occurs when initial infection or

suppuration has been inadequately managed

Causes

Microbacterial infections e.g. Viruses, exotoxins or endotoxins released by

bacteria

Chemical agents

Physical agents e.g. Trauma

Hypersensitivity reactions

Tissue necrosis

Presence of neutrophil polymorphs is a histological diagnostic feature of acute

inflammation As a busy surgical trainee on the colorectal unit you are given the unenviable task of

reviewing the unit's histopathology results for colonic polyps. Which of the polyp

types described below has the greatest risk of malignancy?

A. Hyperplastic polyp

B. Tubular adenoma

C. Villous adenoma

D. Hamartomatous polyp

E. Serrated polyp

Villous adenomas carry the highest risk of malignant transformation. Hyperplastic

polyps carry little in the way of increased risk. Although, patients with

hamartomatous polyp syndromes may have a high risk of malignancy, the polyps

themselves have little malignant potential.

Colonic polyps

Colonic Polyps May occur in isolation of greater numbers as part of the polyposis syndromes. In FAP

greater than 100 polyps are typically present. The risk of malignancy in association

with adenomas is related to size and is the order of 10% in a 1cm adenoma. Isolated

adenomas seldom give risk of symptoms (unless large and distal). Distally sited

villous lesions may produce mucous and if very large electrolyte disturbances may

occur.

Follow up of colonic polyps

Low risk

1 or 2 adenomas <1cm. No follow up or re-colonoscopy at 5 years.

Moderate risk

3 or 4 small adenomas or 1 adenoma >1cm. Re-scope at 3 years.

High risk

>5 small adenomas or >3 with 1 of them >1cm. Re scope at 1 year.

From Atkins and Saunders Gut 2002 51 (suppl V:V6-V9). It is important to stratify

patients appropriately and ensure that a complete colonoscopy with good views was

performed.

Segmental resection or complete colectomy should be considered when:

1. Incomplete excision of malignant polyp

2. Malignant sessile polyp

3. Malignant pedunculated polyp with submucosal invasion

4. Polyps with poorly differentiated carcinoma

5. Familial polyposis coli

-Screening from teenager up to 40 years by 2 yearly sigmoidoscopy/colonoscopy

-Panproctocolectomy and Ileostomy or Restorative Panproctocolectomy.

Rectal polypoidal lesions may be amenable to trans anal endoscopic microsurgery.

A 23 year old man presents to the surgical clinic with an inguinal hernia. On

examination he has a small direct hernia. However, you also notice that he has

pigmented spots around his mouth, on his palms and soles. In his history he

underwent a reduction of an intussusception aged 12 years. Which of the following

lesions is most likely to be identified if a colonoscopy were performed?

A. Hamartomas

B. Tubulovillous adenoma

C. Colorectal cancer

D. Crohns disease

E. Hyperplastic polyps

Theme from April 2012 Exam

He is most likely to have Peutz-Jeghers syndrome which is associated with

Hamartomas.

Peutz-Jeghers syndrome

Peutz-Jeghers syndrome is an autosomal dominant condition characterised by

numerous benign hamartomatous polyps in the gastrointestinal tract. It is also

associated with pigmented freckles on the lips, face, palms and soles. Around 50% of

patients will have died from a gastrointestinal tract cancer by the age of 60 years.

Genetics

Autosomal dominant

Responsible gene encodes serine threonine kinase LKB1 or STK11

Features

Hamartomatous polyps in GI tract (mainly small bowel)

Pigmented lesions on lips, oral mucosa, face, palms and soles

Intestinal obstruction e.g. intussusception (which may lead to diagnosis)

Gastrointestinal bleeding

Management

Conservative unless complications develop

A 56 year old surgeon has been successfully operating for many years. Over

the past few weeks she has begun to notice that her hands are becoming

blistering and weepy. A latex allergy is diagnosed. Which of the following

pathological processes accounts for this scenario?

A. Type 1 hypersensitivity reaction

B. Type 2 hypersensitivity reaction

C. Type 4 hypersensitivity reaction

D. Type 3 hypersensitivity reaction

E. None of the above

Hypersensitivity

reactions: ACID

type 1 --Anaphylactic

type 2 --Cytotoxic

type 3 --Immune complex

type 4 --Delayed

hypersensitivity

Theme from 2012 Exam

Contact dermatitis of a chronic nature is an example of a type 4

hypersensitivity reaction. Type 4 hypersensitivity reactions are cell mediated

rather than antibody mediated.

Hypersensitivity reactions

The Gell and Coombs classification divides hypersensitivity reactions into 4

types

Type I Type II Type III Type IV

Description Anaphylactic Cytotoxic Immune

complex

Delayed type

Mediator IgE IgG, IgM IgG, IgM T-cells

Antigen Exogenous Cell surface Soluble Tissues

Response

time

Minutes Hours Hours 2-3 days

Examples Asthma

Hay fever

Autoimmune

haemolytic anaemia

Pemphigus

Goodpasture's

Serum

sickness

SLE

Aspergillosis

Graft versus host

disease

Contact

dermatitis

A 56 year old motorcyclist is involved in a road traffic accident and sustains a

displaced femoral shaft fracture. Not other injuries are identified on the primary or

secondary surveys. The fracture is treated with closed, antegrade intramedullary

nailing. The following day the patient becomes increasingly agitated and confused.

On examination he is pyrexial, hypoxic SaO2 90% on 6 litres O2, tachycardic and

normotensive. Systemic examination demonstrates a non blanching petechial rash

present over the torso. What is the most likely explanation for this?

A. Pulmonary embolism with paradoxical embolus

B. Fat embolism

C. Meningococcal sepsis

D. Alcohol withdrawl

E. Chronic sub dural haematoma

This man has a recent injury and physical signs that would be concordant with fat

embolism syndrome. Meningococcal sepsis is not usually associated with hypoxia

initially. Pulmonary emboli are not typically associated with pyrexia.

Fat embolism

Diagnosis and clinical features

System Feature

Cardiothoracic Early persistent tachycardia

Tachypnoea, dyspnoea, hypoxia usually 72 hours following

injury

Pyrexia

Dermatological Red/ brown impalpable petechial rash (usually only in 25-

50%)

Subconjunctival and oral haemorrhage/ petechiae

CNS Confusion and agitation

Retinal haemorrhages and intra-arterial fat globules on

fundoscopy

Imaging

May be normal

Fat emboli tend to lodge distally and therefore CTPA may not show any

vascular occlusion, a ground glass appearance may be seen at the periphery

Treatment

Prompt fixation of long bone fractures

Some debate regarding benefit Vs. risk of medullary reaming in femoral shaft/

tibial fractures in terms of increasing risk (probably does not).

DVT prophylaxis

General supportive care

Which of these tumour markers is most helpful in identifying an individual with

hepatocellular carcinoma?

A. Serum AFP

B. Serum CA19-9

C. CEA

D. Beta HCG

E. CA125

Theme from September 2011 Exam

Hepatocellular carcinoma is commonly diagnosed with imaging and an elevated alpha

fetoprotein. Biopsy may seed the tumour and should be avoided. Up to 80% of

hepatocellular carcinoma arise in cirrhotic livers.

Liver tumours

Primary liver tumours The most common primary tumours are cholangiocarcinoma and hepatocellular

carcinoma. Overall metastatic disease accounts for 95% of all liver malignancies

making the primary liver tumours comparatively rare.

Primary liver tumours include:

Cholangiocarcinoma

Hepatocellular carcinoma

Hepatoblastoma

Sarcomas (Rare)

Lymphomas

Carcinoids (most often secondary although primary may occur)

Hepatocellular carcinoma These account for the bulk of primary liver tumours (75% cases). Its worldwide

incidence reflects its propensity to occur on a background of chronic inflammatory

activity. Most cases arise in cirrhotic livers or those with chronic hepatitis B infection,

especially where viral replication is actively occurring. In the UK it accounts for less

than 5% of all cancers, although in parts of Asia its incidence is 100 per 100,000.

The majority of patients (80%) present with existing liver cirrhosis, with a mass

discovered on screening ultrasound.

Diagnosis

CT/ MRI (usually both) are the imaging modalities of choice

a-fetoprotein is elevated in almost all cases

Biopsy should be avoided as it seeds tumours cells through a resection plane.

In cases of diagnostic doubt serial CT and aFP measurements are the preferred

strategy.

Treatment

Patients should be staged with liver MRI and chest, abdomen and pelvic CT

scan.

The testis should be examined in males (testicular tumours may cause raised

AFP). PET CT may be used to identify occult nodal disease.

Surgical resection is the mainstay of treatment in operable cases. In patients

with a small primary tumour in a cirrhotic liver whose primary disease process

is controlled, consideration may be given to primary whole liver resection and

transplantation.

Liver resections are an option but since most cases occur in an already

diseased liver the operative risks and post-operative hepatic dysfunction are

far greater than is seen following metastectomy.

These tumours are not particularly chemo or radiosensitive however, both may

be used in a palliative setting. Tumour ablation is a more popular strategy.

Survival Poor, overall survival is 15% at 5 years.

Cholangiocarcinoma This is the second most common type of primary liver malignancy. As its name

suggests these tumours arise in the bile ducts. Up to 80% of tumours arise in the extra

hepatic biliary tree. Most patients present with jaundice and by this stage the majority

will have disease that is not resectable.

Primary scelerosing cholangitis is the main risk factor. In deprived countries typhoid

and liver flukes are also major risk factors.

Diagnosis

Patients will typically have an obstructive picture on liver function tests.

CA 19-9, CEA and CA 125 are often elevated

CT/ MRI and MRCP are the imaging methods of choice.

Treatment

Surgical resection offers the best chance of cure. Local invasion of peri hilar

tumours is a particular problem and this coupled with lobar atrophy will often

contra indicate surgical resection.

Palliation of jaundice is important, although metallic stents should be avoided

in those considered for resection.

Survival Is poor, approximately 15% 5 year survival.

A 39 year old man has suffered from terminal ileal Crohns disease for the past 20

years. Which condition is he least likely to develop?

A. Gallstones

B. Malabsorption

C. Pyoderma gangrenosum

D. Amyloidosis

E. Feltys syndrome

Felteys syndrome:

Rheumatoid disease

Splenomegaly

Neutropenia

Feltys syndrome is associated with rheumatoid disease. Individuals with long standing

crohns disease are at risk of gallstones because of impairment of the enterohepatic

recycling of bile salts. Formation of entero-enteric fistulation may produce

malabsorption. Amyloidosis may complicate chronic inflammatory states.

Crohns disease

Crohns disease is a chronic transmural inflammation of a segment(s) of the

gastrointestinal tract and may be associated with extra intestinal manifestations.

Frequent disease patterns observed include ileal, ileocolic and colonic disease. Peri-

anal disease may occur in association with any of these. The disease is often

discontinuous in its distribution. Inflammation may cause ulceration, fissures, fistulas

and fibrosis with stricturing. Histology reveals a chronic inflammatory infiltrate that is

usually patchy and transmural.

Ulcerative colitis Vs Crohns

Crohn's disease Ulcerative colitis

Distribution Mouth to anus Rectum and colon

Macroscopic

changes

Cobblestone appearance, apthoid

ulceration

Contact bleeding

Depth of

disease

Transmural inflammation Superficial inflammation

Distribution

pattern

Patchy Continuous

Histological

features

Granulomas (non caseating epithelioid

cell aggregates with Langhans' giant

cells)

Crypt abscesses,

Inflammatory cells in the

lamina propria

Extraintestinal manifestations of Crohns

Related to disease extent Unrelated to disease extent

Aphthous ulcers (10%) Sacroiliiitis (10-15%)

Erythema nodosum (5-10%) Ankylosing spondylitis (1-2%)

Pyoderma gangrenosum (0.5%) Primary sclerosing cholangitis (Rare)

Acute arthropathy (6-12%) Gallstones (up to 30%)

Ocular complications (up to 10%) Renal calculi (up to 10%)

Theme: Renal stones

A. Calcium oxalate

B. Uric acid

C. Cystine

D. Struvite

E. Calcium phosphate

Please select the most likely stone type for each of the following urinary tract stone

scenarios. Each option may be used once, more than once or not at all.

28. A 73 year old lady is undergoing chemotherapy for treatment of acute

leukaemia. She develops symptoms of renal colic. Her urine tests positive for

blood. A KUB x-ray shows no evidence of stones.

Uric acid

Chemotherapy and cell death can increase uric acid levels. In this acute setting

the uric acid stones are unlikely to be coated with calcium and will therefore be

radiolucent.

29. A 16 year old boy presents with renal colic. His parents both have a similar

history of the condition. His urine tests positive for blood. A KUB style x-ray

shows a relatively radiodense stone in the region of the mid ureter.

Cystine

Cystine stones are associated with an inherited metabolic disorder.

30. A 43 year old lady with episodes of recurrent urinary tract sepsis presents with

a staghorn calculus of the left kidney. Her urinary pH is 7.3. A KUB x-ray

shows a faint outline of the calculus.

Struvite

Theme from April 2012 Exam

Chronic infection with urease producing enzymes can produce an alkaline urine

with formation of struvate stone.

Renal stones

Type of

stones

Features Percentage of

all calculi

Calcium

oxalate

Hypercalciuria is a major risk factor (various causes)

Hyperoxaluria may also increase risk

Hypocitraturia increases risk because citrate forms

complexes with calcium making it more soluble

Stones are radio-opaque (though less than calcium

phosphate stones)

Hyperuricosuria may cause uric acid stones to which

calcium oxalate binds

85%

Cystine Inherited recessive disorder of transmembrane cystine

transport leading to decreased absorption of cystine

from intestine and renal tubule

Multiple stones may form

Relatively radiodense because they contain sulphur

1%

Uric acid Uric acid is a product of purine metabolism

May precipitate when urinary pH low

May be caused by diseases with extensive tissue

breakdown e.g. malignancy

More common in children with inborn errors of

metabolism

Radiolucent

5-10%

Calcium

phosphate

May occur in renal tubular acidosis, high urinary pH

increases supersaturation of urine with calcium and

phosphate

Renal tubular acidosis types 1 and 3 increase risk of

stone formation (types 2 and 4 do not)

Radio-opaque stones (composition similar to bone)

10%

Struvite Stones formed from magnesium, ammonium and

phosphate

Occur as a result of urease producing bacteria (and are

thus associated with chronic infections)

Under the alkaline conditions produced, the crystals

can precipitate

Slightly radio-opaque

2-20%

Effect of urinary pH on stone formation Urine pH will show individual variation (from pH 5-7). Post prandially the pH falls as

purine metabolism will produce uric acid. Then the urine becomes more alkaline

(alkaline tide). When the stone is not available for analysis the pH of urine may help

to determine which stone was present.

Stone type Urine acidity Mean urine pH

Calcium phosphate Normal- alkaline >5.5

Calcium oxalate Variable 6

Uric acid Acid 5.5

Struvate Alkaline >7.2

Cystine Normal 6.5

A 64 year old man presents to the clinic with

right upper quadrant discomfort. He has never attended the hospital previously and is

usually well. He has just retired from full time employment as a machinist in a PVC

factory. CT scanning shows a large irregular tumour in the right lobe of his liver.

Which of the following lesions is the most likely?

A. Liposarcoma

B. Angiosarcoma

C. Hamartoma

D. Hyatid liver disease

E. Benign angioma

Angiosarcoma of the liver is a rare tumour. However, it is linked to working with

vinyl chloride, as in this case. Although modern factories minimise the exposure to

this agent, this has not always been the case.

Occupational cancers

Occupational cancers accounted for 5.3% cancer deaths in 2005.

In men the main cancers include:

Mesothelioma

Bladder cancer

Non melanoma skin cancer

Lung cancer

Sino nasal cancer

Occupations with high levels of occupational tumours include:

Construction industry

Working with coal tar and pitch

Mining

Metalworkers

Working with asbestos (accounts for 98% of all mesotheliomas)

Working in rubber industry

Shift work has been linked to breast cancer in women (Health and safety executive

report RR595).

The latency between exposure and disease is typically 15 years for solid tumours and

20 for leukaemia.

Many occupational cancers are otherwise rare. For example sino nasal cancer is an

uncommon tumour, 50% will be SCC. They are linked to conditions such as wood

dust exposure and unlike lung cancer is not strongly linked to cigarette smoking.

Another typical occupational tumour is angiosarcoma of the liver which is linked to

working with vinyl chloride. Again in the non occupational context this is an

extremely rare sporadic tumour.

A 64 year old man presents to the clinic with right upper quadrant discomfort. He has

never attended the hospital previously and is usually well. He has just retired from full

time employment as a machinist in a PVC factory. CT scanning shows a large

irregular tumour in the right lobe of his liver. Which of the following lesions is the

most likely?

A. Liposarcoma

B. Angiosarcoma

C. Hamartoma

D. Hyatid liver disease

E. Benign angioma

Angiosarcoma of the liver is a rare tumour. However, it is linked to working with

vinyl chloride, as in this case. Although modern factories minimise the exposure to

this agent, this has not always been the case.

Occupational cancers

Occupational cancers accounted for 5.3% cancer deaths in 2005.

In men the main cancers include:

Mesothelioma

Bladder cancer

Non melanoma skin cancer

Lung cancer

Sino nasal cancer

Occupations with high levels of occupational tumours include:

Construction industry

Working with coal tar and pitch

Mining

Metalworkers

Working with asbestos (accounts for 98% of all mesotheliomas)

Working in rubber industry

Shift work has been linked to breast cancer in women (Health and safety executive

report RR595).

The latency between exposure and disease is typically 15 years for solid tumours and

20 for leukaemia.

Many occupational cancers are otherwise rare. For example sino nasal cancer is an

uncommon tumour, 50% will be SCC. They are linked to conditions such as wood

dust exposure and unlike lung cancer is not strongly linked to cigarette smoking.

Another typical occupational tumour is angiosarcoma of the liver which is linked to

working with vinyl chloride. Again in the non occupational context this is an

extremely rare sporadic tumour.

A 32 year old man is involved in a house fire and sustains extensive partial thickness

burns to his torso and thigh. Two weeks post operatively he develops oedema of both

lower legs. The most likely cause of this is:

A. Iliofemoral deep vein thrombosis

B. Venous obstruction due to scarring

C. Hypoalbuminaemia

D. Excessive administration of intravenous fluids

E. None of the above

Theme from 2009 Exam

Loss of plasma proteins is the most common cause of oedema developing in this time

frame.

Burns pathology

Extensive burns

Haemolysis due to damage of erythrocytes by heat and microangiopathy

Loss of capillary membrane integrity causing plasma leakage into interstitial

space

Extravasation of fluids from the burn site causing hypovolaemic shock (up to

48h after injury)- decreased blood volume and increased haematocrit

Protein loss

Secondary infection e.g. Staphylococcus aureus

ARDS

Risk of Curlings ulcer (acute peptic stress ulcers)

Danger of full thickness circumferential burns in an extremity as these may

develop compartment syndrome

Healing

Superficial burns: keratinocytes migrate to form a new layer over the burn site

Full thickness burns: dermal scarring. Usually need keratinocytes from skin

grafts to provide optimal coverage.

What is the diagnostic marker for carcinoid syndrome?

A. B-HCG

B. Histamine

C. Chromogranin A

D. 5-Hydroxyindoleacetic acid

E. 5-Hydroxytryptamine

Urinary measurement of 5- HIAA is an important part of clinical follow up.

Carcinoid syndrome

Carcinoid tumours secrete serotonin

Originate in neuroendocrine cells mainly in the intestine (midgut-distal

ileum/appendix)

Can occur in the rectum, bronchi

Hormonal symptoms mainly occur when disease spreads outside the bowel

Clinical features - Onset: years

- Flushing face

- Palpitations

- Tricuspid stenosis causing dyspnoea

- Asthma

- Severe diarrhoea (secretory, persists despite fasting)

Investigation - 5-HIAA in a 24-hour urine collection

- Scintigraphy

- CT scan

Treatment

Octreotide

Surgical removal

A 42 year old man from Southern India presents with chronic swelling of both lower

legs, they are brawny and indurated with marked skin tophic changes. Which of the

following organisms is the most likely origin of this disease process?

A. Loa loa

B. Wuchereria bancrofti

C. Trypanosoma cruzi

D. Trypanosoma gambiense

E. None of the above

W. Bancrofti is the commonest cause of filariasis leading to lymphatic obstruction.

Infection with Loa loa typically occurs in the African sub continent and usually

results in generalised sub cutaneous infections without lymphatic obstruction.

Trypanosomal infections would not produce this clinical picture.

Wuchereria bancrofti

Parasitic filarial nematode

Accounts for 90% of cases of filariasis

Usually diagnosed by blood smears

Usually transmitted by mosquitos

Treatment is with diethylcarbamazine

A 45 year old lady has recently undergone a thyroidectomy for treatment of medullary

thyroid cancer. Which of the following tumour markers is used clinically to screen for

recurrence?

A. Free T3

B. Thyroglobulin

C. Calcitonin

D. Free T4

E. Thyroid stimulating hormone

Theme from 2011 Exam

Calcitonin is clinically utilised to screen for medullary thyroid cancer recurrence.

Thyroid function testing does not form part of either diagnosis or follow up from a

malignancy perspective. However, routine assessment of TSH may be needed in

patients on thyroxine.

Thyroid malignancy

Papillary carcinoma

Commonest sub-type

Accurately diagnosed on fine needle aspiration cytology

Histologically they may demonstrate psammoma bodies (areas of

calcification) and so called 'orphan Annie' nuclei

They typically metastasise via the lymphatics and thus laterally located

apparently ectopic thyroid tissue is usually a metastasis from a well

differentiated papillary carcinoma.

Follicular carcinoma

Are less common than papillary lesions

Like papillary tumours they may present as a discrete nodule. Although they

appear to be well encapsulated macroscopically there invasion on microscopic

evaluation.

Lymph node metastases are uncommon and these tumours tend to spread

haematogenously. This translates into a higher mortality rate.

Follicular lesions cannot be accurately diagnosed on fine needle aspiration

cytology and thus all follicular FNA's will require at least a hemi

thyroidectomy.

Anaplastic carcinoma

Less common and tend to occur in elderly females

Disease is usually advanced at presentation and often only palliative

decompression and radiotherapy can be offered.

Medullary carcinoma

These are tumours of the parafollicular cells ( C Cells) and are of neural crest

origin.

The serum calcitonin may be elevated which is of use when monitoring for

recurrence.

They may be familial and occur as part of the MEN -2A disease spectrum.

Spread may be either lymphatic or haematogenous and as these tumours are

not derived primarily from thyroid cells they are not responsive to radioiodine.

Lymphoma

These respond well to radiotherapy

Radical surgery is unnecessary once the disease has been diagnosed on biopsy

material. Such biopsy material is not generated by an FNA and thus a core

biopsy has to be obtained (with care!).

A 45 year old lady has recently undergone a thyroidectomy for treatment of medullary

thyroid cancer. Which of the following tumour markers is used clinically to screen for

recurrence?

A. Free T3

B. Thyroglobulin

C. Calcitonin

D. Free T4

E. Thyroid stimulating hormone

Theme from 2011 Exam

Calcitonin is clinically utilised to screen for medullary thyroid cancer recurrence.

Thyroid function testing does not form part of either diagnosis or follow up from a

malignancy perspective. However, routine assessment of TSH may be needed in

patients on thyroxine.

Thyroid malignancy

Papillary carcinoma

Commonest sub-type

Accurately diagnosed on fine needle aspiration cytology

Histologically they may demonstrate psammoma bodies (areas of

calcification) and so called 'orphan Annie' nuclei

They typically metastasise via the lymphatics and thus laterally located

apparently ectopic thyroid tissue is usually a metastasis from a well

differentiated papillary carcinoma.

Follicular carcinoma

Are less common than papillary lesions

Like papillary tumours they may present as a discrete nodule. Although they

appear to be well encapsulated macroscopically there invasion on microscopic

evaluation.

Lymph node metastases are uncommon and these tumours tend to spread

haematogenously. This translates into a higher mortality rate.

Follicular lesions cannot be accurately diagnosed on fine needle aspiration

cytology and thus all follicular FNA's will require at least a hemi

thyroidectomy.

Anaplastic carcinoma

Less common and tend to occur in elderly females

Disease is usually advanced at presentation and often only palliative

decompression and radiotherapy can be offered.

Medullary carcinoma

These are tumours of the parafollicular cells ( C Cells) and are of neural crest

origin.

The serum calcitonin may be elevated which is of use when monitoring for

recurrence.

They may be familial and occur as part of the MEN -2A disease spectrum.

Spread may be either lymphatic or haematogenous and as these tumours are

not derived primarily from thyroid cells they are not responsive to radioiodine.

Lymphoma

These respond well to radiotherapy

Radical surgery is unnecessary once the disease has been diagnosed on biopsy

material. Such biopsy material is not generated by an FNA and thus a core

biopsy has to be obtained (with care!).

A 22 year old man is kicked in the head during a rugby match. He is temporarily

concussed, but then regains consciousness. Half an hour later he develops slurred

speech, ataxia and loses consciousnesses. On arrival in hospital he is intubated and

ventilated. A CT Scan is performed which shows an extradural haematoma. What is

the most likely cause?

A. Basilar artery laceration

B. Middle meningeal artery laceration

C. Laceration of the sigmoid sinus

D. Laceration of the anterior cerebral artery

E. Laceration of the middle cerebral artery

Theme based on September 2011 Exam

The most likely vessel from those in the list to cause an acute extra dural haemorrhage

is the middle meningeal artery. The anterior and middle cerebral arteries may cause

acute sub dural haemorrhage. Acute sub dural haemorrhages usually take slightly

longer to evolve than acute extra dural haemorrhages.

Middle meningeal artery

Middle meningeal artery is typically the third branch of the first part of the

maxillary artery, one of the two terminal branches of the external carotid

artery. After branching off the maxillary artery in the infratemporal fossa, it

runs through the foramen spinosum to supply the dura mater (the outermost

meninges) .

The middle meningeal artery is the largest of the three (paired) arteries which

supply the meninges, the others being the anterior meningeal artery and the

posterior meningeal artery.

The middle meningeal artery runs beneath the pterion. It is vulnerable to

injury at this point, where the skull is thin. Rupture of the artery may give rise

to an extra dural hematoma.

In the dry cranium, the middle meningeal, which runs within the dura mater

surrounding the brain, makes a deep indention in the calvarium.

The middle meningeal artery is intimately associated with the

auriculotemporal nerve which wraps around the artery making the two easily

identifiable in the dissection of human cadavers and also easily damaged in

surgery.

A 22 year old man is kicked in the head during a rugby match. He is temporarily

concussed, but then regains consciousness. Half an hour later he develops slurred

speech, ataxia and loses consciousnesses. On arrival in hospital he is intubated and

ventilated. A CT Scan is performed which shows an extradural haematoma. What is

the most likely cause?

A. Basilar artery laceration

B. Middle meningeal artery laceration

C. Laceration of the sigmoid sinus

D. Laceration of the anterior cerebral artery

E. Laceration of the middle cerebral artery

Theme based on September 2011 Exam

The most likely vessel from those in the list to cause an acute extra dural haemorrhage

is the middle meningeal artery. The anterior and middle cerebral arteries may cause

acute sub dural haemorrhage. Acute sub dural haemorrhages usually take slightly

longer to evolve than acute extra dural haemorrhages.

Middle meningeal artery

Middle meningeal artery is typically the third branch of the first part of the

maxillary artery, one of the two terminal branches of the external carotid

artery. After branching off the maxillary artery in the infratemporal fossa, it

runs through the foramen spinosum to supply the dura mater (the outermost

meninges) .

The middle meningeal artery is the largest of the three (paired) arteries which

supply the meninges, the others being the anterior meningeal artery and the

posterior meningeal artery.

The middle meningeal artery runs beneath the pterion. It is vulnerable to

injury at this point, where the skull is thin. Rupture of the artery may give rise

to an extra dural hematoma.

In the dry cranium, the middle meningeal, which runs within the dura mater

surrounding the brain, makes a deep indention in the calvarium.

The middle meningeal artery is intimately associated with the

auriculotemporal nerve which wraps around the artery making the two easily

identifiable in the dissection of human cadavers and also easily damaged in

surgery.

Which of the following is not characteristic of a granuloma?

A. Altered macrophages

B. Fused macrophages

C. Epithelioid cells

D. Mixture of chronic inflammatory cells

E. Polymorphnuclear leucocytes, cellular debris and fibrin

These are typical components of an abscess cavity. Polymorphonuclear leucocytes

may be found in a granuloma if there is a focus of suppuration.

Chronic inflammation

Overview Chronic inflammation may occur secondary to acute inflammation.In most cases

chronic inflammation occurs as a primary process. These may be broadly viewed as

being one of three main processes:

Persisting infection with certain organisms such as Mycobacterium

tuberculosis which results in delayed type hypersensitivity reactions and

inflammation.

Prolonged exposure to non-biodegradable substances such as silica or suture

materials which may induce an inflammatory response.

Autoimmune conditions involving antibodies formed against host antigens.

Acute vs. Chronic inflammation

Acute inflammation Chronic inflammation

Changes to existing vascular structure and

increased permeability of endothelial cells

Angiogenesis predominates

Infiltration of neutrophils Macrophages, plasma cells and

lymphocytes predominate

Process may resolve with: Healing by fibrosis is the main

result

Suppuration

Complete resolution

Abscess formation

Progression to chronic inflammation

Healing by fibrosis

Granulomatous inflammation A granuloma consists of a microscopic aggregation of macrophages (with epithelial

type arrangement =epitheliod). Large giant cells may be found at the periphery of

granulomas.

Mediators Growth factors released by activated macrophages include agents such as interferon

and fibroblast growth factor (plus many more). Some of these such as interferons may

have systemic features resulting in systemic symptoms and signs, which may be

present in individuals with long standing chronic inflammation.

The finding of granulomas is pathognomonic of chronic inflammation, as illustrated

in this biopsy from a patient with colonic Crohns disease

Image sourced from Wikipedia

A 42 year old man presents with a painless lump in the left testicle that he noticed on

self examination. Clinically there is a firm nodule in the left testicle, ultrasound

appearances show an irregular mass lesion. His serum AFP and HCG levels are both

within normal limits. What is the most likely diagnosis?

A. Yolk sack tumour

B. Seminoma

C. Testicular teratoma

D. Epididymo-orchitis

E. Adenomatoid tumour

Seminomas typically have normal AFP and HCG. These are usually raised in

teratomas and yolk sac tumours

This mans age, presenting symptoms and normal tumour markers make a seminoma

the most likely diagnosis. Epididymo-orchitis does not produce irregular mass lesions

which are painless.

Testicular disorders

Testicular cancer Testicular cancer is the most common malignancy in men aged 20-30 years. Around

95% of cases of testicular cancer are germ-cell tumours. Germ cell tumours may

essentially be divided into:

Tumour type Key features Tumour

markers

Pathology

Seminoma Commonest

subtype (50%)

Average age at

diagnosis = 40

Even advanced

disease

associated with

5 year survival

of 73%

AFP usually

normal

HCG

elevated in 10%

seminomas

Lactate

dehydrogenase;

elevated in 10-

20% seminomas

(but also in

many other

conditions)

Sheet like

lobular

patterns of

cells with

substantial

fibrous

component.

Fibrous septa

contain

lymphocytic

inclusions and

granulomas

may be seen.

Non seminomatous germ

cell tumours (42%)

Teratoma

Yolk sac tumour

Choriocarcinoma

Mixed germ cell

tumours (10%)

Younger age at

presentation =20-30

years

Advanced disease

carries worse prognosis

(48% at 5 years)

Retroperitoneal

lymph node dissection

may be needed for

residual disease after

chemotherapy

AFP elevated

in up to 70% of

cases

HCG

elevated in up

to 40% of cases

Other

markers rarely

helpful

Heterogenous

texture with

occasional

ectopic tissue

such as hair

Image demonstrating a classical seminoma, these tumours are typically more uniform

than teratomas

Image sourced from Wikipedia

Risk factors for testicular cancer

Cryptorchidism

Infertility

Family history

Klinefelter's syndrome

Mumps orchitis

Features

A painless lump is the most common presenting symptom

Pain may also be present in a minority of men

Other possible features include hydrocele, gynaecomastia

Diagnosis

Ultrasound is first-line

CT scanning of the chest/ abdomen and pelvis is used for staging

Tumour markers (see above) should be measured

Management

Orchidectomy (Inguinal approach)

Chemotherapy and radiotherapy may be given depending on staging

Abdominal lesions >1cm following chemotherapy may require retroperitoneal

lymph node dissection.

Prognosis is generally excellent

5 year survival for seminomas is around 95% if Stage I

5 year survival for teratomas is around 85% if Stage I

Benign disease

Epididymo-orchitis Acute epididymitis is an acute inflammation of the epididymis, often involving the

testis and usually caused by bacterial infection.

Infection spreads from the urethra or bladder. In men <35 years, gonorrhoea or

chlamydia are the usual infections.

Amiodarone is a recognised non infective cause of epididymitis, which

resolves on stopping the drug.

Tenderness is usually confined to the epididymis, which may facilitate

differentiating it from torsion where pain usually affects the entire testis.

Testicular torsion

Twist of the spermatic cord resulting in testicular ischaemia and necrosis.

Most common in males aged between 10 and 30 (peak incidence 13-15 years)

Pain is usually severe and of sudden onset.

Cremasteric reflex is lost and elevation of the testis does not ease the pain.

Treatment is with surgical exploration. If a torted testis is identified then both

testis should be fixed as the condition of bell clapper testis is often bilateral.

A baby is born by normal vaginal delivery at 39 weeks gestation. Initially all appears

well and then the clinical staff become concerned because the baby develops recurrent

episodes of cyanosis. These are worse during feeding and improve dramatically when

the baby cries. The most likely underlying diagnosis is:

A. Choanal atresia

B. Oesophageal reflux

C. Tetralogy of Fallot

D. Oesophageal atresia

E. Congenital diaphragmatic hernia

Question theme from 2011 exam

In Choanal atresia the episodes of cyanosis are usually worst during feeding.

Improvement may be seen when the baby cries as the oropharyngeal airway is used.

Choanal atresia

Congenital disorder with an incidence of 1 in 7000 births.

Posterior nasal airway occluded by soft tissue or bone.

Associated with other congenital malformations e.g. coloboma

Babies with unilateral disease may go unnoticed.

Babies with bilateral disease will present early in life as they are obligate nasal

breathers.

Treatment is with fenestration procedures designed to restore patency.

A 28 year old lady presents with a pigmented lesion on her calf. Excisional biopsy

confirms a diagnosis of melanoma measuring 1cm in diameter with a Breslow

thickness of 0.5mm. The lesion is close <1 mm to all resection margins. Which of the

following surgical resection margins is acceptable for this lesion?

A. 5 cm

B. 1 cm

C. 0.5 cm

D. 2 cm

E. 3 cm

Malignant melanoma

The main diagnostic features (major

criteria):

Secondary features (minor

criteria)

Change in size

Change in shape

Change in colour

Diameter >6mm

Inflammation

Oozing or bleeding

Altered sensation

Treatment

Suspicious lesions should undergo excision biopsy. The lesion should be

removed in completely as incision biopsy can make subsequent

histopathological assessment difficult.

Once the diagnosis is confirmed the pathology report should be reviewed to

determine whether further re-exicision of margins is required (see below):

Margins of excision-Related to Breslow thickness

Lesions 0-1mm thick 1cm

Lesions 1-2mm thick 1- 2cm (Depending upon site and pathological features)

Lesions 2-4mm thick 2-3 cm (Depending upon site and pathological features)

Lesions >4 mm thick 3cm

Marsden J et al Revised UK guidelines for management of Melanoma. Br J Dermatol

2010 163:238-256.

Further treatments such as sentinel lymph node mapping, isolated limb perfusion and

block dissection of regional lymph node groups should be selectively applied.

A 20 year old man is involved in a road traffic accident. Following the incident he is

unable to extend his wrist. However, this improves over the following weeks. Which

type of injury is he most likely to have sustained?

A. Radial nerve neurotmesis

B. Radial nerve neuropraxia

C. Axillary nerve axonotmesis

D. Ulnar nerve neuropraxia

E. Ulnar nerve axonotmesis

Theme from April 2011 Exam

Transient loss of function makes neuropraxia the most likely injury. The wrist

extensors are innervated by the radial nerve making this the most likely site of injury.

Neuropraxia

Nerve intact but electrical conduction is affected

Myelin sheath integrity is preserved

Full recovery

Autonomic function preserved

Wallerian degeneration does not occur

A 53 year old lady has undergone a bilateral breast augmentation procedure

many years previously. The implants are tense and uncomfortable and are

removed. During their removal the surgeon encounters a dense membrane

surrounding the implants, it has a coarse granular appearance. The tissue is

sent for histology and it demonstrates fibrosis with the presence of

calcification. The underlying process responsible for these changes is:

A. Hyperplasia

B. Dysplasia

C. Metastatic calcification

D. Dystrophic calcification

E. Necrosis

Breast implants often become surrounded by a pseudocapsule and this may

secondarily then be subjected to a process of dystrophic calcification.

Pathological calcification

Dystrophic

calcification

Deposition of calcium deposits in tissues that have undergone,

degeneration, damage or disease in the presence of normal serum

calcium levels

Metastatic

calcification

Deposition of calcium deposits in tissues that are otherwise normal

in the presence of increased serum calcium levels

A 4 year old girl presents with symptoms of right sided loin pain, lethargy and

haematuria. On examination she is pyrexial and has a large mass in the right upper

quadrant. The most likely underlying diagnosis is:

A. Perinephric abscess

B. Nephroblastoma

C. Renal cortical adenoma

D. Grawitz tumour

E. Squamous cell carcinoma of the kidney

In a child of this age, with the symptoms described a nephroblastoma is the most

likely diagnosis. A perinephric abscess is most unlikely. If an abscess were to occur it

would be confined to Gertotas fascia in the first instance, and hence anterior extension

would be unlikely.

Nephroblastoma

Nephroblastoma (Wilms tumours)

Usually present in first 4 years of life

May often present as a mass associated with haematuria (pyrexia may occur in

50%)

Often metastasise early (usually to lung)

Treated by nephrectomy

Younger children have better prognosis (<1 year of age =80% overall 5 year

survival)

Theme: Thyroid neoplasms

A. Follicular carcinoma

B. Anaplastic carcinoma

C. Medullary carcinoma

D. Papillary carcinoma

E. Lymphoma

F. Hashimotos thyroiditis

G. Graves disease

For the following histological descriptions please select the most likely underlying

thyroid neoplasm. Each option may be used once, more than once or not at all.

45. A 22 year old female undergoes a thyroidectomy. The resected specimen

shows a non encapsulated tumour with papillary projections and pale empty

nuclei.

Papillary carcinoma

Theme from April 2012

The presence of papillary structures together with the cytoplasmic features

described is strongly suggestive of papillary carcinoma. They are seldom

encapsulated.

46. A thyroidectomy specimen from a 43 year old lady shows a mass with

prominent oxyphil cells and scanty thyroid colloid.

You answered Medullary carcinoma

The correct answer is Follicular carcinoma

Hurthle cell tumours are a variant of follicular neoplasms in which oxyphil

cells predominate. They have a poorer prognosis than conventional follicular

neoplasms

47. A 32 year old lady undergoes a thyroidectomy for a mild goitre. The resected

specimen shows an intense lymphocytic infiltrate with acinar destruction and

fibrosis.

You answered Lymphoma

The correct answer is Hashimotos thyroiditis

Lymphocytic infiltrates and fibrosis are typically seen in Hashimotos

thyroiditis. In Lymphoma only dense lymphatic type tissue is usually present.

Thyroid neoplasms

Lesion Common features

Follicular

adenoma

Usually present as a solitary thyroid nodule

Malignancy can only be excluded on formal histological

assessment

Papillary

carcinoma

Usually contain a mixture of papillary and colloidal filled

follicles

Histologically tumour has papillary projections and pale empty

nuclei

Seldom encapsulated

Lymph node metastasis predominate

Haematogenous metastasis rare

Account for 60% of thyroid cancers

Follicular

carcinoma

May appear macroscopically encapsulated, microscopically

capsular invasion is seen. Without this finding the lesion is a

follicular adenoma.

Vascular invasion predominates

Multifocal disease rare

Account for 20% of all thyroid cancers

Anaplastic

carcinoma

Most common in elderly females

Local invasion is a common feature

Account for 10% of thyroid cancers

Treatment is by resection where possible, palliation may be

achieved through isthmusectomy and radiotherapy.

Chemotherapy is ineffective.

Medullary

carcinoma

Tumours of the parafollicular cells (C Cells)

C cells derived from neural crest and not thyroid tissue

Serum calcitonin levels often raised

Familial genetic disease accounts for up to 20% cases

Both lymphatic and haematogenous metastasis are recognised,

nodal disease is associated with a very poor prognosis.

Theme: Thyroid neoplasms

A. Follicular carcinoma

B. Anaplastic carcinoma

C. Medullary carcinoma

D. Papillary carcinoma

E. Lymphoma

F. Hashimotos thyroiditis

G. Graves disease

For the following histological descriptions please select the most likely underlying

thyroid neoplasm. Each option may be used once, more than once or not at all.

45. A 22 year old female undergoes a thyroidectomy. The resected specimen

shows a non encapsulated tumour with papillary projections and pale empty

nuclei.

Papillary carcinoma

Theme from April 2012

The presence of papillary structures together with the cytoplasmic features

described is strongly suggestive of papillary carcinoma. They are seldom

encapsulated.

46. A thyroidectomy specimen from a 43 year old lady shows a mass with

prominent oxyphil cells and scanty thyroid colloid.

You answered Medullary carcinoma

The correct answer is Follicular carcinoma

Hurthle cell tumours are a variant of follicular neoplasms in which oxyphil

cells predominate. They have a poorer prognosis than conventional follicular

neoplasms

47. A 32 year old lady undergoes a thyroidectomy for a mild goitre. The resected

specimen shows an intense lymphocytic infiltrate with acinar destruction and

fibrosis.

You answered Lymphoma

The correct answer is Hashimotos thyroiditis

Lymphocytic infiltrates and fibrosis are typically seen in Hashimotos

thyroiditis. In Lymphoma only dense lymphatic type tissue is usually present.

Thyroid neoplasms

Lesion Common features

Follicular

adenoma

Usually present as a solitary thyroid nodule

Malignancy can only be excluded on formal histological

assessment

Papillary

carcinoma

Usually contain a mixture of papillary and colloidal filled

follicles

Histologically tumour has papillary projections and pale empty

nuclei

Seldom encapsulated

Lymph node metastasis predominate

Haematogenous metastasis rare

Account for 60% of thyroid cancers

Follicular

carcinoma

May appear macroscopically encapsulated, microscopically

capsular invasion is seen. Without this finding the lesion is a

follicular adenoma.

Vascular invasion predominates

Multifocal disease rare

Account for 20% of all thyroid cancers

Anaplastic

carcinoma

Most common in elderly females

Local invasion is a common feature

Account for 10% of thyroid cancers

Treatment is by resection where possible, palliation may be

achieved through isthmusectomy and radiotherapy.

Chemotherapy is ineffective.

Medullary

carcinoma

Tumours of the parafollicular cells (C Cells)

C cells derived from neural crest and not thyroid tissue

Serum calcitonin levels often raised

Familial genetic disease accounts for up to 20% cases

Both lymphatic and haematogenous metastasis are recognised,

nodal disease is associated with a very poor prognosis.

A 43 year old lady with hypertension is suspected of having a phaeochromocytoma.

Which of the following investigations is most likely to be beneficial in this situation?

A. Dexamethasone suppression test

B. Urinary 5-Hydroxyindoleacetic Acid (5-HIAA)

C. Histamine provocation test

D. Tyramine provocation test

E. Urinary vanillymandelic acid measurements

Theme from September 2011 Exam

Theme from September 2012 Exam

Urinary VMA measurements are not completely specific but constitute first line

assessment. Stimulation tests of any sort are not justified in first line assessments.

Phaeochromocytoma and adrenal lesions

Phaeochromocytoma Neuroendocrine tumour of the chromaffin cells of the adrenal medulla. Hypertension

and hyperglycaemia are often found.

10% of cases are bilateral.

10% occur in children.

11% are malignant (higher when tumour is located outside the adrenal).

10% will not be hypertensive.

Familial cases are usually linked to the Multiple endocrine neoplasia syndromes

(considered under its own heading).

Most tumours are unilateral (often right sided) and smaller than 10cm.

Diagnosis

Urine analysis of vanillymandelic acid (VMA) is often used (false positives may

occur e.g. in patients eating vanilla ice cream!)

Blood testing for plasma metanephrine levels.

CT and MRI scanning are both used to localise the lesion.

Treatment Patients require medical therapy first. An irreversible alpha adrenoreceptor blocker

should be given, although minority may prefer reversible bockade(1). Labetolol may

be co-administered for cardiac chronotropic control. Isolated beta blockade should not

be considered as it will lead to unopposed alpha activity.

These patients are often volume depleted and will often require moderate volumes of

intra venous normal saline perioperatively.

Once medically optimised the phaeochromocytoma should be removed. Most

adrenalectomies can now be performed using a laparoscopic approach(2). The

adrenals are highly vascular structures and removal can be complicated by

catastrophic haemorrhage in the hands of the inexperienced. This is particularly true

of right sided resections where the IVC is perilously close. Should the IVC be

damaged a laparotomy will be necessary and the defect enclosed within a Satinsky

style vascular clamp and the defect closed with prolene sutures. Attempting to

interfere with the IVC using any instruments other than vascular clamps will result in

vessel trauma and make a bad situation much worse.

Incidental adrenal lesions Adrenal lesions may be identified on CT scanning performed for other reasons(3).

Factors suggesting benign disease on CT include(4):

Size less than 3cm

Homogeneous texture

Lipid rich tissue

Thin wall to lesion

All patients with incidental lesions should be managed jointly with an endocrinologist

and full work up as described above. Patients with functioning lesions or those with

adverse radiological features (Particularly size >3cm) should proceed to surgery.

References 1. Weingarten TN, Cata JP, O'Hara JF, Prybilla DJ, Pike TL, Thompson GB, et al.

Comparison of two preoperative medical management strategies for laparoscopic

resection of pheochromocytoma. Urology. 2010 Aug;76(2):508 e6-11.

2. Nguyen PH, Keller JE, Novitsky YW, Heniford BT, Kercher KW. Laparoscopic

approach to adrenalectomy: review of perioperative outcomes in a single center. Am

Surg. 2011 May;77(5):592-6.

3. Ng VW, Ma RC, So WY, Choi KC, Kong AP, Cockram CS, et al. Evaluation of

functional and malignant adrenal incidentalomas. Arch Intern Med. 2010 Dec

13;170(22):2017-20.

4. Muth A, Hammarstedt L, Hellstrom M, Sigurjonsdottir HA, Almqvist E, Wangberg

B. Cohort study of patients with adrenal lesions discovered incidentally. Br J Surg.

2011 May 27.

A 46 year old lady presents with symptoms of diarrhoea, weight loss of 10 Kg and a

skin rash of erythematous blisters involving the abdomen and buttocks. The blisters

have an irregular border and both intact and ruptured vesicles. What is the most likely

diagnosis?

A. Colonic adenocarcinoma

B. Pancreatic adenocarcinoma

C. Tropical sprue

D. Glucagonoma

E. Insulinoma

Theme from September 2011 Exam

Theme from September 2012 Exam

Glucagonoma is strongly associated with necrolytic migratory erythema.

Glucagonoma

Rare pancreatic tumours arising from the alpha cells of the pancreas.

Glucagon levels markedly elevated.

Symptoms include diarrhoea, weight loss and necrolytic migratory erythema.

A serum level of glucagon >1000pg/ml usually suggests the diagnosis,

imaging with CT scanning is also required.

Treatment is with surgical resection.

A 56 year old man presents with symptoms of neuropathic facial pain and some

weakness of the muscles of facial expression on the right side. On examination he has

a hard mass approximately 6cm anterior to the right external auditory meatus. What is

the most likely diagnosis?

A. Pleomorphic adenoma

B. Adenocarcinoma

C. Mucoepidermoid carcinoma

D. Adenoid cystic carcinoma

E. Lymphoma

Theme from September 2011 Exam

The patient is most likely to have a malignant lesion within the parotid. Of the

malignancies listed; adenoid cystic carcinoma has the greatest tendency to perineural

invasion.

Parotid gland malignancy

Most parotid neoplasms (80%) are benign lesions

Most commonly present with painless mass in cheek region

Up to 30% may present with pain, when this is associated with a discrete mass

lesion in the parotid it usually indicates perineural invasion.

Perineural invasion is very unlikely to occur in association with benign lesions

80% of patients with facial nerve weakness caused by parotid malignancies

will have nodal metastasis and a 5 year survival of 25%

Types of malignancy

Mucoepidermoid

carcinoma

30% of all parotid malignancies

Usually low potential for local invasiveness and metastasis

(depends mainly on grade)

Adenoid cystic

carcinoma

Unpredictable growth patter

Tendency for perineural spread

Nerve growth may display skip lesions resulting in incomplete

excision

Distant metastasis more common (visceral rather than nodal

spread)

5 year survival 35%

Mixed tumours Often a malignancy occurring in a previously benign parotid

lesion

Acinic cell carcinoma Intermediate grade malignancy

May show perineural invasion

Low potential for distant metastasis

5 year survival 80%

Adenocarcinoma Develops from secretory portion of gland

Risk of regional nodal and distant metastasis

5 year survival depends upon stage at presentation, may be up

to 75% with small lesions with no nodal involvement

Lymphoma Large rubbery lesion, may occur in association with Warthins

tumours

Diagnosis should be based on regional nodal biopsy rather

than parotid resection Treatment is with chemotherapy (and

radiotherapy)

A 20 year old African lady undergoes an open appendicectomy. She is reviewed for

an unrelated problem 8 months later. On abdominal inspection the wound site is

covered by shiny dark protuberant scar tissue that projects beyond the limits of the

skin incision. Which of the following is the most likely underlying process?

A. Hypertrophic scar

B. Keloid scar

C. Marjolins ulcer

D. Repeated episodes of wound sepsis

E. Mycosis fungoides

Keloid scars extend beyond the limits of the incision. Mycosis fungoides is a

cutaneous T cell lymphoma.

Wound healing

Surgical wounds are either incisional or excisional and either clean, clean

contaminated or dirty. Although the stages of wound healing are broadly similar their

contributions will vary according to the wound type.

The main stages of wound healing include:

Haemostasis

Vasospasm in adjacent vessels, platelet plug formation and generation of

fibrin rich clot.

Inflammation

Neutrophils migrate into wound (function impaired in diabetes).

Growth factors released, including basic fibroblast growth factor and vascular

endothelial growth factor.

Fibroblasts replicate within the adjacent matrix and migrate into wound.

Macrophages and fibroblasts couple matrix regeneration and clot substitution.

Regeneration

Platelet derived growth factor and transformation growth factors stimulate

fibroblasts and epithelial cells.

Fibroblasts produce a collagen network.

Angiogenesis occurs and wound resembles granulation tissue.

Remodeling

Longest phase of the healing process and may last up to one year (or longer).

During this phase fibroblasts become differentiated (myofibroblasts) and these

facilitate wound contraction.

Collagen fibres are remodeled.

Microvessels regress leaving a pale scar.

The above description represents an idealised scenario. A number of diseases may

distort this process. It is obvious that one of the key events is the establishing well

vascularised tissue. At a local level angiogenesis occurs, but if arterial inflow and

venous return are compromised then healing may be impaired, or simply nor occur at

all. The results of vascular compromise are all too evidence in those with peripheral

vascular disease or those poorly constructed bowel anastomoses.

Conditions such as jaundice will impair fibroblast synthetic function and overall

immunity with a detrimental effect in most parts of healing.

Problems with scars:

Hypertrophic scars Excessive amounts of collagen within a scar. Nodules may be present histologically

containing randomly arranged fibrils within and parallel fibres on the surface. The

tissue itself is confined to the extent of the wound itself and is usually the result of a

full thickness dermal injury. They may go on to develop contractures.

Image of hypertrophic scarring. Note that it remains confined to the boundaries of the

original wound:

Image sourced from Wikipedia

Keloid scars Excessive amounts of collagen within a scar. Typically a keloid scar will pass beyond

the boundaries of the original injury. They do not contain nodules and may occur

following even trivial injury. They do not regress over time and may recur following

removal.

Image of a keloid scar. Note the extension beyond the boundaries of the original

incision:

Image sourced from Wikipedia

Drugs which impair wound healing:

Non steroidal anti inflammatory drugs

Steroids

Immunosupressive agents

Anti neoplastic drugs

Closure Delayed primary closure is the anatomically precise closure that is delayed for a few

days but before granulation tissue becomes macroscopically evident.

Secondary closure refers to either spontaneous closure or to surgical closure after

granulation tissue has formed.

The pathogenicity of the tubercle bacillus is due to which of the following?

A. Necrosis caused by expanding granulomas

B. Ability to multiply within fibroblasts

C. Delayed hypersensitivity reaction against bacteria

D. Effect of antibody response

E. Direct toxic effect on host cells

Mycobacteria stimulate a specific T cell response of cell mediated immunity. This is

effective in reducing the infection, the delayed hypersensitivity also damages tissues.

Necrosis occurs in TB but is usually within the granuloma.

Tuberculosis pathology

Is a form of primary chronic inflammation, caused by the inability of

macrophages to kill the Mycobacterium tuberculosis.

The macrophages often migrate to regional lymph nodes, the lung lesion plus

affected lymph nodes is referred to as a Ghon complex.

This leads to the formation of a granuloma which is a collection of epithelioid

histiocytes.

There is the presence of caseous necrosis in the centre.

The inflammatory response is mediated by a type 4 hypersensitivity reaction.

In healthy individuals the disease may be contained, in the

immunocompromised disseminated (miliary TB) may occur.

Diagnosis

Waxy membrane of mycobacteria prevents binding with normal stains. Ziehl -

Neelsen staining is typically used.

Culture based methods take far longer.

Image showing acid- alcohol fast mycobacteria stained using the Ziehl- Neelsen

method

Image sourced from Wikipedia

A 45 year old women with a thyroid carcinoma undergoes a total thyroidectomy. The

post operative histology report shows a final diagnosis of medullary type thyroid

cancer. Which of the tests below is most likely to be of clinical use in screening for

disease recurrence?

A. Serum CA 19-9 Levels

B. Serum thyroglobulin levels

C. Serum PTH levels

D. Serum calcitonin levels

E. Serum TSH levels

Theme from September 2012 Exam

Medullary thyroid cancers often secrete calcitonin and monitoring the serum levels of

this hormone is useful in detecting sub clinical recurrence.

Thyroid neoplasms

Lesion Common features

Follicular

adenoma

Usually present as a solitary thyroid nodule

Malignancy can only be excluded on formal histological

assessment

Papillary

carcinoma

Usually contain a mixture of papillary and colloidal filled

follicles

Histologically tumour has papillary projections and pale empty

nuclei

Seldom encapsulated

Lymph node metastasis predominate

Haematogenous metastasis rare

Account for 60% of thyroid cancers

Follicular

carcinoma

May appear macroscopically encapsulated, microscopically

capsular invasion is seen. Without this finding the lesion is a

follicular adenoma.

Vascular invasion predominates

Multifocal disease rare

Account for 20% of all thyroid cancers

Anaplastic

carcinoma

Most common in elderly females

Local invasion is a common feature

Account for 10% of thyroid cancers

Treatment is by resection where possible, palliation may be

achieved through isthmusectomy and radiotherapy.

Chemotherapy is ineffective.

Medullary

carcinoma

Tumours of the parafollicular cells (C Cells)

C cells derived from neural crest and not thyroid tissue

Serum calcitonin levels often raised

Familial genetic disease accounts for up to 20% cases

Both lymphatic and haematogenous metastasis are recognised,

nodal disease is associated with a very poor prognosis.

A 15 year old boy undergoes an emergency splenectomy for trauma. He makes a full

recovery and is discharged home. Eight weeks post operatively the general

practitioner performs a full blood count with a blood film. Which of the following is

most likely to be present?

A. Myofibroblasts

B. Howell-Jolly bodies

C. Multinucleate giant cells

D. Reed Sternberg Cells

E. None of the above

Post splenectomy blood

film features:

Howell- Jolly bodies

Pappenheimer bodies

Target cells

Irregular contracted

erythrocytes

As the filtration function is the spleen is no longer present Howell-Jolly bodies are

found.

Post splenectomy blood film changes

The loss of splenic tissue results in the inability to readily remove immature or

abnormal red blood cells from the circulation. The red cell count does not alter

significantly. However, cytoplasmic inclusions may be seen e.g. Howell-Jolly bodies.

In the first few days after splenectomy target cells, siderocytes and reticulocytes will

appear in the circulation. Immediately following splenectomy a granulocytosis

(mainly composed of neutrophils) is seen, this is replaced by a lymphocytosis and

monocytosis over the following weeks.

The platelet count is usually increased and this may be persistent, oral antiplatelet

agents may be needed in some patients.

Image showing Howell Jolly bodies (arrowed)

Image sourced from Wikipedia

A 43 year old women is identified as being a carrier of a BRCA 1 mutation. Apart

from breast cancer, which of the following malignancies is she at greatest risk of

developing?

A. Colonic cancer

B. Ovarian cancer

C. Follicular carcinoma of the thyroid

D. Pituitary adenoma

E. Phaeochromocytoma

BRCA 1 mutation patients are 55% more likely to get ovarian cancer. Those with

BRCA 2 are 25% more likely. The risk of developing other malignancies is slightly

increased but not to the same extent, and not enough to justify screening.

Genetics and surgical disease

Some of the more commonly occurring genetic conditions occurring in surgical

patients are presented here.

Li-Fraumeni Syndrome

Autosomal dominant

Consists of germline mutations to p53 tumour suppressor gene

High incidence of malignancies particularly sarcomas and leukaemias

Diagnosed when:

*Individual develops sarcoma under 45 years

*First degree relative diagnosed with any cancer below age 45 years and another

family member develops malignancy under 45 years or sarcoma at any age

BRCA 1 and 2

Carried on chromosome 17

Linked to developing breast cancer (60%) risk.

Associated risk of developing ovarian cancer (55% with BRCA 1 and 25%

with BRCA2).

Lynch Syndrome

Autosomal dominant

Develop colonic cancer and endometrial cancer at young age

80% of affected individuals with get colonic and or endometrial cancer

High risk individuals may be identified using the Amsterdam criteria

Amsterdam criteria Three or more family members with a confirmed diagnosis of colorectal cancer, one

of whom is a first degree (parent, child, sibling) relative of the other two.

Two successive affected generations.

One or more colon cancers diagnosed under age 50 years.

Familial adenomatous polyposis (FAP) has been excluded.

Gardners syndrome

Autosomal dominant familial colorectal polyposis

Multiple colonic polyps

Extra colonic diseases include: skull osteoma, thyroid cancer and epidermoid

cysts

Desmoid tumours are seen in 15%

Mutation of APC gene located on chromosome 5

Due to colonic polyps most patients will undergo colectomy to reduce risk of

colorectal cancer

Now considered a variant of familial adenomatous polyposis coli

A 53 year old man is due to undergo a splenectomy as a treatment for

refractory haemolytic anaemia. The underlying pathological basis for

haemolytic anaemia is thought to be a Type 2 hypersensitivity response.

Which of the following mechanisms best describes this process

A. Deposition of immune complexes

B. Cell mediated immune response

C. IgE mediated response

D. Formation of autoantibodies against cell surface

antigens

E. None of the above

Mnemonic for the reactions and

the mediators involved

ACID EGG-T

Type 1 Anaphylactic

Type 2 Cytotoxic

Type 3 Immune complex

Type 4 Delayed type

EGG T (mediators)

IgE

IgG

IgG

T cells

Type 2 hypersensitivity reactions (which includes haemolytic anaemia) are

associated with formation of antibody against cell surface antigens.

Hypersensitivity reactions

The Gell and Coombs classification divides hypersensitivity reactions into 4

types

Type I Type II Type III Type IV

Description Anaphylactic Cytotoxic Immune

complex

Delayed type

Mediator IgE IgG, IgM IgG, IgM T-cells

Antigen Exogenous Cell surface Soluble Tissues

Response

time

Minutes Hours Hours 2-3 days

Examples Asthma

Hay fever

Autoimmune

haemolytic anaemia

Pemphigus

Goodpasture's

Serum

sickness

SLE

Aspergillosis

Graft versus host

disease

Contact

dermatitis

A 25 year old man is injured in a road traffic accident. His right tibia is fractured and

is managed by fasciotomies and application of an external fixator. Over the next 48

hours his serum creatinine rises and urine is sent for microscopy, muddy brown casts

are identified. What is the most likely underlying diagnosis?

A. Acute interstitial nephritis

B. Acute tubular necrosis

C. Glomerulonephritis

D. IgA Nephropathy

E. Thin basement membrane disease

This patient is likely to have had compartment syndrome (tibial fracture +

fasciotomies) which may produce myoglobinuria. The presence of worsening renal

function, together with muddy brown casts is strongly suggestive of acute tubular

necrosis. Acute interstitial nephritis usually arises from drug toxicity and does not

usually produce urinary muddy brown casts. Thin basement membrane disease is an

autosomal dominant condition that causes persistent microscopic haematuria, but not

worsening renal function.

Acute Renal Failure

Final pathway is tubular cell death.

Renal medulla is a relatively hypoxic environment making it susceptible to

renal tubular hypoxia.

Renovascular autoregulation maintains renal blood flow across a range of

arterial pressures.

Estimates of GFR are best indices of level of renal function. Useful clinical

estimates can be obtained by considering serum creatinine, age, race, gender

and body size. eGFR calculations such as the Cockcroft and Gault equation

are less reliable in populations with high GFR's.

Nephrotoxic stimuli such as aminoglycosides and radiological contrast media

induce apoptosis. Myoglobinuria and haemolysis result in necrosis. Overlap

exists and proinflammatory cytokines play and important role in potentiating

ongoing damage.

Post-operative renal failure is more likely to occur in patients who are elderly,

have peripheral vascular disease, high BMI, have COPD, receive

vasopressors, are on nephrotoxic medication or undergo emergency surgery.

Avoiding hypotension will reduce risk of renal tubular damage.

There is no evidence that administration of ACE inhibitors or dopamine

reduces the incidence of post-operative renal failure.

A 56 year old man has undergone a radical nephrectomy. The pathologist bisects the

kidney and identifies a pink fleshy tumour in the renal pelvis. What is the most likely

disease?

A. Renal cell carcinoma

B. Transitional cell carcinoma

C. Angiomyolipoma

D. Phaeochromocytoma

E. Renal adenoma

Most renal tumours are yellow or brown in colour. TCC's are one of the few tumours

to appear pink.

Theme from April 2012

The finding of a TCC in the renal pelvis mandates a nephroureterectomy.

Renal lesions

Lesion Disease specific features Treatment

Renal cell

carcinoma

Most present with

haematuria (50%)

Common renal tumour (85%

cases)

Paraneoplastic features

include hypertension and

polycythaemia

Most commonly has

haematogenous mestastasis

Usually radical or partial

nephrectomy

Nephroblastoma Rare childhood tumour

It accounts for 80% of all

genitourinary malignancies

in those under the age of 15

years

Up to 90% will have a mass

50% will be hypertensive

Diagnostic work up includes

ultrasound and CT scanning

Surgical resection combined

with chemotherapy (usually

vincristine, actinomycin D

and doxorubicin

Neuroblastoma Most common extracranial

tumour of childhood

80% occur in those under 4

years of age

Tumour of neural crest

origin (up to 50% occur in

the adrenal gland)

The tumour is usually

calcified and may be

diagnosed using MIBG

scanning

Staging is with CT

Surgical resection,

radiotherapy and

chemotherapy

Transitional cell

carcinoma

Accounts for 90% of upper

urinary tract tumour, but

only 10% of renal tumours

Males affected 3x more than

females

Occupational exposure to

industrial dyes and rubber

chemicals may increase risk

Up to 80% present with

painless haematuria

Diagnosis and staging is with

CT IVU

Radical nephroureterectomy

Angiomyolipoma 80% of these hamartoma

type lesions occur

sporadically, the remainder

are seen in those with

tuberous sclerosis

Tumour is composed of

blood vessels, smooth

muscle and fat

Massive bleeding may occur

in 10% of cases

50% of patients with lesions

>4cm will have symptoms

and will require surgical

resection

A 65 year old lady presents with a lesion affecting her right breast. On examination

she has a weeping, crusting lesion overling the right nipple, the areolar region is not

involved. There is no palpable mass lesion in the breast, there is a palpable axillary

lymph node. The patients general practitioner has tried treating the lesion with 1%

hydrocortisone cream, with no success. What is the most likely diagnosis?

A. Infection with Staphylococcus aureus

B. Pagets disease of the nipple

C. Phyllodes tumour

D. Nipple eczema

E. Basal cell carcinoma

A weeping, crusty lesion such as this is most likely to represent Pagets disease of the

nipple (especially since the areolar region is spared). Although no mass lesion is

palpable, a proportion of patients will still have an underlying invasive malignancy

(hence the lymphadenopathy).

Pagets disease of the nipple

Pagets disease is an eczematoid change of the nipple associated with an underlying

breast malignancy and it is present in 1-2% of patients with breast cancer. In half of

these patients, it is associated with an underlying mass lesion and 90% of such

patients will have an invasive carcinoma. 30% of patients without a mass lesion will

still be found to have an underlying carcinoma. The remainder will have carcinoma in

situ.

Pagets disease differs from eczema of the nipple in that it involves the nipple

primarily and only latterly spreads to the areolar (the opposite occurs in eczema).

Diagnosis is made by punch biopsy, mammography and ultrasound of the breast.

Treatment will depend on the underlying lesion.

A 73 year old man presents with haemoptysis and is suspected of suffering from lung

cancer. On examination he has an enlarged supraclavicular lymph node. Which of the

following features is most likely to be present on histological examination?

A. Increased mitoses

B. Apoptosis

C. Barr Bodies

D. Multinucleate giant cells

E. Granuloma

Theme from 2011 Exam

Increased mitoses are commonly seen in association with malignant transformation of

cells. Apoptosis is not a common feature of metastatic cancer. Barr Bodies are formed

during X chromosome inactivation in female somatic cells.

Histopathology of malignancy

Abnormal tissue architecture

Coarse chromatin

Invasion of basement membrane*

Abnormal mitoses

Angiogenesis

De-differentiation

Areas of necrosis

Nuclear pleomorphism

*= Those features that distinguish invasive malignancy from in situ disease

Which of the following pathological explanations best describes the initial

pathological processes occurring in an abdominal aortic aneurysm in an otherwise

well 65 year old, hypertensive male?

A. Loss of elastic fibres from the adventitia

B. Loss of collagen from the adventitia

C. Loss of collagen from the media

D. Loss of elastic fibres from the media

E. Decreased matrix metalloproteinases in the adventitia

Theme from April 2012 Exam

In established aneurysmal disease there is dilation of all layers of the arterial wall and

loss of both elastin and collagen. The primary event is loss of elastic fibres with

subsequent degradation of collagen fibres.

Pathology of abdominal aortic aneurysm

Abdominal aortic aneurysms occur primarily as a result of the failure of elastic

proteins within the extracellular matrix. Anuerysms typically represent dilation of all

layers of the arterial wall. Most aneurysms are caused by degenerative disease. After

the age of 50 years the normal diameter of the infrarenal aorta is 1.5cm in females and

1.7cm in males. Diameters of 3cm and greater, are considered aneurysmal. The

pathophysiology involved in the development of aneurysms is complex and the

primary event is loss of the intima with loss of elastic fibres from the media. This

process is associated with, and potentiated by, increased proteolytic activity and

lymphocytic infiltration.

Major risk factors for the development of aneurysms include smoking and

hypertension. Rare but important causes include syphilis and connective tissues

diseases such as Ehlers Danlos type 1 and Marfans syndrome.

Layers of arterial wall

Image sourced from Wikipedia

A 28 year old lady has a malignant melanoma removed from her calf. Which of the

following pathological criteria carries the greatest prognostic weighting?

A. Vascular invasion

B. Abnormal mitoses

C. Breslow thickness

D. Perineural invasion

E. Lymphocytic infiltrates

Theme from April 2012 Exam

The Breslow thickness has considerable prognostic importance. Lymphocytic

infiltrates may be associated with an improved prognosis, but do not carry nearly the

same weight as increased thickness.

Malignant melanoma

The main diagnostic features (major

criteria):

Secondary features (minor

criteria)

Change in size

Change in shape

Change in colour

Diameter >6mm

Inflammation

Oozing or bleeding

Altered sensation

Treatment

Suspicious lesions should undergo excision biopsy. The lesion should be

removed in completely as incision biopsy can make subsequent

histopathological assessment difficult.

Once the diagnosis is confirmed the pathology report should be reviewed to

determine whether further re-exicision of margins is required (see below):

Margins of excision-Related to Breslow thickness

Lesions 0-1mm thick 1cm

Lesions 1-2mm thick 1- 2cm (Depending upon site and pathological features)

Lesions 2-4mm thick 2-3 cm (Depending upon site and pathological features)

Lesions >4 mm thick 3cm

Marsden J et al Revised UK guidelines for management of Melanoma. Br J Dermatol

2010 163:238-256.

Further treatments such as sentinel lymph node mapping, isolated limb perfusion and

block dissection of regional lymph node groups should be selectively applied.

A 34 year old lady undergoes an elective cholecystectomy for attacks of recurrent

cholecystitis due to gallstones. Microscopic assessment of the gallbladder is most

likely to show which of the following?

A. Dysplasia of the fundus

B. Widespread necrosis

C. Ashoff-Rokitansky sinuses

D. Metaplasia of the fundus

E. None of the above

Aschoff-Rokitansky sinuses are the result of hyperplasia and herniation of epithelial

cells through the fibromuscular layer of the gallbladder wall. They may be

macroscopic or microscopic. Ashoff-Rokitansky sinuses may be identified in cases of

chronic cholecystitis and gallstones. Although gallstones may predispose to the

development of gallbladder cancer the actual incidence of dysplasia and metaplastic

change is rare. In the elective setting described above necrosis would be rare.

Gallbladder

Fibromuscular sac with capacity of 50ml

Columnar epithelium

Relations of the gallbladder

Anterior Liver

Posterior Covered by peritoneum

Transverse colon

1st part of the duodenum

Laterally Right lobe of liver

Medially Quadrate lobe of liver

Arterial supply Cystic artery (branch of Right hepatic artery)

Venous drainage Cystic vein

Nerve supply Sympathetic- mid thoracic spinal cord, Parasympathetic- anterior vagal trunk

Common bile duct

Origin Confluence of cystic and common hepatic ducts

Relations at

origin

Medially - Hepatic artery

Posteriorly- Portal vein

Relations

distally

Duodenum - anteriorly

Pancreas - medially and laterally

Right renal vein - posteriorly

Arterial supply Branches of hepatic artery and retroduodenal branches of

gastroduodenal artery

Calot's triangle

Medially Common hepatic duct

Inferiorly Cystic duct

Superiorly Inferior edge of liver

Contents Cystic artery

Which of the following are not true of Li-Fraumeni syndrome?

A. It consists of mutations to the p53 tumour suppressor gene

B. Is likely to be present in a teenager presenting with a liposarcoma

C. It has an autosomal dominant inheritance pattern

D. Affected individuals are unlikely to develop acute myeloid leukaemia

E. Adrenal malignancies are more common than in normal population

They are at high risk of developing leukaemia.

Genetics and surgical disease

Some of the more commonly occurring genetic conditions occurring in surgical

patients are presented here.

Li-Fraumeni Syndrome

Autosomal dominant

Consists of germline mutations to p53 tumour suppressor gene

High incidence of malignancies particularly sarcomas and leukaemias

Diagnosed when:

*Individual develops sarcoma under 45 years

*First degree relative diagnosed with any cancer below age 45 years and another

family member develops malignancy under 45 years or sarcoma at any age

BRCA 1 and 2

Carried on chromosome 17

Linked to developing breast cancer (60%) risk.

Associated risk of developing ovarian cancer (55% with BRCA 1 and 25%

with BRCA2).

Lynch Syndrome

Autosomal dominant

Develop colonic cancer and endometrial cancer at young age

80% of affected individuals with get colonic and or endometrial cancer

High risk individuals may be identified using the Amsterdam criteria

Amsterdam criteria Three or more family members with a confirmed diagnosis of colorectal cancer, one

of whom is a first degree (parent, child, sibling) relative of the other two.

Two successive affected generations.

One or more colon cancers diagnosed under age 50 years.

Familial adenomatous polyposis (FAP) has been excluded.

Gardners syndrome

Autosomal dominant familial colorectal polyposis

Multiple colonic polyps

Extra colonic diseases include: skull osteoma, thyroid cancer and epidermoid

cysts

Desmoid tumours are seen in 15%

Mutation of APC gene located on chromosome 5

Due to colonic polyps most patients will undergo colectomy to reduce risk of

colorectal cancer

Now considered a variant of familial adenomatous polyposis coli

A 35 year old type 1 diabetic presents with difficulty mobilising and back pain

radiating to the thigh. He has a temperature of 39 oC and has pain on extension of the

hip. He is diagnosed with an iliopsoas abscess. Which of the following statements is

false in relation to his diagnosis?

A. Staphylococcus aureus is the most likely primary cause

B. Recurrence occurs in 60% cases

C. More common in males

D. Crohn's is the most likely secondary cause

E. CT guided drainage is preferable first line management

Classical features include: a limp, back pain and fever. Recurrence rates are about 15-

20%. Staphylococcus is the commonest primary cause, others include Streptococcus

and E.coli. Management is ideally by CT guided drainage.

Iliopsoas abscess

Collection of pus in iliopsoas compartment (iliopsoas and iliacus)

Causes:

Primary

Haematogenous spread of bacteria

Staphylococcus aureus: most common

Secondary

Crohn's (commonest cause in this category)

Diverticulitis, Colorectal cancer

UTI, GU cancers

Vertebral osteomyelitis

Femoral catheter, lithotripsy

Endocarditis

Note the mortality rate can be up to 19-20% in secondary iliopsoas abscesses

compared with 2.4% in primary abscesses.

Clinical features

Fever

Back/flank pain

Limp

Weight loss

Clinical examination

Patient in the supine position with the knee flexed and the hip mildly

externally rotated

Specific tests to diagnose iliopsoas inflammation:

Place hand proximal to the patient's ipsilateral knee and ask patient to lift thigh

against your hand. This will cause pain due to contraction of the psoas muscle.

Lie the patient on the normal side and hyperextend the affected hip. In inflammation

this should elicit pain as the psoas muscle is stretched.

Investigation

CT is gold standard

Management

Antibiotics

Percutaneous drainage

Surgery is indicated if:

1. Failure of percutaneous drainage

2. Presence of an another intra-abdominal pathology which requires surgery

Surgical approach The authors technique for draining these collections is given here.

Review the CT scans and plan surgical approach. An extraperitoneal approach is

important.

The collection usually extends inferiorly and can be accessed from an incision at a

level of L4 on the affected side.

GA

Transverse laterally placed incision.

Incise external oblique.

Split the subsequent muscle layers.

As you approach the peritoneum use blunt dissection to pass laterally around it.

Remember the ureter and gonadal veins lie posterior at this level.

Eventually you will enter the abscess cavity, a large amount of pus is usually released

at this point. Drain the area with suction and washout with saline.

Place a corrugated drain well into the abscess cavity.

If you have made a small skin incision it is reasonable to bring the drain up through

the skin wound. Otherwise place a lateral exit site and close the skin and external

oblique. If you do this ensure that you use interrupted sutures.

Anchor the drain with strong securely tied silk sutures (it is extremely tiresome if it

falls out!)

Reference

Iliopsoas abscesses

I H Mallick, M H Thoufeeq, T P Rajendran

Postgrad Med J 2004;80:459-462

Which of the following statements relating to parathyroid neoplasms is incorrect?

A. 15% of cases are due to parathyroid carcinoma

B. 80% of cases are due to parathyroid adenomas

C. Parathyroid adenomas are often encapsulated

D. 10% of parathyroid adenomas develop in ectopically located glands

E. 85% of cases of primary hyperparathyroidism are due to solitary

adenomas

Parathyroid carcinomas account for up to 5% of tumours. Adenomas are often

encapsulated .Lesions that are fibrotic and densely adherent to the gland may be a

carcinoma. 85% cases of primary hyperparathyroidism are due to a single adenoma

and this is the reason some surgeons favour a focussed parathyroidectomy.

Parathyroid glands and disorders of calcium metabolism

Hyperparathyroidism

Disease type Hormone profile Clinical features Cause

Primary

hyperparathyroidism

PTH (Elevated)

Ca2+ (Elevated)

Phosphate

(Low)

Serum Calcium

: Creatinine

clearance ratio

> 0.01

May be

asymptomatic if

mild

Recurrent

abdominal pain

(pancreatitis, renal

colic)

Changes to

emotional or

cognitive state

Most cases due to

solitary adenoma

(80%), multifocal

disease occurs in

10-15% and

parathyroid

carcinoma in 1% or

less

Secondary

hyperparathyroidism

PTH (Elevated)

Ca2+ (Low or

normal)

Phosphate

(Elevated)

Vitamin D

levels (Low)

May have few

symptoms

Eventually may

develop bone

disease, osteitis

fibrosa cystica and

soft tissue

calcifications

Parathyroid gland

hyperplasia occurs

as a result of low

calcium, almost

always in a setting

of chronic renal

failure

Tertiary

hyperparathyroidism

Ca2+ (Normal

or high)

PTH (Elevated)

Phosphate

levels

(Decreased or

Normal)

Vitamin D

(Normal or

decreased)

Alkaline

phosphatase

(Elevated)

Metastatic

calcification

Bone pain and /

or fracture

Nephrolithiasis

Pancreatitis

Occurs as a result

of ongoing

hyperplasia of the

parathyroid glands

after correction of

underlying renal

disorder,

hyperplasia of all 4

glands is usually

the cause

Differential diagnoses It is important to consider the rare but relatively benign condition of benign familial

hypocalciuric hypercalcaemia, caused by an autosomal dominant genetic disorder.

Diagnosis is usually made by genetic testing and concordant biochemistry (Serum

Calcium : Creatinine clearance ratio <0.01-distinguished from primary

hyperparathyroidism).

Treatment

Primary hyperparathyroidism

Indications for surgery

Elevated serum Calcium > 1mg/dL above normal

Hypercalciuria > 400mg/day

Creatinine clearance < 30% compared with normal

Episode of life threatening hypercalcaemia

Nephrolithiasis

Age < 50 years

Neuromuscular symptoms

Reduction in bone mineral density of the femoral neck, lumbar spine, or distal

radius of more than 2.5 standard deviations below peak bone mass (T score

lower than -2.5)

Secondary hyperparathyroidism Usually managed with medical therapy.

Indications for surgery in secondary (renal) hyperparathyroidism:

Bone pain

Persistent pruritus

Soft tissue calcifications

Tertiary hyperparathyroidism Usually treatment is surgical

The presence of an autonomously functioning parathyroid gland may require surgery.

If the culprit gland can be identified then it should be excised. Otherwise total

parathyroidectomy and re-implantation of part of the gland may be required.

References 1. Pitt S et al. Secondary and Tertiary Hyperparathyroidism, State of the Art Surgical

Management. Surg Clin North Am 2009 Oct;89(5):1227-39.

2. MacKenzie-Feder J et al. Primary Hyperparathyroidism: An Overview. Int J

Endocrinol 2011; 2011: 251410.

A 20 year old girl presents with a thyroid cancer, she is otherwise well with no

significant family history. On examination she has a nodule in the left lobe of the

thyroid with a small discrete mass separate from the gland itself. Which of the

following is the most likely cause?

A. Follicular carcinoma

B. Anaplastic carcinoma

C. Medullary carcinoma

D. Papillary carcinoma

E. B Cell Lymphoma

Theme from September 2011 Exam

Papillary carcinoma is the most common subtype and may cause lymph node

metastasis (mass separate from the gland itself) that is rare with follicular tumours.

Anaplastic carcinoma would cause more local symptoms and would be rare in this age

group.

Thyroid malignancy

Papillary carcinoma

Commonest sub-type

Accurately diagnosed on fine needle aspiration cytology

Histologically they may demonstrate psammoma bodies (areas of

calcification) and so called 'orphan Annie' nuclei

They typically metastasise via the lymphatics and thus laterally located

apparently ectopic thyroid tissue is usually a metastasis from a well

differentiated papillary carcinoma.

Follicular carcinoma

Are less common than papillary lesions

Like papillary tumours they may present as a discrete nodule. Although they

appear to be well encapsulated macroscopically there invasion on microscopic

evaluation.

Lymph node metastases are uncommon and these tumours tend to spread

haematogenously. This translates into a higher mortality rate.

Follicular lesions cannot be accurately diagnosed on fine needle aspiration

cytology and thus all follicular FNA's will require at least a hemi

thyroidectomy.

Anaplastic carcinoma

Less common and tend to occur in elderly females

Disease is usually advanced at presentation and often only palliative

decompression and radiotherapy can be offered.

Medullary carcinoma

These are tumours of the parafollicular cells ( C Cells) and are of neural crest

origin.

The serum calcitonin may be elevated which is of use when monitoring for

recurrence.

They may be familial and occur as part of the MEN -2A disease spectrum.

Spread may be either lymphatic or haematogenous and as these tumours are

not derived primarily from thyroid cells they are not responsive to radioiodine.

Lymphoma

These respond well to radiotherapy

Radical surgery is unnecessary once the disease has been diagnosed on biopsy

material. Such biopsy material is not generated by an FNA and thus a core

biopsy has to be obtained (with care!).

A 28 year old lady is breast feeding her first child. She presents with discomfort of the

right breast. Clinical examination demonstrates erythema and an area that is fluctuant.

Aspiration and culture of the fluid is most likely to demonstrate infection with which

of the following organisms?

A. Clostridium perfringens

B. Staphylococcus aureus

C. Streptococcus pyogenes

D. Staphylococcus epidermidis

E. Actinomycosis

Theme from 2010 and 2009 Exam

Staphylococcus aureus is the commonest cause. The infants mouth is usually the

source as it damages the nipple areolar complex allowing entry of bacteria.

Breast abscess

In lactational women Staphylococcus aureus is the most common cause

Typical presentation is with a tender mass in a lactating women

There is often tenderness and pain and a fluctuant mass

Diagnosis and treatment is performed using USS and associated drainage of

the abscess cavity. Antibiotics should also be administered

Where there is necrotic skin overlying the abscess, the patient should undergo

surgery

An 18 year old rock climber falls onto his left arm and sustains a large haematoma of

the left upper arm. Unfortunately the wound associated with the injury is neglected

and it becomes infected. Which of the following changes is least likely to occur?

A. Axillary lymphadenopathy

B. Leucopenia

C. Tenderness

D. Mild pyrexia

E. Local formation of yellow pus

Leucopenia would be unusual and should prompt a search for another cause.

Acute inflammation

Inflammation is the reaction of the tissue elements to injury. Vascular changes occur,

resulting in the generation of a protein rich exudate. So long as the injury does not

totally destroy the existing tissue architecture, the episode may resolve with

restoration of original tissue architecture.

Vascular changes

Vasodilation occurs and persists throughout the inflammatory phase.

Inflammatory cells exit the circulation at the site of injury.

The equilibrium that balances Starlings forces within capillary beds is

disrupted and a protein rich exudate will form as the vessel walls also become

more permeable to proteins.

The high fibrinogen content of the fluid may form a fibrin clot. This has

several important immunomodulatory functions.

Sequelae

Resolution Typically occurs with minimal initial injury

Stimulus removed and normal tissue architecture

results

Organisation Delayed removed of exudate

Tissues undergo organisation and usually fibrosis

Suppuration Typically formation of an abscess or an empyema

Sequestration of large quantities of dead

neutrophils

Progression to chronic

inflammation

Coupled inflammatory and reparative activities

Usually occurs when initial infection or

suppuration has been inadequately managed

Causes

Microbacterial infections e.g. Viruses, exotoxins or endotoxins released by

bacteria

Chemical agents

Physical agents e.g. Trauma

Hypersensitivity reactions

Tissue necrosis

Presence of neutrophil polymorphs is a histological diagnostic feature of acute

inflammation Theme: Tumour markers

A. Invasive ductal carcinoma of the breast

B. Prostate cancer

C. Gastric cancer

D. Ovarian cancer

E. Colorectal cancer

F. Pancreatic adenocarcinoma

G. Seminoma testicular cancer

H. Non-seminomatous testicular cancer

I. Hepatocellular carcinoma

For each tumour marker please select the most likely underlying malignancy. Each

option may be used once, more than once or not at all.

13. Raised beta-human chorionic gonadotropin with a raised alpha-feto protein

level

Non-seminomatous testicular cancer

Theme from April 2012 Exam

A raised alpha-feto protein level excludes a seminoma

14. Elevated CA 19-9

Pancreatic adenocarcinoma

15. Raised alpha-feto protein level in a 54-year-old woman

Hepatocellular carcinoma

Tumour markers

Tumour markers may be divided into:

monoclonal antibodies against carbohydrate or glycoprotein tumour antigens

tumour antigens

enzymes (alkaline phosphatase, neurone specific enolase)

hormones (e.g. calcitonin, ADH)

It should be noted that tumour markers usually have a low specificity

Monoclonal antibodies

Tumour marker Association

CA 125 Ovarian cancer

CA 19-9 Pancreatic cancer

CA 15-3 Breast cancer

NB: The breast cancer tumour marker is not specific or sensitive enough to be used

routinely.

Tumour antigens

Tumour marker Association

Prostate specific antigen (PSA) Prostatic carcinoma

Alpha-feto protein (AFP) Hepatocellular carcinoma, teratoma

Carcinoembryonic antigen (CEA) Colorectal cancer

Theme: Benign breast lesions

A. Fibroadenoma

B. Breast abscess

C. Cyst of Montgomery's gland

D. Galactocele

E. Lipoma

F. Duct ectasia

G. Intraductal papilloma

H. Fat necrosis

What is the most likely diagnosis for the scenario given? Each option may be used

once, more than once or not at all.

16. A 64 year old obese female presents with a breast lump. She was hit on the

breast by a cricket ball when playing with her grandson.

Fat necrosis

An obese, post menopausal woman, with a history of trauma points towards

fat necrosis. Trauma causes inflammation of fat cells, leading to formation of

a lump. Mammography will be needed to differentiate it from breast disease.

17. A 21 year old female notices a bloody discharge from the nipple. She is

otherwise well. On examination there are no discrete lesions to feel and

mammography shows dense breast tissue but no mass lesion.

Intraductal papilloma

Intraductal papillomata are the commonest cause of blood stained nipple

discharge in younger women. There is seldom any palpable mass. An

ultrasound is required and possibly a galactogram.

18. A 18 year old female notices a non tender mobile breast lump. Clinically

there is a smooth lump which is not tethered to the skin.

Fibroadenoma

Also called a breast 'mouse' due to its mobility. It is a benign condition

arising from the breast lobule. May enlarge in pregnancy.

Benign Breast lesions

Benign Breast Diseases

Lesion Features Treatment

Fibroadenoma Develop from a whole

lobule

Mobile, firm breast lumps

If >3cm surgical excision is

usual, Phyllodes tumours

should be widely excised

12% of all breast masses

Over a 2 year period up to

30% will get smaller

No increase in risk of

malignancy

(mastectomy if the lesion is

large)

Breast cyst 7% of all Western

females will present with

a breast cyst

Usually presents as a

smooth discrete lump

(may be fluctuant)

Small increased risk of

breast cancer (especially

if younger)

Cysts should be aspirated,

those which are blood stained

or persistantly refill should be

biopsied or excised

Sclerosing adenosis,

(radial scars and

complex sclerosing

lesions)

Usually presents as a

breast lump or breast pain

Causes mammographic

changes which may

mimic carcinoma

Cause distortion of the

distal lobular unit,

without hyperplasia

(complex lesions will

show hyperplasia)

Considered a disorder of

involution, no increase in

malignancy risk

Lesions should be biopsied,

excision is not mandatory

Epithelial

hyperplasia

Variable clinical

presentation ranging from

generalised lumpiness

through to discrete lump

Disorder consists of

increased cellularity of

terminal lobular unit,

atypical features may be

present

Atypical features and

family history of breast

cancer confers greatly

increased risk of

malignancy

If no atypical features then

conservative, those with

atypical features require either

close monitoring or surgical

resection

Fat necrosis Up to 40% cases usually

have a traumatic

aetiology

Physical features usually

Imaging and core biopsy

mimic carcinoma

Mass may increase in size

initially

Duct papilloma Usually present with

nipple discharge

Large papillomas may

present with a mass

The discharge usually

originates from a single

duct

No increase risk of

malignancy

Microdochectomy

A 17 year old man is identified as having a Meckels diverticulum. From which of the

following embryological structures is it derived?

A. Foregut

B. Hindgut

C. Uranchus

D. Cloaca

E. Vitello-intestinal duct

Rule of 2's

2% of population

2 inches (5cm) long

2 feet (60 cm) from the

ileocaecal valve

2 x's more common in men

2 tissue types involved

The Meckels diverticulum is a persistence of the vitello-intestinal duct.

Meckel's diverticulum

Congenital abnormality resulting in incomplete obliteration of the vitello-

intestinal duct

Normally, in the foetus, there is an attachment between the vitello-intestinal

duct and the yolk sac.This disappears at 6 weeks gestation.

The tip is free in majority of cases.

Associated with enterocystomas, umbilical sinuses, and omphaloileal fistulas.

Arterial supply: omphalomesenteric artery.

2% of population, 2 inches long, 2 feet from the ileocaecal valve.

Typically lined by ileal mucosa but ectopic gastric mucosa can occur, with the

risk of peptic ulceration. Pancreatic and jejunal mucosa can also occur.

Clinical

Normally asymptomatic and an incidental finding.

Complications are the result of obstruction, ectopic tissue, or inflammation.

Removal if narrow neck or symptomatic. Options are between wedge excision

or formal small bowel resection and anastomosis.

Theme: Rectal bleeding

A. Solitary rectal ulcer syndrome

B. Haemorroidal disease

C. Fissure in ano

D. Fistula in ano

E. Anal cancer

F. Ulcerative colitis

Please select the most likely diagnosis for the scenario given. Each option may be

used once, more than once or not at all.

20. A 22 year old man presents with a 6 day history of passage of bloody

diarrhoea with passage of mucous and slime. He is passing an average of 8 to

9 bowel movements per day. On digital rectal examination there is no discrete

abnormality to feel, but there is some blood stained mucous on the glove.

Ulcerative colitis

The passage of bloody diarrhoea together with mucous and a short history

makes this a likely first presentation of inflammatory bowel disease.

21. A 17 year old man presents with a 2 week history of significant pain on

defecation accompanied by the presence of a small amount of blood which is

noticed on toilet paper.

Fissure in ano

Young patients with painful rectal bleeding may have a fissure. Treatment is

with stool softeners and either GTN or Diltiazem initially.

22. A 24 year old women presents with a long history of obstructed defecation

and chronic constipation. She often strains to open her bowels for long

periods and occasionally notices that she has passed a small amount of blood.

On examination she has an indurated area located anteriorly approximately

3cm proximal to the anal verge.

Solitary rectal ulcer syndrome

Solitary rectal ulcers are associated with chronic constipation and straining. It

will need to be biopsied to exclude malignancy (the histological appearances

are characteristic). Diagnostic work up should include endoscopy and

probably defecating proctogram and ano-rectal manometry studies.

Rectal bleeding

Rectal bleeding is a common cause for patients to be referred to the surgical clinic. In

the clinical history it is useful to try and localise the anatomical source of the blood.

Bright red blood is usually of rectal anal canal origin, whilst dark red blood is more

suggestive of a proximally sited bleeding source. Blood which has entered the GI tract

from a gastro-duodenal source will typically resemble malaena due to the effects of

the digestive enzymes on the blood itself.

In the table below we give some typical bleeding scenarios together with physical

examination findings and causation.

Cause Type of

bleeding

Features in history Examination findings

Fissure in

ano

Bright red

rectal

bleeding

Painful bleeding that occurs

post defecation in small

volumes. Usually antecedent

features of constipation

Muco-epithelial defect

usually in the midline

posteriorly (anterior fissures

more likely to be due to

underlying disease)

Haemorroids Bright red

rectal

bleeding

Post defecation bleeding

noted both on toilet paper

and drips into pan. May be

alteration of bowel habit and

history of straining. No

blood mixed with stool. No

local pain.

Normal colon and rectum.

Proctoscopy may show

internal haemorrhoids.

Internal haemorrhoids are

usually impalpable.

Crohns

disease

Bright red

or mixed

blood

Bleeding that is

accompanied by other

symptoms such as altered

bowel habit, malaise, history

of fissures (especially

Perineal inspection may

show fissures or fistulae.

Proctoscopy may

demonstrate indurated

mucosa and possibly

anterior) and abscesses. strictures. Skip lesions may

be noted at colonoscopy.

Ulcerative

colitis

Bright red

bleeding

often mixed

with stool

Diarrhoea, weight loss,

nocturnal incontinence,

passage of mucous PR.

Proctitis is the most marked

finding. Peri anal disease is

usually absent. Colonoscopy

will show continuous

mucosal lesion.

Rectal

cancer

Bright red

blood

mixed

volumes

Alteration of bowel habit.

Tenesmus may be present.

Symptoms of metastatic

disease.

Usually obvious mucosal

abnormality. Lesion may be

fixed or mobile depending

upon disease extent.

Surrounding mucosa often

normal, although polyps may

be present.

Image showing a fissure in ano. Typically these are located posteriorly and in the

midline. Fissures at other sites may be associated with underlying disease.

Image sourced from Wikipedia

Colonoscopic image of internal haemorroids. Note these may often be impalpable.

Image sourced from Wikipedia

Investigation

All patients presenting with rectal bleeding require digital rectal examination

and procto-sigmoidoscopy as a minimal baseline.

Remember that haemorrhoids are typically impalpable and to attribute

bleeding to these in the absence of accurate internal inspection is

unsatisfactory.

In young patients with no other concerning features in the history a carefully

performed sigmoidoscopy that demonstrates clear haemorrhoidal disease may

be sufficient. If clear views cannot be obtained then patients require bowel

preparation with an enema and a flexible sigmoidscopy performed.

In those presenting with features of altered bowel habit or suspicion of

inflammatory bowel disease a colonoscopy is the best test.

Patients with excessive pain who are suspected of having a fissure may require

an examination under general or local anaesthesia.

In young patients with external stigmata of fissure and a compatible history it

is acceptable to treat medically and defer internal examination until the fissure

is healed. If the fissure fails to heal then internal examination becomes

necessary along the lines suggested above to exclude internal disease.

Special tests

In patients with a malignancy of the rectum the staging investigations

comprise an MRI of the rectum to identify circumferential resection margin

compromise and to identify mesorectal nodal disease. In addition to this CT

scanning of the chest abdomen and pelvis is necessary to stage for more

distant disease. Some centres will still stage the mesorectum with endo rectal

ultrasound but this is becoming far less common.

Patients with fissure in ano who are being considered for surgical

sphincterotomy and are females who have an obstetric history should probably

have ano rectal manometry testing performed together with endo anal

ultrasound. As this service is not universally available it is not mandatory but

in the absence of such information there are continence issues that may arise

following sphincterotomy.

Management

Disease Management

Fissure in ano GTN ointment 0.2% or diltiazem cream applied topically is the usual

first line treatment. Botulinum toxin for those who fail to respond.

Internal sphincterotomy for those who fail with botox, can be

considered at the botox stage in males.

Haemorroids Lifestyle advice, for small internal haemorrhoids can consider

injection sclerotherapy or rubber band ligation. For external

haemorrhoids consider haemorrhoidectomy. Modern options include

HALO procedure and stapled haemorrhoidectomy.

Inflammatory

bowel disease

Medical management- although surgery may be needed for

fistulating Crohns (setons).

Rectal cancer Anterior resection or abdomino-perineal excision of the colon and

rectum. Total mesorectal excision is now standard of care. Most

resections below the peritoneal reflection will require defunctioning

ileostomy. Most patients will require preoperative radiotherapy.

Which of the following is the most common childhood brain tumour?

A. Glioblastoma multiforme

B. Astrocytoma

C. Medulloblastoma

D. Ependymoma

E. Meningioma

Glioblastoma multiforme is rare in childhood. In contrast, medulloblastoma (more

correctly termed primitive neuroectodermal tumours) is the commonest brain tumour

in children, and the 2nd commonest malignant solid neoplasm in children.

CNS tumours

60% = Glioma and metastatic disease

20% = Meningioma

10% = Pituitary lesions

In paediatric practice medulloblastomas (neuroectodermal tumours) are the

commonest lesion, they are very rare in adults.

Tumours arising in right temporal and frontal lobe may reach considerable size before

becoming symptomatic. Whereas tumours in the speech and visual areas will typically

produce early symptoms.

Diagnosis MRI Scanning provides the best resolution.

Treatment Usually surgery, even if tumour cannot be completely resected conditions such as

rising ICP can be addressed with tumour debulking and survival and quality of life

prolonged.

Curative surgery can usually be undertaken with lesions such as meningiomas.

Gliomas have a marked propensity to invade normal brain and resection of these

lesions is nearly always incomplete.

A keen surgical trainee is about to embark on her first hemi arthroplasty for a

fractured neck of femur. In the anaesthetic room the patient is given 1.2g intravenous

co-amoxiclav. There is a possible history of penicillin allergy but the patient is

demented and the history is not checked. The patient then develops severe respiratory

compromise and haemodynamic collapse. Which of the following pathological

processes accounts for this event?

A. Binding of the drug to circulating IgG class antibodies

B. Recognition of the drug by IgE receptors on mast cells

C. Drug initiated formation of hapten-protein complexes

D. Binding of the drug to circulating IgM class antibodies

E. None of the above

Anaphylactic shock:

Antigen recognised by IgE molecules on the surface of mast cells resulting in rapid

degranulation with release of histamine and other inflammatory cytokines.

This is a case of anaphylactic shock. In anaphylaxis the mast cells degranulate.

Anaphylactic shock

Suspect if there has been exposure to an allergen

Management

- Remove allergen

- ABCD

- Drugs:

Adrenaline 1:1000 0.5ml INTRAMUSCULARLY (not IV). Repeat after 5 mins if no

response.

Then Chlorpheniramine 10mg IV

Then Hydrocortisone 100-200mg IV

Reference Emergency treatment of anaphylactic reactions. Guidelines for healthcare providers.

Working Group of the Resuscitation Council (UK).2008

Which of the following hepatobiliary disorders are most classically associated with

ulcerative colitis?

A. Gallstones

B. Primary sclerosing cholangitis

C. Bile duct stones

D. Liver hamartomas

E. Hepatocellular carcinoma

Primary sclerosing cholangitis is an idiopathic inflammation of the bile ducts. It may

result in episodes of cholestasis and cholangitis and ultimately result in the need for

liver transplantation. It carries a 10% risk of malignant transformation. Crohns disease

is associated with gallstones due to impaired entero-hepatic circulation. Apart from

PSC, ulcerative colitis does not increase the risk of other liver lesions.

Ulcerative colitis

Ulcerative colitis is a form of inflammatory bowel disease. Inflammation always starts

at rectum, never spreads beyond ileocaecal valve and is continuous. The peak

incidence of ulcerative colitis is in people aged 15-25 years and in those aged 55-65

years. It is less common in smokers.

The initial presentation is usually following insidious and intermittent symptoms.

Features include:

bloody diarrhoea

urgency

tenesmus

abdominal pain, particularly in the left lower quadrant

extra-intestinal features (see below)

Questions regarding the 'extra-intestinal' features of inflammatory bowel disease are

common. Extra-intestinal features include sclerosing cholangitis, iritis and ankylosing

spondylitis.

{Common to both Crohn's

disease (CD) and Ulcerative

colitis (UC)}

{Notes}

{Related to

disease activity}

Arthritis: pauciarticular,

asymmetric

Erythema nodosum

Episcleritis

Osteoporosis

Arthritis is the most common

extra-intestinal feature in both

CD and UC

Episcleritis is more common in

CD

{Unrelated to

disease activity}

Arthritis: polyarticular,

symmetric

Uveitis

Pyoderma gangrenosum

Clubbing

Primary sclerosing cholangitis

Primary sclerosing cholangitis is

much more common in UC

Uveitis is more common in UC

Pathology

red, raw mucosa, bleeds easily

no inflammation beyond submucosa (unless fulminant disease)

widespread superficial ulceration with preservation of adjacent mucosa which

has the appearance of polyps ('pseudopolyps')

inflammatory cell infiltrate in lamina propria

neutrophils migrate through the walls of glands to form crypt abscesses

depletion of goblet cells and mucin from gland epithelium

granulomas are infrequent

Barium enema

loss of haustrations

superficial ulceration, 'pseudopolyps'

long standing disease: colon is narrow and short -'drainpipe colon'

Endoscopy

Superficial inflammation of the colonic and rectal mucosa

Continuous disease from rectum proximally

Superifical ulceration, mucosal islands, loss of vascular definition and

continuous ulceration pattern.

Management

Patients with long term disease are at increased risk of development of

malignancy

Acute exacerbations are generally managed with steroids, in chronic patients

agents such as azathioprine and infliximab may be used

Individuals with medically unresponsive disease usually require surgery- in

the acute phase a sub total colectomy and end ileostomy. In the longer term a

proctectomy will be required. An ileoanal pouch is an option for selected

patients

Which of the following hepatobiliary disorders are most classically associated with

ulcerative colitis?

A. Gallstones

B. Primary sclerosing cholangitis

C. Bile duct stones

D. Liver hamartomas

E. Hepatocellular carcinoma

Primary sclerosing cholangitis is an idiopathic inflammation of the bile ducts. It may

result in episodes of cholestasis and cholangitis and ultimately result in the need for

liver transplantation. It carries a 10% risk of malignant transformation. Crohns disease

is associated with gallstones due to impaired entero-hepatic circulation. Apart from

PSC, ulcerative colitis does not increase the risk of other liver lesions.

Ulcerative colitis

Ulcerative colitis is a form of inflammatory bowel disease. Inflammation always starts

at rectum, never spreads beyond ileocaecal valve and is continuous. The peak

incidence of ulcerative colitis is in people aged 15-25 years and in those aged 55-65

years. It is less common in smokers.

The initial presentation is usually following insidious and intermittent symptoms.

Features include:

bloody diarrhoea

urgency

tenesmus

abdominal pain, particularly in the left lower quadrant

extra-intestinal features (see below)

Questions regarding the 'extra-intestinal' features of inflammatory bowel disease are

common. Extra-intestinal features include sclerosing cholangitis, iritis and ankylosing

spondylitis.

{Common to both Crohn's

disease (CD) and Ulcerative

colitis (UC)}

{Notes}

{Related to

disease activity}

Arthritis: pauciarticular,

asymmetric

Erythema nodosum

Episcleritis

Osteoporosis

Arthritis is the most common

extra-intestinal feature in both

CD and UC

Episcleritis is more common in

CD

{Unrelated to

disease activity}

Arthritis: polyarticular,

symmetric

Uveitis

Pyoderma gangrenosum

Clubbing

Primary sclerosing cholangitis

Primary sclerosing cholangitis is

much more common in UC

Uveitis is more common in UC

Pathology

red, raw mucosa, bleeds easily

no inflammation beyond submucosa (unless fulminant disease)

widespread superficial ulceration with preservation of adjacent mucosa which

has the appearance of polyps ('pseudopolyps')

inflammatory cell infiltrate in lamina propria

neutrophils migrate through the walls of glands to form crypt abscesses

depletion of goblet cells and mucin from gland epithelium

granulomas are infrequent

Barium enema

loss of haustrations

superficial ulceration, 'pseudopolyps'

long standing disease: colon is narrow and short -'drainpipe colon'

Endoscopy

Superficial inflammation of the colonic and rectal mucosa

Continuous disease from rectum proximally

Superifical ulceration, mucosal islands, loss of vascular definition and

continuous ulceration pattern.

Management

Patients with long term disease are at increased risk of development of

malignancy

Acute exacerbations are generally managed with steroids, in chronic patients

agents such as azathioprine and infliximab may be used

Individuals with medically unresponsive disease usually require surgery- in

the acute phase a sub total colectomy and end ileostomy. In the longer term a

proctectomy will be required. An ileoanal pouch is an option for selected

patients

Which of the following is not associated with thrombosis?

A. Endothelial cell damage

B. Use of tourniquets in surgery

C. Formation of platelet aggregates

D. Thrombocytopenia

E. Carcinoma of the stomach

All the other options either act directly to promote thrombosis e.g. endothelial cell

damage or via changes in consistency or flow of blood.

Abnormal coagulation

Cause Factors affected

Heparin Prevents activation factors 2,9,10,11

Warfarin Affects synthesis of factors 2,7,9,10

DIC Factors 1,2,5,8,11

Liver disease Factors 1,2,5,7,9,10

Interpretation blood clotting test results

Disorder APTT PT Bleeding time

Haemophilia Increased Normal Normal

von Willebrand's disease Increased Normal Increased

Vitamin K deficiency Increased Increased Normal

Which of the following is not associated with thrombosis?

A. Endothelial cell damage

B. Use of tourniquets in surgery

C. Formation of platelet aggregates

D. Thrombocytopenia

E. Carcinoma of the stomach

All the other options either act directly to promote thrombosis e.g. endothelial cell

damage or via changes in consistency or flow of blood.

Abnormal coagulation

Cause Factors affected

Heparin Prevents activation factors 2,9,10,11

Warfarin Affects synthesis of factors 2,7,9,10

DIC Factors 1,2,5,8,11

Liver disease Factors 1,2,5,7,9,10

Interpretation blood clotting test results

Disorder APTT PT Bleeding time

Haemophilia Increased Normal Normal

von Willebrand's disease Increased Normal Increased

Vitamin K deficiency Increased Increased Normal

A 16 year old boy develops a painful swelling of his distal femur. An osteoblastic

sarcoma is diagnosed. To which of the following sites is this lesion most likely to

metastasise?

A. Inguinal lymph nodes

B. Common iliac lymph nodes

C. Liver

D. Brain

E. Lung

Sarcomas in which Lymphatic

Metastasis is seen:

'RACE For MS'

R: Rhabdomyosarcoma

A: Angiosarcoma

C: Clear cell sarcoma

E: Epithelial cell sarcoma

For: Fibrosarcoma

M: Malignant fibrous histiocytoma

S: Synovial cell sarcoma

Or

'SCARE'

Synovial sarcoma

Clear cell sarcoma

Angiosarcoma

Rhabdomyosarcoma

Epithelioid sarcoma

Sarcomas often metastasise via the haematogenous route and the lung is a common

site for sarcoma metastasis. The liver and brain are often spared (at least initially). A

smaller number may develop lymphatic metastasis (see above).

Sarcomas

Malignant tumours of mesenchymal origin

Types May be either bone or soft tissue in origin.

Bone sarcoma include:

Osteosarcoma

Ewings sarcoma (although non bony sites recognised)

Chrondrosarcoma - originate from Chondrocytes

Soft tissue sarcoma are a far more heterogeneous group and include:

Liposarcoma-adipocytes

Rhabdomyosarcoma-striated muscle

Leiomyosarcoma-smooth muscle

Synovial sarcomas- close to joints (cell of origin not known but not synovium)

Malignant fibrous histiocytoma is a sarcoma that may arise in both soft tissue and

bone.

Features Certain features of a mass or swelling should raise suspicion for a sarcoma these

include:

Large >5cm soft tissue mass

Deep tissue location or intra muscular location

Rapid growth

Painful lump

Assessment Imaging of suspicious masses should utilise a combination of MRI, CT and USS.

Blind biopsy should not be performed prior to imaging and where required should be

done in such a way that the biopsy tract can be subsequently included in any

resection.

Ewings sarcoma

Commoner in males

Incidence of 0.3 / 1, 000, 000

Onset typically between 10 and 20 years of age

Location by femoral diaphysis is commonest site

Histologically it is a small round tumour

Blood borne metastasis is common and chemotherapy is often combined with

surgery

Osteosarcoma

Mesenchymal cells with osteoblastic differentiation

20% of all primary bone tumours

Incidence of 5 per 1,000,000

Peak age 15-30, commoner in males

Limb preserving surgery may be possible and many patients will receive

chemotherapy

Liposarcoma

Malignancy of adipocytes

Rare approximately 2.5 per 1,000,000. They are the second most common soft

tissue sarcoma

Typically located in deep locations such as retroperitoneum

Affect older age group usually >40 years of age

May be well differentiated and thus slow growing although may undergo

dedifferentiation and disease progression

Many tumours will have a pseudocapsule that can misleadingly allow

surgeons to feel that they can 'shell out' these lesions. In reality tumour may

invade at the edge of the pseudocapsule and result in local recurrence if this

strategy is adopted

Usually resistant to radiotherapy although this is often used in a palliative

setting

Malignant Fibrous Histiocytoma

Tumour with large number of histiocytes

Most common sarcoma in adults

Also described as undifferentiated pleomorphic sarcoma NOS (i.e. Cell of

origin is not known)

Four major subtypes are recognised: storiform-pleomorphic (70% cases),

myxoid (less aggressive), giant cell and inflammatory

Treatment is usually with surgical resection and adjuvant radiotherapy as this

reduces the likelihood of local recurrence

A 16 year old boy develops a painful swelling of his distal femur. An osteoblastic

sarcoma is diagnosed. To which of the following sites is this lesion most likely to

metastasise?

A. Inguinal lymph nodes

B. Common iliac lymph nodes

C. Liver

D. Brain

E. Lung

Sarcomas in which Lymphatic

Metastasis is seen:

'RACE For MS'

R: Rhabdomyosarcoma

A: Angiosarcoma

C: Clear cell sarcoma

E: Epithelial cell sarcoma

For: Fibrosarcoma

M: Malignant fibrous histiocytoma

S: Synovial cell sarcoma

Or

'SCARE'

Synovial sarcoma

Clear cell sarcoma

Angiosarcoma

Rhabdomyosarcoma

Epithelioid sarcoma

Sarcomas often metastasise via the haematogenous route and the lung is a common

site for sarcoma metastasis. The liver and brain are often spared (at least initially). A

smaller number may develop lymphatic metastasis (see above).

Sarcomas

Malignant tumours of mesenchymal origin

Types May be either bone or soft tissue in origin.

Bone sarcoma include:

Osteosarcoma

Ewings sarcoma (although non bony sites recognised)

Chrondrosarcoma - originate from Chondrocytes

Soft tissue sarcoma are a far more heterogeneous group and include:

Liposarcoma-adipocytes

Rhabdomyosarcoma-striated muscle

Leiomyosarcoma-smooth muscle

Synovial sarcomas- close to joints (cell of origin not known but not synovium)

Malignant fibrous histiocytoma is a sarcoma that may arise in both soft tissue and

bone.

Features Certain features of a mass or swelling should raise suspicion for a sarcoma these

include:

Large >5cm soft tissue mass

Deep tissue location or intra muscular location

Rapid growth

Painful lump

Assessment

Imaging of suspicious masses should utilise a combination of MRI, CT and USS.

Blind biopsy should not be performed prior to imaging and where required should be

done in such a way that the biopsy tract can be subsequently included in any

resection.

Ewings sarcoma

Commoner in males

Incidence of 0.3 / 1, 000, 000

Onset typically between 10 and 20 years of age

Location by femoral diaphysis is commonest site

Histologically it is a small round tumour

Blood borne metastasis is common and chemotherapy is often combined with

surgery

Osteosarcoma

Mesenchymal cells with osteoblastic differentiation

20% of all primary bone tumours

Incidence of 5 per 1,000,000

Peak age 15-30, commoner in males

Limb preserving surgery may be possible and many patients will receive

chemotherapy

Liposarcoma

Malignancy of adipocytes

Rare approximately 2.5 per 1,000,000. They are the second most common soft

tissue sarcoma

Typically located in deep locations such as retroperitoneum

Affect older age group usually >40 years of age

May be well differentiated and thus slow growing although may undergo

dedifferentiation and disease progression

Many tumours will have a pseudocapsule that can misleadingly allow

surgeons to feel that they can 'shell out' these lesions. In reality tumour may

invade at the edge of the pseudocapsule and result in local recurrence if this

strategy is adopted

Usually resistant to radiotherapy although this is often used in a palliative

setting

Malignant Fibrous Histiocytoma

Tumour with large number of histiocytes

Most common sarcoma in adults

Also described as undifferentiated pleomorphic sarcoma NOS (i.e. Cell of

origin is not known)

Four major subtypes are recognised: storiform-pleomorphic (70% cases),

myxoid (less aggressive), giant cell and inflammatory

Treatment is usually with surgical resection and adjuvant radiotherapy as this

reduces the likelihood of local recurrence

Infection with which of the following micro-organisms may result in a clinical picture

resembling achalasia of the oesphagus?

A. Epstein Barr virus

B. Wuchereria Bancrofti

C. Candida Spp

D. Trypanosoma Cruzi

E. Helicobacter Pylori

Infection with Trypanosoma Cruzi may result in destruction of the ganglion cells of

the myenteric plexus, resulting in a clinical picture similar to achalasia.

Trypanosoma Cruzi

Protozoan

Causes Chagas disease

Carried by bugs which infect the skin whilst feeding

Penetrate through open wounds and mucous membranes

Intracellular proliferation

Major infective sites include CNS, intestinal myenteric plexus, spleen, lymph

nodes and cardiac muscle

Chronic disease is irreversible, nifurtimox is used to treat acute infection

A 45-year-old man presents to surgical outpatients with a long history of recurrent

abdominal pain and vomiting. He is noted to have a peripheral motor neuropathy on

examination. What is the most likely diagnosis?

A. Huntington's disease

B. Myeloma

C. Acute intermittent porphyria

D. Lawrence-Moon-Biedl syndrome

E. Friedreich's ataxia

Neurological signs combined with abdominal pain is acute intermittent porphyria or

lead poisoning until proven otherwise.

Acute intermittent porphyria

Acute intermittent porphyria (AIP) is a rare autosomal dominant condition caused by

a defect in porphobilinogen deaminase, an enzyme involved in the biosynthesis of

haem. The results in the toxic accumulation of delta aminolaevulinic acid and

porphobilinogen. It characteristically presents with abdominal and neuropsychiatric

symptoms in 20-40 year olds. AIP is more common in females (5:1)

Features

abdominal: abdominal pain, vomiting

neurological: motor neuropathy

psychiatric: e.g. depression

hypertension and tachycardia common

Diagnosis

classically urine turns deep red on standing

raised urinary porphobilinogen (elevated between attacks and to a greater

extent during acute attacks)

assay of red cells for porphobilinogen deaminase

raised serum levels of delta aminolaevulinic acid and porphobilinogen

A 56 year old man presents with episodic facial pain and discomfort whilst eating. He

has suffered from halitosis recently and he frequently complains of a dry mouth. He

has a smooth swelling underneath his right mandible. What is the most likely

underlying diagnosis?

A. Stone impacted in Whartons duct

B. Stone impacted in Stensens duct

C. Benign adenoma of the submandibular gland

D. Adenocarcinoma of the submandibular gland

E. Squamous cell carcinoma of the submandibular gland

The symptoms are typical for sialolithiasis. The stones most commonly form in the

submandibular gland and therefore may occlude Whartons duct. Stensens duct drains

the parotid gland.

Submandibular glands- disease

Physiology The submandibular glands secrete approximately 800- 1000ml saliva per day. They

typically produce mixed seromucinous secretions. When paraympathetic activity is

dominant the secretions will be more serous. The parasympathetic fibres are derived

from the chorda tympani nerves and the submandibular ganglion, they travel to the

glands via the lingual nerves.

Sialolithiasis

80% of all salivary gland calculi occur in the submandibular gland

70% of the these calculi are radio-opaque

Stones are usually composed of calcium phosphate or calcium carbonate

Patients typically develop colicky pain and post prandial swelling of the gland

Investigation involves sialography to demonstrate the site of obstruction and

associated other stones

Stones impacted in the distal aspect of Whartons duct may be removed orally,

other stones and chronic inflammation will usually require gland excision

Sialadenitis

Usually occurs as a result of Staphylococcus aureus infection

Pus may be seen leaking from the duct, erythema may also be noted

Development of a sub mandibular abscess is a serious complication as it may

spread through the other deep fascial spaces and occlude the airway

Submandibular tumours

Only 8% of salivary gland tumours affect the sub mandibular gland

Of these 50% are malignant (usually adenoid cystic carcinoma)

Diagnosis usually involves fine needle aspiration cytology

Imaging is with CT and MRI

In view of the high prevalence of malignancy, all masses of the submandibular

glands should generally be excised.

Which of the following cellular types or features is not seen in sarcoidosis?

A. Reed Sternberg Cells

B. T lymphocytes

C. Macrophages

D. Asteroid bodies

E. B lymphocytes

Reed Sternberg cells are seen in Hodgkins disease. All of the other cell types are seen

in sarcoid.

Chronic inflammation

Overview Chronic inflammation may occur secondary to acute inflammation.In most cases

chronic inflammation occurs as a primary process. These may be broadly viewed as

being one of three main processes:

Persisting infection with certain organisms such as Mycobacterium

tuberculosis which results in delayed type hypersensitivity reactions and

inflammation.

Prolonged exposure to non-biodegradable substances such as silica or suture

materials which may induce an inflammatory response.

Autoimmune conditions involving antibodies formed against host antigens.

Acute vs. Chronic inflammation

Acute inflammation Chronic inflammation

Changes to existing vascular structure and

increased permeability of endothelial cells

Angiogenesis predominates

Infiltration of neutrophils Macrophages, plasma cells and

lymphocytes predominate

Process may resolve with:

Suppuration

Complete resolution

Abscess formation

Progression to chronic inflammation

Healing by fibrosis

Healing by fibrosis is the main

result

Granulomatous inflammation A granuloma consists of a microscopic aggregation of macrophages (with epithelial

type arrangement =epitheliod). Large giant cells may be found at the periphery of

granulomas.

Mediators Growth factors released by activated macrophages include agents such as interferon

and fibroblast growth factor (plus many more). Some of these such as interferons may

have systemic features resulting in systemic symptoms and signs, which may be

present in individuals with long standing chronic inflammation.

The finding of granulomas is pathognomonic of chronic inflammation, as illustrated

in this biopsy from a patient with colonic Crohns disease

Image sourced from Wikipedia

Which of the following diseases is not considered a risk factor for gastric cancer?

A. Polya gastrectomy for antral ulcer

B. Atrophic gastritis

C. Intestinal metaplasia of columnar type at the gastric cardia

D. Patient with polyp showing medium grade dysplasia

E. Long term therapy with H2 blockers

Although some acid lowering procedures increase the risk of gastric cancer the use of

H2 blockers does not, at the present time, seem to increase the risk.

Gastric cancer

Overview There are 700,000 new cases of gastric cancer worldwide each year. It is most

common in Japan and less common in western countries. It is more common in men

and incidence rises with increasing age. The exact cause of many sporadic cancer is

not known, however, familial cases do occur in HNPCC families. In addition,

smoking and smoked or preserved foods increase the risk. Japanese migrants retain

their increased risk (decreased in subsequent generations). The distribution of the

disease in western countries is changing towards a more proximal location (perhaps

due to rising obesity).

Pathology There is some evidence of support a stepwise progression of the disease through

intestinal metaplasia progressing to atrophic gastritis and subsequent dysplasia,

through to cancer. The favoured staging system is TNM. The risk of lymph node

involvement is related to size and depth of invasion; early cancers confined to

submucosa have a 20% incidence of lymph node metastasis. Tumours of the gastro-

oesophageal junction are classified as below:

Type

1

True oesophageal cancers and may be associated with Barrett's oesophagus.

Type

2

Carcinoma of the cardia, arising from cardiac type epithelium

or short segments with intestinal metaplasia at the oesophagogastric junction.

Type

3

Sub cardial cancers that spread across the junction. Involve similar nodal

stations to gastric cancer.

Groups for close endoscopic monitoring

Intestinal metaplasia of columnar type

Atrophic gastritis

Low to medium grade dysplasia

Patients who have previously undergone resections for benign peptic ulcer

disease (except highly selective vagotomy).

Referral to endoscopy

Patients of any age with

dyspepsia and any of the

following

Patients without

dyspepsia

Worsening dyspepsia

Chronic gastrointestinal

bleeding

Dysphagia Barretts oesophagus

Dysphagia Unexplained

abdominal pain or

weight loss

Intestinal metaplasia

Weight loss Vomiting Dysplasia

Iron deficiency anaemia Upper abdominal mass Atrophic gastritis

Upper abdominal mass Jaundice Patient aged over 55 years

with unexplained or persistent

dyspepsia

Upper GI endoscopy performed for dyspepsia. The addition of dye spraying (as

shown in the bottom right) may facilitate identification of smaller tumours

Image sourced from Wikipedia

Staging

CT scanning of the chest abdomen and pelvis is the routine first line staging

investigation in most centres.

Laparoscopy to identify occult peritoneal disease

PET CT (particularly for junctional tumours)

Treatment

Proximally sited disease greater than 5-10cm from the OG junction may be

treated by sub total gastrectomy

Total gastrectomy if tumour is <5cm from OG junction

For type 2 junctional tumours (extending into oesophagus)

oesophagogastrectomy is usual

Endoscopic sub mucosal resection may play a role in early gastric cancer

confined to the mucosa and perhaps the sub mucosa (this is debated)

Lymphadenectomy should be performed. A D2 lymphadenectomy is widely

advocated by the Japanese, the survival advantages of extended

lymphadenectomy have been debated. However, the overall recommendation

is that a D2 nodal dissection be undertaken.

Most patients will receive chemotherapy either pre or post operatively.

Prognosis

UK Data

Disease extent Percentage 5 year survival

All RO resections 54%

Early gastric cancer 91%

Stage 1 87%

Stage 2 65%

Stage 3 18%

Operative procedure

Total Gastrectomy , lymphadenectomy and Roux en Y anastomosis

General anaesthesia

Prophylactic intravenous antibiotics

Incision: Rooftop.

Perform a thorough laparotomy to identify any occult disease.

Mobilise the left lobe of the liver off the diaphragm and place a large pack over it.

Insert a large self retaining retractor e.g. omnitract or Balfour (take time with this, the

set up should be perfect). Pack the small bowel away.

Begin by mobilising the omentum off the transverse colon.

Proceed to detach the short gastric vessels.

Mobilise the pylorus and divide it at least 2cm distally using a linear cutter stapling

device.

Continue the dissection into the lesser sac taking the lesser omentum and left gastric

artery flush at its origin.

The lymph nodes should be removed en bloc with the specimen where possible.

Place 2 stay sutures either side of the distal oesophagus. Ask the anaesthetist to pull

back on the nasogastric tube. Divide the distal oesophagus and remove the stomach.

The oesphago jejunal anastomosis should be constructed. Identify the DJ flexure and

bring a loop of jejunum up to the oesophagus (to check it will reach). Divide the

jejunum at this point. Bring the divided jejunum either retrocolic or antecolic to the

oesophagus. Anastamose the oesophagus to the jejunum, using either interrupted 3/0

vicryl or a stapling device. Then create the remainder of the Roux en Y reconstruction

distally.

Place a jejunostomy feeding tube.

Wash out the abdomen and insert drains (usually the anastomosis and duodenal

stump). Help the anaesthetist insert the nasogastric tube (carefully!)

Close the abdomen and skin.

Enteral feeding may commence on the first post-operative day. However, most

surgeons will leave patients on free NG drainage for several days and keep them nil

by mouth.

A 56 year old man is diagnosed as having a glioma. From which of the following cell

types do these tumours usually originate?

A. Astrocytes

B. Oligodendrocytes

C. Ependymal cells

D. Squamous cells

E. Neuroglial cells

Theme from January 2012 Exam

Gliomas originate from glial (otherwise known as neuroglial) cells. These serve a

structural function in the CNS. The tumours produced may resemble a number of

CNS cell types. Tumours are therefore named according to the cells they resemble

rather than the origin. Where this is not possible they are termed gliomas.

Glioma

Glioma is a tumour that is typically found in the CNS. These tumours arise from glial

cells. They are sub categorised according to the cell type they most closely resemble.

Glioma sub types

Ependymomas- Ependymal cells

Astocytomas- Astrocytes (including glioblastoma)

Oligodendrogliomas- Oligodendrocytes

Mixed- e.g. oligoastrocytomas

Gliomas are categorised as being either high or low grade lesions (the former has the

worse prognosis). They may be either supra or infra tentorial. Their symptoms will

typically reflect their site of origin. Glioblastoma multiforme has the worst prognosis

and few patients will survive beyond 12 months.

A 78 year old man presents with unilateral deafness which has been present for the

past 3 months. On examination Webers test localises to the contralateral side and a

CT scan of his head shows a thickened calvarium with areas of sclerosis and

radiolucency. His blood tests show an elevated alkaline phosphatase, normal serum

calcium and normal PTH levels. Which of the following is the most likely underlying

diagnosis?

A. Multiple myeloma with skull involvement

B. Osteoporosis

C. Pagets disease with skull involvement

D. Lung cancer with skull metastasis

E. Osteopetrosis with skull involvement

Of the conditions listed Pagets disease is the most likely diagnosis (skull vault

expansion and sensorineural hearing loss). Multiple myeloma would typically result

in multiple areas of radiolucency and usually raised calcium in this setting.

Osteopetrosis is a recognised cause of the features described. However, it is a rare

inherited disorder and usually presents in children in young adults. Presentation at this

stage with no prior symptoms would be extremely rare and therefore this is not the

most likely diagnosis.

Pagets disease

Paget's disease is a disease of increased but uncontrolled bone turnover and is

characterised by architecturally abnormal bones. It is thought to be primarily a

disorder of osteoclasts, with excessive osteoclastic resorption followed by increased

osteoblastic activity causing areas of sclerosis and deformity. Paget's disease is

common (UK prevalence 5%) but symptomatic in only 1 in 20 patients

Predisposing factors

increasing age

male sex

northern latitude

family history

Clinical features

bone pain (e.g. pelvis, lumbar spine, femur)

classical, untreated features: bowing of tibia, bossing of skull

raised alkaline phosphatase (ALP) - calcium* and phosphate are typically

normal

skull x-ray: thickened vault, osteoporosis circumscripta

Indications for treatment include bone pain, skull or long bone deformity, fracture,

periarticular Paget's

bisphosphonate (either oral risedronate or IV zoledronate)

calcitonin is less commonly used now

Complications

deafness (cranial nerve entrapment)

bone sarcoma (1% if affected for > 10 years)

fractures

skull thickening

high-output cardiac failure

*usually normal in this condition but hypercalcaemia may occur with prolonged

immobilisation

Theme: Genetic causes of cancer

A. Multiple endocrine neoplasia type I

B. Multiple endocrine neoplasia type II

C. Gardner's syndrome

D. Lynch Syndrome

E. Kartagener's syndrome

F. Von Recklinghausen's disease

Please select the most likely condition for the disease process described. Each option

may be used once, more than once or not at all

13. A 5 year old boy presents with recurrent episodes of sinusitis. The casualty

staff are surprised to find his liver lying in the left upper quadrant of the

abdomen

Kartagener's syndrome

This is a case of Kartagener's syndrome. The primary problem is of immotile

cilia syndrome. When associated with situs inversus Kartagener's syndrome is

diagnosed.

14. A 22 year old man presents with carcinoma of the caecum. His brother died

from colorectal cancer aged 25 and his father died from the disease aged 30.

Lynch Syndrome

This is a case of Lynch syndrome HNPCC. It is transmitted in an autosomal

dominant fashion.

15. A tall 32 year old lady presents with a diffuse neck swelling a carcinoma of

the thyroid medullary type is diagnosed.

Multiple endocrine neoplasia type II

This is a case MEN type IIb. It is associated with phaeochromocytomas and is

transmitted in an autosomal dominant pattern if inherited. All MEN II tend to

have medullary carcinoma of the thyroid as a presenting feature

Genetics and surgical disease

Some of the more commonly occurring genetic conditions occurring in surgical

patients are presented here.

Li-Fraumeni Syndrome

Autosomal dominant

Consists of germline mutations to p53 tumour suppressor gene

High incidence of malignancies particularly sarcomas and leukaemias

Diagnosed when:

*Individual develops sarcoma under 45 years

*First degree relative diagnosed with any cancer below age 45 years and another

family member develops malignancy under 45 years or sarcoma at any age

BRCA 1 and 2

Carried on chromosome 17

Linked to developing breast cancer (60%) risk.

Associated risk of developing ovarian cancer (55% with BRCA 1 and 25%

with BRCA2).

Lynch Syndrome

Autosomal dominant

Develop colonic cancer and endometrial cancer at young age

80% of affected individuals with get colonic and or endometrial cancer

High risk individuals may be identified using the Amsterdam criteria

Amsterdam criteria Three or more family members with a confirmed diagnosis of colorectal cancer, one

of whom is a first degree (parent, child, sibling) relative of the other two.

Two successive affected generations.

One or more colon cancers diagnosed under age 50 years.

Familial adenomatous polyposis (FAP) has been excluded.

Gardners syndrome

Autosomal dominant familial colorectal polyposis

Multiple colonic polyps

Extra colonic diseases include: skull osteoma, thyroid cancer and epidermoid

cysts

Desmoid tumours are seen in 15%

Mutation of APC gene located on chromosome 5

Due to colonic polyps most patients will undergo colectomy to reduce risk of

colorectal cancer

Now considered a variant of familial adenomatous polyposis coli

A 45 year old man presents with symptoms of urinary colic. In the history he has

suffered from recurrent episodes of frank haematuria over the past week or so. On

examination he has a left loin mass and a varicocele. The most likely diagnosis is:

A. Renal adenocarcinoma

B. Renal cortical adenoma

C. Squamous cell carcinoma of the renal pelvis

D. Retroperitoneal fibrosis

E. Nephroblastoma

Renal adenocarcinoma are the most common renal malignancy and account

for 75% cases.

Patients may develop frank haematuria and have episodes of clot colic.

A Grawitz tumour is an eponymous name for Renal Adenocarcinoma.

May metastasise to bone.

Renal tumours

Renal cell carcinoma Renal cell carcinoma is an adenocarcinoma of the renal cortex and is believed to arise

from the proximal convoluted tubule. They are usually solid lesions, up to 20% may

be multifocal, 20% may be calcified and 20% may have either a cystic component or

be wholly cystic. They are often circumscribed by a pseudocapsule of compressed

normal renal tissue. Spread may occur either by direct extension into the adrenal

gland, renal vein or surrounding fascia. More distant disease usually occurs via the

haematogenous route to lung, bone or brain.

Renal cell carcinoma comprise up to 85% of all renal malignancies. Males are more

commonly affected than females and sporadic tumours typically affect patients in

their sixth decade.

Patients may present with a variety of symptoms including; haematuria (50%), loin

pain (40%), mass (30%) and up to 25% may have symptoms of metastasis.Less than

10% have the classic triad of haematuria, pain and mass.

Investigation Many cases will present as haematuria and be discovered during diagnostic work up.

Benign renal tumours are rare, so renal masses should be investigated with multislice

CT scanning. Some units will add and arterial and venous phase to the scan to

demonstrate vascularity and evidence of caval ingrowth.

CT scanning of the chest and abdomen to detect distant disease should also be

undertaken.

Routine bone scanning is not indicated in the absence of symptoms.

Biopsy should not be performed when a nephrectomy is planned but is mandatory

before any ablative therapies are undertaken.

Assessment of the functioning of the contra lateral kidney.

Management T1 lesions may be managed by partial nephrectomy and this gives equivalent

oncological results to total radical nephrectomy. Partial nephrectomy may also be

performed when there is inadequate reserve in the remaining kidney.

For T2 lesions and above a radical nephrectomy is standard practice and this may be

performed via a laparoscopic or open approach. Preoperative embolisation is not

indicated nor is resection of uninvolved adrenal glands. During surgery early venous

control is mandatory to avoid shedding of tumour cells into the circulation.

Patients with completely resected disease do not benefit from adjuvant therapy

with either chemotherapy or biological agents. These should not be administered

outside the setting of clinical trials.

Patients with transitional cell cancer will require a nephroureterectomy with

disconnection of the ureter at the bladder.

References Lungberg B et al. EAU guidelines on renal cell carcinoma: The 2010 update.

European Urology 2010 (58): 398-406.

A 63 year old man finds that he has to stop walking after 100 yards due to bilateral

calf pain. He finds that bending forwards and walking up hill helps. He is able to ride

a bike without any pain. What is the most likely underlying cause?

A. Lumbar canal stenosis

B. Diabetic neuropathy

C. Aorto-iliac occlusion

D. Occlusion of the superficial femoral artery

E. Pelvic rheumatoid arthritis

Theme from April 2012 Exam The positional nature of the pain and the fact that improves with walking uphill makes

an underlying vascular aetiology far less likely.

Lumbar spinal stenosis

Lumbar spinal stenosis is a condition in which the central canal is narrowed by

tumour, disk prolapse or other similar degenerative changes.

Patients may present with a combination of back pain, neuropathic pain and

symptoms mimicking claudication. One of the main features that may help to

differentiate it from true claudication in the history is the positional element to the

pain. Sitting is better than standing and patients may find it easier to walk uphill rather

than downhill. The neurogenic claudication type history makes lumbar spinal stenosis

a likely underlying diagnosis, the absence of such symptoms makes it far less likely.

Pathology Degenerative disease is the commonest underlying cause. Degeneration is believed to

begin in the intervertebral disk where biochemical changes such as cell death and loss

of proteoglycan and water content lead to progressive disk bulging and collapse. This

process leads to an increased stress transfer to the posterior facet joints, which

accelerates cartilaginous degeneration, hypertrophy, and osteophyte formation; this is

associated with thickening and distortion of the ligamentum flavum. The combination

of the ventral disk bulging, osteophyte formation at the dorsal facet, and ligamentum

flavum hyptertrophy combine to circumferentially narrow the spinal canal and the

space available for the neural elements. The compression of the nerve roots of the

cauda equina leads to the characteristic clinical signs and symptoms of lumbar spinal

stenosis.

Diagnosis MRI scanning is the best modality for demonstrating the canal narrowing. Historically

a bicycle test was used as true vascular claudicants could not complete the test.

Treatment Laminectomy

A 73 year old lady is admitted for a laparoscopic cholecystectomy. During her pre-

operative assessment it is noted that she is receiving furosemide for the treatment of

hypertension. Approximately what proportion of the sodium that is filtered at the

glomerulus will be subsequently excreted?

A. Up to 25%

B. Upt to 75%

C. Between 3 and 5%

D. <2%

E. Between 1 and 2%

Theme from 2010 Exam

The loop diuretics can lead to marked increases in the amount of sodium excreted.

They act in the medullary and cortical aspects of the thick ascending limb of the loop

of Henle. This results in a decreased medullary osmolal gradient and increases free

water excretion (as well as loss of sodium). Because loop diuretics result in the loss of

both sodium and water they are less frequently associated with hyponatraemia than

thiazide diuretics (these latter agents act in the cortex and do not affect urine

concentrating ability).

Diuretic agents

The diuretic drugs are divided into three major classes, which are distinguished

according to the site at which they impair sodium reabsorption: loop diuretics in the

thick ascending loop of Henle, thiazide type diuretics in the distal tubule and

connecting segment; and potassium sparing diuretics in the aldosterone - sensitive

principal cells in the cortical collecting tubule.

In the kidney, sodium is reabsorbed through Na+/ K+ ATPase pumps located on the

basolateral membrane. These pumps return reabsorbed sodium to the circulation and

maintain low intracellular sodium levels. This latter effect ensures a constant

concentration gradient.

Physiological effects of commonly used diuretics

Site of action Diuretic Carrier or

channel inhibited

Percentage of filtered

sodium excreted

Ascending limb of loop

of Henle

Frusemide Na+/K+ 2Cl -

carrier

Upt to 25%

Distal tubule and

connecting segment

Thiazides Na+Cl- carrier Between 3 and 5%

Cortical collecting

tubule

Spironolactone Na+ channel Between 1 and 2%

A 59 year old man presents with recurrent episodes of urinary sepsis. In his history he

mentions that he has suffered from recurrent attacks of left iliac fossa pain over the

past few months. He has also notices bubbles in his urine. He undergoes a CT scan

which shows a large inflammatory mass in the left iliac fossa. No other abnormality is

detected. The most likely diagnosis is:

A. Ulcerative colitis

B. Crohns disease

C. Mesenteric ischaemia

D. Diverticular disease

E. Rectal cancer

Theme from 2009 and 2011 Exam

Recurrent attacks of diverticulitis may cause the development of local abscesses

which may erode into the bladder resulting in urinary sepsis and pneumaturia. This

would be an unusual presentation from Crohns disease and rectal cancer would be

more distally sited and generally evidence of extra colonic disease would be present if

the case were malignant and this advanced.

Diverticular disease

Diverticular disease is a common surgical problem. It consists of herniation of colonic

mucosa through the muscular wall of the colon. The usual site is between the taenia

coli which vessels pierce the muscle to supply the mucosa.

Symptoms

Altered bowel habit

Bleeding

Abdominal pain

Complications

Diverticulitis

Haemorrhage

Development of fistula

Perforation and faecal peritonitis

Perforation and development of abscess

Development of diverticular phlegmon

Diagnosis Patients presenting in clinic will typically undergo either a colonoscopy or barium

enema as part of their diagnostic work up. Both tests will identify diverticular disease.

It can be far more difficult to confidently exclude cancer, particularly in diverticular

strictures.

Acutely unwell surgical patients should be investigated in a systematic way. Plain

abdominal films and an erect chest x-ray will identify perforation. An abdominal CT

scan with oral and intravenous contrast will help to identify whether acute

inflammation is present but also the presence of local complications such as abscess

formation.

Severity Classification- Hinchey

I Para-colonic abscess

II Pelvic abscess

III Purulent peritonitis

IV Faecal peritonitis

Treatment

Increase dietary fibre intake.

Mild attacks of diverticulitis may be managed conservatively with antibiotics.

Peri colonic abscesses should be drained either surgically or radiologically.

Recurrent episodes of acute diverticulitis requiring hospitalisation are a

relative indication for a segmental resection.

Hinchey IV perforations (generalised faecal peritonitis) will require a

resection and usually a stoma. This group have a very high risk of post

operative complications and usually require HDU admission. Less severe

perforations may be managed by laparoscopic washout and drain insertion.

A 78 year old man is referred to the clinic by his general practitioner. For many years

he noticed a smooth swelling approximately 5cm anterior to the tragus of his right ear.

Apart from being a heavy smoker he has no co-morbidities. What is the most likely

diagnosis?

A. Pleomorphic adenoma

B. Liposarcoma

C. Warthins tumour

D. Adenocarcinoma

E. None of the above

Warthins tumours are most common in elderly smokers. They have a relatively

benign and indolent course. They are usually well circumscribed as illustrated below:

Image sourced from Wikipedia

Parotid gland clinical

Causes of bilateral parotid enlargement

Mumps: Associated with meningoencephalitis, pancreatitis, orchitis, or

deafness

Parotitis

Sialectasis - especially if related to eating

Sjogren's syndrome: dry eyes or mouth, connective tissue disease

Sarcoidosis

Tuberculosis

Alcoholism

Myxoedema

Cushing's disease

Diabetes/insulin resistance

Liver cirrhosis

Gout

Bulimia nervosa

Drugs

Severe dehydration

Malnutrition

Causes of unilateral parotid enlargement

Salivary calculus

Tumour

Parotid gland tumours

Pleomorphic adenomas are the most common.

Incisional biopsy of parotid masses is not recommended, so superficial

parotidectomy is the usual procedure of choice.

Signs of facial nerve palsy and a parotid mass should raise suspicion of

malignancy.

Warthins tumours are relatively benign lesions that are slow growing and

occur most commonly in elderly male smokers.

Adenoid cystic carcinoma have a tendency for perineural invasion.

Theme: Lung cancer

A. Adenocarcinoma

B. Small cell lung cancer

C. Large cell lung cancer

D. Squamous cell carcinoma

Please select the most likely lung cancer variant for the scenario described. Each

option may be used once, more than once or not at all.

21. A 73 year old heavy smoker presents with haemoptysis. On examination he is

cachectic and shows evidence of clubbing. Imaging shows a main bronchial

tumour with massive mediastinal lymphadenopathy together with widespread

visceral metastases.

Small cell lung cancer

Theme from April 2012

Small cell carcinoma is associated with disseminated disease at presentation

in the majority of cases. Most cases occur in the main airways and

paraneoplastic features are common.

22. A 68 year old female who has never smoked presents with a mass at the

periphery of her right lung.

You answered Large cell lung cancer

The correct answer is Adenocarcinoma

Adenocarcinomas are the most common tumour type present in never

smokers. They are usually located at the periphery.

23. An 85 year old man presents with a cough and haemoptysis. He has a modest

smoking history of 15 pack years. He is found to have a tumour located in the

right main bronchus, with no evidence of metastatic disease. He decides no

undergo any treatment and he remains well for a further 12 months before

developing symptomatic metastasis.

You answered Large cell lung cancer

The correct answer is Squamous cell carcinoma

Squamous cell carcinomas are reported to be more slow growing and are

typically centrally located. Small cell carcinomas are usually centrally

located. However, small cell carcinomas would seldom be associated with a

survival of a year without treatment.

Lung cancer

Lung cancers may be classified according to histological subtypes. The main

distinction is between small cell and non small cell lung cancer. Non small cell lung

cancer is the most common variant and accounts for 80% of all lung cancers.

Non small cell lung cancer These share common features of prognosis and management. They comprise the

following tumours:

Squamous cell carcinoma (25% cases)

Adenocarcinoma (40% cases)

Large cell carcinoma (10% cases)

Paraneoplastic features and early disease dissemination are less likely than with small

cell lung carcinoma. Adenocarcinoma is the most common lung cancer type

encountered in never smokers.

Small cell lung carcinoma Small cell lung carcinomas are comprised of cells with a neuro endocrine

differentiation. The neuroendocrine hormones may be released from these cells with a

wide range of paraneoplastic associations. These tumours are strongly associated with

smoking and will typically arise in the larger airways. They disseminate early in the

course of the disease and although they are usually chemosensitive this seldom results

in long lasting remissions.

Which of the following cells is not found on a blood film post splenectomy?

A. Pappenheimer bodies

B. Stipple cells

C. Erythrocyte containing siderotic granules

D. Howell-Jolly bodies

E. Target cells

Stipple cells are found in lead poisoning/haemoglobinopathies.

Blood film in hyposplenism:

Howell-Jolly bodies

Pappenheimer bodies

Poikilocytes (Target cells)

Erythrocyte containing siderotic granules

Heinz bodies

Post splenectomy blood film changes

The loss of splenic tissue results in the inability to readily remove immature or

abnormal red blood cells from the circulation. The red cell count does not alter

significantly. However, cytoplasmic inclusions may be seen e.g. Howell-Jolly bodies.

In the first few days after splenectomy target cells, siderocytes and reticulocytes will

appear in the circulation. Immediately following splenectomy a granulocytosis

(mainly composed of neutrophils) is seen, this is replaced by a lymphocytosis and

monocytosis over the following weeks.

The platelet count is usually increased and this may be persistent, oral antiplatelet

agents may be needed in some patients.

Image showing Howell Jolly bodies (arrowed)

Image sourced from Wikipedia

A 45 year old man with long standing ulcerative colitis and rectal dysplasia presents

with a DALM lesion in the rectum. What is the most appropriate management option?

A. Snare polypectomy

B. Repeat endoscopy in 2 years

C. Discharge

D. Anterior resection

E. Panproctocolectomy

DALM lesions complicating ulcerative colitis should be managed with

panproctocolectomy. An anterior resection is inadequate since it will only remove the

rectum and ulcerative colitis affects the entire colon. Since many will be associated

with invasion a snare polypectomy is not sufficient either.

Colonic lesions - DALM

The term DALM lesion refers to a Dysplasia Associated Lesion or Mass.

They may complicate dysplasia occurring in patients with longstanding

ulcerative colitis.

They have a high incidence of invasive foci.

When they complicate longstanding ulcerative colitis, they should be treated

by panproctocolectomy.

Which of the metastatic bone tumours described below is at the greatest risk of

pathological fracture ?

A. Proximal humeral lesion from a prostate cancer

B. Vertebral body lesions from a prostate cancer

C. Peritrochanteric lesion from a carcinoma of the breast

D. Proximal humeral lesion from a carcinoma of the breast

E. Peritrochanteric lesion from a prostate cancer

Peritrochanteric lesions have the greatest risks of fracture (due to loading).

The lesions from breast cancer are usually lytic and therefore at higher risk

rather than the sclerotic lesions from prostate cancer.

Metastatic bone disease- risk of fracture

Metastatic bone tumours may be described as blastic, lytic or mixed.

Osteoblastic metastatic disease has the lowest risk of spontaneous fracture

when compared to osteolytic lesions of a similar size.

Lesions affecting the peritrochanteric region are most prone to spontaneous

fracture (because of loading forces at that site).

The factors are incorportated into the Mirel Scoring system to stratify the risk

of spontaneous fracture for bone metastasis of varying types.

Mirel Scoring system

Score

points

Site Radiographic

appearance

Width of bone

involved

Pain

1 Upper

extremity

Blastic Less than 1/3 Mild

2 Lower

extremity

Mixed 1/3 to 2/3 Moderate

3 Peritrochanteric Lytic More than 2/3 Aggravated by

function

Depending upon the score the treatment should be as follows:

Score Risk of fracture Treatment

9 or greater Impending (33%) Prophylactic fixation

8 Borderline Consider fixation

7 or less Not impending (4%) Non operative management

A 63 year old male presents with several episodes of haematuria. He suffers from

COPD secondary to long term smoking. Which is the most likely underlying cause?

A. Renal cortical adenoma

B. Renal adenocarcinoma

C. Nephroblastoma

D. Transitional cell carcinoma of the bladder

E. Adenocarcinoma of the bladder

Theme from 2009 Exam

TCC is the most common subtype and is strongly linked to smoking. The important

point to note in this question is the term most likely as renal adenocarcinoma may

produce similar symptoms but is less likely.

Bladder cancer

Bladder cancer is the second most common urological cancer. It most commonly

affects males aged between 50 and 80 years of age. Those who are current, or

previous (within 20 years), smokers have a 2-5 fold increased risk of the disease.

Occupational exposure to hydrocarbons such as alanine increases the risk. Although

rare in the UK, chronic bladder inflammation arising from Schistosomiasis infection

remains a common cause of squamous cell carcinomas, in those countries where the

disease is endemic.

Benign tumours Benign tumours of the bladder including inverted urothelial papilloma and

nephrogenic adenoma are uncommon.

Bladder malignancies

Transitional cell carcinoma (>90% of cases)

Squamous cell carcinoma ( 1-7% -except in regions affected by

schistosomiasis)

Adenocarcinoma (2%)

Transitional cell carcinomas may arise as solitary lesions, or may be multifocal,

owing to the effect of "field change" within the urothelium. Up to 70% of TCC's will

have a papillary growth pattern. These tumours are usually superficial in location and

accordingly have a better prognosis. The remaining tumours show either mixed

papillary and solid growth or pure solid growths. These tumours are typically more

prone to local invasion and may be of higher grade, the prognosis is therefore worse.

Those with T3 disease or worse have a 30% (or higher) risk of regional or distant

lymph node metastasis.

TNM Staging

Stage Description

T0 No evidence of tumour

Ta Non invasive papillary carcinoma

T1 Tumour invades sub epithelial connective tissue

T2a Inner half of detrusor invaded

T2b Outer half of detrusor invaded

T3 Tumour extends to perivesical fat

T4a Invasion of uterus, prostate or bowel

T4b Invasion of other abdominal organs

N0 No nodal disease

N1 Single lymph node metastasis (up to 2cm)

N2 Single node >2cm or multiple nodes up to 5cm

N3 Nodes over 5cm

M1 Distant disease

Presentation Most patients (85%) will present with painless, macroscopic haematuria. In those

patients with incidental microscopic haematuria, up to 10% of females aged over 50

will be found to have a malignancy (once infection excluded).

Staging Most will undergo a cystoscopy and biopsies or TURBT, this provides histological

diagnosis and information relating to depth of invasion. Locoregional spread is best

determined using pelvic MRI and distant disease CT scanning. Nodes of uncertain

significance may be investigated using PET CT.

Treatment Those with superficial lesions may be managed using TURBT in isolation. Those

with recurrences or higher grade/ risk on histology may be offered intravesical

chemotherapy. Those with T2 disease are usually offered either surgery (radical

cystectomy and ileal conduit) or radical radiotherapy.

Prognosis

T1 90%

T2 60%

T3 35%

T4a 10-25%

Any T, N1-N2 30%

Theme: Neck lumps

A. Cystic hygroma

B. Bartonella infection

C. Mycobacterium tuberculosis infection

D. Branchial cyst

E. Thyroglossal cyst

F. Pharyngeal pouch

G. Follicular thyroid cyst

H. Parathyroid adenoma

I. None of the above

Please select the most likely underlying disease process for the scenario given. Each

option may be used once, more than once or not at all.

28. A 25 year old cat lover presents with symptoms of abdominal pain, lethargy

and sweats. These have been present for the past two weeks. On examination

she has lymphadenopathy in the posterior triangle.

You answered Mycobacterium tuberculosis infection

The correct answer is Bartonella infection

Bartonella infection may occur following a cat scratch. The organism is

intracellular. Generalised systemic symptoms may occur for a week or so

prior to clinical presentation.

29. A 25 year old lady presents with an swelling located at the anterior border of

the sternocleidomastoid muscle. The swelling is intermittent and on

examination it is soft and fluctuant.

Branchial cyst

Branchial cysts are remnants of the branchial cleft. They may become

infected.

30. A 38 year old lady presents with a mass in the midline of the neck

immediately below the hyoid bone. It moves upwards on tongue protrusion.

Thyroglossal cyst

Thyroglossal cysts are usually located in the midline and are linked to the

foramen caecum and will thus move upwards on tongue protrusion.

Neck lumps

The table below gives characteristic exam question features for conditions causing

neck lumps:

Reactive

lymphadenopathy

By far the most common cause of neck swellings. There may

be a history of local infection or a generalised viral illness

Lymphoma Rubbery, painless lymphadenopathy

The phenomenon of pain whilst drinking alcohol is very

uncommon

There may be associated night sweats and splenomegaly

Thyroid swelling May be hypo-, eu- or hyperthyroid symptomatically

Moves upwards on swallowing

Thyroglossal cyst More common in patients < 20 years old

Usually midline, between the isthmus of the thyroid and the

hyoid bone

Moves upwards with protrusion of the tongue

May be painful if infected

Pharyngeal pouch More common in older men

Represents a posteromedial herniation between

thyropharyngeus and cricopharyngeus muscles

Usually not seen, but if large then a midline lump in the neck

that gurgles on palpation

Typical symptoms are dysphagia, regurgitation, aspiration

and chronic cough

Cystic hygroma A congenital lymphatic lesion (lymphangioma) typically

found in the neck, classically on the left side

Most are evident at birth, around 90% present before 2 years

of age

Branchial cyst An oval, mobile cystic mass that develops between the

sternocleidomastoid muscle and the pharynx

Develop due to failure of obliteration of the second branchial

cleft in embryonic development

Usually present in early adulthood

Cervical rib More common in adult females

Around 10% develop thoracic outlet syndrome

Carotid aneurysm Pulsatile lateral neck mass which doesn't move on swallowing

A 22 year old man presents with a discharging area on his lower back. On

examination there is an epithelial defect located 6cm proximal to the tip of his coccyx

and located in the midline. There are two further defects located about 2cm superiorly

in the same position. He is extremely hirsute. What is the most likely diagnosis?

A. Pre sacral tumour

B. Sacrococcygeal teratoma

C. Pilonidal sinus

D. Fistula in ano

E. Occult spina bifida

Pilonidal sinuses are extremely common in hirsute individuals and typically present as

midline sinuses in the natal cleft.

Pilonidal sinus

Occur as a result of hair debris creating sinuses in the skin (Bascom theory).

Usually in the natal cleft of male patients after puberty.

It is more common in Caucasians related to their hair type and growth

patterns.

The opening of the sinus is lined by squamous epithelium, but most of its wall

consists of granulation tissue. Up to 50 cases of squamous cell carcinoma has

been described in patients with chronic pilonidal sinus disease.

Hairs become trapped within the sinus.

Clinically the sinus presents when acute inflammation occurs, leading to an

abscess. Patients may describe cycles of being asymptomatic and periods of

pain and discharge from the sinus.

Treatment is difficult and opinions differ. Definitive treatment should never be

undertaken when acute infection or abscess is present as this will result in

failure.

Definitive treatments include the Bascom procedure with excision of the pits

and obliteration of the underlying cavity. The Karydakis procedure involves

wide excision of the natal cleft such that the surface is recontoured once the

wound is closed. This avoids the shearing forces that break off the hairs and

has reasonable results.

Pilonidal sinuses are most commonly located in the midline of the natal cleft, as

illustrated below

Image sourced from Wikipedia

A 43 year old man from Greece presents with colicky right upper quadrant pain,

jaundice and a pruritic, erythematous rash over his body. He is initially treated with

ciprofloxacin, but does not improve. What is the most likely diagnosis?

A. Infection with Wucheria bancrofti

B. Infection with Echinococcus granulosus

C. Type III hypersentivity reaction

D. Allergy to ciprofloxacin

E. Common bile duct stones

Infection with Echinococcus granulosus will typically produce a type I

hypersensitivity reaction which is characterised by a urticarial rash. With biliary

rupture a classical triad of biliary colic, jaundice and urticaria occurs. Whilst jaundice

and biliary colic may be a feature of CBD stones they do not produce an urticarial

rash. Antibiotic sensitivity with ciprofloxacin may produce jaundice and a rash,

however it was not present at the outset and does not cause biliary colic.

Hydatid cysts

Hydatid cysts are endemic in Mediterranean and Middle Eastern countries. They are

caused by the tapeworm parasite Echinococcus granulosus. An outer fibrous capsule

is formed containing multiple small daughter cysts. These cysts are allergens which

precipitate a type 1 hypersensitivity reaction.

Clinical features are as follows:

Up to 90% cysts occur in the liver and lungs

Can be asymtomatic, or symptomatic if cysts > 5cm in diameter

Morbidity caused by cyst bursting, infection and organ dysfunction (biliary,

bronchial, renal and cerebrospinal fluid outflow obstruction)

In biliary ruputure there may be the classical triad of; biliary colic, jaundice,

and urticaria

CT is the best investigation to differentiate hydatid cysts from amoebic and pyogenic

cysts.

Surgery is the mainstay of treatment (the cyst walls must not be ruptured during

removal and the contents sterilised first).

A 22 year old lady presents with an episode of renal colic and following investigation

is suspected of suffering from MEN IIa. Which of the following abnormalities of the

parathyroid glands are most often found in this condition?

A. Hypertrophy

B. Hyperplasia

C. Adenoma

D. Carcinoma

E. Metaplasia

MEN IIa

Medullary thyroid cancer

Hyperparathyroidism (usually

hyperplasia)

Phaeochromocytoma

In MEN IIa the commonest lesion is medullary thyroid cancer, with regards to the

parathyroid glands the most common lesion is hyperplasia. In MEN I a parathyroid

adenoma is the most common lesion.

Multiple Endocrine Neoplasia

Multiple endocrine neoplasia (MEN) is inherited as an autosomal dominant disorder.

The table below summarises the three main types of MEN:

MEN type I MEN type IIa MEN type IIb

Mnemonic 'three P's':

Parathyroid (95%): Parathyroid adenoma

Pituitary (70%):

Prolactinoma/ACTH/Growth Hormone

secreting adenoma

Pancreas (50%): Islet cell

tumours/Zollinger Ellison syndrome

also: Adrenal (adenoma) and thyroid

(adenoma)

Phaeochromocytoma

Medullary thyroid

cancer (70%)

Hyperparathyroidism

(60%)

Same as MEN IIa

with addition of:

Marfanoid body

habitus

Mucosal neuromas

MENIN gene (chromosome 11)

Most common presentation =

hypercalcaemia

RET oncogene

(chromosome 10)

RET oncogene

(chromosome 10)

A male infant is born prematurely at 34 weeks gestation by emergency cesarean

section. He initially appears to be stable. However, over the ensuing 24 hours he

develops worsening neurological function. Which of the following processes is most

likely to have occurred?

A. Extra dural haemorrhage

B. Sub dural haemorrhage

C. Sub arachnoid haemorrhage

D. Intraventricular haemorrhage

E. Arteriovenous malformation

Theme from April 2012 Exam

Intraventricular haemorrhage

Intraventricular haemorrhage is a haemorrhage that occurs into the ventricular system

of the brain. It is relatively rare in adult surgical practice and when it does occur, it is

typically associated with severe head injuries. In premature neonates it may occur

spontaneously. The blood may clot and occlude CSF flow, hydrocephalus may result.

In neonatal practice the vast majority of IVH occur in the first 72 hours after birth, the

aetiology is not well understood and it is suggested to occur as a result of birth trauma

combined with cellular hypoxia, together the with the delicate neonatal CNS.

Treatment Is largely supportive, therapies such as intraventricular thrombolysis and prophylactic

CSF drainage have been trialled and not demonstrated to show benefit.

Hydrocephalus and rising ICP is an indication for shunting.

A 22 year old man is admitted to hospital with a lower respiratory chest infection. He

had a splenectomy after being involved in a car accident. What is the most likely

infective organism?

A. Haemophilus influenzae

B. Staphylococcus aureus

C. Rhinovirus

D. Mycobacterium tuberculosis

E. Moraxella catarrhalis

Organisms causing post

splenectomy sepsis:

Streptococcus pneumoniae

Haemophilus influenzae

Meningococci

Encapsulated organisms carry the greatest pathogenic risk following splenectomy.

The effects of sepsis following splenectomy are variable. This may be the result of

small isolated fragments of splenic tissue that retain some function following

splenectomy. These may implant spontaneously following splenic rupture (in trauma)

or be surgically implanted at the time of splenectomy.

Post splenectomy sepsis

The loss of splenic function renders individuals at increased risk of fulminant sepsis.

Young children are at the highest risk, especially in the first 2 years following

surgery. Surgery for trauma is associated with a lower risk than when splenectomy is

performed as a treatment for haematological disorders.

Infection with encapsulated organisms poses the greatest risk, these organisms may be

opsonised, but this then goes undetected at an immunological level due to loss of the

spleen.

Prophylactic vaccinations are usually administered to reduce the risk of pneumococcal

septicaemia. Since the vaccine only covers up to 80% of pneumococcal infections,

patients will usually recieve long term, low dose penicillin prophylaxis in addition to

vaccination.

A 24 year old man presents with symptoms of malaise, weight loss and

lymphadenopathy. A lymph node biopsy is performed and the subsequent histology

report states that there is evidence of granuloma formation and central necrosis. What

is the most likely underlying cause?

A. Non Hodgkins lymphoma

B. Churg Strauss syndrome

C. Epstein Barr Virus infection

D. Rheumatoid nodule

E. Infection with Mycobacterium tuberculosis

These histological features are typically seen in TB. Necrosis occurring in granulomas

is usually indicative of an underlying infective cause. Churg Strauss syndrome is a

form of vasculitis, which is the usual histological finding. Granulomas are reported in

the condition, but it is rare for them to demonstrate necrosis.

Tuberculosis pathology

Is a form of primary chronic inflammation, caused by the inability of

macrophages to kill the Mycobacterium tuberculosis.

The macrophages often migrate to regional lymph nodes, the lung lesion plus

affected lymph nodes is referred to as a Ghon complex.

This leads to the formation of a granuloma which is a collection of epithelioid

histiocytes.

There is the presence of caseous necrosis in the centre.

The inflammatory response is mediated by a type 4 hypersensitivity reaction.

In healthy individuals the disease may be contained, in the

immunocompromised disseminated (miliary TB) may occur.

Diagnosis

Waxy membrane of mycobacteria prevents binding with normal stains. Ziehl -

Neelsen staining is typically used.

Culture based methods take far longer.

Image showing acid- alcohol fast mycobacteria stained using the Ziehl- Neelsen

method

Image sourced from Wikipedia

A 20 year old man develops acute appendicitis, his appendix is removed and he

makes a full recovery. Which of the following pathological processes is least likely to

be present in the acutely inflamed tissues?

A. Altered Starlings forces.

B. Seqestration of neurophils

C. Formation of fluid exudate

D. Formation of granulomas

E. None of the above

Neutrophil polymorphs=Acute

inflammation.

Granuloma = Chronic

inflammation.

Acute inflammation:

3 phases:

1. Changes in blood vessel and flow: flush, flare, wheal

2. Fluid exudates (rich in protein i.e. Ig, coagulation factors) produced via increased

vascular permeability

3. Cellular exudates mainly containing neutrophil polymorphs pass into extravascular

space.

Neutrophils are then transported to tissues via:

a. Margination of neutrophils to the peripheral plasmatic of the vessel rather than the

central axial stream

b. Pavementing: Adhesion of neutrophils to endothelial cells in venules at site of

acute inflammation

c. Emigration: neutrophils pass between endothelial cells into the tissue

Acute inflammation

Inflammation is the reaction of the tissue elements to injury. Vascular changes occur,

resulting in the generation of a protein rich exudate. So long as the injury does not

totally destroy the existing tissue architecture, the episode may resolve with

restoration of original tissue architecture.

Vascular changes

Vasodilation occurs and persists throughout the inflammatory phase.

Inflammatory cells exit the circulation at the site of injury.

The equilibrium that balances Starlings forces within capillary beds is

disrupted and a protein rich exudate will form as the vessel walls also become

more permeable to proteins.

The high fibrinogen content of the fluid may form a fibrin clot. This has

several important immunomodulatory functions.

Sequelae

Resolution Typically occurs with minimal initial injury

Stimulus removed and normal tissue architecture

results

Organisation Delayed removed of exudate

Tissues undergo organisation and usually fibrosis

Suppuration Typically formation of an abscess or an empyema

Sequestration of large quantities of dead

neutrophils

Progression to chronic

inflammation

Coupled inflammatory and reparative activities

Usually occurs when initial infection or

suppuration has been inadequately managed

Causes

Microbacterial infections e.g. Viruses, exotoxins or endotoxins released by

bacteria

Chemical agents

Physical agents e.g. Trauma

Hypersensitivity reactions

Tissue necrosis

Presence of neutrophil polymorphs is a histological diagnostic feature of acute

inflammation Theme: Liver lesions

A. Haemangioma

B. Hepatocellular carcinoma

C. Hepatic metastasis

D. Polycystic liver disease

E. Simple liver cyst

F. Hyatid cyst

G. Amoebic abscess

H. Mesenchymal hamartoma

Please select the most likely liver lesion for the scenario given. Each option may be

used once, more than once or not at all.

38. A 42 year old lady has suffered from hepatitis C for many years and has also

developed cirrhosis. On routine follow up, an ultrasound has demonstrated a

2.5cm lesion in the right lobe of the liver.

Hepatocellular carcinoma

In patients with cirrhosis the presence of a lesion >2cm is highly suggestive

of malignancy. The diagnosis is virtually confirmed if the AFP is

>400ng/mL.

39. A 25 year old man from the far east presents with a fever and right upper

quadrant pain. As part of his investigations a CT scan shows an ill defined

lesion in the right lobe of the liver.

You answered Mesenchymal hamartoma

The correct answer is Amoebic abscess

Amoebic abscesses will tend to present in a similar fashion to other pyogenic

liver abscesses. They should be considered in any individual presenting from

a region where Entamoeba histiolytica is endemic. Treatment with

metronidazole usually produces a marked clinical response.

40. A 42 year old lady presents with right upper quadrant pain and a sensation of

abdominal fullness. An ultrasound scan demonstrates a 6.5 cm hyperechoic

lesion in the right lobe of the liver. Serum AFP is normal.

Haemangioma

A large hyperechoic lesion in the presence of normal AFP is likely to be a

haemangioma. An HCC of equivalent size will almost always result in rise in

AFP.

Benign liver lesions

Benign liver lesions

Haemangioma Most common benign tumours of mesenchymal origin

Incidence in autopsy series is 8%

Cavernous haemangiomas may be enormous

Clinically they are reddish purple hypervascular lesions

Lesions are normally separated from normal liver by ring of

fibrous tissue

On ultrasound they are typically hyperechoic

Liver cell

adenoma

90% develop in women in their third to fifth decade

Linked to use of oral contraceptive pill

Lesions are usually solitary

They are usually sharply demarcated from normal liver

although they usually lack a fibrous capsule

On ultrasound the appearances are of mixed echoity and

heterogeneous texture. On CT most lesions are hypodense

when imaged prior to administration of IV contrast agents

In patients with haemorrhage or symptoms removal of the

adenoma may be required

Mesenchymal

hamartomas

Congential and benign, usually present in infants. May compress

normal liver

Liver abscess Biliary sepsis is a major predisposing factor

Structures drained by the portal venous system form the

second largest source

Common symptoms include fever, right upper quadrant

pain. Jaundice may be seen in 50%

Ultrasound will usually show a fluid filled cavity,

hyperechoic walls may be seen in chronic abscesses

Amoebic abscess Liver abscess is the most common extra intestinal

manifestation of amoebiasis

Between 75 and 90% lesions occur in the right lobe

Presenting complaints typically include fever and right

upper quadrant pain

Ultrasonography will usually show a fluid filled structure

with poorly defined boundaries

Aspiration yield sterile odourless fluid which has an

anchovy paste consistency

Treatment is with metronidazole

Hyatid cysts Seen in cases of Echinococcus infection

Typically an intense fibrotic reaction occurs around sites of

infection

The cyst has no epithelial lining

Cysts are commonly unilocular and may grow to 20cm in

size. The cyst wall is thick and has an external laminated

hilar membrane and an internal enucleated germinal layer

Typically presents with malaise and right upper quadrant

pain. Secondary bacterial infection occurs in 10%.

Liver function tests are usually abnormal and eosinophilia is

present in 33% cases

Ultrasound may show septa and hyatid sand or daughter

cysts.

Percutaneous aspiration is contra indicated

Treatment is by sterilisation of the cyst with mebendazole

and may be followed by surgical resection. Hypertonic

swabs are packed around the cysts during surgery

Polycystic liver

disease

Usually occurs in association with polycystic kidney disease

Autosomal dominant disorder

Symptoms may occur as a result of capsular stretch

Cystadenoma Rare lesions with malignant potential

Usually solitary multiloculated lesions

Liver function tests usually normal

Ultrasonography typically shows a large anechoic, fluid

filled area with irregular margins. Internal echos may result

from septa

Surgical resection is indicated in all cases

Which of the following disorders is associated with massive splenomegaly?

A. Acute lymphoblastic leukaemia

B. Acute myeloblastic leukaemia

C. Acute myelomonocytic leukaemia

D. Acute monoblastic leukaemia

E. Chronic granulocytic leukaemia

Chronic leukaemia is more likely to be associated with splenomegaly than acute

leukaemia.

Spleen

Embryology: derived from mesenchymal tissue

Shape: orange segment

Position: below 9th-12th ribs

Weight: 75-150g

Relations

Superiorly- diaphragm

Anteriorly- gastric impression

Posteriorly- kidney

Inferiorly- colon

Hilum: tail of pancreas and splenic vessels (splenic artery divides here,

branches pass to the white pulp transporting plasma)

Forms apex of lesser sac (containing short gastric vessels)

Contents - White pulp: immune function. Contains central trabecular artery. The germinal

centres are supplied by arterioles called penicilliary radicles.

- Red pulp: filters abnormal red blood cells

Function

Filtration of abnormal blood cells and foreign bodies such as bacteria.

Immunity: IgM. Production of properdin, and tuftsin which help target fungi

and bacteria for phagocytosis.

Haematopoiesis: up to 5th month gestation or in haematological disorders.

Pooling: storage of 40% platelets.

Iron reutilisation

Storage red blood cells-animals, not humans.

Storage monocytes

Disorders of the spleen Massive splenomegaly

Myelofibrosis

Chronic myeloid leukaemia

Visceral leishmaniasis (kala-azar)

Malaria

Gaucher's syndrome

Other causes (as above plus)

Portal hypertension e.g. secondary to cirrhosis

Lymphoproliferative disease e.g. CLL, Hodgkin's

Haemolytic anaemia

Infection: hepatitis, glandular fever

Infective endocarditis

Sickle-cell*, thalassaemia

Rheumatoid arthritis (Felty's syndrome)

*the majority of adults patients with sickle-cell will have an atrophied spleen due to

repeated infarction

Causes of primary chronic inflammation do not include which of the following?

A. Sarcoidosis

B. Tuberculosis

C. Ulcerative colitis

D. Prostheses

E. Chronic cholecystitis

Chronic cholecystitis is caused by recurrent episodes of acute inflammation.

Prosthetic implants may be the site of primary chronic inflammation. A common

example clinically is breast implants which may become encapsulated. The

subsequent fibrosis then results in distortion and may be painful.

Chronic inflammation

Overview Chronic inflammation may occur secondary to acute inflammation.In most cases

chronic inflammation occurs as a primary process. These may be broadly viewed as

being one of three main processes:

Persisting infection with certain organisms such as Mycobacterium

tuberculosis which results in delayed type hypersensitivity reactions and

inflammation.

Prolonged exposure to non-biodegradable substances such as silica or suture

materials which may induce an inflammatory response.

Autoimmune conditions involving antibodies formed against host antigens.

Acute vs. Chronic inflammation

Acute inflammation Chronic inflammation

Changes to existing vascular structure and

increased permeability of endothelial cells

Angiogenesis predominates

Infiltration of neutrophils Macrophages, plasma cells and

lymphocytes predominate

Process may resolve with:

Suppuration

Complete resolution

Abscess formation

Progression to chronic inflammation

Healing by fibrosis

Healing by fibrosis is the main

result

Granulomatous inflammation A granuloma consists of a microscopic aggregation of macrophages (with epithelial

type arrangement =epitheliod). Large giant cells may be found at the periphery of

granulomas.

Mediators Growth factors released by activated macrophages include agents such as interferon

and fibroblast growth factor (plus many more). Some of these such as interferons may

have systemic features resulting in systemic symptoms and signs, which may be

present in individuals with long standing chronic inflammation.

The finding of granulomas is pathognomonic of chronic inflammation, as illustrated

in this biopsy from a patient with colonic Crohns disease

Image sourced from Wikipedia

A 30 year old man is trapped in a house fire and sustains 30% partial and full

thickness burns to his torso and limbs. Three days following admission he has a brisk

haematemesis. Which of the following is the most likely explanation for this event?

A. Dieulafoy lesion

B. Curlings ulcers

C. Mallory Weiss tear

D. Depletion of platelets

E. Depletion of clotting factors

Stress ulcers in burns patients are referred to as Curlings ulcers and may cause

haematemesis.

Burns

Types of burn

Type of burn Skin layers

affected

Skin

appearance

Blanching Management

Epidermal/Superficial Epidermis Red, moist Yes

Superficial partial

thickness

Epidermis and

part of papillary

dermis affected

Pale, dry Yes Normally heals

with no

intervention

Deep partial thickness Epidermis, whole

papillary dermis

affected

Mottled red

colour

No Needs surgical

intervention

(depending on

site)

Full thickness Whole skin layer

and subcutaneous

tissue affected

Dry, leathery

hard wound

No Burns centre

Depth of burn assessment

Bleeding on needle prick

Sensation

Appearance

Blanching to pressure

Percentage burn estimation Lund Browder chart: most accurate even in children

Wallace rule of nines

Palmar surface: surface area palm = 0.8% burn

>15% body surface area burns in adults needs urgent burn fluid resuscitation

Transfer to burn centre if:

Need burn shock resuscitation

Face/hands/genitals affected

Deep partial thickness or full thickness burns

Significant electrical/chemical burns

Escharotomies

Indicated in circumferential full thickness burns to the torso or limbs.

Careful division of the encasing band of burn tissue will potentially improve

ventilation (if the burn involves the torso), or relieve compartment syndrome

and oedema (where a limb is involved)

References www.euroburn.org/e107files/downloads/guidelinesburncare.pdf

Hettiaratchy S & Papini R. Initial management of a major burn: assessment and

resuscitation. BMJ 2004;329:101-103

Theme: Adrenal gland disorders

A. Nelsons syndrome

B. Conns syndrome

C. Cushings syndrome

D. Benign incidental adenoma

E. Malignant adrenal adenoma

F. Waterhouse- Friderichsen syndrome

G. Metastatic lesion

H. Walker - Warburg syndrome

I. Phaeochromocytoma

Please select the most appropriate adrenal disorder for the scenario given. Each

disorder may be selected once, more than once or not at all.

44. A 19 year old lady is admitted to ITU with severe meningococcal sepsis. She

is on maximal inotropic support and a CT scan of her chest and abdomen is

performed. The adrenal glands show evidence of diffuse haemorrhage.

Waterhouse- Friderichsen syndrome

This is often a pre-terminal event and is associated with profound sepsis and

coagulopathy.

45. A 34 year old lady is admitted with recurrent episodes of non-specific

abdominal pain. On each admission all blood investigations are normal, as are

her observations. On this admission a CT scan was performed. This

demonstrates a 1.5cm nodule in the right adrenal gland. This is associated

with a lipid rich core. Urinary VMA is within normal limits. Other hormonal

studies are normal.

You answered Nelsons syndrome

The correct answer is Benign incidental adenoma

This is typical for a benign adenoma.Benign adenomas often have a lipid rich

core that is readily identifiable on CT scanning. In addition the nodules are

often well circumscribed.

46. A 38 year old man is noted to have a blood pressure of 175/110 on routine

screening. On examination there are no physical abnormalities of note. CT

scanning shows a left sided adrenal mass. Plasma metanephrines are elevated.

Phaeochromocytoma

Hypertension in a young patient without any obvious cause should be

investigated. Urinary VMA and plasma metanephrines are typically elevated.

Phaeochromocytoma and adrenal lesions

Phaeochromocytoma Neuroendocrine tumour of the chromaffin cells of the adrenal medulla. Hypertension

and hyperglycaemia are often found.

10% of cases are bilateral.

10% occur in children.

11% are malignant (higher when tumour is located outside the adrenal).

10% will not be hypertensive.

Familial cases are usually linked to the Multiple endocrine neoplasia syndromes

(considered under its own heading).

Most tumours are unilateral (often right sided) and smaller than 10cm.

Diagnosis Urine analysis of vanillymandelic acid (VMA) is often used (false positives may

occur e.g. in patients eating vanilla ice cream!)

Blood testing for plasma metanephrine levels.

CT and MRI scanning are both used to localise the lesion.

Treatment Patients require medical therapy first. An irreversible alpha adrenoreceptor blocker

should be given, although minority may prefer reversible bockade(1). Labetolol may

be co-administered for cardiac chronotropic control. Isolated beta blockade should not

be considered as it will lead to unopposed alpha activity.

These patients are often volume depleted and will often require moderate volumes of

intra venous normal saline perioperatively.

Once medically optimised the phaeochromocytoma should be removed. Most

adrenalectomies can now be performed using a laparoscopic approach(2). The

adrenals are highly vascular structures and removal can be complicated by

catastrophic haemorrhage in the hands of the inexperienced. This is particularly true

of right sided resections where the IVC is perilously close. Should the IVC be

damaged a laparotomy will be necessary and the defect enclosed within a Satinsky

style vascular clamp and the defect closed with prolene sutures. Attempting to

interfere with the IVC using any instruments other than vascular clamps will result in

vessel trauma and make a bad situation much worse.

Incidental adrenal lesions Adrenal lesions may be identified on CT scanning performed for other reasons(3).

Factors suggesting benign disease on CT include(4):

Size less than 3cm

Homogeneous texture

Lipid rich tissue

Thin wall to lesion

All patients with incidental lesions should be managed jointly with an endocrinologist

and full work up as described above. Patients with functioning lesions or those with

adverse radiological features (Particularly size >3cm) should proceed to surgery.

References 1. Weingarten TN, Cata JP, O'Hara JF, Prybilla DJ, Pike TL, Thompson GB, et al.

Comparison of two preoperative medical management strategies for laparoscopic

resection of pheochromocytoma. Urology. 2010 Aug;76(2):508 e6-11.

2. Nguyen PH, Keller JE, Novitsky YW, Heniford BT, Kercher KW. Laparoscopic

approach to adrenalectomy: review of perioperative outcomes in a single center. Am

Surg. 2011 May;77(5):592-6.

3. Ng VW, Ma RC, So WY, Choi KC, Kong AP, Cockram CS, et al. Evaluation of

functional and malignant adrenal incidentalomas. Arch Intern Med. 2010 Dec

13;170(22):2017-20.

4. Muth A, Hammarstedt L, Hellstrom M, Sigurjonsdottir HA, Almqvist E, Wangberg

B. Cohort study of patients with adrenal lesions discovered incidentally. Br J Surg.

2011 May 27.

A 15 year old boy is admitted with colicky abdominal pain of 6 hours duration. On

examination he has a soft abdomen, on systemic examination he has brownish spots

around his mouth, feet and hands. His mother underwent surgery for intussusception,

aged 12, and has similar lesions. What is the most likely underlying diagnosis?

A. Li Fraumeni syndrome

B. Peutz-Jeghers syndrome

C. Addisons disease

D. McCune -Albright syndrome

E. Appendicitis

This is most likely to be Peutz-Jeghers syndrome. Addisons and McCune Albright

syndrome may produce similar skin changes but the intussusception resulting from

polyps combined with the autosomal inheritance pattern makes this the most likely

diagnosis.

Peutz-Jeghers syndrome

Peutz-Jeghers syndrome is an autosomal dominant condition characterised by

numerous benign hamartomatous polyps in the gastrointestinal tract. It is also

associated with pigmented freckles on the lips, face, palms and soles. Around 50% of

patients will have died from a gastrointestinal tract cancer by the age of 60 years.

Genetics

Autosomal dominant

Responsible gene encodes serine threonine kinase LKB1 or STK11

Features

Hamartomatous polyps in GI tract (mainly small bowel)

Pigmented lesions on lips, oral mucosa, face, palms and soles

Intestinal obstruction e.g. intussusception (which may lead to diagnosis)

Gastrointestinal bleeding

Management

Conservative unless complications develop

Which of the following is not included in Multiple Endocrine Neoplasia Type 2b?

A. Phaeochromocytoma

B. Visceral ganglioneuromas

C. Thyroid medullary carcinoma

D. Zollinger Ellison syndrome

E. Marfanoid features

MEN IIB

Medullary thyroid cancer

Phaeochromocytoma

Mucosal neuroma

Marfanoid appearance

Multiple Endocrine Neoplasia

Multiple endocrine neoplasia (MEN) is inherited as an autosomal dominant disorder.

The table below summarises the three main types of MEN:

MEN type I MEN type IIa MEN type IIb

Mnemonic 'three P's':

Parathyroid (95%): Parathyroid adenoma

Pituitary (70%):

Prolactinoma/ACTH/Growth Hormone

secreting adenoma

Pancreas (50%): Islet cell

tumours/Zollinger Ellison syndrome

Phaeochromocytoma

Medullary thyroid

cancer (70%)

Hyperparathyroidism

(60%)

Same as MEN IIa

with addition of:

Marfanoid body

habitus

Mucosal neuromas

also: Adrenal (adenoma) and thyroid

(adenoma)

MENIN gene (chromosome 11)

Most common presentation =

hypercalcaemia

RET oncogene

(chromosome 10)

RET oncogene

(chromosome 10)

Which virus is associated with Kaposi's sarcoma?

A. Human herpes virus 8

B. Human papillomavirus 16

C. Human T-lymphotropic virus 1

D. Epstein-Barr virus

E. Human papillomavirus 18

Oncoviruses

Viruses which cause cancer

These may be detected on blood test and prevented by vaccine

These are the main types of oncoviruses and their diseases:

Oncovirus Cancer

Epstein-Barr virus Burkitt's lymphoma

Hodgkin's lymphoma

Post transfusion lymphoma

Nasopharyngeal carcinoma

Human papillomavirus 16/18 Cervical cancer

Anal cancer

Penile cancer

Vulval cancer

Oropharyneal cancer

Human herpes virus 8 Kaposi's sarcoma

Hepatitis B virus Hepatocellular carcinoma

Hepatitis C virus Hepatocellular carcinoma

Human T-lymphotropic virus 1 Tropical spastic paraparesis

Adult T cell leukaemia

Which of the following is not a feature of Wallerian Degeneration?

A. May result from an axonotmesis

B. May occur in either the central or peripheral nervous systems

C. The axon remains excitable throughout the whole process

D. The distal neuronal stump is affected

E. Is a component of the healing process following neuronal injury

The axon loses its excitability once the process is established.

Wallerian degeneration

- Is the process that occurs when a nerve is cut or crushed.

- It occurs when the part of the axon separated from the neuron's cell nucleus

degenerates.

- It usually begins 24 hours following neuronal injury and the distal axon remains

excitable up until this time.

- The degeneration of the axon is following by breakdown of the myelin sheath, a

process that occurs by infiltration of the site with macrophages.

- Eventually regeneration of the nerve may occur although recovery will depend on

the extent and manner of injury

A 45 year old woman complains of painful tingling in her fingers. The pain is relieved

by hanging the arm over the side of the bed. She has a positive Tinel's sign. Which of

the following is most likely to contribute to her diagnosis?

A. Methotrexate use

B. Crohn's disease

C. Hyperthyroidism

D. Tuberculosis

E. Rheumatoid arthritis

This woman has a diagnosis of carpal tunnel syndrome. Rheumatological disorders

are a common cause. Clinical examination should focus on identifying stigmata of

rheumatoid arthritis, such as rheumatoid nodules, vasculitic lesions and

metacarpophalangeal joint arthritis.

Carpal tunnel syndome

Carpal tunnel syndrome is caused by compression of median nerve in the carpal

tunnel

History

pain/pins and needles in thumb, index, middle finger e.g. at night

patient flicks hand to obtain relief

Examination

weakness of thumb abduction

wasting of thenar eminence (NOT hypothenar)

Tinel's sign: tapping causes paraesthesia

Phalen's sign: flexion of wrist causes symptoms

Causes of carpal tunnel syndrome

MEDIAN TRAP Mnemonic

Myxoedema

Edema premenstrually

Diabetes

Idiopathic

Agromegaly

Neoplasm

Trauma

Rheumatoid arthritis

Amyloidosis

Pregnancy

Management

Non surgical

treatment

Surgery

May resolve

spontaneously

Avoid precipitants and

reassurance

Night-time splints

Local steroid

injections

Complete division of the flexor retinaculum and

decompression of the tunnel (successful in approximately

80% of patients)

Which of the following is not an oncogene?

A. ras

B. myc

C. sis

D. Ki 67

E. erb-B

Ki 67 is a nuclear proliferation marker (used in immunohistochemistry). Although,

Ki67 positivity is a marker of malignancy, it is not itself, an oncogene.

Oncogenes

Oncogenes are cancer promoting genes that are derived from normal genes (proto-

oncogenes). Proto-oncogenes play an important physiological role in cellular growth.

They are implicated in the development of up to 20% of human cancers.

Proto-oncogenes may become oncogenes via the following processes:

Mutation (point mutation)

Chromosomal translocation

Increased protein expression

Only one mutated copy of the gene is needed for cancer to occur - a dominant effect

Classification of oncogenes

Growth factors e.g. Sis

Transcription factors e.g. Myc

Receptor tyrosine kinase e.g. RET

Cytoplasmic tyrosine kinase e.g. Src

Regulatory GTPases e.g. Ras

Tumour supressor genes Tumour supressor genes restrict or repress cellular proliferation in normal cells. Their

inactivation through mutation or germ line incorporation is implicated in renal,

colonic, breast, bladder and many other cancers. One of the best known tumour

supressor genes is p53. p53 gene offers protection by causing apoptosis of damaged

cells. Other well known genes include BRCA 1 and 2.

A 30 year old male presents with a painless swelling of the testis. Histologically the

stroma has lymphocytic infiltrate. The most likely diagnosis is :

A. Differentiated teratoma

B. Malignant undifferentiated teratoma

C. Classical seminoma

D. Spermatocytic seminoma

E. Anaplastic seminoma

Seminoma is the commonest type of testicular tumour and is more common in males

aged between 30-40 years. Classical seminoma is the commonest subtype and

histology shows lymphocytic stromal infiltrate. Other subtypes include:

1. Spermatocytic: tumour cells resemble spermatocytes. Excellent prognosis.

2. Anaplastic

3. Syncytiotrophoblast giant cells: beta hCG present in cells

A teratoma is common in males aged 20-30 years.

Testicular disorders

Testicular cancer Testicular cancer is the most common malignancy in men aged 20-30 years. Around

95% of cases of testicular cancer are germ-cell tumours. Germ cell tumours may

essentially be divided into:

Tumour type Key features Tumour

markers

Pathology

Seminoma Commonest

subtype (50%)

Average age at

diagnosis = 40

Even advanced

disease

associated with

5 year survival

of 73%

AFP usually

normal

HCG

elevated in 10%

seminomas

Lactate

dehydrogenase;

elevated in 10-

20% seminomas

(but also in

many other

conditions)

Sheet like

lobular

patterns of

cells with

substantial

fibrous

component.

Fibrous septa

contain

lymphocytic

inclusions and

granulomas

may be seen.

Non seminomatous germ

cell tumours (42%)

Teratoma

Yolk sac tumour

Choriocarcinoma

Mixed germ cell

tumours (10%)

Younger age at

presentation =20-30

years

Advanced disease

carries worse prognosis

(48% at 5 years)

Retroperitoneal

lymph node dissection

may be needed for

residual disease after

chemotherapy

AFP elevated

in up to 70% of

cases

HCG

elevated in up

to 40% of cases

Other

markers rarely

helpful

Heterogenous

texture with

occasional

ectopic tissue

such as hair

Image demonstrating a classical seminoma, these tumours are typically more uniform

than teratomas

Image sourced from Wikipedia

Risk factors for testicular cancer

Cryptorchidism

Infertility

Family history

Klinefelter's syndrome

Mumps orchitis

Features

A painless lump is the most common presenting symptom

Pain may also be present in a minority of men

Other possible features include hydrocele, gynaecomastia

Diagnosis

Ultrasound is first-line

CT scanning of the chest/ abdomen and pelvis is used for staging

Tumour markers (see above) should be measured

Management

Orchidectomy (Inguinal approach)

Chemotherapy and radiotherapy may be given depending on staging

Abdominal lesions >1cm following chemotherapy may require retroperitoneal

lymph node dissection.

Prognosis is generally excellent

5 year survival for seminomas is around 95% if Stage I

5 year survival for teratomas is around 85% if Stage I

Benign disease

Epididymo-orchitis Acute epididymitis is an acute inflammation of the epididymis, often involving the

testis and usually caused by bacterial infection.

Infection spreads from the urethra or bladder. In men <35 years, gonorrhoea or

chlamydia are the usual infections.

Amiodarone is a recognised non infective cause of epididymitis, which

resolves on stopping the drug.

Tenderness is usually confined to the epididymis, which may facilitate

differentiating it from torsion where pain usually affects the entire testis.

Testicular torsion

Twist of the spermatic cord resulting in testicular ischaemia and necrosis.

Most common in males aged between 10 and 30 (peak incidence 13-15 years)

Pain is usually severe and of sudden onset.

Cremasteric reflex is lost and elevation of the testis does not ease the pain.

Treatment is with surgical exploration. If a torted testis is identified then both

testis should be fixed as the condition of bell clapper testis is often bilateral.

A 48 year old women presents with recurrent loin pain and fevers. Investigation

reveals a staghorn calculus of the left kidney. Infection with which of the following

organisms is most likely?

A. Staphylococcus saprophyticus

B. Proteus mirabilis

C. Klebsiella

D. E-Coli

E. Staphylococcus epidermidis

Theme from April 2012 Exam

Infection with Proteus mirabilis accounts for 90% of all proteus infections. It has a

urease producing enzyme. This will tend to favor urinary alkalinisation which is a

relative per-requisite for the formation of staghorn calculi.

Renal stones

Type of

stones

Features Percentage of

all calculi

Calcium

oxalate

Hypercalciuria is a major risk factor (various causes)

Hyperoxaluria may also increase risk

Hypocitraturia increases risk because citrate forms

complexes with calcium making it more soluble

Stones are radio-opaque (though less than calcium

phosphate stones)

Hyperuricosuria may cause uric acid stones to which

calcium oxalate binds

85%

Cystine Inherited recessive disorder of transmembrane cystine

transport leading to decreased absorption of cystine

from intestine and renal tubule

Multiple stones may form

Relatively radiodense because they contain sulphur

1%

Uric acid Uric acid is a product of purine metabolism

May precipitate when urinary pH low

May be caused by diseases with extensive tissue

breakdown e.g. malignancy

More common in children with inborn errors of

metabolism

Radiolucent

5-10%

Calcium

phosphate

May occur in renal tubular acidosis, high urinary pH

increases supersaturation of urine with calcium and

phosphate

10%

Renal tubular acidosis types 1 and 3 increase risk of

stone formation (types 2 and 4 do not)

Radio-opaque stones (composition similar to bone)

Struvite Stones formed from magnesium, ammonium and

phosphate

Occur as a result of urease producing bacteria (and are

thus associated with chronic infections)

Under the alkaline conditions produced, the crystals

can precipitate

Slightly radio-opaque

2-20%

Effect of urinary pH on stone formation Urine pH will show individual variation (from pH 5-7). Post prandially the pH falls as

purine metabolism will produce uric acid. Then the urine becomes more alkaline

(alkaline tide). When the stone is not available for analysis the pH of urine may help

to determine which stone was present.

Stone type Urine acidity Mean urine pH

Calcium phosphate Normal- alkaline >5.5

Calcium oxalate Variable 6

Uric acid Acid 5.5

Struvate Alkaline >7.2

Cystine Normal 6.5

Causes of granulomatous disease do not include:

A. Amiodarone

B. Allopurinol

C. Sulphonamides

D. Beryllium

E. Wegener's granulomatosis

Allopurinol and sulphonamides cause hepatic granulomas.

Chronic inflammation

Overview Chronic inflammation may occur secondary to acute inflammation.In most cases

chronic inflammation occurs as a primary process. These may be broadly viewed as

being one of three main processes:

Persisting infection with certain organisms such as Mycobacterium

tuberculosis which results in delayed type hypersensitivity reactions and

inflammation.

Prolonged exposure to non-biodegradable substances such as silica or suture

materials which may induce an inflammatory response.

Autoimmune conditions involving antibodies formed against host antigens.

Acute vs. Chronic inflammation

Acute inflammation Chronic inflammation

Changes to existing vascular structure and

increased permeability of endothelial cells

Angiogenesis predominates

Infiltration of neutrophils Macrophages, plasma cells and

lymphocytes predominate

Process may resolve with:

Suppuration

Complete resolution

Abscess formation

Progression to chronic inflammation

Healing by fibrosis

Healing by fibrosis is the main

result

Granulomatous inflammation A granuloma consists of a microscopic aggregation of macrophages (with epithelial

type arrangement =epitheliod). Large giant cells may be found at the periphery of

granulomas.

Mediators Growth factors released by activated macrophages include agents such as interferon

and fibroblast growth factor (plus many more). Some of these such as interferons may

have systemic features resulting in systemic symptoms and signs, which may be

present in individuals with long standing chronic inflammation.

The finding of granulomas is pathognomonic of chronic inflammation, as illustrated

in this biopsy from a patient with colonic Crohns disease

Image sourced from Wikipedia

Theme: Facial nerve palsy

A. Adenoid cystic carcinoma

B. Cerebrovascular accident

C. Petrous temporal fracture

D. Warthins tumour

E. Sarcoidosis

F. Pleomorphic adenoma

G. Cholesteatoma

Please select the most likely cause of facial nerve palsy for the scenario given. Each

option may be used once, more than once or not at all.

56. A 22 year old man presents with symptoms of lethargy and bilateral facial

nerve palsy. On examination he has bilateral parotid gland enlargement.

You answered Adenoid cystic carcinoma

The correct answer is Sarcoidosis

Theme from 2011 Exam

Facial nerve palsy is the commonest neurological manifestation of sarcoid. It

usually resolves. The absence of ear discharge or discrete lesion on palpation

is against the other causes.

57. A 21 year old man presents with a unilateral facial nerve palsy after being hit

in the head. On examination he has a right sided facial nerve palsy and a

watery discharge from his nose.

Petrous temporal fracture

Nasal discharge of clear fluid and recent head injury makes a basal skull

fracture the most likely underlying diagnosis.

58. A 43 year old lady presents with symptoms of chronic ear discharge and a

right sided facial nerve palsy. On examination she has foul smelling fluid

draining from her right ear and a complete right sided facial nerve palsy.

Cholesteatoma

Foul smelling ear discharge and facial nerve weakness is likely to be due to

cholesteatoma. The presence of a neurological deficit is a sinister feature.

Facial nerve palsy

Sarcoid Facial nerve palsy is the most frequent neurological manifestation

of sarcoid

Affects right and left side with equal frequency, may be bilateral

Typically resolves in up to 80% of cases

Cholesteatoma Destructive and expanding growth of keratinised squamous

epithelium

Patients often complain of chronic ear discharge

Infection with Pseudomonas may occur resulting in foul smell to

discharge

Aquired lesions usually arise from the Pars flaccida region of the

tympanic membrane

Surgical removal and mastoidectomy may be needed

Recurrence rates of 20% may be seen following surgery

Basal skull

fracture

History of head injury

Presence of features such as Battles sign on examination

Clinical presence of CSF leak strongly supports diagnosis

Assessment is by CT and MRI scan

Prophylactic antibiotics should be given in cases of CSF leak

A 59 year old lady is referred from the NHS breast screening program. A recent

mammogram is reported as showing linear, branching microcalcification with coarse

granules. Which disease process is the most likely underlying cause of these

appearances?

A. Invasive lobular cancer

B. Lobular carcinoma in situ

C. Cribriform type ductal carcinoma in situ

D. Comedo type ductal carcinoma in situ

E. Fibroadenosis

Comedo type DCIS is usually associated with microcalcifications. Cribriform lesions

are usually multifocal but less likely to form microcalcifications. Lobular cancers and

in situ lesions rarely form microcalcifications and are difficult to detect using

mammography.

Breast cancer - In situ disease

Breast cancer that has yet to invade the basement membrane is referred to as in situ

disease. Both ductal and lobular in situ variants are recognised.

Ductal carcinoma in situ

Sub types include; comedo, cribriform, micropapillary and solid

Comdeo DCIS is most likely to form microcalcifications

Cribriform and micropapillary are most likely to be multifocal

Most lesions are mixed (composed of multiple subtypes)

High nuclear grade DCIS is associated with more malignant characteristics

(loss of p53, increase erbB2 expression)

Local excision of low nuclear grade DCIS will usually produce satisfactory

outcomes.

Multifocal lesions, large and high nuclear grade lesions will usually require

mastectomy

Lobular carcinoma in situ

Much rarer than DCIS

Does not form microcalcifications

Usually single growth pattern

When an invasive component is found it is less likely to be associated with

axillary nodal metastasis than with DCIS

Low grade LCIS is usually treated by monitoring rather than excision

In patients with an annular pancreas where is the most likely site of obstruction?

A. The first part of the duodenum

B. The second part of the duodenum

C. The fourth part of the duodenum

D. The third part of the duodenum

E. The duodeno-jejunal flexure

Theme from 2011 Exam

Theme from September 2012 Exam

The pancreas develops from two foregut outgrowths (ventral and dorsal). During rotation

the ventral bud and adjacent gallbladder and bile duct lie together and fuse. When the

pancreas fails to rotate normally it can compress the duodenum with development of

obstruction. Usually occurring as a result of associated duodenal malformation. The second

part of the duodenum is the commonest site.

Pancreas

The pancreas is a retroperitoneal organ and lies posterior to the stomach. It may be

accessed surgically by dividing the peritoneal reflection that connects the greater omentum

to the transverse colon. The pancreatic head sits in the curvature of the duodenum. It's tail

lies close to the hilum of the spleen, a site of potential injury during splenectomy.

Relations

Posterior to the pancreas

Pancreatic head Inferior vena cava

Common bile duct

Right and left renal veins

Superior mesenteric vein and artery

Pancreatic neck Superior mesenteric vein, portal vein

Pancreatic body- Left renal vein

Crus of diaphragm

Psoas muscle

Adrenal gland

Kidney

Aorta

Pancreatic tail Left kidney

Anterior to the pancreas

Pancreatic head 1st part of the duodenum

Pylorus

Gastroduodenal artery

SMA and SMV(uncinate process)

Pancreatic body Stomach

Duodenojejunal flexure

Pancreatic tail Splenic hilum

Superior to the pancreas

Coeliac trunk and its branches common hepatic artery and splenic artery

Grooves of the head of the pancreas

2nd and 3rd part of the duodenum

Arterial supply

Head: pancreaticoduodenal artery Rest: splenic artery

Venous drainage

Head: superior mesenteric vein Body and tail: splenic vein

Ampulla of Vater

Merge of pancreatic duct and common bile duct Is an important landmark, halfway along the second part of the duodenum, that

marks the anatomical transition from foregut to midgut (also the site of transition between regions supplied by coeliac trunk and SMA).

Image sourced from Wikipedia

Rate question:

1

2

3

4

5

Theme: Chest pain

A. Achalasia

B. Pulmonary embolus

C. Dissection of thoracic aorta

D. Boerhaaves syndrome

E. Gastro-oesophageal reflux

F. Carcinoma of the oesophagus

G. Oesophageal candidiasis

Please select the most likely cause for chest pain for the scenario given. Each option

may be used once, more than once or not at all.

3. A 43 year old man who has a long term history of alcohol misuse is admitted

with a history of an attack of vomiting after an episode of binge drinking. After

vomiting he developed sudden onset left sided chest pain, which is pleuritic in

nature. On examination he is profoundly septic and drowsy with severe

epigastric tenderness and left sided chest pain.

Boerhaaves syndrome

In patients with Boerhaaves the rupture is often on the left side. The story here

is typical. All patients should have a contrast study to confirm the diagnosis

and the affected site prior to thoracotomy.

4. A 22 year old man is admitted with severe retrosternal chest pain and recurrent

episodes of dysphagia. These occur sporadically and often resolve

spontaneously. On examination there are no physical abnormalities and the

patient seems well.

You answered Dissection of thoracic aorta

The correct answer is Achalasia

Achalasia may produce severe chest pain and many older patients may undergo

cardiac investigations prior to endoscopy.

Endoscopic injection with botulinum toxin is a popular treatment (although the

benefit is not long lasting). Cardiomyotomy is a more durable alternative.

5. An obese 53 year old man presents with symptoms of recurrent retrosternal

discomfort and dyspepsia. This is typically worse at night after eating a large

meal. On examination there is no physical abnormality to find.

Gastro-oesophageal reflux

Patients with GORD often have symptoms that are worse at night. In this age

group an Upper GI endoscopy should probably be performed.

Surgical chest pain

Dissection of

thoracic aorta

Tearing interscapular pain

Discrepancy in arterial blood pressures taken in both arms

May show mediastinal widening on chest x-ray

Diffuse

oesophageal

spasm

Spectrum of oesophageal motility disorders

Caused by uncoordinated contractions of oesphageal muscles

May show "nutcracker oesophagus" on barium swallow

Symptoms include dysphagia, retrosternal discomfort and

dyspepsia

Gastro-

oesphageal

reflux

Common cause of retrosternal discomfort

Usually associated with symptoms of regurgitation,

odynophagia and dyspepsia

Symptoms usually well controlled with PPI therapy

Risk factors include obesity, smoking and excess alcohol

consumption

Boerhaaves

syndrome

Spontaneous rupture of the oesophagus

Caused by episodes of repeated vomiting often in association

with alcohol excess

Typically there is an episode of repetitive vomiting followed

by severe chest and epigastric pain

Diagnosis is by CT and contrast studies

Treatment is surgical; during first 12 hours primary repair,

beyond this usually creation of controlled fistula with a T

Tube, delay beyond 24 hours is associated with fulminent

mediastinitis and is usually fatal.

Achalasia Difficulty swallowing, dysphagia to both liquids and solids

and sometimes chest pain

Usually caused by failure of distal oesphageal inhibitory

neurones

Diagnosis is by pH and manometry studies together with

contrast swallow and endoscopy

Treatment is with either botulinum toxin, pneumatic

dilatation or cardiomyotomy

A 32 year old woman presents with an episode of haemoptysis and is found to have

metastatic tumour present within the parenchyma of the lungs. This is biopsied and

subsequent histology shows clear cells. What is the most likely primary site?

A. Kidney

B. Breast

C. Liver

D. Adrenal

E. Bone

Clear cell tumours are a sub type of renal cell cancer it is associated with specific

genetic changes localised to chromosome 3.

Renal lesions

Lesion Disease specific features Treatment

Renal cell

carcinoma

Most present with

haematuria (50%)

Common renal tumour (85%

cases)

Paraneoplastic features

include hypertension and

polycythaemia

Most commonly has

haematogenous mestastasis

Usually radical or partial

nephrectomy

Nephroblastoma Rare childhood tumour

It accounts for 80% of all

genitourinary malignancies

in those under the age of 15

years

Up to 90% will have a mass

50% will be hypertensive

Diagnostic work up includes

ultrasound and CT scanning

Surgical resection combined

with chemotherapy (usually

vincristine, actinomycin D

and doxorubicin

Neuroblastoma Most common extracranial

tumour of childhood

80% occur in those under 4

years of age

Tumour of neural crest

origin (up to 50% occur in

the adrenal gland)

The tumour is usually

calcified and may be

diagnosed using MIBG

scanning

Staging is with CT

Surgical resection,

radiotherapy and

chemotherapy

Transitional cell

carcinoma

Accounts for 90% of upper

urinary tract tumour, but

only 10% of renal tumours

Males affected 3x more than

females

Occupational exposure to

industrial dyes and rubber

chemicals may increase risk

Up to 80% present with

Radical nephroureterectomy

painless haematuria

Diagnosis and staging is with

CT IVU

Angiomyolipoma 80% of these hamartoma

type lesions occur

sporadically, the remainder

are seen in those with

tuberous sclerosis

Tumour is composed of

blood vessels, smooth

muscle and fat

Massive bleeding may occur

in 10% of cases

50% of patients with lesions

>4cm will have symptoms

and will require surgical

resection

A laceration of the wrist produces a median nerve transaction. The wound is clean and

seen immediately after injury. Collateral soft tissue damage is absent. The patient asks

what the prognosis is. You indicate that the nerve should regrow at approximately:

A. 0.1 mm per day

B. 1 mm per day

C. 5 mm per day

D. 1 cm per day

E. None of the above

Transaction of a peripheral nerve results in hemorrhage and in retraction of the

several nerve ends. Almost immediately, degeneration of the axon distal to the injury

begins. Degeneration also occurs in the proximal fragment back to the first node of

Ranvier. Phagocytosis of the degenerated axonal fragments leaves neurilemmal sheath

with empty cylindrical spaces where the axons were. Several days following the

injury, axons from the proximal fragment begin to regrow. If they make contact with

the distal neurilemmal sheath, regrowth occurs at about the rate of 1 mm/day.

However, if associated trauma, fracture, infection, or separation of neurilemmal

sheath ends precludes contact between axons, growth is haphazard and a traumatic

neuroma is formed. When neural transaction is associated with widespread soft tissue

damage and hemorrhage (with increased probability of infection), many surgeons

choose to delay reapproximation of the severed nerve end for 3 to 4 weeks.

Nerve injury

There are 3 types of nerve injury:

Neuropraxia Nerve intact but electrical conduction is affected

Full recovery

Autonomic function preserved

Wallerian degeneration does not occur

Axonotmesis Axon is damaged and the myelin sheath is preserved. The

connective tissue framework is not affected.

Wallerian degeneration occurs.

Neurotmesis Disruption of the axon, myelin sheath and surrounding

connective tissue.

Wallerian degeneration occurs.

Wallerian Degeneration

Axonal degeneration distal to the site of injury.

Typically begins 24-36 hours following injury.

Axons are excitable prior to degeneration occurring.

Myelin sheath degenerates and is phagocytosed by tissue macrophages.

Nerve repair

Neuronal repair may only occur physiologically where nerves are in direct

contact. Where a large defect is present the process of nerve regeneration is

hampered and may not occur at all or result in the formation of a neuroma.

Where nerve regrowth occurs it typically occurs at a rate of 1mm per day.

Which of the following statements relating to gastric cancer is untrue?

A. It is associated with chronic helicobacter pylori infection

B. 5% of gastric malignancies are due to lymphoma

C. In the Lauren classification the diffuse type of adenocarcinoma typically

presents as a large exophytic growth in the antrum

D. Smoking is a risk factor

E. It is associated with acanthosis nigricans

The Lauren classification describes a diffuse type of adenocarcinoma (Linitis plastica type

lesion) and an intestinal type. The diffuse type is often deeply infiltrative and may be difficult

to detect on endoscopy. Barium meal appearances can be characteristic.

Gastric cancer

Overview

There are 700,000 new cases of gastric cancer worldwide each year. It is most common in

Japan and less common in western countries. It is more common in men and incidence rises

with increasing age. The exact cause of many sporadic cancer is not known, however,

familial cases do occur in HNPCC families. In addition, smoking and smoked or preserved

foods increase the risk. Japanese migrants retain their increased risk (decreased in

subsequent generations). The distribution of the disease in western countries is changing

towards a more proximal location (perhaps due to rising obesity).

Pathology

There is some evidence of support a stepwise progression of the disease through intestinal

metaplasia progressing to atrophic gastritis and subsequent dysplasia, through to cancer.

The favoured staging system is TNM. The risk of lymph node involvement is related to size

and depth of invasion; early cancers confined to submucosa have a 20% incidence of lymph

node metastasis. Tumours of the gastro-oesophageal junction are classified as below:

Type

1

True oesophageal cancers and may be associated with Barrett's oesophagus.

Type

2

Carcinoma of the cardia, arising from cardiac type epithelium

or short segments with intestinal metaplasia at the oesophagogastric junction.

Type

3

Sub cardial cancers that spread across the junction. Involve similar nodal stations to

gastric cancer.

Groups for close endoscopic monitoring

Intestinal metaplasia of columnar type Atrophic gastritis Low to medium grade dysplasia Patients who have previously undergone resections for benign peptic ulcer disease

(except highly selective vagotomy).

Referral to endoscopy

Patients of any age with

dyspepsia and any of the

following

Patients without

dyspepsia

Worsening dyspepsia

Chronic gastrointestinal bleeding Dysphagia Barretts oesophagus

Dysphagia Unexplained abdominal

pain or weight loss

Intestinal metaplasia

Weight loss Vomiting Dysplasia

Iron deficiency anaemia Upper abdominal mass Atrophic gastritis

Upper abdominal mass Jaundice Patient aged over 55 years with

unexplained or persistent

dyspepsia

Upper GI endoscopy performed for dyspepsia. The addition of dye spraying (as shown in the

bottom right) may facilitate identification of smaller tumours

Image sourced from Wikipedia

Staging

CT scanning of the chest abdomen and pelvis is the routine first line staging investigation in most centres.

Laparoscopy to identify occult peritoneal disease PET CT (particularly for junctional tumours)

Treatment

Proximally sited disease greater than 5-10cm from the OG junction may be treated by sub total gastrectomy

Total gastrectomy if tumour is <5cm from OG junction For type 2 junctional tumours (extending into oesophagus) oesophagogastrectomy is

usual Endoscopic sub mucosal resection may play a role in early gastric cancer confined to

the mucosa and perhaps the sub mucosa (this is debated) Lymphadenectomy should be performed. A D2 lymphadenectomy is widely

advocated by the Japanese, the survival advantages of extended lymphadenectomy have been debated. However, the overall recommendation is that a D2 nodal dissection be undertaken.

Most patients will receive chemotherapy either pre or post operatively.

Prognosis

UK Data

Disease extent Percentage 5 year survival

All RO resections 54%

Early gastric cancer 91%

Stage 1 87%

Stage 2 65%

Stage 3 18%

Operative procedure

Total Gastrectomy , lymphadenectomy and Roux en Y anastomosis

General anaesthesia

Prophylactic intravenous antibiotics

Incision: Rooftop.

Perform a thorough laparotomy to identify any occult disease.

Mobilise the left lobe of the liver off the diaphragm and place a large pack over it. Insert a

large self retaining retractor e.g. omnitract or Balfour (take time with this, the set up should

be perfect). Pack the small bowel away.

Begin by mobilising the omentum off the transverse colon.

Proceed to detach the short gastric vessels.

Mobilise the pylorus and divide it at least 2cm distally using a linear cutter stapling device.

Continue the dissection into the lesser sac taking the lesser omentum and left gastric artery

flush at its origin.

The lymph nodes should be removed en bloc with the specimen where possible.

Place 2 stay sutures either side of the distal oesophagus. Ask the anaesthetist to pull back on

the nasogastric tube. Divide the distal oesophagus and remove the stomach.

The oesphago jejunal anastomosis should be constructed. Identify the DJ flexure and bring a

loop of jejunum up to the oesophagus (to check it will reach). Divide the jejunum at this

point. Bring the divided jejunum either retrocolic or antecolic to the oesophagus.

Anastamose the oesophagus to the jejunum, using either interrupted 3/0 vicryl or a stapling

device. Then create the remainder of the Roux en Y reconstruction distally.

Place a jejunostomy feeding tube.

Wash out the abdomen and insert drains (usually the anastomosis and duodenal stump).

Help the anaesthetist insert the nasogastric tube (carefully!)

Close the abdomen and skin.

Enteral feeding may commence on the first post-operative day. However, most surgeons will

leave patients on free NG drainage for several days and keep them nil by mouth.

Which of the following statements relating to Gardners syndrome variant of familial

adenomatous polyposis coli is false?

A. It is an autosomal dominant condition

B. Patients may develop retroperitoneal desmoid tumours

C. The vast majority of the polyps are benign and thus the risk of

colorectal cancer is small

D. Patients are at increased risk of thyroid cancer

E. It is characterised by a mutation in the APC gene

The multiple polyps increase the risk of malignancy and most patients should undergo

a colectomy.

Genetics and surgical disease

Some of the more commonly occurring genetic conditions occurring in surgical

patients are presented here.

Li-Fraumeni Syndrome

Autosomal dominant

Consists of germline mutations to p53 tumour suppressor gene

High incidence of malignancies particularly sarcomas and leukaemias

Diagnosed when:

*Individual develops sarcoma under 45 years

*First degree relative diagnosed with any cancer below age 45 years and another

family member develops malignancy under 45 years or sarcoma at any age

BRCA 1 and 2

Carried on chromosome 17

Linked to developing breast cancer (60%) risk.

Associated risk of developing ovarian cancer (55% with BRCA 1 and 25%

with BRCA2).

Lynch Syndrome

Autosomal dominant

Develop colonic cancer and endometrial cancer at young age

80% of affected individuals with get colonic and or endometrial cancer

High risk individuals may be identified using the Amsterdam criteria

Amsterdam criteria Three or more family members with a confirmed diagnosis of colorectal cancer, one

of whom is a first degree (parent, child, sibling) relative of the other two.

Two successive affected generations.

One or more colon cancers diagnosed under age 50 years.

Familial adenomatous polyposis (FAP) has been excluded.

Gardners syndrome

Autosomal dominant familial colorectal polyposis

Multiple colonic polyps

Extra colonic diseases include: skull osteoma, thyroid cancer and epidermoid

cysts

Desmoid tumours are seen in 15%

Mutation of APC gene located on chromosome 5

Due to colonic polyps most patients will undergo colectomy to reduce risk of

colorectal cancer

Now considered a variant of familial adenomatous polyposis coli

Which of the following statements relating to Gardners syndrome variant of familial

adenomatous polyposis coli is false?

A. It is an autosomal dominant condition

B. Patients may develop retroperitoneal desmoid tumours

C. The vast majority of the polyps are benign and thus the risk of

colorectal cancer is small

D. Patients are at increased risk of thyroid cancer

E. It is characterised by a mutation in the APC gene

The multiple polyps increase the risk of malignancy and most patients should undergo

a colectomy.

Genetics and surgical disease

Some of the more commonly occurring genetic conditions occurring in surgical

patients are presented here.

Li-Fraumeni Syndrome

Autosomal dominant

Consists of germline mutations to p53 tumour suppressor gene

High incidence of malignancies particularly sarcomas and leukaemias

Diagnosed when:

*Individual develops sarcoma under 45 years

*First degree relative diagnosed with any cancer below age 45 years and another

family member develops malignancy under 45 years or sarcoma at any age

BRCA 1 and 2

Carried on chromosome 17

Linked to developing breast cancer (60%) risk.

Associated risk of developing ovarian cancer (55% with BRCA 1 and 25%

with BRCA2).

Lynch Syndrome

Autosomal dominant

Develop colonic cancer and endometrial cancer at young age

80% of affected individuals with get colonic and or endometrial cancer

High risk individuals may be identified using the Amsterdam criteria

Amsterdam criteria Three or more family members with a confirmed diagnosis of colorectal cancer, one

of whom is a first degree (parent, child, sibling) relative of the other two.

Two successive affected generations.

One or more colon cancers diagnosed under age 50 years.

Familial adenomatous polyposis (FAP) has been excluded.

Gardners syndrome

Autosomal dominant familial colorectal polyposis

Multiple colonic polyps

Extra colonic diseases include: skull osteoma, thyroid cancer and epidermoid

cysts

Desmoid tumours are seen in 15%

Mutation of APC gene located on chromosome 5

Due to colonic polyps most patients will undergo colectomy to reduce risk of

colorectal cancer

Now considered a variant of familial adenomatous polyposis coli

A 60-year-old man presents with lower urinary tract symptoms and is offered a PSA

test. Which one of the following could interfere with the PSA level?

A. Vigorous exercise in the past 48 hours

B. Poorly controlled diabetes mellitus

C. Drinking more than 4 units of alcohol in the past 48 hours

D. Smoking

E. Recent cholecystectomy

PSA testing

Prostate specific antigen (PSA) is a serine protease enzyme produced by normal and

malignant prostate epithelial cells. It has become an important tumour marker but

much controversy still exists regarding its usefulness as a screening tool.

The NHS Prostate Cancer Risk Management Programme (PCRMP) has published

updated guidelines in 2009 on how to handle requests for PSA testing in

asymptomatic men. A recent European trial (ERSPC) showed a statistically

significant reduction in the rate of death prostate cancer by 20% in men aged 55 to 69

years but this was associated with a high risk of over-diagnosis and over-treatment.

Having reviewed this and other data the National Screening Committee have decided

not to introduce a prostate cancer screening programme yet but rather allow men to

make an informed choice.

Age-adjusted upper limits for PSA were recommended by the PCRMP*:

{Age} {PSA level (ng/ml)}

50-59 years 3.0

60-69 years 4.0

> 70 years 5.0

PSA levels may also be raised by**:

benign prostatic hyperplasia (BPH)

prostatitis and urinary tract infection (NICE recommend to postpone the PSA

test for at least 1 month after treatment)

ejaculation (ideally not in the previous 48 hours)

vigorous exercise (ideally not in the previous 48 hours)

urinary retention

instrumentation of the urinary tract

Poor specificity and sensitivity

around 33% of men with a PSA of 4-10 ng/ml will be found to have prostate

cancer. With a PSA of 10-20 ng/ml this rises to 60% of men

around 20% with prostate cancer have a normal PSA

various methods are used to try and add greater meaning to a PSA level

including age-adjusted upper limits and monitoring change in PSA level with

time (PSA velocity or PSA doubling time)

*aide memoire for upper PSA limit: (age - 20) / 10

**whether digital rectal examination actually causes a rise in PSA levels is a matter of

debate

Which of the symptoms below is least typical of pancreatic cancer?

A. Painless jaundice

B. Hyperamylasaemia

C. Hyperglycaemia

D. Weight loss

E. Classical Courvoisier syndrome

Raised serum amylase is relatively uncommon. The typical Courvoisier syndrome

typically occurs in 20% and hyperglycaemia occurs in 15-20%.

Pancreatic cancer

Adenocarcinoma

Risk factors: Smoking, diabetes, Adenoma, Familial adenomatous polyposis

Mainly occur in the head of the pancreas (70%)

Spread locally and metastasizes to the liver

Carcinoma of the pancreas should be differentiated from other periampullary

tumours with better prognosis

Clinical features

Weight loss

Painless jaundice

Epigastric discomfort (pain usually due to invasion of the coeliac plexus is a

late feature)

Pancreatitis

Trousseau's sign: migratory superficial thrombophlebitis

Investigations

USS: May miss small lesions

CT Scanning (pancreatic protocol). If unresectable on CT then no further

staging needed.

PET/CT for those with operable disease on CT alone

ERCP/ MRI for bile duct assessment.

Staging laparoscopy to exclude peritoneal disease.

Management

Head of pancreas: Whipple's resection (SE dumping and ulcers). Newer

techniques include pylorus preservation and SMA/ SMV resection.

Carcinoma body and tail: poor prognosis, distal pancreatectomy if operable.

Usually adjuvent chemotherapy for resectable disease

ERCP and stent for jaundice and palliation.

Surgical bypass may be needed for duodenal obstruction.

A 53 year old man presents with dyspepsia. An upper GI endoscopy is performed and

Helicobacter pylori is identified. A duodenal ulcer is present in the first part of the

duodenum. Duodenal biopsies are taken and demonstrate epithelium that resembles

cells of the gastric antrum. Which of the following is the most likely explanation for

this process?

A. Hyperplasia of the crypts of Lieberkhun

B. Duodenal metaplasia

C. Duodenal dysplasia

D. Duodenal carcinoma

E. Hyptertrophy of Brunners glands

Metaplasia = cell type

conversion

Theme in January 2012 exam

The process involved is metaplasia. During metaplasia there is no direct

carcinogenesis, however the persistent presence of precipitants of metaplasia will lead

to malignant changes in cells.

Metaplastic changes in the duodenal cap are frequently seen in association with H-

Pylori induced ulcers. It typically resolves after ulcer healing and eradication therapy.

Metaplasia

Definition: reversible change of differentiated cells to another cell type.

May represent an adaptive substitution of cells that are sensitive to stress by

cell types better able to withstand the adverse environment.

Can be a normal physiological response (ossification of cartilage to form

bone)

Most common epithelial metaplasia occurs with transformation of columnar

cells to squamous cells (smoking causes ciliated columnar cells to be replaced

by squamous epithelial cells; Schistosomiasis).

Metaplasia from squamous to columnar cells occurs in Barrett oesophagus.

If the metaplastic stimulus is removed, the cells will return to their original

pattern of differentiation. However, if the stimulus is not removed then

progression to dysplasia may occur.

Not considered directly carcinogenic, however the factors which predispose to

metaplasia, if persistent may induce malignant transformation.

The pathogenesis involves a reprogramming of stem cells that are known to

exist in normal tissues, or of undifferentiated mesenchymal cells present in

connective tissue. In a metaplastic change, these precursor cells differentiate

along a new pathway.

A 22 year old lady presents with symptoms and signs of hyperthyroidism. Her

diagnostic work up results in a diagnosis of Graves disease. Which of the following

best describes the pathophysiology of the condition?

A. Formation of IgG antibodies to the TSH receptors on the thyroid

gland

B. Formation of IgG antibodies to the TRH receptors on the anterior

pituitary

C. Formation of IgM antibodies to the TSH receptors on the thyroid

gland

D. Formation of IgA antibodies to the TSH receptors on the thyroid

gland

E. Formation of IgM antibodies to the TRH receptors on the anterior

pituitary

Usually IgG antibodies are formed against the TSH receptors on the thyroid gland.

Which is why the TSH level is often very low in Graves disease.

Thyroid hormones

Hormones of the thyroid gland

Triiodothyronine T3 Major hormone active in target cells

Thyroxine T4 Most prevalent form in plasma, less biologically active than T3

Calcitonin Lowers plasma calcium

Synthesis and secretion of thyroid hormones

Thyroid actively concentrates iodide to twenty five times the plasma

concentration.

Iodide is oxidised by peroxidase in the follicular cells to atomic iodine which

then iodinates tyrosine residues contained in thyroglobulin.

Iodinated tyrosine residues in thyroglobulin undergo coupling to either T3 or

T4.

Process is stimulated by TSH, which stimulates secretion of thyroid hormones.

The normal thyroid has approximately 3 month reserves of thyroid hormones.

LATS and Graves disease In Graves disease patients develop IgG antibodies to the TSH receptors on the thyroid

gland. This results in chronic and long term stimulation of the gland with release of

thyroid hormones. The typically situation is raised thyroid hormones and low TSH.

Thyroid receptor autoantibodies should be checked in individuals presenting with

hyperthyroidism as they are present in up to 85% cases.

Theme: Breast disease

A. Tuberculosis

B. Actinomycosis

C. Duct ectasia

D. Fibroadenoma

E. Fat necrosis

F. Intraductal papilloma

G. Breast abscess

What is the most likely diagnosis for each scenario given? Each diagnosis may be

used once, more than once or not at all.

14. A 32 year old Indian woman presents with a tender breast lump. She has a 2

month old child. Clinically there is a tender, fluctuant mass of the breast.

Breast abscess

Similar theme in September 2011 Exam

This lady is likely to be breast feeding and is at risk of mastitis. This may lead

to an abscess if not treated. Staphylococcus aureus is usually the causative

organism.

15. A 53 year old lady presents with a green nipple discharge. On examination

she has discharge originating from multiple ducts and associated nipple

inversion.

Duct ectasia

Duct ectasia is common during the period of breast involution that occurs

during the menopausal period. As the ducts shorten they may contain

insipiated material. The discharge will often discharge from several ducts.

16. A 52 year old lady presents with an episode of nipple discharge. It is usually

clear in nature. On examination the discharge is seen to originate from a

single duct and although it appears clear, when the discharge is tested with a

labstix it is shown to contain blood. Imaging and examination shows no

obvious mass lesion,

Intraductal papilloma

Intraductal papilloma usually cause single duct discharge. The fluid is often

clear, although it may be blood stained. If the fluid is tested with a labstix

(little point in routiine practice) then it will usually contain small amounts of

blood. A microdocechtomy may be performed.

Non malignant breast disease

Duct ectasia

Mammary duct ectasia may be seen in up to 25% of normal female breasts

Patients usually present with nipple discharge, which may be from single or

multiple ducts (usually present age >50 years)

The discharge is often thick and green

Duct ectasia is a normal varient of breast involution and is not the same

condition as periductal mastitis

Periductal mastitis

Present at younger age than duct ectasia

May present with features of inflammation, abscess or mammary duct fistula

Stongly associated with smoking

Usually treated with antibiotics, abscess will require drainage

Intraductal papilloma

Growth of papilloma in a single duct

Usually presents with clear or blood stained discharge originating from a

single duct

No increase in risk of malignancy

Breast abscess

Lactational mastitis is common

Infection is usually with Staphylococcus aureus

On examination there is usually a tender fluctuant mass

Treatment is with antibiotics and ultrasound guided aspiration

Overlying skin necrosis is an indication for surgical debridement, which may

be complicated by the development of a subsequent mammary duct fistula.

Tuberculosis

Rare in western countries, usually secondary TB

Affects women later in child bearing period

Chronic breast or axillary sinus is present in up to 50% cases

Diagnosis is by biopsy culture and histology

A 45 year old man has widespread metastatic adenocarcinoma of the colon. Which of

the following tumour markers is most likely to be elevated?

A. CA19-9

B. Carcinoembryonic antigen

C. Alpha Feto Protein

D. CA 125

E. Beta BCG

Screening for colonic cancer

using CEA is not justified

Carcinoembryonic antigen is elevated in colonic cancer, typically in relation to

disease extent with highest serum levels noted in metastatic disease. It is falsely

elevated in a number of non-malignant disease states such as cirrhosis and colitis and

for this reason it has no role in monitoring colitics for colonic cancer[1].

Reference

1. Sturgeon, C.M., L.C. Lai, and M.J. Duffy, Serum tumour markers: how to order

and interpret them. BMJ, 2009. 339: p. b3527.

Colorectal cancer screening and diagnosis

Overview

Most cancers develop from adenomatous polyps. Screening for colorectal

cancer has been shown to reduce mortality by 16%

The NHS now has a national screening programme offering screening every 2

years to all men and women aged 60 to 69 years. Patients aged over 70 years

may request screening

Eligible patients are sent faecal occult blood (FOB) tests through the post

Patients with abnormal results are offered a colonoscopy

At colonoscopy, approximately:

5 out of 10 patients will have a normal exam

4 out of 10 patients will be found to have polyps which may be removed due

to their premalignant potential

1 out of 10 patients will be found to have cancer

Diagnosis

Essentially the following patients need referral:

- Altered bowel habit for more than six weeks

- New onset of rectal bleeding

- Symptoms of tenesmus

Colonoscopy is the gold standard, provided it is complete and good mucosal

visualisation is achieved. Other options include double contrast barium enema and CT

colonography.

Staging

Once a malignant diagnosis is made patients with colonic cancer will be staged using

chest / abdomen and pelvic CT. Patients with rectal cancer will also undergo

evaluation of the mesorectum with pelvic MRI scanning.

For examination purposes the Dukes and TNM systems are preferred.

Tumour markers Carcinoembryonic antigen (CEA) is the main tumour marker in colorectal cancer. Not

all tumours secrete this, and it may be raised in conditions such as IBD. However,

absolute levels do correlate (roughly) with disease burden and whilst this marker may

not be used extensively in follow up, it can be useful for investigation of patients with

cancer of unknown primary.

Which of the following does not cause hypercalcaemia?

A. Thiazides

B. DiGeorge syndrome

C. Vitamin A

D. Rhabdomylosis

E. Sarcoidosis

VITAMINS TRAP

V - Vitamins A & D

I - Immobilization

T - Thyrotoxicosis

A - Addison's disease

M - Milk-alkali syndrome

I - Inflammatory disorders

N - Neoplastic diseases

S - Sarcoidosis

T - Thiazides and other drugs

R - Rhabdomyolysis

A - AIDS

P - Paget's disease, Parenteral

nutrition, Parathyroid disease.

The parathyroid glands don't form as a result of this syndrome.

Hypercalcaemia

Main causes

Malignancy

Primary hyperparathyroidism

Less common

Sarcoidosis (extrarenal synthesis of calcitriol )

Thiazides, lithium

Immobilisation

Pagets disease

Vitamin A/D toxicity

Thyrotoxicosis

MEN

Milk alkali syndrome

Clinical features

“Stones, bones, abdominal moans, and psychic groans”

Theme: Diseases affecting the great vessels

A. Aortic coarctation

B. Cervical rib

C. Takayasu's arteritis

D. Subclavian steal syndrome

E. Patent ductus arteriosus

F. Aortic dissection

Please select the most likely underlying cause for the symptoms described. Each

option may be used once, more than once or not at all.

19. A 24 year old lady from Western India presents with symptoms of lethargy

and dizziness, worse on turning her head. On examination her systolic blood

pressure is 176/128. Her pulses are impalpable at all peripheral sites.

Auscultation of her chest reveals a systolic heart murmur.

You answered Aortic coarctation

The correct answer is Takayasu's arteritis

Takayasu's arteritis most commonly affects young Asian females. Pulseless

peripheries are a classical finding. The CNS symptoms may be variable.

20. A 48 year old man notices that he is becoming increasingly dizzy when he

plays squash, in addition he has also developed cramping pain in his left arm.

One day he is inflating his car tyre with a hand held pump, he collapses and is

brought to hospital.

Subclavian steal syndrome

Subclavian steal syndrome is associated with a stenosis or occlusion of the

subclavian artery, proximal to the origin of the vertebral artery. As a result the

increased metabolic needs of the arm then cause retrograde flow and

symptoms of CNS vascular insufficiency.

21. A 25 year old junior doctor has a chest x-ray performed as part of a routine

insurance medical examination. The x-ray shows evidence of rib notching.

Auscultation of his chest reveals a systolic murmur which is loudest at the

posterior aspect of the fourth intercostal space.

Aortic coarctation

Coarctation of the aorta may occur due to the remnant of the ductus arteriosus

acting as a fibrous constrictive band of the aorta. Weak arm pulses may be

seen, radiofemoral delay is the classical physical finding. Collateral flow

through the intercostal vessels may produce notching of the ribs, if the disease

is long standing.

Vascular disease

Aortic dissection Chest pain (anterior chest pain- ascending aorta, back pain

- descending aorta)

Widening of aorta on chest x-ray

Diagnosis made by CT scanning

Treatment is either medical (Type B disease) or surgical

(Type A disease)

Cervical rib Supernumery fibrous band arising from seventh cervical

vertebra

Incidence of 1 in 500

May cause thoracic outlet syndrome

Treatment involves surgical division of rib

Subclavian steal

syndrome

Due to proximal stenotic lesion of the subclavian artery

Results in retrograte flow through vertebral or internal

thoracic arteries

The result is that decrease in cerebral blood flow may

occur and produce syncopal symptoms

A duplex scan and/ or angiogram will delineate the lesion

and allow treatment to be planned

Takayasu's

arteritis

Large vessel granulomatous vasculitis

Results in intimal narrowing

Most commonly affects young asian females

Patients present with features of mild systemic illness,

followed by pulseless phase with symptoms of vascular

insufficiency

Treatment is with systemic steroids

Patent ductus

arteriosus

Ductus arteriosus is a normal foetal vessel that closes

spontaneously after birth

Results in high pressure, oxygenated blood entering the

pulmonary circuit

Untreated patients develop symptoms of congestive cardiac

failure

Coarctation of

the aorta

Aortic stenosis at the site of the ductus arteriosus insertion

Most common in boys and girls with Turners syndrome

Patients may present with symptoms of arterial

insufficiency, such as syncope and claudication

Blood pressure mismatch may be seen, as may mismatch

of pulse pressure in the upper and lower limbs

Treatment is either with angioplasty or surgical resection

(the former is the most common)

A 25 year old man is found to have carcinoid syndrome. Which of the following

hormones is released by carcinoids?

A. Serotonin

B. Dopamine

C. Nor adrenaline

D. Adrenaline

E. Aldosterone

Rule of thirds:

1/3 multiple

1/3 small bowel

1/3 metastasize

1/3 second

tumour

Carcinoids secrete serotonin. Carcinoid syndrome will only occur in the presence of

liver metastasis as the hormone released from primary lesions will typically be

metabolised by the liver.

Carcinoid syndrome

Carcinoid tumours secrete serotonin

Originate in neuroendocrine cells mainly in the intestine (midgut-distal

ileum/appendix)

Can occur in the rectum, bronchi

Hormonal symptoms mainly occur when disease spreads outside the bowel

Clinical features - Onset: years

- Flushing face

- Palpitations

- Tricuspid stenosis causing dyspnoea

- Asthma

- Severe diarrhoea (secretory, persists despite fasting)

Investigation - 5-HIAA in a 24-hour urine collection

- Scintigraphy

- CT scan

Treatment

Octreotide

Surgical removal

Which one of the following genes protects against neoplasms?

A. sis

B. p53

C. ras

D. myc

E. src

p53 is a tumour supressor gene and located on chromosome 17. It plays an important

role in causing cells that are undergoing neoplastic changes to enter an apoptotic

pathway.

Oncogenes

Oncogenes are cancer promoting genes that are derived from normal genes (proto-

oncogenes). Proto-oncogenes play an important physiological role in cellular growth.

They are implicated in the development of up to 20% of human cancers.

Proto-oncogenes may become oncogenes via the following processes:

Mutation (point mutation)

Chromosomal translocation

Increased protein expression

Only one mutated copy of the gene is needed for cancer to occur - a dominant effect

Classification of oncogenes

Growth factors e.g. Sis

Transcription factors e.g. Myc

Receptor tyrosine kinase e.g. RET

Cytoplasmic tyrosine kinase e.g. Src

Regulatory GTPases e.g. Ras

Tumour supressor genes Tumour supressor genes restrict or repress cellular proliferation in normal cells. Their

inactivation through mutation or germ line incorporation is implicated in renal,

colonic, breast, bladder and many other cancers. One of the best known tumour

supressor genes is p53. p53 gene offers protection by causing apoptosis of damaged

cells. Other well known genes include BRCA 1 and 2.

A 55 year old man with a long history of achalasia is successfully treated by a Hellers

Cardiomyotomy. Several years later he develops an oesophageal malignancy. Which

of the following lesions is most likely to be present?

A. Adenocarcinoma

B. Gastrointestinal stromal tumour

C. Leiomyosarcoma

D. Rhabdomyosarcoma

E. Squamous cell carcinoma

Achalasia is a rare condition. However, even once treated there is an increased risk of

malignancy. When it does occur it is most likely to be of squamous cell type.

Oesophageal cancer

Incidence is increasing

In most cases in the Western world this increase is accounted for by a rise in

the number of cases of adenocarcinoma. In the UK adenocarcinomas account

for 65% of cases.

Barretts oesophagus is a major risk factor for most cases of oesophageal

adenocarcinoma.

In other regions of the world squamous cancer is more common and is linked

to smoking, alcohol intake, diets rich in nitrosamines and achalasia.

Surveillance of Barretts is important as it imparts a 30 fold increase in cancer

risk and if invasive malignancy is diagnosed early then survival may approach

85% at 5 years.

Diagnosis

Upper GI endoscopy is the first line test

Contrast swallow may be of benefit in classifying benign motility disorders

but has no place in the assessment of tumours

Staging is initially undertaken with CT scanning of the chest, abdomen and

pelvis. If overt metastatic disease is identified using this modality then further

complex imaging is unnecessary

If CT does not show metastatic disease, then local stage may be more

accurately assessed by use of endoscopic ultrasound.

Staging laparoscopy is performed to detect occult peritoneal disease. PET CT

is performed in those with negative laparoscopy. Thoracoscopy is not

routinely performed.

Treatment Operable disease is best managed by surgical resection. The most standard procedure

is an Ivor- Lewis type oesophagectomy. This procedure involves the mobilisation of

the stomach and division of the oesophageal hiatus. The abdomen is closed and a right

sided thoracotomy performed. The stomach is brought into the chest and the

oesophagus mobilised further. An intrathoracic oesophagogastric anastomosis is

constructed. Alternative surgical strategies include a transhiatal resection (for distal

lesions), a left thoraco-abdominal resection (difficult access due to thoracic aorta) and

a total oesophagectomy (McKeown) with a cervical oesophagogastric anastomosis.

The biggest surgical challenge is that of anastomotic leak, with an intrathoracic

anastomosis this will result in mediastinitis. With high mortality. The McKeown

technique has an intrinsically lower systemic insult in the event of anastmotic leakage.

In addition to surgical resection many patients will be treated with adjuvent

chemotherapy.

Which of the following genes is not implicated in the adenoma-carcinoma sequence in

colorectal cancer?

A. src

B. c-myc

C. APC

D. p53

E. K-ras

Other genes involved are:

MCC

DCC

c-yes

bcl-2

Colorectal cancer

Annually about 150,000 new cases are diagnosed and 50,000 deaths from the

disease

About 75% will have sporadic disease and 25% will have a family history

Colorectal tumours comprise a spectrum of disease ranging from adenomas

through to polyp cancers and frank malignancy.

Polyps may be categorised into: neoplastic polyps, adenomatous polyps and

non neoplastic polyps.

The majority of adenomas are polypoidal lesions, although flat lesions do

occur and may prove to be dysplastic.

Non-neoplastic polyps include hyperplastic, juvenile, hamartomatous,

inflammatory, and lymphoid polyps, which have not generally been thought of

as precursors of cancer.

Three characteristics of adenomas that correlate with malignant potential have

been characterised. These include increased size, villous architecture and

dysplasia. For this reason most polyps identified at colonoscopy should be

removed.

The transformation from polyp to cancer is described by the adenoma -

carcinoma sequence and its principles should be appreciated. Essentially

genetic changes accompany the transition from adenoma to carcinoma; key

changes include APC, c-myc, K RAS mutations and p53 deletions.

A 63 year old lady is suspected as having sarcoidosis. She is sent to the general surgeons and

a lymph node biopsy is performed. Which of the following histological features is most likely

to be identified in a lymph node if sarcoid is present?

A. Psammoma bodies

B. Extensive necrosis

C. Dense eosinophillic infiltrates

D. Asteroid bodies

E. None of the above

Asteroid bodies are often found in the granulomas of individuals with sarcoid. Unlike the

granulomata associated with tuberculosis the granulomas of sarcoid are rarely associated

with extensive necrosis.

An Asteroid body in an individual with sarcoid

Image sourced from Wikipedia

Chronic inflammation

Overview

Chronic inflammation may occur secondary to acute inflammation.In most cases chronic

inflammation occurs as a primary process. These may be broadly viewed as being one of

three main processes:

Persisting infection with certain organisms such as Mycobacterium tuberculosis which results in delayed type hypersensitivity reactions and inflammation.

Prolonged exposure to non-biodegradable substances such as silica or suture materials which may induce an inflammatory response.

Autoimmune conditions involving antibodies formed against host antigens.

Acute vs. Chronic inflammation

Acute inflammation Chronic inflammation

Changes to existing vascular structure and increased

permeability of endothelial cells

Angiogenesis predominates

Infiltration of neutrophils Macrophages, plasma cells and

lymphocytes predominate

Process may resolve with:

Suppuration Complete resolution Abscess formation Progression to chronic inflammation Healing by fibrosis

Healing by fibrosis is the main result

Granulomatous inflammation

A granuloma consists of a microscopic aggregation of macrophages (with epithelial type

arrangement =epitheliod). Large giant cells may be found at the periphery of granulomas.

Mediators

Growth factors released by activated macrophages include agents such as interferon and

fibroblast growth factor (plus many more). Some of these such as interferons may have

systemic features resulting in systemic symptoms and signs, which may be present in

individuals with long standing chronic inflammation.

The finding of granulomas is pathognomonic of chronic inflammation, as illustrated in this

biopsy from a patient with colonic Crohns disease

Image sourced from Wikipedia

Brown tumours of bone are associated with which of the following?

A. Hyperthyroidism

B. Hypothyroidism

C. Hyperparathyroidism

D. Hypoparathyroidism

E. Osteopetrosis

Brown tumors are tumors of bone that arise in settings of excess osteoclast activity,

such as hyperparathyroidism, and consist of fibrous tissue, woven bone and

supporting vasculature, but no matrix. They are radiolucent on x-ray. The osteoclasts

consume the trabecular bone that osteoblasts lay down and this front of reparative

bone deposition followed by additional resorption can expand beyond the usual shape

of the bone, involving the periosteum thus causing bone pain. They appear brown

because haemosiderin is deposited at the site.

Primary hyperparathyroidism

In exams primary hyperparathyroidism is stereotypically seen in elderly females with

an unquenchable thirst and an inappropriately normal or raised parathyroid hormone

level. It is most commonly due to a solitary adenoma

Causes of primary hyperparathyroidism

80%: solitary adenoma

15%: hyperplasia

4%: multiple adenoma

1%: carcinoma

Features - 'bones, stones, abdominal groans and psychic moans'

Polydipsia, polyuria

Peptic ulceration/constipation/pancreatitis

Bone pain/fracture

Renal stones

Depression

Hypertension

Associations

Hypertension

Multiple endocrine neoplasia: MEN I and II

Investigations

Raised calcium, low phosphate

PTH may be raised or normal

Technetium-MIBI subtraction scan

Treatment

Parathyroidectomy, if imaging suggests target gland then a focused approach

may be used

A 73 year old man is recovering following an emergency Hartmans procedure

performed for an obstructing sigmoid cancer. The pathology report shows a

moderately differentiated adenocarcinoma that invades the muscularis propria,

3 of 15 lymph nodes are involved with metastatic disease. What is the correct

stage for this?

A. Astler Coller Stage B2

B. Dukes stage A

C. Dukes stage B

D. Dukes stage C

E. Dukes stage D

Theme from September 2011 Exam

The involvement of lymph nodes makes this Dukes C. In the Astler Coller

system the B and C subsets are split to B1 and B2 and C1 and C2. Where C2

denotes involvement of the nodes in conjunction with penetration of the

muscularis propria.

Dukes classification

Gives the extent of spread of colorectal cancer

Dukes A Tumour confined to the mucosa (90%)

Dukes B Tumour invading bowel wall (70%)

Dukes C Lymph node metastases (45%)

Dukes D Distant metastases (6%)(20% if resectable)

5 year survival in brackets

A 55 year old man with dyspepsia undergoes an upper GI endoscopy. An irregular

erythematous area is seen to protrude proximally from the gastro-oesophageal

junction. Apart from specialised intestinal metaplasia, which of the following cell

types should also be present for a diagnosis of Barretts oesophagus to be made?

A. Goblet cell

B. Neutrophil

C. Lymphocytes

D. Epithelial cells

E. Macrophages

Goblet cells need to be present for a diagnosis of Barrett's oesophagus to be made.

Barrett's oesophagus

Intestinal metaplasia

Squamous epithelium replaced by columnar epithelium in the lower

oesophagus

3 types of columnar epithelium:

1. Junctional

2. Atrophic fundal

3. Specialised

Presence of goblet cells important in identification

Premalignant change (progress to dysplasia)

Risk of adenocarcinoma

Risk factors: middle age, men, smoker, Caucasian, gastro-oesophageal reflux,

obesity

Treatment

Long term proton pump inhibitor

Consider pH and manometry studies in younger patients who may prefer to

consider an anti reflux procedure

Regular endoscopic monitoring (more frequently if moderate dysplasia). With

quadrantic biopsies every 2-3 cm

If severe dysplasia be very wary of small foci of cancer

Which of the following amino acids is present in all types of collagen?

A. Alanine

B. Aspartime

C. Glycine

D. Tyrosine

E. Cysteine

Collagen has a generic structure of Glycine- X- Y, where X and Y are variable sub

units. The relatively small size of the glycine molecule enables collagen to form a

tight helical structure.

Collagen

One of the major connective tissue proteins

Composed of 3 polypeptide strands that are woven into a helix

Numerous hydrogen bonds exist within molecule to provide additional

strength

Many sub types but commonest sub type is I (90% of bodily collagen)

Vitamin c is important in establishing cross links

Collagen Diseases

Osteogenesis imperfecta

Ehlers Danlos

Osteogenesis imperfecta:

-8 Subtypes

-Defect of type I collagen

-In type I the collagen is normal quality but insufficient quantity

-Type II- poor quantity and quality

-Type III- Collagen poorly formed, normal quantity

-Type IV- Sufficient quantity but poor quality

Patients have bones which fracture easily, loose joint and multiple other defects

depending upon which sub type they suffer from

Ehlers Danlos:

-Multiple sub types

-Abnormality of types 1 and 3 collagen

-Patients have features of hypermobility.

-Individuals are prone to joint dislocations and pelvic organ prolapse. In addition to

many other diseases related to connective tissue defects

Which of the following amino acids is present in all types of collagen?

A. Alanine

B. Aspartime

C. Glycine

D. Tyrosine

E. Cysteine

Collagen has a generic structure of Glycine- X- Y, where X and Y are variable sub

units. The relatively small size of the glycine molecule enables collagen to form a

tight helical structure.

Collagen

One of the major connective tissue proteins

Composed of 3 polypeptide strands that are woven into a helix

Numerous hydrogen bonds exist within molecule to provide additional

strength

Many sub types but commonest sub type is I (90% of bodily collagen)

Vitamin c is important in establishing cross links

Collagen Diseases

Osteogenesis imperfecta

Ehlers Danlos

Osteogenesis imperfecta:

-8 Subtypes

-Defect of type I collagen

-In type I the collagen is normal quality but insufficient quantity

-Type II- poor quantity and quality

-Type III- Collagen poorly formed, normal quantity

-Type IV- Sufficient quantity but poor quality

Patients have bones which fracture easily, loose joint and multiple other defects

depending upon which sub type they suffer from

Ehlers Danlos:

-Multiple sub types

-Abnormality of types 1 and 3 collagen

-Patients have features of hypermobility.

-Individuals are prone to joint dislocations and pelvic organ prolapse. In addition to

many other diseases related to connective tissue defects

An 8 year old boy presented with a painless swelling on the superotemporal aspect of

his orbit. It was smooth on examination, produced no visual disturbances. Following

excision it was found to be lined by squamous epithelium and hair follicles. Which of

the following lesions most closely matches these findings?

A. Dermoid cyst

B. Desmoid tumour

C. Lipoma

D. Sebaceous cyst

E. Schwannoma

Dermoid cysts are embryological remnants and may be lined by hair and squamous

epithelium (like teratomas). They are often located in the midline and may be linked

to deeper structures resulting in a dumbbell shape to the lesion. Complete excision is

requires as they have a propensity to local recurrence if not excised.

Desmoid tumours are a different entity, they most commonly develop in ligaments

and tendons. They are also referred to as aggressive fibromatosis and consist of

fibroblast dense lesions (resembling scar tissue). They should be managed in a similar

manner to soft tissue sarcomas.

Skin Diseases

Skin lesions may be referred for surgical assessment, but more commonly will come

via a dermatologist for definitive surgical management.

Skin malignancies include basal cell carcinoma, squamous cell carcinoma and

malignant melanoma.

Basal Cell Carcinoma

Most common form of skin cancer.

Commonly occur on sun exposed sites apart from the ear.

Sub types include nodular, morphoeic, superficial and pigmented.

Typically slow growing with low metastatic potential.

Standard surgical excision, topical chemotherapy and radiotherapy are all

successful.

As a minimum a diagnostic punch biopsy should be taken if treatment other

than standard surgical excision is planned.

Squamous Cell Carcinoma

Again related to sun exposure.

May arise in pre - existing solar keratoses.

May metastasise if left.

Immunosupression (e.g. Following transplant), increases risk.

Wide local excision is the treatment of choice and where a diagnostic excision

biopsy has demonstrated SCC, repeat surgery to gain adequate margins may

be required.

Malignant Melanoma

The main diagnostic features (major

criteria):

Change in size

Change in shape

Change in colour

Secondary features (minor

criteria)

Diameter >6mm

Inflammation

Oozing or bleeding

Altered sensation

Treatment

Suspicious lesions should undergo excision biopsy. The lesion should be

removed in completely as incision biopsy can make subsequent

histopathological assessment difficult.

Once the diagnosis is confirmed the pathology report should be reviewed to

determine whether further re-exicision of margins is required (see below):

Margins of excision-Related to Breslow thickness

Lesions 0-1mm thick 1cm

Lesions 1-2mm thick 1- 2cm (Depending upon site and pathological features)

Lesions 2-4mm thick 2-3 cm (Depending upon site and pathological features)

Lesions >4 mm thick 3cm

Marsden J et al Revised UK guidelines for management of Melanoma. Br J Dermatol

2010 163:238-256.

Further treatments such as sentinel lymph node mapping, isolated limb perfusion and

block dissection of regional lymph node groups should be selectively applied.

Kaposi Sarcoma

Tumour of vascular and lymphatic endothelium.

Purple cutaneous nodules.

Associated with immunosupression.

Classical form affects elderly males and is slow growing.

Immunosupression form is much more aggressive and tends to affect those

with HIV related disease.

Non malignant skin disease

Dermatitis Herpetiformis

Chronic itchy clusters of blisters.

Linked to underlying gluten enteropathy (coeliac disease).

Dermatofibroma

Benign lesion.

Firm elevated nodules.

Usually history of trauma.

Lesion consists of histiocytes, blood vessels and fibrotic changes.

Pyogenic granuloma

Overgrowth of blood vessels.

Red nodules,

Usually follow trauma.

May mimic amelanotic melanoma.

Acanthosis nigricans

Brown to black, poorly defined, velvety hyperpigmentation of the skin.

Usually found in body folds such as the posterior and lateral folds of the neck,

the axilla, groin, umbilicus, forehead, and other areas.

The most common cause of acanthosis nigricans is insulin resistance, which

leads to increased circulating insulin levels. Insulin spillover into the skin

results in its abnormal increase in growth (hyperplasia of the skin).

In the context of a malignant disease, acanthosis nigricans is a paraneoplastic

syndrome and is then commonly referred to as acanthosis nigricans maligna.

Involvement of mucous membranes is rare and suggests a coexisting

malignant condition

A 55 year old man from Hong Kong presents with left sided otalgia and recurrent

episodes of epistaxis. On examination his pharynx appears normal. Examination of

his neck reveals left sided cervical lymphadenopathy. What is the most likely

underlying diagnosis?

A. Antrochoanal polyp

B. Nasopharyngeal carcinoma

C. Adenocarcinoma of the tonsil

D. Angiofibroma

E. Globus syndrome

Given this mans ethnic origin and presenting features a nasopharyngeal carcinoma is

the most likely underlying diagnosis.

Nasopharyngeal carcinoma

Squamous cell carcinoma of the nasopharynx

Rare in most parts of the world, apart from individuals from Southern China

Associated with Epstein Barr virus infection

Presenting features

Systemic Local

Cervical lymphadenopathy Otalgia

Unilateral serous otitis media

Nasal obstruction, discharge and/ or epistaxis

Cranial nerve palsies e.g. III-VI

Imaging Combined CT and MRI.

Treatment Radiotherapy is first line therapy.

Patients with suspected temporal arteritis are often sent for temporal artery biopsy.

Which statement is true?

A. Temporal artery biopsy is only diagnostic if there is visual loss

B. Biopsy is typically taken from the non-symptomatic side to avoid the

risk of blindness

C. Pre-operative localisation with duplex is mandatory

D. Biopsies may be non diagnostic in over 50% of cases

E. Biopsies are usually performed under general anaesthesia

Temporal artery biopsies are frequently non diagnostic. They should be taken from

the symptomatic side and though not mandatory a duplex ultrasound is a helpful

investigation, particularly if they mark the artery. It is usually performed under local

anaesthetic.

Temporal artery biopsy

Superficial temporal artery is a terminal branch of the external carotid artery

Main indication

Temporal arteritis

American College of Rheumatology guidelines recommend a temporal artery biopsy

if:

Age of onset older than 50 years

New-onset headache or localized head pain

Temporal artery tenderness to palpation or reduced pulsation

ESR > 50 mm/h

Histopathology

Vessel wall granulomatous arteritis with mononuclear cell infiltrates and giant

cell formation

Procedure

Position: supine, head 45 degrees

USS doppler to locate the superficial temporal artery or palpate

Local anaesthetic

Artery within temporoparietal fascia

Clamp and ligate the vessel

Cut 3-5cm

Ligate the remaining ends with absorbable suture

Close the skin

Contraindication Glucocorticoid therapy > 30 days

Risks Injury to facial or auriculotemporal nerve

Which of the following best describes the processes underpinning type IV

hypersensitivity reactions?

A. Deposition of immune complexes of IgG and antigen at

the site of inflammation

B. Deposition of IgA complexes at the site of inflammation

C. Deposition of IgM and IgG complexes at the site of

inflammation

D. Degranulation of mast cells at the site of inflammation

E. T cell mediated response at the site of inflammation

Hypersensitivity

reactions: ACID

type 1 --Anaphylactic

type 2 --Cytotoxic

type 3 --Immune complex

type 4 --Delayed

hypersensitivity

T Cells are the mediators of type 4 hypersensitivity reactions which are characterised

by the absence of immune complex deposition.

Hypersensitivity reactions

The Gell and Coombs classification divides hypersensitivity reactions into 4 types

Type I Type II Type III Type IV

Description Anaphylactic Cytotoxic Immune

complex

Delayed type

Mediator IgE IgG, IgM IgG, IgM T-cells

Antigen Exogenous Cell surface Soluble Tissues

Response

time

Minutes Hours Hours 2-3 days

Examples Asthma

Hay fever

Autoimmune

haemolytic anaemia

Pemphigus

Goodpasture's

Serum

sickness

SLE

Aspergillosis

Graft versus host

disease

Contact

dermatitis

Theme: Bone tumours

A. Osteosarcoma

B. Fibrosarcoma

C. Osteoclastoma

D. Ewings sarcoma

E. Leiomyosarcoma

F. Chondrosarcoma

G. Rhabdomyosarcoma

H. Osteoid osteoma

I. Malignant fibrous histiocytoma

Please select the most appropriate lesion for the clinical scenario given. Each option

may be used once, more than once or not at all.

7. A 16 year-old boy presents to his GP with loss of weight, pain and fever. On

examination, a soft tissue mass is palpable over the mid-thigh region

Ewings sarcoma

Ewing's sarcoma is a malignant round cell tumour occurring in the diaphysis of

the long bones in the children. These are not confined to the ends of long

bones. x Rays often show a large soft-tissue mass with concentric layers of

new bone formation ( 'onion-peel' sign). The ESR may be elevated, thus

suggesting an inflammatory or an infective cause such as osteomyelitis;

although osteomyelitis usually affects the metaphyseal region in children.

Treatment is with chemotherapy and surgical excision, an endoprothesis may

be used to conserve the limb.

8. A 75 year old lady presents with weight loss, pain and a swelling over her left

knee. She has been treated for Pagets disease of the bone for some time.

Osteosarcoma

Osteosarcoma may complicate Pagets disease of bone in up to 10% cases.

Radiological appearances include bone destruction coupled with new bone

formation, periosteal elevation may also occur. Surgical resection is the main

treatment.

9. A 17-year-old girl presents with weight loss, fever and a swelling over her

right knee. Movements of her knee are restricted. A plain x-ray of the affected

site shows multiple lytic and lucent lesions with clearly defined borders.

You answered Malignant fibrous histiocytoma

The correct answer is Osteoclastoma

Osteoclastoma has a characteristic appearance on x-ray with multple lytic and

lucent areas (Soap bubble) appearances. Pathological fractures may occur. The

disease is usually indolent.

Sarcomas

Malignant tumours of mesenchymal origin

Types May be either bone or soft tissue in origin.

Bone sarcoma include:

Osteosarcoma

Ewings sarcoma (although non bony sites recognised)

Chrondrosarcoma - originate from Chondrocytes

Soft tissue sarcoma are a far more heterogeneous group and include:

Liposarcoma-adipocytes

Rhabdomyosarcoma-striated muscle

Leiomyosarcoma-smooth muscle

Synovial sarcomas- close to joints (cell of origin not known but not synovium)

Malignant fibrous histiocytoma is a sarcoma that may arise in both soft tissue and

bone.

Features Certain features of a mass or swelling should raise suspicion for a sarcoma these

include:

Large >5cm soft tissue mass

Deep tissue location or intra muscular location

Rapid growth

Painful lump

Assessment Imaging of suspicious masses should utilise a combination of MRI, CT and USS.

Blind biopsy should not be performed prior to imaging and where required should be

done in such a way that the biopsy tract can be subsequently included in any

resection.

Ewings sarcoma

Commoner in males

Incidence of 0.3 / 1, 000, 000

Onset typically between 10 and 20 years of age

Location by femoral diaphysis is commonest site

Histologically it is a small round tumour

Blood borne metastasis is common and chemotherapy is often combined with

surgery

Osteosarcoma

Mesenchymal cells with osteoblastic differentiation

20% of all primary bone tumours

Incidence of 5 per 1,000,000

Peak age 15-30, commoner in males

Limb preserving surgery may be possible and many patients will receive

chemotherapy

Liposarcoma

Malignancy of adipocytes

Rare approximately 2.5 per 1,000,000. They are the second most common soft

tissue sarcoma

Typically located in deep locations such as retroperitoneum

Affect older age group usually >40 years of age

May be well differentiated and thus slow growing although may undergo

dedifferentiation and disease progression

Many tumours will have a pseudocapsule that can misleadingly allow

surgeons to feel that they can 'shell out' these lesions. In reality tumour may

invade at the edge of the pseudocapsule and result in local recurrence if this

strategy is adopted

Usually resistant to radiotherapy although this is often used in a palliative

setting

Malignant Fibrous Histiocytoma

Tumour with large number of histiocytes

Most common sarcoma in adults

Also described as undifferentiated pleomorphic sarcoma NOS (i.e. Cell of

origin is not known)

Four major subtypes are recognised: storiform-pleomorphic (70% cases),

myxoid (less aggressive), giant cell and inflammatory

Treatment is usually with surgical resection and adjuvant radiotherapy as this

reduces the likelihood of local recurrence

A 22 year old man is undergoing an abdominal ultrasound scan as part of a series of

investigations for abdominal pain. The radiologist notes that there is evidence of

splenic atrophy. What is the most likely cause?

A. Letterer-Siwe disease

B. Coeliac disease

C. Malaria

D. Niemann-Pick disease

E. Sarcoidosis

Splenic atrophy may occur in coeliac disease together with the appearance of Howell-

Jolly bodies in erythrocytes. Letterer - Siwe disease is a form of Histiocytosis X in

which macrophages proliferate.

Spleen

Embryology: derived from mesenchymal tissue

Shape: orange segment

Position: below 9th-12th ribs

Weight: 75-150g

Relations

Superiorly- diaphragm

Anteriorly- gastric impression

Posteriorly- kidney

Inferiorly- colon

Hilum: tail of pancreas and splenic vessels (splenic artery divides here,

branches pass to the white pulp transporting plasma)

Forms apex of lesser sac (containing short gastric vessels)

Contents - White pulp: immune function. Contains central trabecular artery. The germinal

centres are supplied by arterioles called penicilliary radicles.

- Red pulp: filters abnormal red blood cells

Function

Filtration of abnormal blood cells and foreign bodies such as bacteria.

Immunity: IgM. Production of properdin, and tuftsin which help target fungi

and bacteria for phagocytosis.

Haematopoiesis: up to 5th month gestation or in haematological disorders.

Pooling: storage of 40% platelets.

Iron reutilisation

Storage red blood cells-animals, not humans.

Storage monocytes

Disorders of the spleen Massive splenomegaly

Myelofibrosis

Chronic myeloid leukaemia

Visceral leishmaniasis (kala-azar)

Malaria

Gaucher's syndrome

Other causes (as above plus)

Portal hypertension e.g. secondary to cirrhosis

Lymphoproliferative disease e.g. CLL, Hodgkin's

Haemolytic anaemia

Infection: hepatitis, glandular fever

Infective endocarditis

Sickle-cell*, thalassaemia

Rheumatoid arthritis (Felty's syndrome)

*the majority of adults patients with sickle-cell will have an atrophied spleen due to

repeated infarction

Which statement relating to phaeochromocytoma is untrue?

A. They are tumours of chromaffin cells in the adrenal medulla.

B. They are bilateral in 10% of cases.

C. When located in an extra adrenal location have a higher incidence of

malignancy.

D. May be associated with an elevated urinary VMA.

E. Up to 40% may have a blood pressure within the normal range.

Normotension is seen in around 10% cases. The remainder show a degree of

hypertension.

Phaeochromocytoma and adrenal lesions

Phaeochromocytoma Neuroendocrine tumour of the chromaffin cells of the adrenal medulla. Hypertension

and hyperglycaemia are often found.

10% of cases are bilateral.

10% occur in children.

11% are malignant (higher when tumour is located outside the adrenal).

10% will not be hypertensive.

Familial cases are usually linked to the Multiple endocrine neoplasia syndromes

(considered under its own heading).

Most tumours are unilateral (often right sided) and smaller than 10cm.

Diagnosis Urine analysis of vanillymandelic acid (VMA) is often used (false positives may

occur e.g. in patients eating vanilla ice cream!)

Blood testing for plasma metanephrine levels.

CT and MRI scanning are both used to localise the lesion.

Treatment Patients require medical therapy first. An irreversible alpha adrenoreceptor blocker

should be given, although minority may prefer reversible bockade(1). Labetolol may

be co-administered for cardiac chronotropic control. Isolated beta blockade should not

be considered as it will lead to unopposed alpha activity.

These patients are often volume depleted and will often require moderate volumes of

intra venous normal saline perioperatively.

Once medically optimised the phaeochromocytoma should be removed. Most

adrenalectomies can now be performed using a laparoscopic approach(2). The

adrenals are highly vascular structures and removal can be complicated by

catastrophic haemorrhage in the hands of the inexperienced. This is particularly true

of right sided resections where the IVC is perilously close. Should the IVC be

damaged a laparotomy will be necessary and the defect enclosed within a Satinsky

style vascular clamp and the defect closed with prolene sutures. Attempting to

interfere with the IVC using any instruments other than vascular clamps will result in

vessel trauma and make a bad situation much worse.

Incidental adrenal lesions Adrenal lesions may be identified on CT scanning performed for other reasons(3).

Factors suggesting benign disease on CT include(4):

Size less than 3cm

Homogeneous texture

Lipid rich tissue

Thin wall to lesion

All patients with incidental lesions should be managed jointly with an endocrinologist

and full work up as described above. Patients with functioning lesions or those with

adverse radiological features (Particularly size >3cm) should proceed to surgery.

References 1. Weingarten TN, Cata JP, O'Hara JF, Prybilla DJ, Pike TL, Thompson GB, et al.

Comparison of two preoperative medical management strategies for laparoscopic

resection of pheochromocytoma. Urology. 2010 Aug;76(2):508 e6-11.

2. Nguyen PH, Keller JE, Novitsky YW, Heniford BT, Kercher KW. Laparoscopic

approach to adrenalectomy: review of perioperative outcomes in a single center. Am

Surg. 2011 May;77(5):592-6.

3. Ng VW, Ma RC, So WY, Choi KC, Kong AP, Cockram CS, et al. Evaluation of

functional and malignant adrenal incidentalomas. Arch Intern Med. 2010 Dec

13;170(22):2017-20.

4. Muth A, Hammarstedt L, Hellstrom M, Sigurjonsdottir HA, Almqvist E, Wangberg

B. Cohort study of patients with adrenal lesions discovered incidentally. Br J Surg.

2011 May 27. A 69 year old man presents with a purple lesion on his forearm. It is excised and an a 3 cm

Merkel cell tumour is diagnosed. Which of the following statements relating to this diagnosis

is false?

A. He should undergo a sentinel lymph node biopsy.

B. Lymphovascular invasion is typically seen histologically

C. They are more common in immunosupressed patients

D. Histologically they may resemble pyogenic granuloma

E. They are associated with visceral metastasis

Merkel cell tumours are rare cutaneous tumours. Histologically they consist of sheets and

nodules of hyperchromatic epithelial cells, with high rates of mitosis and apoptosis. As such

they are relatively easy to distinguish from pyogenic granuloma which has no features of

malignancy and would not show lymphovascular invasion.

Merkel cell tumours of the skin

Rare but aggressive tumour. Develops from intra epidermal Merkel cells. Usually presents on elderly, sun damaged skin. The periorbital area is the

commonest site.

Histologically these tumours appear within the dermis and subcutis. The lesions consist of sheets and nodules of small hyperchromatic epithelial cells with high rates of mitosis and apoptosis. Lymphovascular invasion is commonly seen.

Pre-existing infection with Merkel Cell Polyomavirus is seen in 80% cases.

Treatment

Surgical excision is first line. Margins of 1cm are required. Lesions >10mm in diameter

should undergo sentinel lymph node biopsy. Adjuvant radiotherapy is often given to reduce

the risk of local recurrence.

Prognosis

With lymph node metastasis 5 year survival is 50% or less. Small lesions without nodal spread are usually associated with a 5 year survival of

80%.

Skin biopsy demonstrating a Merkel Cell cancer. Note the hyperchromatic epithelial cells and

high mitotic rate

Image sourced from Wikipedia

A 58 year old lady undergoes a screening mammogram and appearances are

suspicious for ductal carcinoma in situ. A stereotactic core biopsy is performed. If

ductal carcinoma in situ is to be diagnosed, which of the following pathological

features must not be present?

A. Nuclear pleomorphism

B. Coarse chromatin

C. Abnormal mitoses

D. Angiogenesis

E. Dysplastic cells infiltrating the suspensory ligaments of the breast

The presence of invasion is a hallmark of invasive disease and thus would not be a

feature of DCIS. Angiogenesis may occur in association with high grade DCIS.

Histopathology of malignancy

Abnormal tissue architecture

Coarse chromatin

Invasion of basement membrane*

Abnormal mitoses

Angiogenesis

De-differentiation

Areas of necrosis

Nuclear pleomorphism

*= Those features that distinguish invasive malignancy from in situ disease

Which of the following does not occur as a pathological response to extensive burns?

A. Plasma leakage into interstitial space

B. Polycythaemia

C. Increased haematocrit

D. Keratinocyte migration during healing

E. Cardiac output reduction by 50% in first 30 minutes

Haemolysis is the main pathological response.

Burns pathology

Extensive burns

Haemolysis due to damage of erythrocytes by heat and microangiopathy

Loss of capillary membrane integrity causing plasma leakage into interstitial

space

Extravasation of fluids from the burn site causing hypovolaemic shock (up to

48h after injury)- decreased blood volume and increased haematocrit

Protein loss

Secondary infection e.g. Staphylococcus aureus

ARDS

Risk of Curlings ulcer (acute peptic stress ulcers)

Danger of full thickness circumferential burns in an extremity as these may

develop compartment syndrome

Healing

Superficial burns: keratinocytes migrate to form a new layer over the burn site

Full thickness burns: dermal scarring. Usually need keratinocytes from skin

grafts to provide optimal coverage.

Theme: Chest pain

A. Pulmonary embolism

B. Acute exacerbation asthma

C. Physiological

D. Mitral valve stenosis

E. Aortic dissection

F. Mitral regurgitation

G. Bronchopneumonia

H. Tuberculosis

I. None of the above

What is the most likely diagnosis for the scenario given? Each option may be used

once, more than once or not at all.

15. A 28 year old Indian woman, who is 18 weeks pregnant, presents with

increasing shortness of breath, chest pain and coughing clear sputum.

She is apyrexial, blood pressure is 140/80 mmHg, heart rate 130 bpm and

saturations 94% on 15L oxygen. On examination there is a mid diastolic

murmur, there are bibasal crepitations and mild pedal oedema. She

suddenly deteriorates and has a respiratory arrest. Her chest x-ray shows

a whiteout of both of her lungs.

Mitral valve stenosis

Mitral stenosis is the commonest cause of cardiac abnormality occurring

in pregnant women. Mitral stenosis is becoming less common in the UK

population, however should be considered in women from countries were

there is a higher incidence of rheumatic heart disease. Mitral stenosis

causes a mid diastolic murmur which may be difficult to auscultate

unless the patient is placed into the left lateral position. These patients

are at risk of atrial fibrillation (up tp 40%), which can also contribute to

rapid decompensation. Physiological changes in pregnancy may cause an

otherwise asymptomatic patient to suddenly deteriorate. Balloon

valvuloplasty is the treatment of choice.

16. A 28 year old woman, who is 30 weeks pregnant, presents with sudden

onset chest pain associated with loss of consciousness. Her blood

pressure is 170/90 mmHg, saturations on 15L oxygen 93%, heart rate

120 bpm and she is apyrexial. On examination there is an early diastolic

murmur, occasional bibasal creptitations and mild peal oedema. An ECG

shows ST elevation in leads II, III and aVF.

Aortic dissection

Aortic dissection is associated with the 3rd trimester of pregnancy,

connective tissue disorders (Marfan's, Ehlers- Danlos) and bicuspid

valve. Patients may complain of a tearing chest pain or syncope.

Clinically they may be hypertensive. The right coronary artery may

become involved in the dissection, causing myocardial infarct in up to

2% cases (hence ST elevation in the inferior leads). An aortic regurgitant

murmur may be auscultated.

17. A 28 year old woman, who is 18 weeks pregnant, presents with sudden

chest pain. Her blood pressure is 150/70 mmHg, saturations are 92% on

15L oxygen and her heart rate is 130 bpm. There are no murmurs and her

chest is clear. There is signs of thrombophlebitis in the left leg.

Pulmonary embolism

Chest pain, hypoxia and clear chest on auscultation in pregnancy should

lead to a high suspicion of pulmonary embolism.

Theme question in September 2011

exam Pregnant women can decompensate

rapidly from cardiac compromise.

Chest pain in pregnancy

Aortic dissection

Predisposing factors in pregnancy are hypertension, congenital heart disease

and Marfan's syndrome

Mainly Stanford type A dissections

Sudden tearing chest pain, transient syncope

Patient may be cold and clammy, hypertensive and have an aortic

regurgitation murmur

Involvement of the right coronary artery may cause inferior myocardial

infarction

Surgical management

Gestational

timeframe

Management

< 28/40 Aortic repair with the fetus kept in utero

28-32/40 Dependent on fetal condition

> 32/40 Primary Cesarean section followed by aortic repair at the same

operation

Mitral stenosis

Most cases associated with rheumatic heart disease

Becoming less common in British women; suspect in Immigrant women

Commonest cardiac condition in pregnancy

Commonly associated with mortality

Valve surgery; balloon valvuloplasty preferable

Pulmonary embolism

Leading cause of mortality in pregnancy

Half dose scintigraphy; CT chest if underlying lung disease should aid

diagnosis

Treatment with low molecular weight heparin throughout pregnancy and 4-6

weeks after childbirth

Warfarin is contra indicated in pregnancy

References 1. Bates S.M. and Ginsberg J.S. How we manage venous thromboembolism during

pregnancy. Blood 2002 (100): 3470-3478.

2. Scarsbrook A.Fand Gleeson V. Investigating suspected pulmonary embolism in

pregnancy. BMJ 2003 (326) : 1135 doi: 10.1136/bmj.7399.1135.

3. Morley C. A. and Lim B. A. Lesson of the Week: The risks of delay in diagnosis of

breathlessness in pregnancy. BMJ 1995 (311) : 1083. A 67 year old man is investigated for biliary colic and a 4.8 cm abdominal aortic aneurysm is

identified. Which of the following statements relating to this condition is untrue?

A. The wall will be composed of dense fibrous tissue only

B. The majority are located inferior to the renal arteries

C. They occur most often in current or former smokers

D. He should initially be managed by a process of active surveillance

E. Aortoduodenal fistula is a recognised complication following repair.

They are true aneurysms and have all 3 layers of arterial wall.

Abdominal aorta aneurysm

Abdominal aortic aneurysms are a common problem in vascular surgery. They may occur as either true or false aneurysm. With the former all 3 layers of the

arterial wall are involved, in the latter only a single layer of fibrous tissue forms the aneurysm wall.

True abdominal aortic aneurysms have an approximate incidence of 0.06 per 1000 people. They are commonest in elderly men and for this reason the UK is now introducing the aneurysm screening program with the aim of performing an abdominal aortic ultrasound measurement in all men aged 65 years.

Causes

Several different groups of patients suffer from aneurysmal disease. The commonest group is those who suffer from standard arterial disease, i.e. Those

who are hypertensive, have diabetes and have been or are smokers. Other patients such as those suffering from connective tissue diseases such as

Marfan's may also develop aneurysms. In patients with abdominal aortic aneurysms the extracellular matrix becomes disrupted with a change in the balance of collagen and elastic fibres.

Management

Most abdominal aortic aneurysms are an incidental finding. Symptoms most often relate to rupture or impending rupture. 20% rupture anteriorly into the peritoneal cavity. Very poor prognosis. 80% rupture posteriorly into the retroperitoneal space The risk of rupture is related to aneurysm size, only 2% of aneurysms measuring less

than 4cm in diameter will rupture over a 5 year period. This contrasts with 75% of aneurysms measuring over 7cm in diameter.

This is well explained by La Places' law which relates size to transmural pressure. For this reason most vascular surgeons will subject patients with an aneurysm size of

5cm or greater to CT scanning of the chest, abdomen and pelvis with the aim of

delineating anatomy and planning treatment. Depending upon co-morbidities, surgery is generally offered once the aneurysm is between 5.5cm and 6cm.

A CT reconstruction showing an infrarenal abdominal aortic aneurysm. The walls of the sac

are calcified which may facilitate identification on plain x-rays

Image sourced from Wikipedia

Indications for surgery

Symptomatic aneurysms (80% annual mortality if untreated) Increasing size above 5.5cm if asymptomatic Rupture (100% mortality without surgery)

Surgical procedures

Abdominal aortic aneurysm repair

Procedure:

GA

Invasive monitoring (A-line, CVP, catheter)

Incision: Midline or transverse

Bowel and distal duodenum mobilised to access aorta.

Aneurysm neck and base dissected out and prepared for cross clamp

Systemic heparinisation

Cross clamp (distal first)

Longitudinal aortotomy

Atherectomy

Deal with back bleeding from lumbar vessels and inferior mesenteric artery

Insert graft either tube or bifurcated depending upon anatomy

Suture using Prolene (3/0 for proximal , distal anastomosis suture varies according to site)

Clamps off: End tidal CO2 will rise owing to effects of reperfusion, at this point major risk of

myocardial events.

Haemostasis

Closure of aneurysm sac to minimise risk of aorto-enteric fistula

Closure: Loop 1 PDS or Prolene to abdominal wall

Skin- surgeons preference

Post operatively:

ITU (Almost all)

Greatest risk of complications following emergency repair

Complications: Embolic- gut and foot infarcts

Cardiac - owing to premorbid states, reperfusion injury and effects of cross clamp

Wound problems

Later risks related to graft- infection and aorto-enteric fistula

Special groups

Supra renal AAA

These patients will require a supra renal clamp and this carries a far higher risk of

complications and risk of renal failure.

Ruptured AAA

Preoperatively the management depends upon haemodynamic instability. In patients with

symptoms of rupture (typical pain, haemodynamic compromise and risk factors) then ideally

prompt laparotomy. In those with vague symptoms and haemodynamic stability the ideal

test is CT scan to determine whether rupture has occurred or not. Most common rupture

site is retroperitoneal 80%. These patients will tend to develop retroperitoneal haematoma.

This can be disrupted if Bp is allowed to rise too high so aim for Bp 100mmHg.

Operative details are similar to elective repair although surgery should be swift, blind

rushing often makes the situation worse. Plunging vascular clamps blindly into a pool of

blood at the aneurysm neck carries the risk of injury the vena cava that these patients do not

withstand. Occasionally a supracoeliac clamp is needed to effect temporary control,

although leaving this applied for more than 20 minutes tends to carry a dismal outcome.

EVAR

Increasingly patients are now being offered endovascular aortic aneurysm repair. This is

undertaken by surgeons and radiologists working jointly. The morphology of the aneurysm is

important and not all are suitable. Here is a typical list of those features favoring a suitable

aneurysm:

Long neck Straight iliac vessels Healthy groin vessels

Clearly few AAA patients possess the above and compromise has to be made. The use of

fenestrated grafts can allow supra renal AAA to be treated.

Procedure:

GA

Radiology or theatre

Bilateral groin incisions

Common femoral artery dissected out

Heparinisation

Arteriotomy and insertion of guide wire

Dilation of arteriotomy

Insertion of EVAR Device

Once in satisfactory position it is released

Arteriotomy closed once check angiogram shows good position and no endoleak

Complications:

Endoleaks depending upon site are either Type I or 2. These may necessitate re-intervention

and all EVAR patients require follow up . Details are not needed for MRCS.

References

A reasonable review is provided by:

Sakalihasan N, Limet R, Defawe O. Abdominal aortic aneurysm. Lancet 2005 (365):1577-

1589

Which of the following statements in relation to the p53 tumour suppressor protein is

false?

A. It may induce necrosis of cells with non repairable DNA damage

B. It is affected in Li Fraumeni syndrome

C. It can induce DNA repair

D. It can halt the cell cycle

E. It may inhibit angiogenesis

When DNA cannot be repaired it will induce cellular apoptosis (not necrosis)

Genetics and surgical disease

Some of the more commonly occurring genetic conditions occurring in surgical

patients are presented here.

Li-Fraumeni Syndrome

Autosomal dominant

Consists of germline mutations to p53 tumour suppressor gene

High incidence of malignancies particularly sarcomas and leukaemias

Diagnosed when:

*Individual develops sarcoma under 45 years

*First degree relative diagnosed with any cancer below age 45 years and another

family member develops malignancy under 45 years or sarcoma at any age

BRCA 1 and 2

Carried on chromosome 17

Linked to developing breast cancer (60%) risk.

Associated risk of developing ovarian cancer (55% with BRCA 1 and 25%

with BRCA2).

Lynch Syndrome

Autosomal dominant

Develop colonic cancer and endometrial cancer at young age

80% of affected individuals with get colonic and or endometrial cancer

High risk individuals may be identified using the Amsterdam criteria

Amsterdam criteria Three or more family members with a confirmed diagnosis of colorectal cancer, one

of whom is a first degree (parent, child, sibling) relative of the other two.

Two successive affected generations.

One or more colon cancers diagnosed under age 50 years.

Familial adenomatous polyposis (FAP) has been excluded.

Gardners syndrome

Autosomal dominant familial colorectal polyposis

Multiple colonic polyps

Extra colonic diseases include: skull osteoma, thyroid cancer and epidermoid

cysts

Desmoid tumours are seen in 15%

Mutation of APC gene located on chromosome 5

Due to colonic polyps most patients will undergo colectomy to reduce risk of

colorectal cancer

Now considered a variant of familial adenomatous polyposis coli

Which of the following cell types is most likely to be identified in the wall of a fistula

in ano?

A. Squamous cells

B. Goblet cells

C. Columnar cells

D. Ciliated columnar cells

E. None of the above

A fistula is an abnormal connection between two epithelial lined surfaces, in the case

of a fistula in ano it will be lined by squamous cells.

Fistulas

A fistula is defined as an abnormal connection between two epithelial

surfaces.

There are many types ranging from Branchial fistulae in the neck to entero-

cutaneous fistulae abdominally.

In general surgical practice the abdominal cavity generates the majority and

most of these arise from diverticular disease and Crohn's.

As a general rule all fistulae will resolve spontaneously as long as there is no

distal obstruction. This is particularly true of intestinal fistulae.

The four types of fistulae are:

Enterocutaneous These link the intestine to the skin. They may be high (>1L) or low output (<1L)

depending upon source. Duodenal /jejunal fistulae will tend to produce high volume,

electrolyte rich secretions which can lead to severe excoriation of the skin. Colo-

cutaneous fistulae will tend to leak faeculent material. Both fistulae may result from

the spontaneous rupture of an abscess cavity onto the skin (such as following perianal

abscess drainage) or may occur as a result of iatrogenic input. In some cases it may

even be surgically desirable e.g. mucous fistula following sub total colectomy for

colitis.

Suspect if there is excess fluid in the drain.

Enteroenteric or Enterocolic This is a fistula that involves the large or small intestine. They may originate in a

similar manner to enterocutaneous fistulae. A particular problem with this fistula type

is that bacterial overgrowth may precipitate malabsorption syndromes. This may be

particularly serious in inflammatory bowel disease.

Enterovaginal Aetiology as above.

Enterovesicular This type of fistula goes to the bladder. These fistulas may result in frequent urinary

tract infections, or the passage of gas from the urethra during urination.

Management Some rules relating to fistula management:

They will heal provided there is no underlying inflammatory bowel disease

and no distal obstruction, so conservative measures may be the best option

Where there is skin involvement, protect the overlying skin, often using a well

fitted stoma bag- skin damage is difficult to treat

A high output fistula may be rendered more easily managed by the use of

octreotide, this will tend to reduce the volume of pancreatic secretions.

Nutritional complications are common especially with high fistula (e.g. high

jejunal or duodenal) these may necessitate the use of TPN to provide

nutritional support together with the concomitant use of octreotide to reduce

volume and protect skin.

When managing perianal fistulae surgeons should avoid probing the fistula

where acute inflammation is present, this almost always worsens outcomes.

When perianal fistulae occur secondary to Crohn's disease the best

management option is often to drain acute sepsis and maintain that drainage

through the judicious use of setons whilst medical management is

implemented.

Always attempt to delineate the fistula anatomy, for abscesses and fistulae that

have an intra abdominal source the use of barium and CT studies should show

a track. For perianal fistulae surgeons should recall Goodsall's rule in relation

to internal and external openings.

22 year old man is referred to the surgical clinic. He has been complaining of varicose

veins for many years. On examination he has extensive varicosities of the right leg,

there are areas of marked port wine staining. The saphenofemoral junction is

competent on doppler assessment. The most likely underlying diagnosis is:

A. Deep vein thrombosis

B. Klippel-Trenaunay syndrome

C. Varicose veins due to sapheno-popliteal junction incompetence

D. Sturge - Weber syndrome

E. Angiosarcoma

Sturge - Weber syndrome is a an arteriovenous malformation affecting the face and

CNS, the peripheral vessels are not affected. Simple varicose veins should not

typically be associated with port wine staining, not should a DVT or angiosarcoma.

Klippel-Trenaunay syndrome

Klippel-Trenaunay-Weber syndrome generally affects a single extremity, although

cases of multiple affected limbs have been reported. The leg is the most common site

followed by the arms, the trunk, and rarely the head and the neck

Signs and symptoms The birth defect is diagnosed by the presence of a combination of these symptoms:

One or more distinctive port-wine stains with sharp borders

Varicose veins

Hypertrophy of bony and soft tissues, that may lead to local gigantism or

shrinking.

An improperly developed lymphatic system

In some cases, port-wine stains (capillary port wine type) may be absent. Such cases

are very rare and may be classified as "atypical Klippel-Trenaunay syndrome".

KTS can either affect blood vessels, lymph vessels, or both. The condition most

commonly presents with a mixture of the two. Those with venous involvement

experience increased pain and complications.

A 68 year old man presents with an ulcerated lesion on his right cheek. It is excised and on

histological assessment a squamous cell carcinoma is diagnosed. It measures 25mm in

diameter and is 4mm deep. Which of the following statements relating to this condition is

false?

A. In this particular case margins of at least 6mm are

required

B. Use of cryosurgery to treat this patients lesion

would have been unsafe

C. Use of radiotherapy to treat this lesion would have

been unsafe

D. This patients local recurrence rate may approach

15%

E. The disese usually spreads via lymphatics

Poor prognostic factors in SCC:

Size >20mm (local recurrence rate of up to 15%)

Depth greater than 4mm (risk of metastasis up to 30%)

This man has an SCC with significant risk of metastasis. Although cryotherapy may be used to

treat SCC it would be most unsafe in this setting as the lesion extends deeply. However,

radiotherapy is a safe treatment modality for SCC and may be used in selected cases. It is

unwise to use radiotherapy in areas prone to radionecrosis e.g. the nose.

Squamous cell carcinoma of the skin

Second most common skin malignancy Derived from epidermal keratinocytes Commonest in fair skinned individuals in sun exposed sites May occur in perianal and genital skin especially in association with Human

Papilloma Virus 16 and 18 infections.

Groups at high risk

Renal transplant and on immunosuppression

Individuals with HIV

Those who have received psoralen UVA therapy

Chronic wounds (Marjolins ulcer)

Xeroderma pigmentosum

Oculocutaneous albinism

Prognosis

Good Prognosis Poor prognosis

Well differentiated tumours Poorly differentiated tumours

<20mm diameter >20mm in diameter

<2mm deep >4mm deep

No associated diseases Immunosupression for whatever reason

Treatment

Surgical excision with 4mm margins if lesion <20mm in diameter. If tumour >20mm then

margins should be 6mm.

Squamous cell carcinomas of the skin typically arise in areas of sun exposure as shown here

Image sourced from Wikipedia

References

The British Association of Dermatology provides guidelines for the diagnosis and treatment

of SCC.

http://www. bad.org.uk

A 23 year old man presents with weight loss fatigue and lymphadenopathy. He is

diagnosed with tuberculosis. Which of the following processes most closely matches

the underlying pathological process?

A. Type 1 hypersensitivity reaction

B. Type 2 hypersensitivity reaction

C. Type 3 hypersensitivity reaction

D. Type 4 hypersensitivity reaction

E. None of the above

Granulomas (which occur in tuberculosis) are a feature of Type 4 hypersensitivity

reactions.

Hypersensitivity reactions

The Gell and Coombs classification divides hypersensitivity reactions into 4 types

Type I Type II Type III Type IV

Description Anaphylactic Cytotoxic Immune

complex

Delayed type

Mediator IgE IgG, IgM IgG, IgM T-cells

Antigen Exogenous Cell surface Soluble Tissues

Response

time

Minutes Hours Hours 2-3 days

Examples Asthma

Hay fever

Autoimmune

haemolytic anaemia

Pemphigus

Goodpasture's

Serum

sickness

SLE

Aspergillosis

Graft versus host

disease

Contact

dermatitis

A 73 year old man undergoes an emergency amputation for severe lower limb sepsis

and gangrene. Post operatively he develops disseminated intravascular coagulation.

Which of the following clotting factors will be most rapidly consumed in this

process?

A. Factor V and VIII

B. Factor I

C. Factor I and III

D. Factor III and VII

E. Factor VI and VIII

D-I-S-S-E-M-I-N-A-T-E-D

D-Dx: D dimer

I-Immune complexes

S-Snakebite, shock, heatstroke

S-SLE

E-Eclampsia, HELLP syndrome

M-Massive tissue damage

I-Infections: viral and bacterial

N-Neoplasms

A-Acute promyelocytic leukemia

T-Tumor products: Tissue Factor (TF) and TF-like factors released by carcinomas of

pancreas, prostate, lung,

colon, stomach

E-Endotoxins (bacterial)

D-Dead fetus (retained)

DIC Will tend to consume factors five and eight intially (and platelets).

Disseminated intravascular coagulation

Simultaneous coagulation and haemorrhage caused by initially formation of thrombi

which consume clotting factors (factors 5,8) and platelets, ultimately leading to

bleeding

Causes include:

Infection

Malignancy

Trauma e.g. major surgery, burns, shock, dissecting aortic aneurysm

Liver disease

Obstetric complications

Key points

Clinically bleeding is usually a dominant feature, bruising, ischaemia and

organ failure

Blood tests: prolonged clotting times, thrombocytopenia, decreased

fibrinogen, increased fibrinogen degradation products

Treat the underlying cause and supportive management

A 63 year old Japanese man presents with epigastric discomfort and iron deficiency

anaemia. He undergoes an upper GI endoscopy, where the following appearances are found:

Image sourced from Wikipedia

The most likely diagnosis is:

A. Squamous cell carcinoma

B. Linitis plastica

C. Leiomyosarcoma

D. Gastric varices

E. None of the above

During upper GI endoscopy, a linitis plastica lesion may prevent gastric distension.

Linitis plastica produces a diffuse infiltrating lesion, the stomach is fibrotic and rigid and will

not typically distend. This may be described as a 'leather bottle stomach'. Diagnosis is made

with a combination of pathology examination with endoscopy, radiological or surgical

assessment. Pathologically signet-ring cell proliferation occurs.

At the present time clinical images do not form part of the MRCS Part A exam

Gastric cancer

Overview

There are 700,000 new cases of gastric cancer worldwide each year. It is most common in

Japan and less common in western countries. It is more common in men and incidence rises

with increasing age. The exact cause of many sporadic cancer is not known, however,

familial cases do occur in HNPCC families. In addition, smoking and smoked or preserved

foods increase the risk. Japanese migrants retain their increased risk (decreased in

subsequent generations). The distribution of the disease in western countries is changing

towards a more proximal location (perhaps due to rising obesity).

Pathology

There is some evidence of support a stepwise progression of the disease through intestinal

metaplasia progressing to atrophic gastritis and subsequent dysplasia, through to cancer.

The favoured staging system is TNM. The risk of lymph node involvement is related to size

and depth of invasion; early cancers confined to submucosa have a 20% incidence of lymph

node metastasis. Tumours of the gastro-oesophageal junction are classified as below:

Type

1

True oesophageal cancers and may be associated with Barrett's oesophagus.

Type

2

Carcinoma of the cardia, arising from cardiac type epithelium

or short segments with intestinal metaplasia at the oesophagogastric junction.

Type

3

Sub cardial cancers that spread across the junction. Involve similar nodal stations to

gastric cancer.

Groups for close endoscopic monitoring

Intestinal metaplasia of columnar type Atrophic gastritis Low to medium grade dysplasia Patients who have previously undergone resections for benign peptic ulcer disease

(except highly selective vagotomy).

Referral to endoscopy

Patients of any age with

dyspepsia and any of the

following

Patients without

dyspepsia

Worsening dyspepsia

Chronic gastrointestinal bleeding Dysphagia Barretts oesophagus

Dysphagia Unexplained abdominal

pain or weight loss

Intestinal metaplasia

Weight loss Vomiting Dysplasia

Iron deficiency anaemia Upper abdominal mass Atrophic gastritis

Upper abdominal mass Jaundice Patient aged over 55 years with

unexplained or persistent

dyspepsia

Upper GI endoscopy performed for dyspepsia. The addition of dye spraying (as shown in the

bottom right) may facilitate identification of smaller tumours

Image sourced from Wikipedia

Staging

CT scanning of the chest abdomen and pelvis is the routine first line staging investigation in most centres.

Laparoscopy to identify occult peritoneal disease PET CT (particularly for junctional tumours)

Treatment

Proximally sited disease greater than 5-10cm from the OG junction may be treated by sub total gastrectomy

Total gastrectomy if tumour is <5cm from OG junction For type 2 junctional tumours (extending into oesophagus) oesophagogastrectomy is

usual Endoscopic sub mucosal resection may play a role in early gastric cancer confined to

the mucosa and perhaps the sub mucosa (this is debated) Lymphadenectomy should be performed. A D2 lymphadenectomy is widely

advocated by the Japanese, the survival advantages of extended lymphadenectomy have been debated. However, the overall recommendation is that a D2 nodal dissection be undertaken.

Most patients will receive chemotherapy either pre or post operatively.

Prognosis

UK Data

Disease extent Percentage 5 year survival

All RO resections 54%

Early gastric cancer 91%

Stage 1 87%

Stage 2 65%

Stage 3 18%

Operative procedure

Total Gastrectomy , lymphadenectomy and Roux en Y anastomosis

General anaesthesia

Prophylactic intravenous antibiotics

Incision: Rooftop.

Perform a thorough laparotomy to identify any occult disease.

Mobilise the left lobe of the liver off the diaphragm and place a large pack over it. Insert a

large self retaining retractor e.g. omnitract or Balfour (take time with this, the set up should

be perfect). Pack the small bowel away.

Begin by mobilising the omentum off the transverse colon.

Proceed to detach the short gastric vessels.

Mobilise the pylorus and divide it at least 2cm distally using a linear cutter stapling device.

Continue the dissection into the lesser sac taking the lesser omentum and left gastric artery

flush at its origin.

The lymph nodes should be removed en bloc with the specimen where possible.

Place 2 stay sutures either side of the distal oesophagus. Ask the anaesthetist to pull back on

the nasogastric tube. Divide the distal oesophagus and remove the stomach.

The oesphago jejunal anastomosis should be constructed. Identify the DJ flexure and bring a

loop of jejunum up to the oesophagus (to check it will reach). Divide the jejunum at this

point. Bring the divided jejunum either retrocolic or antecolic to the oesophagus.

Anastamose the oesophagus to the jejunum, using either interrupted 3/0 vicryl or a stapling

device. Then create the remainder of the Roux en Y reconstruction distally.

Place a jejunostomy feeding tube.

Wash out the abdomen and insert drains (usually the anastomosis and duodenal stump).

Help the anaesthetist insert the nasogastric tube (carefully!)

Close the abdomen and skin.

Enteral feeding may commence on the first post-operative day. However, most surgeons will

leave patients on free NG drainage for several days and keep them nil by mouth.

Rate question:

1

2

3

4

5

Theme: Genetics and cancer

A. Multiple endocrine neoplasia type I

B. Multiple endocrine neoplasia type II

C. Gardner's syndrome

D. Lynch Syndrome

E. Kartagener's syndrome

F. Neurofibromatosis Type I

G. Neurofibromatosis Type II

Please select the most likely condition for the disease process described. Each option

may be used once, more than once or not at all.

26. A 40 year old male is found to have multiple colonic polyps during a

colonoscopy. He mentions that he has extra teeth.

You answered Kartagener's syndrome

The correct answer is Gardner's syndrome

Gardner's syndrome is an AD disorder, characterised by: Colonic polyps,

supernumerary teeth, jaw osteomas, congenital hypertrophy of retinal

pigment. osteomas of the skull, thyroid cancer, epidermoid cysts, fibromas

and sebaceous cysts.

27. A 10 year old boy who has learning difficulties, reports a difference in size

between his two legs.

You answered Gardner's syndrome

The correct answer is Neurofibromatosis Type I

Neurofibromatosis type I. A hallmark finding is a plexiform neurofibroma,

which is a sheet of neurofibromatosis tissue which encases major nerves. In

children this attracts extra blood circulation, which can accelerate growth of

the affected limb.

Other features include:

Schwannoma, > 6

Cafe au lait spots, axillary freckling, Lisch nodules, Optic glioma.

Meningiomas, Glioma, or Schwannoma.

28. A 22 year old is found to have bilateral acoustic neuromas.

You answered Neurofibromatosis Type I

The correct answer is Neurofibromatosis Type II

In NF2 bilateral acoustic neuromas are characteristic with a FH of

Neurofibroma,

Genetics and surgical disease

Some of the more commonly occurring genetic conditions occurring in surgical

patients are presented here.

Li-Fraumeni Syndrome

Autosomal dominant

Consists of germline mutations to p53 tumour suppressor gene

High incidence of malignancies particularly sarcomas and leukaemias

Diagnosed when:

*Individual develops sarcoma under 45 years

*First degree relative diagnosed with any cancer below age 45 years and another

family member develops malignancy under 45 years or sarcoma at any age

BRCA 1 and 2

Carried on chromosome 17

Linked to developing breast cancer (60%) risk.

Associated risk of developing ovarian cancer (55% with BRCA 1 and 25%

with BRCA2).

Lynch Syndrome

Autosomal dominant

Develop colonic cancer and endometrial cancer at young age

80% of affected individuals with get colonic and or endometrial cancer

High risk individuals may be identified using the Amsterdam criteria

Amsterdam criteria Three or more family members with a confirmed diagnosis of colorectal cancer, one

of whom is a first degree (parent, child, sibling) relative of the other two.

Two successive affected generations.

One or more colon cancers diagnosed under age 50 years.

Familial adenomatous polyposis (FAP) has been excluded.

Gardners syndrome

Autosomal dominant familial colorectal polyposis

Multiple colonic polyps

Extra colonic diseases include: skull osteoma, thyroid cancer and epidermoid

cysts

Desmoid tumours are seen in 15%

Mutation of APC gene located on chromosome 5

Due to colonic polyps most patients will undergo colectomy to reduce risk of

colorectal cancer

Now considered a variant of familial adenomatous polyposis coli

A 52 year old lady is referred to the breast clinic with symptoms of nipple discharge.

The discharge is usually thick and green. Which of the following statements relating

to the most likely underlying diagnosis is untrue?

A. The majority of patients will be smokers

B. Typically produces blood stained nipple

discharge

C. It is not associated with increased risk of breast

cancer

D. May result in development of mammary duct

fistula

E. May require total duct excision (Hadfields

operation) if it fails to resolve

Blood stained nipple discharge

should always be investigated.

Nipple fluid cytology is generally

unhelpful.

Discharge of this type of material is most likely to be due to duct ectasia. Green or

brown discharge is most common. Blood stained discharge should raise concern of

intraductal papilloma or cancer.

Nipple discharge

Causes of nipple discharge

Physiological During breast feeding

Galactorrhoea Commonest cause may be response to emotional events,

drugs such as histamine receptor anatagonists are also

implicated

Hyperprolactinaemia Commonest type of pituitary tumour

Microadenomas <1cm in diameter

Macroadenomas >1cm in diameter

Pressure on optic chiasm may cause bitemporal

hemianopia

Mammary duct

ectasia

Dilatation breast ducts.

Most common in enopausal women

Discharge typically thick and green in colour

Most common in smokers

Carcinoma Often blood stained

May be underlying mass or axillary lymphadenopathy

Intraductal

papilloma

Commoner in younger patients

May cause blood stained discharge

There is usually no palpable lump

Assessment of patients

Examine breast and determine whether there is mass lesion present

All mass lesions should undergo Triple assessment.

Reporting of investigations Where a mass lesion is suspected or investigations are requested these are prefixed

using a system that denotes the investigation type e.g. M for mammography, followed

by a numerical code as shown below:

1 No abnormality

2 Abnormality with benign features

3 Indeterminate probably benign

4 Indeterminate probably malignant

5 Malignant

Management of non malignant nipple discharge

Exclude endocrine disease

Nipple cytology unhelpful

Smoking cessation advice for duct ectasia

For duct ectasia with severe symptoms, total duct excision may be warrented.

Which of the following statements relating to gas gangrene is untrue?

A. There is necrosis with putrefaction

B. The causative pathogens may be detected on normal perineal skin

C. Treatment with low dose penicillin is indicated

D. Hyperbaric oxygen may be beneficial

E. Clostridium perfringens is a recognised cause

Rapid surgery and high dose antibiotics are indicated in the treatment of gas gangrene.

Meleney's Gangrene and Necrotising Fasciitis

Necrotising fasciitis

Advancing soft tissue infection associated with fascial necrosis Uncommon, but can be fatal In many cases there is underlying background immunosuppression e.g. Diabetes Caused by polymicrobial flora (aerobic and anaerobic) and MRSA is seen increasingly

in cases of necrotising fasciitis Streptococcus is the commonest organism in isolated pathogen infection (15%)

Meleneys gangrene

Meleneys is a similar principle but the infection is more superficially sited than necrotising fasciitis and often confined to the trunk

Fournier gangrene

Necrotising fasciitis affecting the perineum Polymicrobial with E.coli and Bacteroides acting in synergy

Clinical features

Fever

Pain

Cellulitis

Oedema

Induration

Numbness

Late findings

Purple/black skin discolouration

Blistering

Haemorrhagic bullae

Crepitus

Dirty Dishwater fluid discharge

Septic shock

A typical case of gas gangrene presenting late demonstrating some of the features described

above

Image sourced from gangrene" target="_blank" style =

"font-size:11px; color:#777;" >Wikipedia

Diagnosis is mainly clinical

Management

Radical surgical debridement forms the cornerstone of management Sterile dressing is used to dress the wound Reconstructive surgery is considered once the infection is completely treated

Reference

Necrotising fasciitis

Saiidy Hasham, Paolo Matteucci, Paul R W Stanley, Nick B Hart

BMJ 2005;330:830-833

A 30 year old man presents with abdominal distension, a laparotomy is performed, at

operation the abdomen is filled with a large amount of gelatinous fluid. What is the

most likely underlying diagnosis?

A. Infection with entamoeba histolytica

B. Pseudomxyoma peritonei

C. Metastatic colonic cancer

D. Chylous ascites

E. None of the above

Pseudomyxoma is associated with the deposition of large amounts of gelatinous

material. The appendix is the commonest organ or origin.

Pseudomyxoma Peritonei

Rare mucinous tumour

Most commonly arising from the appendix (other abdominal viscera are also

recognised as primary sites)

Incidence of 1-2/1,000,000 per year

The disease is characterised by the accumulation of large amounts of

mucinous material in the abdominal cavity

Treatment Is usually surgical and consists of cytoreductive surgery (and often peritonectomy c.f

Sugarbaker procedure) combined with intra peritoneal chemotherapy with mitomycin

C.

Survival is related to the quality of primary treatment and in Sugarbakers own centre

5 year survival rates of 75% have been quoted. Patients with disseminated

intraperitoneal malignancy from another source fare far worse.

In selected patients a second look laparotomy is advocated and some practice this

routinely.

A 30 year old man is suspected of having appendicitis. At operation an inflamed

Meckels diverticulum is found. Which of the following vessels is responsible for the

blood supply to a Meckels diverticulum?

A. Right colic artery

B. Vitelline artery

C. Appendicular artery

D. Internal iliac artery

E. External iliac artery

The vitelline arteries supply a Meckels these are usually derived from the ileal

arcades.

Meckel's diverticulum

Congenital abnormality resulting in incomplete obliteration of the vitello-

intestinal duct

Normally, in the foetus, there is an attachment between the vitello-intestinal

duct and the yolk sac.This disappears at 6 weeks gestation.

The tip is free in majority of cases.

Associated with enterocystomas, umbilical sinuses, and omphaloileal fistulas.

Arterial supply: omphalomesenteric artery.

2% of population, 2 inches long, 2 feet from the ileocaecal valve.

Typically lined by ileal mucosa but ectopic gastric mucosa can occur, with the

risk of peptic ulceration. Pancreatic and jejunal mucosa can also occur.

Clinical

Normally asymptomatic and an incidental finding.

Complications are the result of obstruction, ectopic tissue, or inflammation.

Removal if narrow neck or symptomatic. Options are between wedge excision

or formal small bowel resection and anastomosis.

A 23 year old man suffers a thermal injury to his left hand. It becomes red and

painful. Which of the following mediators are not involved in this process?

A. Histamine

B. Free radicals

C. Prostaglandins

D. Leukotrienes

E. Serotonin

Acute inflammation is not

mediated by free radicals

Chemical mediators facilitate the spread of inflammation into normal tissue

Chemical mediators include:

Lysosomal compounds

Chemokines such as serotinin and histamine (released by platelets and mast

cells)

Other enzyme cascades producing inflammatory mediators include:

Complement, kinin, coagulation system & fibrinolytic system

Acute inflammation

Inflammation is the reaction of the tissue elements to injury. Vascular changes occur,

resulting in the generation of a protein rich exudate. So long as the injury does not

totally destroy the existing tissue architecture, the episode may resolve with

restoration of original tissue architecture.

Vascular changes

Vasodilation occurs and persists throughout the inflammatory phase.

Inflammatory cells exit the circulation at the site of injury.

The equilibrium that balances Starlings forces within capillary beds is

disrupted and a protein rich exudate will form as the vessel walls also become

more permeable to proteins.

The high fibrinogen content of the fluid may form a fibrin clot. This has

several important immunomodulatory functions.

Sequelae

Resolution Typically occurs with minimal initial injury

Stimulus removed and normal tissue architecture

results

Organisation Delayed removed of exudate

Tissues undergo organisation and usually fibrosis

Suppuration Typically formation of an abscess or an empyema

Sequestration of large quantities of dead

neutrophils

Progression to chronic

inflammation

Coupled inflammatory and reparative activities

Usually occurs when initial infection or

suppuration has been inadequately managed

Causes

Microbacterial infections e.g. Viruses, exotoxins or endotoxins released by

bacteria

Chemical agents

Physical agents e.g. Trauma

Hypersensitivity reactions

Tissue necrosis

Presence of neutrophil polymorphs is a histological diagnostic feature of acute

inflammation

Which of the following associations are incorrect?

A. Afro-Caribbean skin and keloid scarring

B. Extensive third degree burns and wound contraction

C. Chemotherapy and dehisence of healed wounds

D. Poor healing at the site of previous radiotherapy

E. Zinc deficiency and delayed healing

Wound healing

Surgical wounds are either incisional or excisional and either clean, clean contaminated or

dirty. Although the stages of wound healing are broadly similar their contributions will vary

according to the wound type.

The main stages of wound healing include:

Haemostasis

Vasospasm in adjacent vessels, platelet plug formation and generation of fibrin rich clot.

Inflammation

Neutrophils migrate into wound (function impaired in diabetes). Growth factors released, including basic fibroblast growth factor and vascular

endothelial growth factor. Fibroblasts replicate within the adjacent matrix and migrate into wound. Macrophages and fibroblasts couple matrix regeneration and clot substitution.

Regeneration

Platelet derived growth factor and transformation growth factors stimulate fibroblasts and epithelial cells.

Fibroblasts produce a collagen network. Angiogenesis occurs and wound resembles granulation tissue.

Remodeling

Longest phase of the healing process and may last up to one year (or longer). During this phase fibroblasts become differentiated (myofibroblasts) and these

facilitate wound contraction. Collagen fibres are remodeled. Microvessels regress leaving a pale scar.

The above description represents an idealised scenario. A number of diseases may distort

this process. It is obvious that one of the key events is the establishing well vascularised

tissue. At a local level angiogenesis occurs, but if arterial inflow and venous return are

compromised then healing may be impaired, or simply nor occur at all. The results of

vascular compromise are all too evidence in those with peripheral vascular disease or those

poorly constructed bowel anastomoses.

Conditions such as jaundice will impair fibroblast synthetic function and overall immunity

with a detrimental effect in most parts of healing.

Problems with scars:

Hypertrophic scars

Excessive amounts of collagen within a scar. Nodules may be present histologically

containing randomly arranged fibrils within and parallel fibres on the surface. The tissue

itself is confined to the extent of the wound itself and is usually the result of a full thickness

dermal injury. They may go on to develop contractures.

Image of hypertrophic scarring. Note that it remains confined to the boundaries of the

original wound:

Image sourced from Wikipedia

Keloid scars

Excessive amounts of collagen within a scar. Typically a keloid scar will pass beyond the

boundaries of the original injury. They do not contain nodules and may occur following even

trivial injury. They do not regress over time and may recur following removal.

Image of a keloid scar. Note the extension beyond the boundaries of the original incision:

Image sourced from Wikipedia

Drugs which impair wound healing:

Non steroidal anti inflammatory drugs Steroids Immunosupressive agents Anti neoplastic drugs

Closure

Delayed primary closure is the anatomically precise closure that is delayed for a few days but

before granulation tissue becomes macroscopically evident.

Secondary closure refers to either spontaneous closure or to surgical closure after

granulation tissue has formed.

A 23 year old man fractures his right tibia in a sporting accident. At which point in the

healing process is fracture callus most likely to be visible radiologically?

A. 1 day

B. 7 days

C. 8 weeks

D. 6 weeks

E. 3 weeks

Fracture callus is composed of fibroblasts and chondroblasts and the synthesis of

fibrocartilage. It is typically visible on radiographs approximately 3 weeks following

injury. If delayed then there may be risk of non union.

Fracture healing

Bone fracture - Bleeding vessels in the bone and periosteum

- Clot and haematoma formation

- The clot organises over a week (improved structure and collagen)

- The periosteum contains osteoblasts which produce new bone

- Mesenchymal cells produce cartilage (fibrocartilage and hyaline cartilage) in the soft

tissue around the fracture

- Connective tissue + hyaline cartilage = callus

- As the new bone approaches the new cartilage, endochondral ossification occurs to

bridge the gap

- Trabecular bone forms

- Trabecular bone is resorbed by osteoclasts and replaced with compact bone

Factors Affecting Fracture Healing

Age

Malnutrition

Bone disorders: osteoporosis

Systemic disorders: diabetes, Marfan's syndrome and Ehlers-Danlos syndrome

cause abnormal musculoskeletal healing.

Drugs: steroids, non steroidal anti inflammatory agents.

Type of bone: Cancellous (spongy) bone fractures are usually more stable,

involve greater surface areas, and have a better blood supply than cortical

(compact) bone fractures.

Degree of Trauma: The more extensive the injury to bone and surrounding soft

tissue, the poorer the outcome.

Vascular Injury: Especially the femoral head, talus, and scaphoid bones.

Degree of Immobilization

Intra-articular Fractures: These fractures communicate with synovial fluid,

which contains collagenases that retard bone healing.

Separation of Bone Ends: Normal apposition of fracture fragments is needed

for union to occur. Inadequate reduction, excessive traction, or interposition of

soft tissue will prevent healing.

Infection

Of the options below, which does not cause lymphadenopathy?

A. Kawasaki disease

B. Systemic Lupus Erthematosus

C. Phenytoin

D. Hydrallazine

E. Amiodarone

Lymphadenopathy

Lymphadenopathy in the neck, axillae, groins and abdomen

Need to note: solitary/multiple, defined/indistinct, hard/rubbery/soft,

tender/painless

Causes of lymphadenopathy

Mnemonic: Hodgkins disease

H aematological: Hodgkins lymphoma, NHL, Leukaemia

O ncological: metastases

D ermatopathic lympadenitis

G aucher's disease

K awasaki disease

I nfections: TB, glandular fever, Syphilis

N iemann Pick disease

S erum sickness

D rug reaction (phenytoin)

I mmunological (SLE)

S arcoidosis

E ndocrinological (Hyperthyroidism)

A ngioimmunoplastic lymphadenopathy

S LE

E osinophilic granulomatosis A 23 year old man is reviewed on the ward 10 days following a laparotomy. The wound is

inspected and is healing well. Which of the following processes is least likely to be occurring

in the wound at this stage?

A. Angiogenesis

B. Synthesis of collagen

C. Necrosis of fibroblasts

D. Secretion of matrix metalloproteinases by

fibroblasts

E. Proliferation of fibroblasts

Vasculogenesis vs Angiogenesis

Vascu is new. Angi is pre

Vasculogenesis is new vessels developing

in situ from existing mesenchyme.

Angiogenesis is vessels develop from

sprouting off pre-existing arteries.

Fibroblasts are an important cell type in healing wounds. They typically proliferate in the

early phases of wound healing. They release matrix metalloproteinases and these facilitate

in the remodelling of the matrix within the healing wound. Necrosis in a healing wound

would be unusual as wounds will tend to show clinical evidence of angiognesis by this time.

Wound healing

Surgical wounds are either incisional or excisional and either clean, clean contaminated or

dirty. Although the stages of wound healing are broadly similar their contributions will vary

according to the wound type.

The main stages of wound healing include:

Haemostasis

Vasospasm in adjacent vessels, platelet plug formation and generation of fibrin rich clot.

Inflammation

Neutrophils migrate into wound (function impaired in diabetes). Growth factors released, including basic fibroblast growth factor and vascular

endothelial growth factor. Fibroblasts replicate within the adjacent matrix and migrate into wound. Macrophages and fibroblasts couple matrix regeneration and clot substitution.

Regeneration

Platelet derived growth factor and transformation growth factors stimulate fibroblasts and epithelial cells.

Fibroblasts produce a collagen network. Angiogenesis occurs and wound resembles granulation tissue.

Remodeling

Longest phase of the healing process and may last up to one year (or longer). During this phase fibroblasts become differentiated (myofibroblasts) and these

facilitate wound contraction. Collagen fibres are remodeled. Microvessels regress leaving a pale scar.

The above description represents an idealised scenario. A number of diseases may distort

this process. It is obvious that one of the key events is the establishing well vascularised

tissue. At a local level angiogenesis occurs, but if arterial inflow and venous return are

compromised then healing may be impaired, or simply nor occur at all. The results of

vascular compromise are all too evidence in those with peripheral vascular disease or those

poorly constructed bowel anastomoses.

Conditions such as jaundice will impair fibroblast synthetic function and overall immunity

with a detrimental effect in most parts of healing.

Problems with scars:

Hypertrophic scars

Excessive amounts of collagen within a scar. Nodules may be present histologically

containing randomly arranged fibrils within and parallel fibres on the surface. The tissue

itself is confined to the extent of the wound itself and is usually the result of a full thickness

dermal injury. They may go on to develop contractures.

Image of hypertrophic scarring. Note that it remains confined to the boundaries of the

original wound:

Image sourced from Wikipedia

Keloid scars

Excessive amounts of collagen within a scar. Typically a keloid scar will pass beyond the

boundaries of the original injury. They do not contain nodules and may occur following even

trivial injury. They do not regress over time and may recur following removal.

Image of a keloid scar. Note the extension beyond the boundaries of the original incision:

Image sourced from Wikipedia

Drugs which impair wound healing:

Non steroidal anti inflammatory drugs Steroids Immunosupressive agents Anti neoplastic drugs

Closure

Delayed primary closure is the anatomically precise closure that is delayed for a few days but

before granulation tissue becomes macroscopically evident.

Secondary closure refers to either spontaneous closure or to surgical closure after

granulation tissue has formed.

A 25 year old women presents with a slowly enlarging mass on the side of the face.

Clinical examination demonstrates that the mass is located in the tail of the parotid

gland. There is no evidence of facial nerve involvement. What is the most likely

cause?

A. Sialolithiasis

B. Adenocarcinoma

C. Warthins tumour

D. Oncocytoma

E. Pleomorphic adenoma

Pleomorphic adenomas are the commonest tumours of the parotid gland and are often

slow growing, smooth and mobile. Warthins tumours are typically found in elderly

males and are composed of multiple cysts and solid components consisting of

lymphoid tissue. Warthins tumours are most often found in the tail of the parotid

gland, but not in 25 year old females, where a pleomorphic adenoma remains the most

likely lesion.

Parotid gland clinical

Causes of bilateral parotid enlargement

Mumps: Associated with meningoencephalitis, pancreatitis, orchitis, or

deafness

Parotitis

Sialectasis - especially if related to eating

Sjogren's syndrome: dry eyes or mouth, connective tissue disease

Sarcoidosis

Tuberculosis

Alcoholism

Myxoedema

Cushing's disease

Diabetes/insulin resistance

Liver cirrhosis

Gout

Bulimia nervosa

Drugs

Severe dehydration

Malnutrition

Causes of unilateral parotid enlargement

Salivary calculus

Tumour

Parotid gland tumours

Pleomorphic adenomas are the most common.

Incisional biopsy of parotid masses is not recommended, so superficial

parotidectomy is the usual procedure of choice.

Signs of facial nerve palsy and a parotid mass should raise suspicion of

malignancy.

Warthins tumours are relatively benign lesions that are slow growing and

occur most commonly in elderly male smokers.

Adenoid cystic carcinoma have a tendency for perineural invasion.

Beta-naphthalamine is associated with which of the following malignancies?

A. Lung cancer

B. Bowel cancer

C. Bladder cancer

D. Liver cancer

E. Renal cancer

Beta-naphthalamine is used in the rubber industry.

The following factors are associated with the development of bladder cancer:

smoking

occupational: aniline dyes used in printing and textile industry, rubber

manufacture

schistosomiasis

drugs: cyclophosphamide

Occupational cancers

Occupational cancers accounted for 5.3% cancer deaths in 2005.

In men the main cancers include:

Mesothelioma

Bladder cancer

Non melanoma skin cancer

Lung cancer

Sino nasal cancer

Occupations with high levels of occupational tumours include:

Construction industry

Working with coal tar and pitch

Mining

Metalworkers

Working with asbestos (accounts for 98% of all mesotheliomas)

Working in rubber industry

Shift work has been linked to breast cancer in women (Health and safety executive

report RR595).

The latency between exposure and disease is typically 15 years for solid tumours and

20 for leukaemia.

Many occupational cancers are otherwise rare. For example sino nasal cancer is an

uncommon tumour, 50% will be SCC. They are linked to conditions such as wood

dust exposure and unlike lung cancer is not strongly linked to cigarette smoking.

Another typical occupational tumour is angiosarcoma of the liver which is linked to

working with vinyl chloride. Again in the non occupational context this is an

extremely rare sporadic tumour.

A 56 year old man with Wilsons disease presents with right upper quadrant

discomfort. An ultrasound scan is performed and this demonstrates a mass lesion in

the right lobe of the liver. What is the most appropriate method of establishing the

underlying diagnosis?

A. PET CT scan

B. Ultrasound guided biopsy

C. Measurement of serum alpha feto protein

D. MRI scan of the liver

E. CT scan of the liver

High AFP + chronic liver

inflammation = Hepatocellular

carcinoma.

This is likely to be a hepatocellulcar carcinoma. Diagnosis is usually made by AFP

measurement (with further imaging depending on the result). Biopsy should not be

performed as it may seed the tumour. Chronic liver diseases such as Wilsons disease

(Hepato-lenticular degeneration) increase the risk.

Liver tumours

Primary liver tumours The most common primary tumours are cholangiocarcinoma and hepatocellular

carcinoma. Overall metastatic disease accounts for 95% of all liver malignancies

making the primary liver tumours comparatively rare.

Primary liver tumours include:

Cholangiocarcinoma

Hepatocellular carcinoma

Hepatoblastoma

Sarcomas (Rare)

Lymphomas

Carcinoids (most often secondary although primary may occur)

Hepatocellular carcinoma These account for the bulk of primary liver tumours (75% cases). Its worldwide

incidence reflects its propensity to occur on a background of chronic inflammatory

activity. Most cases arise in cirrhotic livers or those with chronic hepatitis B infection,

especially where viral replication is actively occurring. In the UK it accounts for less

than 5% of all cancers, although in parts of Asia its incidence is 100 per 100,000.

The majority of patients (80%) present with existing liver cirrhosis, with a mass

discovered on screening ultrasound.

Diagnosis

CT/ MRI (usually both) are the imaging modalities of choice

a-fetoprotein is elevated in almost all cases

Biopsy should be avoided as it seeds tumours cells through a resection plane.

In cases of diagnostic doubt serial CT and aFP measurements are the preferred

strategy.

Treatment

Patients should be staged with liver MRI and chest, abdomen and pelvic CT

scan.

The testis should be examined in males (testicular tumours may cause raised

AFP). PET CT may be used to identify occult nodal disease.

Surgical resection is the mainstay of treatment in operable cases. In patients

with a small primary tumour in a cirrhotic liver whose primary disease process

is controlled, consideration may be given to primary whole liver resection and

transplantation.

Liver resections are an option but since most cases occur in an already

diseased liver the operative risks and post-operative hepatic dysfunction are

far greater than is seen following metastectomy.

These tumours are not particularly chemo or radiosensitive however, both may

be used in a palliative setting. Tumour ablation is a more popular strategy.

Survival Poor, overall survival is 15% at 5 years.

Cholangiocarcinoma This is the second most common type of primary liver malignancy. As its name

suggests these tumours arise in the bile ducts. Up to 80% of tumours arise in the extra

hepatic biliary tree. Most patients present with jaundice and by this stage the majority

will have disease that is not resectable.

Primary scelerosing cholangitis is the main risk factor. In deprived countries typhoid

and liver flukes are also major risk factors.

Diagnosis

Patients will typically have an obstructive picture on liver function tests.

CA 19-9, CEA and CA 125 are often elevated

CT/ MRI and MRCP are the imaging methods of choice.

Treatment

Surgical resection offers the best chance of cure. Local invasion of peri hilar

tumours is a particular problem and this coupled with lobar atrophy will often

contra indicate surgical resection.

Palliation of jaundice is important, although metallic stents should be avoided

in those considered for resection.

Survival Is poor, approximately 15% 5 year survival.

A 55 year old man has suffered from reflux oesophagitis for many years. During a

recent endoscopy a biopsy is taken from the distal oesophagus. The histopathology

report indicates that cells are identified with features of coarse chromatin and

abnormal mitoses. The cells are confined to the superficial epithelial layer only.

Which of the following accounts for this process?

A. Metaplasia

B. Apoptosis

C. Autoimmune oesophagitis

D. Dysplasia

E. Infection with Helicobacter pylori

Dysplasia = pre

cancerous

Dysplasia tends to develop as a result of prolonged stimulation by precipitants.

Removal of these precipitants may possibly reverse these changes. Replacement of

differentiated cells wth another cell type describes metaplasia rather than dysplasia.

The absence of invasion distinguishes this from malignancy.

Dysplasia

Premalignant condition

Disordered growth and differentiation of calls

Alteration in size, shape, and organization of cells

Features increased abnormal cell growth (increased number of

mitoses/abnormal mitoses and cellular differentiation)

Underlying connective tissue is not invaded

Causes include smoking, Helicobacter pylori, Human papilloma virus

Main differences to metaplasia is that dysplasia is considered to be part of

carcinogenesis (pre cancerous) and is associated with a delay in maturation of

cells rather than differentiated cells replacing one another.

The absence of invasion differentiates dysplasia from invasive malignancy.

Severe dysplasia with foci of invasion are well recognised.

A 63 year old man has a history of claudication that has been present for many years.

He is recently evaluated in the clinic and a duplex scan shows that he has an 85%

stenosis of the superficial femoral artery. Two weeks later he presents with a 1 hour

history of severe pain in his leg. On examination he has absent pulses in the affected

limb and it is much cooler than the contra-lateral limb. Which process best accounts

for this presentation?

A. Thrombosis

B. Embolus

C. Atheroma growth

D. Sub intimal dissection

E. Anaemia

Theme from April 2012 Exam

In an existing lesion a complication such as thrombosis is more likely than embolus.

These patients should receive heparin and imaging with duplex scanning. Whilst an

early surgical bypass or intra-arterial thrombolysis may be indicated, an embolectomy

should not generally be performed as the lesion is not an embolus and the operation

therefore ineffective.

Claudication

Claudication is a condition in which patients develop pain in a limb during periods of

exercise. The underlying disorder is usually that of arterial insufficiency. Atheroma

develops in the arterial wall and once this occludes >50-75% of the lumenal diameter

the supply to metabolising tissues distally may become compromised. The typical

claudicant complains of calf pain that is worse on exercise and relieved by rest. This

typical description assumes that the SFA is the site of disease, more proximal disease

may present with other symptoms such as buttock claudication and impotence.

The history is usually a progressive one, patients presenting as an emergency with

severe pain, diminished sensation, pallor and absent pulses have critical limb

ischaemia. This may complicate claudication and usually indicates a plaque related

complication, such as thrombosis.

Risk factors Risk factors for claudication include smoking, diabetes and hyperlipidaemia.

Diagnosis Diagnostic work -up includes measurement of ankle- brachial pressure indices, duplex

scanning and formal angiography.

Treatment Those with long claudication distances, no ulceration or gangrene may be managed

conservatively. Patients with rest pain, ulceration or gangrene will almost always

require intervention. All patients should receive an antiplatelet agent and a statin

unless there are compelling contra-indications.

The following are true of carcinoid tumours except:

A. When present in the appendix tip and measure less than 2 cm have

an excellent prognosis

B. Even when metastatic disease is present it tends to follow a

protracted course

C. When present in the appendix body tend to present with carcinoid

syndrome even when liver metastases are not present

D. May be imaged using 5 HIAA radionucleotide scanning

E. Advanced appendiceal carcinoids may require right hemicolectomy.

Rule of thirds:

1/3 multiple

1/3 small bowel

1/3 metastasize

1/3 second

tumour

Liver metastases are necessary for the presence of carcinoid syndrome.

Carcinoid syndrome

Carcinoid tumours secrete serotonin

Originate in neuroendocrine cells mainly in the intestine (midgut-distal

ileum/appendix)

Can occur in the rectum, bronchi

Hormonal symptoms mainly occur when disease spreads outside the bowel

Clinical features - Onset: years

- Flushing face

- Palpitations

- Tricuspid stenosis causing dyspnoea

- Asthma

- Severe diarrhoea (secretory, persists despite fasting)

Investigation - 5-HIAA in a 24-hour urine collection

- Scintigraphy

- CT scan

Treatment

Octreotide

Surgical removal

During a difficult femoro-popliteal bypass operation the surgeon inadvertently places

a clamp across the femoral nerve. It remains there for most of the procedure. At the

end of the operation the nerve is inspected, it is in continuity but has evidence of

being crushed. Which of the following is most likely to occur over the following

months?

A. Wallerian degeneration

B. Rapid restoration of neuronal function because the axon itself is

intact

C. Normal but delayed neuronal transmission due to disruption of the

myelin

D. Absence of neuroma formation

E. None of the above

A neuronal injury such as this will result in Wallerian degeneration even though the

nerve remains in continuity. Neuromas may well form.

Nerve injury

There are 3 types of nerve injury:

Neuropraxia Nerve intact but electrical conduction is affected

Full recovery

Autonomic function preserved

Wallerian degeneration does not occur

Axonotmesis Axon is damaged and the myelin sheath is preserved. The

connective tissue framework is not affected.

Wallerian degeneration occurs.

Neurotmesis Disruption of the axon, myelin sheath and surrounding

connective tissue.

Wallerian degeneration occurs.

Wallerian Degeneration

Axonal degeneration distal to the site of injury.

Typically begins 24-36 hours following injury.

Axons are excitable prior to degeneration occurring.

Myelin sheath degenerates and is phagocytosed by tissue macrophages.

Nerve repair

Neuronal repair may only occur physiologically where nerves are in direct

contact. Where a large defect is present the process of nerve regeneration is

hampered and may not occur at all or result in the formation of a neuroma.

Where nerve regrowth occurs it typically occurs at a rate of 1mm per day.

A 38 year old lady who smokes heavily presents with recurrent episodes of

infection in the right breast. On examination she has an indurated area at the

lateral aspect of the nipple areaolar complex. Imaging shows no mass lesions.

What is the most likely diagnosis?

A. Duct ectasia

B. Periductal mastitis

C. Pagets disease of the nipple

D. Mondors disease of the breast

E. Radial scar

Periductal mastitis is common in smokers and may present with recurrent

infections. Treatment is with co-amoxyclav. Mondors disease of the breast is a

localised thrombophlebitis of a breast vein.

Duct ectasia

Duct ectasia is a dilatation and shortening of the terminal breast ducts within

3cm of the nipple. It is common and the incidence increases with age. It

typically presents with nipple retraction and occaisionally creamy nipple

discharge. It may be confused with periductal mastitis, which presents in

younger women, the vast majority of which are smokers. Periductal mastitis

typically presents with periareolar or sub areolar infections and may be

recurrent.

Patients with troublesome nipple discharge may be treated by

microdochectomy (if young) or total duct excision (if older). Which of the following statements relating to chronic inflammation is true?

A. Chronic inflammation is mainly secondary to acute inflammation

B. Neutrophils are the predominant cells involved

C. Growth factors are not involved in the process

D. Appendicitis is mainly a form of chronic inflammation

E. Fibrosis is a macroscopic feature

Macroscopic features include:

Ulcers Fibrosis Granulomatous process

It most commonly occurs as a primary event rather than as a result of acute inflammation.

Chronic inflammation

Overview

Chronic inflammation may occur secondary to acute inflammation.In most cases chronic

inflammation occurs as a primary process. These may be broadly viewed as being one of

three main processes:

Persisting infection with certain organisms such as Mycobacterium tuberculosis which results in delayed type hypersensitivity reactions and inflammation.

Prolonged exposure to non-biodegradable substances such as silica or suture materials which may induce an inflammatory response.

Autoimmune conditions involving antibodies formed against host antigens.

Acute vs. Chronic inflammation

Acute inflammation Chronic inflammation

Changes to existing vascular structure and increased

permeability of endothelial cells

Angiogenesis predominates

Infiltration of neutrophils Macrophages, plasma cells and

lymphocytes predominate

Process may resolve with:

Suppuration Complete resolution Abscess formation Progression to chronic inflammation Healing by fibrosis

Healing by fibrosis is the main result

Granulomatous inflammation

A granuloma consists of a microscopic aggregation of macrophages (with epithelial type

arrangement =epitheliod). Large giant cells may be found at the periphery of granulomas.

Mediators

Growth factors released by activated macrophages include agents such as interferon and

fibroblast growth factor (plus many more). Some of these such as interferons may have

systemic features resulting in systemic symptoms and signs, which may be present in

individuals with long standing chronic inflammation.

The finding of granulomas is pathognomonic of chronic inflammation, as illustrated in this

biopsy from a patient with colonic Crohns disease

Image sourced from Wikipedia

Theme: Paediatric ano-rectal disorders

A. Ulcerative colitis

B. Juvenile polyps

C. Haemorroids

D. Intussceception

E. Rectal cancer

F. Anal fissure

G. Arteriovenous malformation

Please select the most likely cause for the condition described. Each option may be

used once, more than once or not at all.

6. A 4 year old boy is brought to the clinic. He gives a history of difficult, painful

defecation with bright red rectal bleeding.

Anal fissure

Theme from April 2012 Exam

Painful rectal bleeding in this age group is typically due to a fissure. Treatment

should include stool softeners and lifestyle advice.

7. A 2 year old has a history of rectal bleeding. The parents notice that post

defecation, a cherry red lesion is present at the anal verge.

Juvenile polyps

Theme from September 2012 Exam

These lesions are usually hamartomas and this accounts for the colour of the

lesions. Although the lesions are not themselves malignant they serve as a

marker of an underlying polyposis disorder.

8. A 12 year old is brought to the colorectal clinic with a history of rectal

bleeding, altered bowel habit, weight loss and malaise. Abdominal

examination is normal.

Ulcerative colitis

The systemic features in the history are strongly suggestive of inflammatory

bowel disease rather than the other causes.

Paediatric proctology

Children may present with altered bowel habit and/ or rectal bleeding. Classical

haemorroidal disease is relatively rare in children. Painful bright red rectal bleeding is

much more common since constipation is a relatively common childhood disorder.

The hard stool causes a tear of the ano-rectal mucosa with subsequent fissure. The

pain from the fissure must be addressed promptly or the child will delay defecation

and this fissure will worsen.

Inflammatory bowel disease may present in a similar pattern in paediatric practice

with altered bowel habit (usually diarrhoea) and bleeding. Systemic features may be

present and investigation with an endoscopy may be required.

Children with intussceception usually present at a relatively young age and the history

is usually one of colicky abdominal pain, together with a mass on clinical

examination.The often cited red current jelly type stool is a rare but classical feature.

Juvenile polyps may occur as part of the familial polyposis coli syndromes. The

lesions, which are hamartomas, are often cherry red if they protrude externally

You review a 42-year-old woman 8 months following a renal transplant for focal

segmental glomerulosclerosis. She is on a combination of tacrolimus, mycophenolate,

and prednisolone. She has now presented with a five day history of feeling generally

unwell with jaundice, fatigue and arthralgia. On examination she has jaundice,

widespread lymphadenopathy and hepatomegaly. What is the most likely diagnosis?

A. Hepatitis C

B. Epstein-Barr virus

C. HIV

D. Hepatitis B

E. Cytomegalovirus

Post transplant complications

CMV: 4 weeks to 6 months post

transplant

EBV: post transplant lymphoproliferative

disease. > 6 months post transplant

Post transplant lymphoproliferative disorder is most commonly associated with

Epstein-Barr virus. It typically occurs 6 months post transplant and is associated with

high dose immunosupressant therapy. Remember cytomegalovirus presents within the

first 4 weeks to 6 months post transplant.

Renal transplant:HLA typing and graft failure

The human leucocyte antigen (HLA) system is the name given to the major

histocompatibility complex (MHC) in humans. It is coded for on chromosome 6.

Some basic points on the HLA system

Class 1 antigens include A, B and C. Class 2 antigens include DP,DQ and DR

When HLA matching for a renal transplant the relative importance of the HLA

antigens are as follows DR > B > A

Graft survival

1 year = 90%, 10 years = 60% for cadaveric transplants

1 year = 95%, 10 years = 70% for living-donor transplants

Post-op problems

ATN of graft

Vascular thrombosis

Urine leakage

UTI

Hyperacute acute rejection

Due to antibodies against donor HLA type 1 antigens

Rarely seen due to HLA matching

Acute graft failure (< 6 months)

Usually due to mismatched HLA

Other causes include cytomegalovirus infection

Management: give steroids, if resistant use monoclonal antibodies

Causes of chronic graft failure (> 6 months)

Chronic allograft nephropathy

Ureteric obstruction

Recurrence of original renal disease (MCGN > IgA > FSGS)

Which of the following processes facilitates phagocytosis?

A. Apoptosis

B. Opsonisation

C. Proteolysis

D. Angiogenesis

E. Necrosis

Theme from 2008 Exam

Opsonisation will facilitate phagocytosis. The micro-organism becomes coated with

antibody, C3b and certain acute phase proteins. The macrophages and neutrophils

have up regulation of phagocytic cell surface receptors in these circumstances, a

process mediated by pro-inflammatory cytokines. These cells then engulf the micro

organism.

Phagocytosis

Ingestion of pathogens or foreign materials by cells

First step is opsonisation whereby the organism is coated by antibody

Second step is adhesion to cell surface

Third step is pseudopodial extension to form a phagocytic vacuole

Lysosomes fuse with vacuole and degrade contents

Features which are evaluated for the grading of breast cancer include all the

following, except:

A. Tubule formation

B. Mitoses

C. Nuclear pleomorphism

D. Tumour necrosis

E. Coarse chromatin

The necrosis of a tumour may be suggestive of a high grade tumour which has

out grown its blood supply. However, the grading of breast cancer which

classically follows the Bloom -Richardson grading model will tend to favor

nuclear appearances (which include mitoses, coarse chromatin and

pleomorphism). Tubule formation is an important marker of the degree of

differentiation with formation of tubular structures being associated with well

differentiated tumours.

Tumour grading

Tumours may be graded according to their degree of differentiation, mitotic

activity and other features. Grade 1 tumours are the most differentiated and

grade 4 the least. The assessment is subjective, in most cases high grade

equates to poor prognosis, or at least rapid growth.

Tumours of glandular epithelium will tend to arrange themselves into acinar

type structures containing a central lumen. Well differentiated tumours may

show excellent acinar formation and poorly differentiated tumours simply

clumps of cells around a desmoplastic stroma. Sometimes tumours

demonstrate mucous production without evidence of acinar formation. Since

mucous production is evidence of a glandular function such tumours are often

termed mucinous adenocarcinoma.

Squamous cell tumours will typically produce structures resembling epithelial

cell components. Well differentiated tumours may also produce keratin

(depending upon tissue of origin). Which of the following is associated with poor wound healing?

A. Jaundice

B. Patients taking carbamazepine

C. General anaesthesia using thiopentone

D. General anaesthesia using ketamine

E. Multiple sclerosis

Mnemonic to remember factors

affecting wound healing: DID NOT HEAL

D iabetes

I nfection, irradiation

D rugs eg steroids, chemotherapy

N utritional deficiencies (vitamin A, C &

zinc, manganese), Neoplasia

O bject (foreign material)

T issue necrosis

H ypoxia

E xcess tension on wound

A nother wound

L ow temperature, Liver jaundice

Multiple sclerosis is associated with pressure sores, however the cellular healing process is

not affected.

Wound healing

Surgical wounds are either incisional or excisional and either clean, clean contaminated or

dirty. Although the stages of wound healing are broadly similar their contributions will vary

according to the wound type.

The main stages of wound healing include:

Haemostasis

Vasospasm in adjacent vessels, platelet plug formation and generation of fibrin rich clot.

Inflammation

Neutrophils migrate into wound (function impaired in diabetes). Growth factors released, including basic fibroblast growth factor and vascular

endothelial growth factor. Fibroblasts replicate within the adjacent matrix and migrate into wound. Macrophages and fibroblasts couple matrix regeneration and clot substitution.

Regeneration

Platelet derived growth factor and transformation growth factors stimulate fibroblasts and epithelial cells.

Fibroblasts produce a collagen network. Angiogenesis occurs and wound resembles granulation tissue.

Remodeling

Longest phase of the healing process and may last up to one year (or longer).

During this phase fibroblasts become differentiated (myofibroblasts) and these facilitate wound contraction.

Collagen fibres are remodeled. Microvessels regress leaving a pale scar.

The above description represents an idealised scenario. A number of diseases may distort

this process. It is obvious that one of the key events is the establishing well vascularised

tissue. At a local level angiogenesis occurs, but if arterial inflow and venous return are

compromised then healing may be impaired, or simply nor occur at all. The results of

vascular compromise are all too evidence in those with peripheral vascular disease or those

poorly constructed bowel anastomoses.

Conditions such as jaundice will impair fibroblast synthetic function and overall immunity

with a detrimental effect in most parts of healing.

Problems with scars:

Hypertrophic scars

Excessive amounts of collagen within a scar. Nodules may be present histologically

containing randomly arranged fibrils within and parallel fibres on the surface. The tissue

itself is confined to the extent of the wound itself and is usually the result of a full thickness

dermal injury. They may go on to develop contractures.

Image of hypertrophic scarring. Note that it remains confined to the boundaries of the

original wound:

Image sourced from Wikipedia

Keloid scars

Excessive amounts of collagen within a scar. Typically a keloid scar will pass beyond the

boundaries of the original injury. They do not contain nodules and may occur following even

trivial injury. They do not regress over time and may recur following removal.

Image of a keloid scar. Note the extension beyond the boundaries of the original incision:

Image sourced from Wikipedia

Drugs which impair wound healing:

Non steroidal anti inflammatory drugs Steroids Immunosupressive agents Anti neoplastic drugs

Closure

Delayed primary closure is the anatomically precise closure that is delayed for a few days but

before granulation tissue becomes macroscopically evident.

Secondary closure refers to either spontaneous closure or to surgical closure after

granulation tissue has formed.

A 55 year old man undergoes a colonoscopy and a colonic polyp is identified. It has a

lobular appearance and is located on a stalk in the sigmoid colon. Which of the

processes below best accounts for this disease?

A. Apoptosis

B. Metaplasia

C. Dysplasia

D. Calcification

E. Degeneration

Theme from April 2012 Exam

Most colonic polyps described above are adenomas. These may have associated

dysplasia. The more high grade the dysplasia the greater the level of clinical concern.

Colonic polyps

Colonic Polyps May occur in isolation of greater numbers as part of the polyposis syndromes. In FAP

greater than 100 polyps are typically present. The risk of malignancy in association

with adenomas is related to size and is the order of 10% in a 1cm adenoma. Isolated

adenomas seldom give risk of symptoms (unless large and distal). Distally sited

villous lesions may produce mucous and if very large electrolyte disturbances may

occur.

Follow up of colonic polyps

Low risk

1 or 2 adenomas <1cm. No follow up or re-colonoscopy at 5 years.

Moderate risk

3 or 4 small adenomas or 1 adenoma >1cm. Re-scope at 3 years.

High risk

>5 small adenomas or >3 with 1 of them >1cm. Re scope at 1 year.

From Atkins and Saunders Gut 2002 51 (suppl V:V6-V9). It is important to stratify

patients appropriately and ensure that a complete colonoscopy with good views was

performed.

Segmental resection or complete colectomy should be considered when:

1. Incomplete excision of malignant polyp

2. Malignant sessile polyp

3. Malignant pedunculated polyp with submucosal invasion

4. Polyps with poorly differentiated carcinoma

5. Familial polyposis coli

-Screening from teenager up to 40 years by 2 yearly sigmoidoscopy/colonoscopy

-Panproctocolectomy and Ileostomy or Restorative Panproctocolectomy.

Rectal polypoidal lesions may be amenable to trans anal endoscopic microsurgery.

56 year old lady has just undergone a colonoscopy and a lesion was identified in the

caecum. The histology report states that biopsies have been taken from a sessile

serrated polyp with traditional features? What is the best management option?

A. Perform a right hemicolectomy

B. List the patient for colonoscopic polypectomy

C. Discharge the patient

D. Re scope the patient in 6 months

E. Re scope the patient at 3 years

These polyps represent an alternative pathway to progression to carcinoma and may

be diagnostically confused with hyperplastic polyps. Hyperplastic polyps are more

common in the left colon and confer no increased risk. SSA's are more common in the

right colon and are usually larger. Those with "traditional features" on histology have

dysplasia with increased risk of malignant transformation.

Colonic polyps

Colonic Polyps May occur in isolation of greater numbers as part of the polyposis syndromes. In FAP

greater than 100 polyps are typically present. The risk of malignancy in association

with adenomas is related to size and is the order of 10% in a 1cm adenoma. Isolated

adenomas seldom give risk of symptoms (unless large and distal). Distally sited

villous lesions may produce mucous and if very large electrolyte disturbances may

occur.

Follow up of colonic polyps

Low risk

1 or 2 adenomas <1cm. No follow up or re-colonoscopy at 5 years.

Moderate risk

3 or 4 small adenomas or 1 adenoma >1cm. Re-scope at 3 years.

High risk

>5 small adenomas or >3 with 1 of them >1cm. Re scope at 1 year.

From Atkins and Saunders Gut 2002 51 (suppl V:V6-V9). It is important to stratify

patients appropriately and ensure that a complete colonoscopy with good views was

performed.

Segmental resection or complete colectomy should be considered when:

1. Incomplete excision of malignant polyp

2. Malignant sessile polyp

3. Malignant pedunculated polyp with submucosal invasion

4. Polyps with poorly differentiated carcinoma

5. Familial polyposis coli

-Screening from teenager up to 40 years by 2 yearly sigmoidoscopy/colonoscopy

-Panproctocolectomy and Ileostomy or Restorative Panproctocolectomy.

Rectal polypoidal lesions may be amenable to trans anal endoscopic microsurgery. A 30 year old male presents with gynaecomastia. Clinically he is noted to have a nodule in

the left testis. The most likely diagnosis is:

A. Oestrogen abuse

B. Seminoma with syncytiotrophoblast giant cells

C. Teratoma

D. Choriocarcinoma

E. Leydig cell tumour

Leydig cell tumours are rare testicular sex cord stromal tumours (which also include sertoli

cell tumours) which are associated with hormonal activity.

Patients with Leydig cell tumours may present with gynaecomastia before they notice testicular enlargement.

Majority are benign Histology: eosinophilic cells in columns

Testicular disorders

Testicular cancer

Testicular cancer is the most common malignancy in men aged 20-30 years. Around 95% of

cases of testicular cancer are germ-cell tumours. Germ cell tumours may essentially be

divided into:

Tumour type Key features Tumour markers Pathology

Seminoma Commonest subtype (50%)

Average age at diagnosis = 40

Even advanced disease associated with 5 year survival of 73%

AFP usually

normal

HCG elevated

in 10%

seminomas

Lactate

dehydrogenase;

elevated in 10-

20% seminomas

(but also in many

other conditions)

Sheet like lobular

patterns of cells

with substantial

fibrous

component.

Fibrous septa

contain

lymphocytic

inclusions and

granulomas may

be seen.

Non seminomatous germ cell

tumours (42%)

Teratoma Yolk sac tumour Choriocarcinoma Mixed germ cell

tumours (10%)

Younger age at

presentation =20-30

years

Advanced disease

carries worse prognosis

(48% at 5 years)

Retroperitoneal lymph

node dissection may be

needed for residual

disease after

chemotherapy

AFP elevated in

up to 70% of

cases

HCG elevated

in up to 40% of

cases

Other markers

rarely helpful

Heterogenous

texture with

occasional

ectopic tissue

such as hair

Image demonstrating a classical seminoma, these tumours are typically more uniform than

teratomas

Image sourced from Wikipedia

Risk factors for testicular cancer

Cryptorchidism Infertility Family history Klinefelter's syndrome Mumps orchitis

Features

A painless lump is the most common presenting symptom Pain may also be present in a minority of men

Other possible features include hydrocele, gynaecomastia

Diagnosis

Ultrasound is first-line CT scanning of the chest/ abdomen and pelvis is used for staging Tumour markers (see above) should be measured

Management

Orchidectomy (Inguinal approach) Chemotherapy and radiotherapy may be given depending on staging Abdominal lesions >1cm following chemotherapy may require retroperitoneal lymph

node dissection.

Prognosis is generally excellent

5 year survival for seminomas is around 95% if Stage I 5 year survival for teratomas is around 85% if Stage I

Benign disease

Epididymo-orchitis

Acute epididymitis is an acute inflammation of the epididymis, often involving the testis and

usually caused by bacterial infection.

Infection spreads from the urethra or bladder. In men <35 years, gonorrhoea or chlamydia are the usual infections.

Amiodarone is a recognised non infective cause of epididymitis, which resolves on stopping the drug.

Tenderness is usually confined to the epididymis, which may facilitate differentiating it from torsion where pain usually affects the entire testis.

Testicular torsion

Twist of the spermatic cord resulting in testicular ischaemia and necrosis. Most common in males aged between 10 and 30 (peak incidence 13-15 years) Pain is usually severe and of sudden onset. Cremasteric reflex is lost and elevation of the testis does not ease the pain. Treatment is with surgical exploration. If a torted testis is identified then both testis

should be fixed as the condition of bell clapper testis is often bilateral.

Which of the following features are not typical of Crohns disease?

A. Complex fistula in ano

B. Small bowel strictures

C. Skip lesions

D. 'Rose thorn ulcers' on barium studies

E. Pseudopolyps on colonoscopy

Pseudopolyps are a feature of ulcerative colitis and occur when there is severe

mucosal ulceration. The remaining islands of mucosa may then appear to be

isolated and almost polypoidal.

Crohns disease

Crohns disease is a chronic transmural inflammation of a segment(s) of the

gastrointestinal tract and may be associated with extra intestinal

manifestations. Frequent disease patterns observed include ileal, ileocolic and

colonic disease. Peri-anal disease may occur in association with any of these.

The disease is often discontinuous in its distribution. Inflammation may cause

ulceration, fissures, fistulas and fibrosis with stricturing. Histology reveals a

chronic inflammatory infiltrate that is usually patchy and transmural.

Ulcerative colitis Vs Crohns

Crohn's disease Ulcerative colitis

Distribution Mouth to anus Rectum and colon

Macroscopic

changes

Cobblestone appearance, apthoid

ulceration

Contact bleeding

Depth of

disease

Transmural inflammation Superficial inflammation

Distribution

pattern

Patchy Continuous

Histological

features

Granulomas (non caseating epithelioid

cell aggregates with Langhans' giant

cells)

Crypt abscesses,

Inflammatory cells in the

lamina propria

Extraintestinal manifestations of Crohns

Related to disease extent Unrelated to disease extent

Aphthous ulcers (10%) Sacroiliiitis (10-15%)

Erythema nodosum (5-10%) Ankylosing spondylitis (1-2%)

Pyoderma gangrenosum (0.5%) Primary sclerosing cholangitis (Rare)

Acute arthropathy (6-12%) Gallstones (up to 30%)

Ocular complications (up to 10%) Renal calculi (up to 10%)

A 32 year old lady presents with a 1.5cm pigmented lesion on her back. The surgeon

is concerned that this may be a melanoma. What is the most appropriate course of

action?

A. 2mm punch biopsy from the centre of the lesion

B. 4mm punch biopsy from the centre of the lesion

C. Wide excision of the lesion with 3cm margins

D. Excisional biopsy of the lesion

E. Wide excision of the lesion with 1cm margins

Suspicious naevi should NOT be partially sampled as histological interpretation is

severely compromised. Complete excision is mandatory where lesions fulfil

diagnostic criteria. However, wide excision for margins may be deferred until

definitive histology is available.

Lesions that are suspicious for melanoma should be excised with complete margins.

Radical excision is not routinely undertaken for diagnostic purposes and therefore if

subsequent histopathological assessment determines that the lesion is a melanoma a

re-exicision of margins may be required. Incisional punch biopsies of potential

melanomas makes histological interpretation difficult and is best avoided.

Malignant melanoma

The main diagnostic features (major

criteria):

Change in size

Change in shape

Change in colour

Secondary features (minor

criteria)

Diameter >6mm

Inflammation

Oozing or bleeding

Altered sensation

Treatment

Suspicious lesions should undergo excision biopsy. The lesion should be

removed in completely as incision biopsy can make subsequent

histopathological assessment difficult.

Once the diagnosis is confirmed the pathology report should be reviewed to

determine whether further re-exicision of margins is required (see below):

Margins of excision-Related to Breslow thickness

Lesions 0-1mm thick 1cm

Lesions 1-2mm thick 1- 2cm (Depending upon site and pathological features)

Lesions 2-4mm thick 2-3 cm (Depending upon site and pathological features)

Lesions >4 mm thick 3cm

Marsden J et al Revised UK guidelines for management of Melanoma. Br J Dermatol

2010 163:238-256.

Further treatments such as sentinel lymph node mapping, isolated limb perfusion and

block dissection of regional lymph node groups should be selectively applied.

A 70 year old male presents with painless frank haematuria. Clinical examination is

unremarkable. Routine blood tests reveal a haemoglobin of 18g/dl but are otherwise

normal. What is the most likely underlying diagnosis?

A. Squamous cell carcinoma of the bladder

B. Adenocarcinoma of the prostate

C. Adenocarcinoma of the kidney

D. Wilms tumour

E. Transitional cell carcinoma of the renal pelvis

Theme from April 2012 Exam

Polycythaemia is a recognised feature of renal cell carcinoma. Wilms tumours most

commonly occur in children.

Haematuria

Causes of haematuria

Trauma Injury to renal tract

Renal trauma commonly due to blunt injury (others

penetrating injuries)

Ureter trauma rare: iatrogenic

Bladder trauma: due to RTA or pelvic fractures

Infection Remember TB

Malignancy Renal cell carcinoma (remember paraneoplastic

syndromes): painful or painless

Urothelial malignancies: 90% are transitional cell

carcinoma, can occur anywhere along the urinary tract.

Painless haematuria.

Squamous cell carcinoma and adenocarcinoma: rare

bladder tumours

Prostate cancer

Penile cancers: SCC

Renal disease Glomerulonephritis

Stones Microscopic haematuria common

Structural

abnormalities

Benign prostatic hyperplasia (BPH) causes haematuria

due to hypervascularity of the prostate gland

Cystic renal lesions e.g. polycystic kidney disease

Vascular malformations

Renal vein thrombosis due to renal cell carcinoma

Coagulopathy Causes bleeding of underlying lesions

Drugs Cause tubular necrosis or interstitial nephritis:

aminoglycosides, chemotherapy

Interstitial nephritis: penicillin, sulphonamides, and

NSAIDs

Anticoagulants

Benign Exercise

Gynaecological Endometriosis: flank pain, dysuria, and haematuria that

is cyclical

Iatrogenic Catheterisation

Radiotherapy; cystitis, severe haemorrhage, bladder

necrosis

Pseudohaematuria

References Http://bestpractice.bmj.com/best-practice/monograph/316/overview/aetiology.html

Theme: Thyroid neoplasms

A. Follicular carcinoma

B. Follicular adenoma

C. Papillary carcinoma

D. Papillary adenoma

E. Anaplastic carcinoma

F. Medullary carcinoma

Please select the most likely underlying diagnosis for the thyroid masses described.

Each option may be used once, more than once or not at all.

19. A 78 year old lady presents to the surgical clinic with symptoms of both

dysphagia and dyspnoea. On examination there is a large mass in the neck

that moves on swallowing. CT scanning of the neck shows a locally

infiltrative lesion arising from the thyroid and invading the strap muscles.

Anaplastic carcinoma

Marked local invasion is a feature of anaplastic carcinoma. These tumours are

more common in elderly females.

20. A 25 year old female presents with a lump in her neck. On examination she

has a discrete nodule in the right lobe of the thyroid. A fine needle aspirate

shows papillary cells. An adjacent nodule is also sampled which shows

similar well differentiated papillary cells.

Papillary carcinoma

Multifocal disease is a recognised feature of papillary lesions. Papillary

adenomas are not really recognised and most well differentiated lesions are

papillary carcinomas.

21. A 45 year old man presents with a fracture of his right humerus. On

examination there is a lytic lesion of the proximal humerus and a mass in the

neck, this moves on swallowing.

You answered Medullary carcinoma

The correct answer is Follicular carcinoma

Follicular carcinomas are a recognised source of bone metastasis. Up to 60%

will show vascular invasion histologically.

Thyroid neoplasms

Lesion Common features

Follicular

adenoma

Usually present as a solitary thyroid nodule

Malignancy can only be excluded on formal histological

assessment

Papillary

carcinoma

Usually contain a mixture of papillary and colloidal filled

follicles

Histologically tumour has papillary projections and pale empty

nuclei

Seldom encapsulated

Lymph node metastasis predominate

Haematogenous metastasis rare

Account for 60% of thyroid cancers

Follicular

carcinoma

May appear macroscopically encapsulated, microscopically

capsular invasion is seen. Without this finding the lesion is a

follicular adenoma.

Vascular invasion predominates

Multifocal disease rare

Account for 20% of all thyroid cancers

Anaplastic

carcinoma

Most common in elderly females

Local invasion is a common feature

Account for 10% of thyroid cancers

Treatment is by resection where possible, palliation may be

achieved through isthmusectomy and radiotherapy.

Chemotherapy is ineffective.

Medullary

carcinoma

Tumours of the parafollicular cells (C Cells)

C cells derived from neural crest and not thyroid tissue

Serum calcitonin levels often raised

Familial genetic disease accounts for up to 20% cases

Both lymphatic and haematogenous metastasis are recognised,

nodal disease is associated with a very poor prognosis.

Rate questi

Which of the following lesions is most closely associated with Barretts oesophagus?

A. Squamous cell carcinoma

B. Gastro intestinal stromal tumours

C. Carcinoid tumours

D. Leiomyosarcoma

E. Adenocarcinoma

Barretts oesphagus is most closely associated with adenocarcinoma, and it confers at

30 fold increased risk of developing the condition.

Oesophageal cancer

Incidence is increasing

In most cases in the Western world this increase is accounted for by a rise in

the number of cases of adenocarcinoma. In the UK adenocarcinomas account

for 65% of cases.

Barretts oesophagus is a major risk factor for most cases of oesophageal

adenocarcinoma.

In other regions of the world squamous cancer is more common and is linked

to smoking, alcohol intake, diets rich in nitrosamines and achalasia.

Surveillance of Barretts is important as it imparts a 30 fold increase in cancer

risk and if invasive malignancy is diagnosed early then survival may approach

85% at 5 years.

Diagnosis

Upper GI endoscopy is the first line test

Contrast swallow may be of benefit in classifying benign motility disorders

but has no place in the assessment of tumours

Staging is initially undertaken with CT scanning of the chest, abdomen and

pelvis. If overt metastatic disease is identified using this modality then further

complex imaging is unnecessary

If CT does not show metastatic disease, then local stage may be more

accurately assessed by use of endoscopic ultrasound.

Staging laparoscopy is performed to detect occult peritoneal disease. PET CT

is performed in those with negative laparoscopy. Thoracoscopy is not

routinely performed.

Treatment Operable disease is best managed by surgical resection. The most standard procedure

is an Ivor- Lewis type oesophagectomy. This procedure involves the mobilisation of

the stomach and division of the oesophageal hiatus. The abdomen is closed and a right

sided thoracotomy performed. The stomach is brought into the chest and the

oesophagus mobilised further. An intrathoracic oesophagogastric anastomosis is

constructed. Alternative surgical strategies include a transhiatal resection (for distal

lesions), a left thoraco-abdominal resection (difficult access due to thoracic aorta) and

a total oesophagectomy (McKeown) with a cervical oesophagogastric anastomosis.

The biggest surgical challenge is that of anastomotic leak, with an intrathoracic

anastomosis this will result in mediastinitis. With high mortality. The McKeown

technique has an intrinsically lower systemic insult in the event of anastmotic leakage.

In addition to surgical resection many patients will be treated with adjuvent

chemotherapy.

A 56 year old man presents with epigastric discomfort and episodes of migratory

thrombophlebitis. On examination he is mildly jaundiced. A CT scan shows peri hilar

lymphadenopathy and a mass in the pancreatic head. Which of the following is the

most likely underlying diagnosis?

A. Squamous cell carcinoma of the pancreas

B. Adenocarcinoma of the pancreas

C. Insulinoma

D. Glucagonoma

E. Gastrinoma

Adenocarcinoma of the pancreas is the most likely diagnosis and migratory

thrombophlebitis is associated with the condition. Squamous cells carcinoma is

extremely uncommon in the pancreas. Gastrinoma are extremely rare and thus not the

most likely diagnosis.

Pancreatic cancer

Adenocarcinoma

Risk factors: Smoking, diabetes, Adenoma, Familial adenomatous polyposis

Mainly occur in the head of the pancreas (70%)

Spread locally and metastasizes to the liver

Carcinoma of the pancreas should be differentiated from other periampullary

tumours with better prognosis

Clinical features

Weight loss

Painless jaundice

Epigastric discomfort (pain usually due to invasion of the coeliac plexus is a

late feature)

Pancreatitis

Trousseau's sign: migratory superficial thrombophlebitis

Investigations

USS: May miss small lesions

CT Scanning (pancreatic protocol). If unresectable on CT then no further

staging needed.

PET/CT for those with operable disease on CT alone

ERCP/ MRI for bile duct assessment.

Staging laparoscopy to exclude peritoneal disease.

Management

Head of pancreas: Whipple's resection (SE dumping and ulcers). Newer

techniques include pylorus preservation and SMA/ SMV resection.

Carcinoma body and tail: poor prognosis, distal pancreatectomy if operable.

Usually adjuvent chemotherapy for resectable disease

ERCP and stent for jaundice and palliation.

Surgical bypass may be needed for duodenal obstruction.

A 22 year old man presents with symptoms of headache, lethargy and confusion. On

examination he is febrile and has a right sided weakness. A CT scan shows a ring

enhancing lesion affecting the left motor cortex. Which of the following is the most

likely diagnosis?

A. Arteriovenous malformation

B. Cerebral abscess

C. Herpes simplex encephalitis

D. Metastatic renal adenocarcinoma

E. Glioblastoma multiforme

The combination of rapidly progressive neurology, fever and headache is highly

suggestive of cerebral abscess. CT scanning will show a ring enhancing lesion

because the intravenous contrast cannot penetrate the centre of the abscess cavity.

HSV encephalitis does not produce ring enhancing lesions.

Brain abscess

CNS abscesses may result from a number of causes including, extension of

sepsis from middle ear or sinuses, trauma or surgery to the scalp, penetrating

head injuries and embolic events from endocarditis.

The presenting symptoms will depend upon the site of the abscess (those in

critical areas e.g.motor cortex) will present earlier. Abscesses have a

considerable mass effect in the brain and raised intra cranial pressure is

common.

Although fever, headache and focal neurology are highly suggestive of a brain

abscess the absence of one or more of these does not exclude the diagnosis,

fever may be absent and even if present, is usually not the swinging pyrexia

seen with abscesses at other sites.

Assessment of the patient includes imaging with CT scanning.

Treatment is usually surgical, a craniotomy is performed and the abscess

cavity debrided. The abscess may reform because the head is closed following

abscess drainage.

A 56 year old man from Egypt has suffered from recurrent attacks of haematuria for

many years. He presents with suprapubic discomfort and at cystoscopy is found to

have a mass lesion within the bladder. What is the most likely diagnosis?

A. Squamous cell carcinoma

B. Transitional cell papilloma

C. Adenocarcinoma

D. Leiomyosarcoma

E. Rhabdomyosarcoma

In Egypt Schistosomiasis is more common than in the UK and may cause recurrent

episodes of haematuria. In those affected with the condition who develop a bladder

neoplasm, an SCC is the most common type.

Bladder cancer

Bladder cancer is the second most common urological cancer. It most commonly

affects males aged between 50 and 80 years of age. Those who are current, or

previous (within 20 years), smokers have a 2-5 fold increased risk of the disease.

Occupational exposure to hydrocarbons such as alanine increases the risk. Although

rare in the UK, chronic bladder inflammation arising from Schistosomiasis infection

remains a common cause of squamous cell carcinomas, in those countries where the

disease is endemic.

Benign tumours Benign tumours of the bladder including inverted urothelial papilloma and

nephrogenic adenoma are uncommon.

Bladder malignancies

Transitional cell carcinoma (>90% of cases)

Squamous cell carcinoma ( 1-7% -except in regions affected by

schistosomiasis)

Adenocarcinoma (2%)

Transitional cell carcinomas may arise as solitary lesions, or may be multifocal,

owing to the effect of "field change" within the urothelium. Up to 70% of TCC's will

have a papillary growth pattern. These tumours are usually superficial in location and

accordingly have a better prognosis. The remaining tumours show either mixed

papillary and solid growth or pure solid growths. These tumours are typically more

prone to local invasion and may be of higher grade, the prognosis is therefore worse.

Those with T3 disease or worse have a 30% (or higher) risk of regional or distant

lymph node metastasis.

TNM Staging

Stage Description

T0 No evidence of tumour

Ta Non invasive papillary carcinoma

T1 Tumour invades sub epithelial connective tissue

T2a Inner half of detrusor invaded

T2b Outer half of detrusor invaded

T3 Tumour extends to perivesical fat

T4a Invasion of uterus, prostate or bowel

T4b Invasion of other abdominal organs

N0 No nodal disease

N1 Single lymph node metastasis (up to 2cm)

N2 Single node >2cm or multiple nodes up to 5cm

N3 Nodes over 5cm

M1 Distant disease

Presentation Most patients (85%) will present with painless, macroscopic haematuria. In those

patients with incidental microscopic haematuria, up to 10% of females aged over 50

will be found to have a malignancy (once infection excluded).

Staging Most will undergo a cystoscopy and biopsies or TURBT, this provides histological

diagnosis and information relating to depth of invasion. Locoregional spread is best

determined using pelvic MRI and distant disease CT scanning. Nodes of uncertain

significance may be investigated using PET CT.

Treatment Those with superficial lesions may be managed using TURBT in isolation. Those

with recurrences or higher grade/ risk on histology may be offered intravesical

chemotherapy. Those with T2 disease are usually offered either surgery (radical

cystectomy and ileal conduit) or radical radiotherapy.

Prognosis

T1 90%

T2 60%

T3 35%

T4a 10-25%

Any T, N1-N2 30%

A newborn infant is noted to have a unilateral cleft lip only. What is the most likely

explanation for this process?

A. Incomplete fusion of the second branchial arch

B. Incomplete fusion of the nasolabial muscle rings

C. Incomplete fusion of the first branchial arch

D. Incomplete fusion of the third branchial arch

E. Incomplete fusion of the secondary palate

Theme from April 2012 Exam

Unilateral isolated cleft lip represents a failure of nasolabial ring fusion. It is not

related to branchial arch fusion. Arch disorders have a far more profound phenotype

and malformation sequences.

Cleft lip and palate

Cleft lip and palate are the most common congenital deformity affecting the orofacial

structures. Whilst they may be an isolated developmental malformation they are also a

recognised component of more than 200 birth defects. The incidence is as high as 1 in

600 live births. The commonest variants are:

Isolated cleft lip (15%)

Isolated cleft palate (40%)

Combined cleft lip and palate (45%)

The aetiology of the disorder is multifactorial; both genetic (affected first degree

relative increases risk) and environmental factors play a role.

Cleft lip Cleft lip occurs as a result of disruption of the muscles of the upper lip and nasolabial

region. These muscles comprise a chain of muscles viz; nasolabial, bilabial and

labiomental. Defects may be unilateral or bilateral.

Cleft palate The primary palate consists of all anatomical structures anterior to the incisive

foramen. The seconday palate lies more posteriorly and is sub divided into the hard

and soft palate. Cleft palate occurs as a result of non fusion of the two palatine

shelves. Both hard and soft palate may be involved. Complete cases are associated

with complete separation of the nasal septum and vomer from the palatine processes.

Treatment Surgical reconstruction is the mainstay of management. The procedures are planned

according to the extent of malformation and child age. Simple defects are managed as

a single procedure. Complex malformations are usually corrected in stages. Affected

individuals have a higher incidence of hearing and speech problems.

A 72 year old woman with back pain and chronic renal failure has the following

results:

Reference range

Ca2+ 2.03 2.15-2.55 mmol/l

Parathyroid hormone 10.4 1-6.5 pmol/l

Phosphate 0.80 0.6-1.25 mmol/l

What is the most likely diagnosis?

A. Hypoparathyroidism

B. Primary hyperparathyroidism

C. Secondary hyperparathyroidism

D. Tertiary hyperparathyroidism

E. Pseudohypoparathyroidism

In relation to secondary hyperparathyroidism; there is a HIGH PTH and the Ca2+ is

NORMAL or LOW. In secondary hyperparathyroidism there in hyperplasia of the

parathyroid glands in response to chronic hypocalcaemia (or hyperphosphataemia)

and is a normal physiological response. Calcium is restored from bone, kidneys and

the gastrointestinal system.

Parathyroid glands and disorders of calcium metabolism

Hyperparathyroidism

Disease type Hormone profile Clinical features Cause

Primary

hyperparathyroidism

PTH (Elevated)

Ca2+ (Elevated)

Phosphate

(Low)

Serum Calcium

: Creatinine

clearance ratio

> 0.01

May be

asymptomatic if

mild

Recurrent

abdominal pain

(pancreatitis, renal

colic)

Changes to

emotional or

cognitive state

Most cases due to

solitary adenoma

(80%), multifocal

disease occurs in

10-15% and

parathyroid

carcinoma in 1% or

less

Secondary

hyperparathyroidism

PTH (Elevated)

Ca2+ (Low or

normal)

Phosphate

(Elevated)

Vitamin D

May have few

symptoms

Eventually may

develop bone

disease, osteitis

fibrosa cystica and

Parathyroid gland

hyperplasia occurs

as a result of low

calcium, almost

always in a setting

of chronic renal

levels (Low) soft tissue

calcifications

failure

Tertiary

hyperparathyroidism

Ca2+ (Normal

or high)

PTH (Elevated)

Phosphate

levels

(Decreased or

Normal)

Vitamin D

(Normal or

decreased)

Alkaline

phosphatase

(Elevated)

Metastatic

calcification

Bone pain and /

or fracture

Nephrolithiasis

Pancreatitis

Occurs as a result

of ongoing

hyperplasia of the

parathyroid glands

after correction of

underlying renal

disorder,

hyperplasia of all 4

glands is usually

the cause

Differential diagnoses It is important to consider the rare but relatively benign condition of benign familial

hypocalciuric hypercalcaemia, caused by an autosomal dominant genetic disorder.

Diagnosis is usually made by genetic testing and concordant biochemistry (Serum

Calcium : Creatinine clearance ratio <0.01-distinguished from primary

hyperparathyroidism).

Treatment

Primary hyperparathyroidism

Indications for surgery

Elevated serum Calcium > 1mg/dL above normal

Hypercalciuria > 400mg/day

Creatinine clearance < 30% compared with normal

Episode of life threatening hypercalcaemia

Nephrolithiasis

Age < 50 years

Neuromuscular symptoms

Reduction in bone mineral density of the femoral neck, lumbar spine, or distal

radius of more than 2.5 standard deviations below peak bone mass (T score

lower than -2.5)

Secondary hyperparathyroidism Usually managed with medical therapy.

Indications for surgery in secondary (renal) hyperparathyroidism:

Bone pain

Persistent pruritus

Soft tissue calcifications

Tertiary hyperparathyroidism Usually treatment is surgical

The presence of an autonomously functioning parathyroid gland may require surgery.

If the culprit gland can be identified then it should be excised. Otherwise total

parathyroidectomy and re-implantation of part of the gland may be required.

References 1. Pitt S et al. Secondary and Tertiary Hyperparathyroidism, State of the Art Surgical

Management. Surg Clin North Am 2009 Oct;89(5):1227-39.

2. MacKenzie-Feder J et al. Primary Hyperparathyroidism: An Overview. Int J

Endocrinol 2011; 2011: 251410.

A 72 year old woman with back pain and chronic renal failure has the following

results:

Reference range

Ca2+ 2.03 2.15-2.55 mmol/l

Parathyroid hormone 10.4 1-6.5 pmol/l

Phosphate 0.80 0.6-1.25 mmol/l

What is the most likely diagnosis?

A. Hypoparathyroidism

B. Primary hyperparathyroidism

C. Secondary hyperparathyroidism

D. Tertiary hyperparathyroidism

E. Pseudohypoparathyroidism

In relation to secondary hyperparathyroidism; there is a HIGH PTH and the Ca2+ is

NORMAL or LOW. In secondary hyperparathyroidism there in hyperplasia of the

parathyroid glands in response to chronic hypocalcaemia (or hyperphosphataemia)

and is a normal physiological response. Calcium is restored from bone, kidneys and

the gastrointestinal system.

Parathyroid glands and disorders of calcium metabolism

Hyperparathyroidism

Disease type Hormone profile Clinical features Cause

Primary

hyperparathyroidism

PTH (Elevated)

Ca2+ (Elevated)

Phosphate

May be

asymptomatic if

mild

Most cases due to

solitary adenoma

(80%), multifocal

(Low)

Serum Calcium

: Creatinine

clearance ratio

> 0.01

Recurrent

abdominal pain

(pancreatitis, renal

colic)

Changes to

emotional or

cognitive state

disease occurs in

10-15% and

parathyroid

carcinoma in 1% or

less

Secondary

hyperparathyroidism

PTH (Elevated)

Ca2+ (Low or

normal)

Phosphate

(Elevated)

Vitamin D

levels (Low)

May have few

symptoms

Eventually may

develop bone

disease, osteitis

fibrosa cystica and

soft tissue

calcifications

Parathyroid gland

hyperplasia occurs

as a result of low

calcium, almost

always in a setting

of chronic renal

failure

Tertiary

hyperparathyroidism

Ca2+ (Normal

or high)

PTH (Elevated)

Phosphate

levels

(Decreased or

Normal)

Vitamin D

(Normal or

decreased)

Alkaline

phosphatase

(Elevated)

Metastatic

calcification

Bone pain and /

or fracture

Nephrolithiasis

Pancreatitis

Occurs as a result

of ongoing

hyperplasia of the

parathyroid glands

after correction of

underlying renal

disorder,

hyperplasia of all 4

glands is usually

the cause

Differential diagnoses It is important to consider the rare but relatively benign condition of benign familial

hypocalciuric hypercalcaemia, caused by an autosomal dominant genetic disorder.

Diagnosis is usually made by genetic testing and concordant biochemistry (Serum

Calcium : Creatinine clearance ratio <0.01-distinguished from primary

hyperparathyroidism).

Treatment

Primary hyperparathyroidism

Indications for surgery

Elevated serum Calcium > 1mg/dL above normal

Hypercalciuria > 400mg/day

Creatinine clearance < 30% compared with normal

Episode of life threatening hypercalcaemia

Nephrolithiasis

Age < 50 years

Neuromuscular symptoms

Reduction in bone mineral density of the femoral neck, lumbar spine, or distal

radius of more than 2.5 standard deviations below peak bone mass (T score

lower than -2.5)

Secondary hyperparathyroidism Usually managed with medical therapy.

Indications for surgery in secondary (renal) hyperparathyroidism:

Bone pain

Persistent pruritus

Soft tissue calcifications

Tertiary hyperparathyroidism Usually treatment is surgical

The presence of an autonomously functioning parathyroid gland may require surgery.

If the culprit gland can be identified then it should be excised. Otherwise total

parathyroidectomy and re-implantation of part of the gland may be required.

References 1. Pitt S et al. Secondary and Tertiary Hyperparathyroidism, State of the Art Surgical

Management. Surg Clin North Am 2009 Oct;89(5):1227-39.

2. MacKenzie-Feder J et al. Primary Hyperparathyroidism: An Overview. Int J

Endocrinol 2011; 2011: 251410. Theme: Pharyngitis

A. Infectious mononucleosis

B. Acute bacterial tonsillitis

C. Quinsy

D. Lymphoma

E. Diptheria

Please select the most likely underlying cause for the following patients presenting with

pharyngitis. Each option may be used once, more than once or not at all.

2. An 8 year old child presents with enlarged tonsils that meet in the midline and are

covered with a white film that bleeds when you attempt to remove it. He is pyrexial

but otherwise well.

Acute bacterial tonsillitis

Theme from April 2012 Exam

In acute tonsillitis the tonsils will often meet in the midline and may be covered with

a membrane. Individuals who are systemically well are unlikely to have diptheria.

3. A 10 year old child presents with enlarged tonsils that meet in the midline.

Oropharyngeal examination confirms this finding and you also notice peticheal

haemorrhages affecting the oropharynx. On systemic examination he is noted to

have splenomegaly.

Infectious mononucleosis

A combination of pharyngitis and tonsillitis is often seen in glandular fever.

Antibiotics containing penicillin may produce a rash when given in this situation,

leading to a mistaken label of allergy.

4. A 19 year old man has had a sore throat for the past 5 days. Over the past 24 hours

he has notices increasing and severe throbbing pain in the region of his right tonsil.

He is pyrexial and on examination he is noted to have a swelling of this area.

Quinsy

Unilateral swelling and fever is usually indicative of quinsy. Surgical drainage usually

produces prompt resolution of symptoms.

Acute tonsillitis

Characterised by pharyngitis, fever, malaise and lymphadenopathy. Over half of all cases are bacterial with Streptococcus pyogenes the most common

organism The tonsils are typically oedematous and yellow or white pustules may be present Infectious mononucleosis may mimic the condition. Treatment with penicillin type antibiotics is indicated for bacterial tonsillitis. Bacterial tonsillitis may result in local abscess formation (quinsy)

Acute streptococcal tonsillitis

Image sourced from Wikipedia

A 4 year old boy is brought to the clinic by his mother who has noticed a small lesion

at the external angle of his eye. On examination there is a small cystic structure which

has obviously been recently infected. On removal of the scab, there is hair visible

within the lesion. What is the most likely diagnosis?

A. Dermoid cyst

B. Desmoid cyst

C. Sebaceous cyst

D. Epidermoid cyst

E. Keratoacanthoma

Dermoid cysts occur at sites of embryonic fusion and may contain multiple cell types.

They occur most often in children.

The lesion is unlikely to be a desmoid cyst as these are seldom located either at this

site or in this age group. In addition they do not contain hair. Sebaceous cysts will

usually have a punctum and contain a cheesy material. Epidermoid cysts contain

keratin plugs.

Dermoid cysts

A cutaneous dermoid cyst may develop at sites of embryonic developmental fusion.

They are most common in the midline of the neck, external angle of the eye and

posterior to the pinna of the ear. They typically have multiple inclusions such as hair

follicles that bud out from its walls. They may develop at other sites such as the ovary

and in these sites are synonymous with teratomas.

A desmoid tumour is a completely different entity and may be classified either as low

grade fibrosarcomas or non aggressive fibrous tumours. They commonly present as

large infiltrative masses. They may be divided into abdominal, extra abdominal and

intra abdominal. All types share the same biological features. Extra abdominal

desmoids have an equal sex distribution and primarily arise in the musculature of the

shoulder, chest wall, back and thigh. Abdominal desmoids usually arise in the

musculoaponeurotic structures of the abdominal wall. Intra abdominal desmoids tend

to occur in the mesentery or pelvic side walls and occur most frequently in patients

with familial adenomatous polyposis coli syndrome.

Which of the following tumours are most likely to give rise to para-aortic nodal

metastasis early?

A. Ovarian

B. Bladder

C. Rectal

D. Caecal

E. Cervical

Theme from 2009 Exam

Ovarian tumours are supplied by the ovarian vessels, these branch directly from the

aorta. The cervix drains to the internal and external iliac nodes.

Para-aortic lymphadenopathy

Organ sites that may metastasise (early) to the para-aortic lymph nodes:

Testis

Ovary

Uterine fundus

Many other organs may result in para-aortic nodal disease. However, these deposits

will represent a much later stage of the disease, since other nodal stations are involved

earlier.

Which of the following statements relating to malignant mesothelioma is false?

A. It may be treated by extrapleural pneumonectomy.

B. It is linked to asbestos exposure.

C. It is linked to cigarette smoking independent of asbestos exposure.

D. It may occur intra abdominally.

E. It is relatively resistant to radiotherapy

It is not linked to cigarette smoking. When identified at an early stage a radical

resection is the favored option. Radiotherapy is often given perioperatively. However,

it is not a particularly radiosensitive tumour. Combination chemotherapy gives some

of the best results and most regimes are cisplatin based.

Occupational cancers

Occupational cancers accounted for 5.3% cancer deaths in 2005.

In men the main cancers include:

Mesothelioma

Bladder cancer

Non melanoma skin cancer

Lung cancer

Sino nasal cancer

Occupations with high levels of occupational tumours include:

Construction industry

Working with coal tar and pitch

Mining

Metalworkers

Working with asbestos (accounts for 98% of all mesotheliomas)

Working in rubber industry

Shift work has been linked to breast cancer in women (Health and safety executive

report RR595).

The latency between exposure and disease is typically 15 years for solid tumours and

20 for leukaemia.

Many occupational cancers are otherwise rare. For example sino nasal cancer is an

uncommon tumour, 50% will be SCC. They are linked to conditions such as wood

dust exposure and unlike lung cancer is not strongly linked to cigarette smoking.

Another typical occupational tumour is angiosarcoma of the liver which is linked to

working with vinyl chloride. Again in the non occupational context this is an

extremely rare sporadic tumour.

Theme: Thyroid disorders

A. Sick euthyroid

B. Hyperthyroidism

C. Hypothyroidism

D. Normal euthyroid

E. Anxiety state

F. Factitious hyperthyroidism

For each of the scenarios please match the scenario with the most likely underlying

diagnosis. Each answer may be used once, more than once or not at all.

8. A 33 year old man is recovering following a protracted stay on the intensive

care unit recovering from an anastomotic leak following a difficult Trans

hiatal oesophagectomy. His progress is slow, and the intensive care doctors

receive the following thyroid function test results:

TSH 1.0 u/L

Free T4 8

T3 1.0 (1.2-3.1 normal)

You answered Normal euthyroid

The correct answer is Sick euthyroid

Theme from April 2012 Exam

Sick euthyroid syndrome is caused by systemic illness. With this, the patient

may have an apparently low total and free T4 and T3, with a normal or low

TSH. Note that the levels are only mildly below normal.

9. A 28 year old female presents to the general practitioner with symptoms of

fever and diarrhoea. As part of her diagnostic evaluation the following

thyroid function tests are obtained:

TSH < 0.01

Free T4 30

T3 4.0

Hyperthyroidism

The symptoms are suggestive of hyperthyroidism. This is supported by the

abnormal blood results; suppressed TSH with an elevated T3 and T4.

10. A 19 year old lady presents with palpitations. The medical officer takes a

blood sample for thyroid function tests. The following results are obtained:

TSH > 6.0

Free T4 20

T3 2.0

Hypothyroidism

An elevated TSH with normal T4 indicates partial thyroid failure. This is

caused by Hashimotos, drugs (lithium, antithyroids) and dyshormogenesis.

Hyperthyroidism

Causes of hyperthyroidism include:

Diffuse toxic goitre (Graves Disease)

Toxic nodular goitre

Toxic nodule

Rare causes

Graves disease Graves disease is characterised by a diffuse vascular goitre that appears at the same

time as the clinical manifestations of hyperthyroidism. It is commonest in younger

females and may be associated with eye signs. Thyrotoxic symptoms will

predominate. Up to 50% of patients will have a familial history of autoimmune

disorders. The glandular hypertrophy and hyperplasia occur as a result of the thyroid

stimulating effects of the TSH receptor antibodies.

Toxic nodular goitre In this disorder the goitre is present for a long period of time prior to the development

of clinical symptoms. In most goitres the nodules are inactive and in some cases it is

the internodular tissue that is responsible for the goitre.

Toxic nodule Overactive, autonomously functioning nodule. It may occur as part of generalised

nodularity or be a true toxic adenoma. The TSH levels are usually low as the

autonomously functioning thyroid tissue will exert a negative feedback effect.

Signs and symptoms

Symptoms Signs

Lethargy Tachycardia

Emotionally labile Agitation

Heat intolerance Hot, moist palms

Weight loss Exopthalmos

Excessive appetite Thyroid goitre and bruit

Palpitations Lid lag/retraction

Diagnosis The most sensitive test for diagnosing hyperthyroidism is plasma T3 (which is raised).

Note in hypothyroidism the plasma T4 and TSH are the most sensitive tests. A TSH

level of <0.5U/L suggests hyperthyroidism. TSH receptor antibodies may be tested

for in the diagnosis of Graves.

Treatment First line treatment for Graves disease is usually medical and the block and replace

regime is the favored option. Carbimazole is administered at higher doses and

thyroxine is administered orally. Patient are maintained on this regime for between 6

and 12 months. Attempts are then made to wean off medication. Where relapse then

occurs the options are between ongoing medical therapy, radioiodine or surgery.

Which of the following conditions is least likely to exhibit the Koebner phenomenon?

A. Vitiligo

B. Molluscum contagiosum

C. Lichen planus

D. Psoriasis

E. Lupus vulgaris

Lupus vulgaris is not associated with the Koebner phenomenon.

Koebner phenomenon

The Koebner phenomenon describes skin lesions which appear at the site of injury. It

is seen in:

Psoriasis

Vitiligo

Warts

Lichen planus

Lichen sclerosus

Molluscum contagiosum

Which of the tumour types listed below is found most frequently in a person

with aggressive fibromatosis?

A. Medullary thyroid cancer

B. Basal cell carcinoma of the skin

C. Desmoid tumours

D. Dermoid tumours

E. Malignant melanoma

Aggressive fibromatosis

Aggressive fibromatosis is a disorder consisting of desmoid tumours, which

behave in a locally aggressive manner. Desmoid tumours may be identified in

both abdominal and extra-abdominal locations. Metastatic disease is rare. The

main risk factor (for abdominal desmoids) is having APC variant of familial

adenomatous polyposis coli. Most cases are sporadic.

Treatment is by surgical excision.

A 20 year old man is suspected of having an inflamed Meckels diverticulum. At

which of the following locations is it most likely to be found?

A. Approximately 60 cm distal to the ileo-caecal valve

B. Approximately 60 cm proximal to the ileocaecal valve

C. Approximately 200cm distal to the ileocaecal valve

D. Approximately 200cm proximal to the ileocaecal valve

E. 50cm distal to the DJ flexure

Rule of 2's

2% of population

2 inches (5cm) long

2 feet (60 cm) from the

ileocaecal valve

2 x's more common in men

2 tissue types involved

They are typically found 2 feet proximal to the ileocaecal valve (or approximately

60cm).

Meckel's diverticulum

Congenital abnormality resulting in incomplete obliteration of the vitello-

intestinal duct

Normally, in the foetus, there is an attachment between the vitello-intestinal

duct and the yolk sac.This disappears at 6 weeks gestation.

The tip is free in majority of cases.

Associated with enterocystomas, umbilical sinuses, and omphaloileal fistulas.

Arterial supply: omphalomesenteric artery.

2% of population, 2 inches long, 2 feet from the ileocaecal valve.

Typically lined by ileal mucosa but ectopic gastric mucosa can occur, with the

risk of peptic ulceration. Pancreatic and jejunal mucosa can also occur.

Clinical

Normally asymptomatic and an incidental finding.

Complications are the result of obstruction, ectopic tissue, or inflammation.

Removal if narrow neck or symptomatic. Options are between wedge excision

or formal small bowel resection and anastomosis.

A 43 year old lady presents with severe chest pain. Investigations demonstrate a

dissecting aneurysm of the ascending aorta which originates at the aortic valve. What

is the optimal long term treatment?

A. Endovascular stent

B. Medical therapy with beta blockers

C. Medical therapy with ACE inhibitors

D. Sutured aortic repair

E. Aortic root replacement

Proximal aortic dissections are generally managed with surgical aortic root

replacement. The proximal origin of the dissection together with chest pain (which

may occur in all types of aortic dissection) raises concerns about the possibility of

coronary ostial involvement (which precludes stenting). There is no role for attempted

suture repair in this situation.

Aortic dissection

More common than rupture of the abdominal aorta

33% of patients die within the first 24 hours, and 50% die within 48 hours if

no treatment received

Associated with hypertension

Features of aortic dissection: tear in the intimal layer, followed by formation

and propagation of a subintimal hematoma. Cystic medial necrosis (Marfan's)

Most common site of dissection: 90% occurring within 10 centimetres of the

aortic valve

Stanford Classification

Type Location Treatment

A Ascending aorta/ aortic root Surgery- aortic root replacement

B Descending aorta Medical therapy with antihypertensives

DeBakey classification

Type Site affected

I Ascending aorta, aortic arch, descending aorta

II Ascending aorta only

III Descending aorta distal to left subclavian artery

Clinical features

Tearing, sudden onset chest pain (painless 10%)

Hypertension or Hypotension

A blood pressure difference greater than 20 mm Hg

Neurologic deficits (20%)

Investigations

CXR: widened mediastinum, abnormal aortic knob, ring sign, deviation

trachea/oesophagus

CT (spiral)

MRI

Angiography (95% of patients diagnosed)

Management

Beta-blockers: aim HR 60-80 bpm and systolic BP 100-120 mm Hg.

Urgent surgical intervention: type A dissections. This will usually involve

aortic root replacement.

A 45 year old lady presents with a pathological fracture of her femoral shaft. She is a

poor historian, but it transpires that she underwent a thyroidectomy 1 year previously.

She has no other illness or co-morbidities. What is the most likely underlying

diagnosis?

A. Hyperparathyroidism

B. Metastatic papillary carcinoma of the thyroid

C. Metastatic medullary carcinoma of the thyroid

D. Metastatic follicular carcinoma of the thyroid

E. None of the above

Follicular carcinomas are a recognised cause of bone metastasis. Papillary lesions

typically spread via the lymphatics.

Thyroid malignancy

Papillary carcinoma

Commonest sub-type

Accurately diagnosed on fine needle aspiration cytology

Histologically they may demonstrate psammoma bodies (areas of

calcification) and so called 'orphan Annie' nuclei

They typically metastasise via the lymphatics and thus laterally located

apparently ectopic thyroid tissue is usually a metastasis from a well

differentiated papillary carcinoma.

Follicular carcinoma

Are less common than papillary lesions

Like papillary tumours they may present as a discrete nodule. Although they

appear to be well encapsulated macroscopically there invasion on microscopic

evaluation.

Lymph node metastases are uncommon and these tumours tend to spread

haematogenously. This translates into a higher mortality rate.

Follicular lesions cannot be accurately diagnosed on fine needle aspiration

cytology and thus all follicular FNA's will require at least a hemi

thyroidectomy.

Anaplastic carcinoma

Less common and tend to occur in elderly females

Disease is usually advanced at presentation and often only palliative

decompression and radiotherapy can be offered.

Medullary carcinoma

These are tumours of the parafollicular cells ( C Cells) and are of neural crest

origin.

The serum calcitonin may be elevated which is of use when monitoring for

recurrence.

They may be familial and occur as part of the MEN -2A disease spectrum.

Spread may be either lymphatic or haematogenous and as these tumours are

not derived primarily from thyroid cells they are not responsive to radioiodine.

Lymphoma

These respond well to radiotherapy

Radical surgery is unnecessary once the disease has been diagnosed on biopsy

material. Such biopsy material is not generated by an FNA and thus a core

biopsy has to be obtained (with care!).

Theme: Cardiac murmurs

A. Pulmonary stenosis

B. Mitral regurgitation

C. Tricuspid regurgitation

D. Aortic stenosis

E. Mitral stenosis

F. Aortic sclerosis

What is the most likely cause of the cardiac murmur in the following patients? Each

option may be used once, more than once or not at all.

16. A 35 year old Singaporean female attends a varicose vein pre operative clinic.

On auscultation a mid diastolic murmur is noted at the apex. The murmur is

enhanced when the patient lies in the left lateral position.

Mitral stenosis

Theme from September 2011 exam

A mid diastolic murmur at the apex is a classical description of a mitral

stenosis murmur. The most common cause is rheumatic heart disease.

Complications of mitral stenosis include atrial fibrillation, stroke, myocardial

infarction and infective endocarditis.

17. A 22 year old intravenous drug user is found to have a femoral abscess. The

nursing staff contact the on call doctor as the patient has a temperature of

39oC. He is found to have a pan systolic murmur loudest at the left sternal

edge at the 4th intercostal space.

You answered Mitral regurgitation

The correct answer is Tricuspid regurgitation

Intravenous drug users are at high risk of right sided cardiac valvular

endocarditis. The character of the murmur fits with a diagnosis of tricuspid

valve endocarditis.

18. An 83 year old woman is admitted with a left intertrochanteric neck of femur

fracture. On examination the patient is found to have an ejection systolic

murmur loudest in the aortic region. There is no radiation of the murmur to

the carotid arteries. Her ECG is normal.

You answered Aortic stenosis

The correct answer is Aortic sclerosis

The most likely diagnosis is aortic sclerosis. The main differential diagnosis

is of aortic stenosis, however as there is no radiation of the murmur to the

carotids and the ECG is normal, this is less likely.

Cardiac murmurs

Type of Murmur Conditions

Ejection systolic Aortic stenosis

Pulmonary stenosis, HOCM

ASD, Fallot's

Pan-systolic Mitral regurgitation

Tricuspid regurgitation

VSD

Late systolic Mitral valve prolapse

Coarctation of aorta

Early diastolic Aortic regurgitation

Graham-Steel murmur (pulmonary regurgitation)

Mid diastolic Mitral stenosis

Austin-Flint murmur (severe aortic regurgitation)

A 55 year old lady presents with discomfort in the right breast. On clinical

examination a small lesion is identified and clinical appearances suggest

fibroadenoma. Imaging confirms the presence of a fibroadenoma alone. A core biopsy

is taken, this confirms the presence of the fibroadenoma. However, the pathologist

notices that a small area of lobular carcinoma in situ is also present in the biopsy.

What is the best management?

A. Whole breast irradiation

B. Simple mastectomy

C. Mastectomy and sentinal lymph node biopsy

D. Wide local excision and sentinel lymph node biopsy

E. Breast MRI scan

Lobular carcinoma in situ has a low association with invasive malignancy. It is

seldom associated with microcalcification and therefore MRI is the best tool for

determining disease extent. Resection of in situ disease is not generally recommended

and most surgeons would simply pursue a policy of close clinical and radiological

follow up.

Lobular carcinoma of the breast

Lobular breast cancers are less common than their ductal counterparts. They typically

present differently, the mass is usually more diffuse and less obvious on the usual

imaging modalities of ultrasound and mammography. This is significant since the

disease may be understaged resulting in inadequate treatment when wide local

excision is undertaken.

In women with invasive lobular carcinoma it is usually safest to perform an MRI scan

of the breast, if breast conserving surgery is planned.

Lobular carcinomas are also more likely to be multifocal and metastasise to the

contralateral breast.

Lobular carcinoma in situ is occasionally diagnosed incidentally on core biopsies.

Unlike DCIS, lobular carcinoma in situ is far less strongly associated with foci of

invasion and is usually managed by close monitoring.

Which one of the following may be associated with an increased risk of venous

thromboembolism?

A. Diabetes

B. Cannula

C. Hyperthyroidism

D. Tamoxifen

E. Amiodarone

Consider thromboembolism in breast cancer patients on tamoxifen!

Venous thromboembolism: risk factors

ommon predisposing factors include malignancy, pregnancy and the period following

an operation. The comprehensive list below is partly based on the 2010 SIGN venous

thromboembolism (VTE) guidelines:

General

increased risk with advancing age

obesity

family history of VTE

pregnancy (especially puerperium)

immobility

hospitalisation

anaesthesia

central venous catheter: femoral >> subclavian

Underlying conditions

malignancy

thrombophilia: e.g. Activated protein C resistance, protein C and S deficiency

heart failure

antiphospholipid syndrome

Behcet's

polycythaemia

nephrotic syndrome

sickle cell disease

paroxysmal nocturnal haemoglobinuria

hyperviscosity syndrome

homocystinuria

Medication

combined oral contraceptive pill: 3rd generation more than 2nd generation

hormone replacement therapy

raloxifene and tamoxifen

antipsychotics (especially olanzapine) have recently been shown to be a risk

factor

SIGN also state that the following are risk factors for recurrent VTE:

previous unprovoked VTE

male sex

obesity

thrombophilias

A 42 year old lady is investigated for symptoms of irritability and altered bowel habit.

On examination she is noted to have a smooth enlargement of the thyroid gland. As

part of her investigations thyroid function tests are requested, these are as follows:

TSH 0.1 mug/l

Free T4 35 pmol/l

The most likely underlying diagnosis is:

A. Multinodular goitre

B. Follicular carcinoma of the thyroid gland

C. Graves disease

D. Pregnancy

E. None of the above

TSH receptor antibodies will cause stimulation of the thyroid to synthesise T4.

However, this will have a negative feedback effect on the pituitary causing decrease

in TSH levels.

Where hyperthyroidism occurs secondary to pregnancy the TSH is typically elevated.

Thyroid disease

Patients may present with a number of different manifestations of thyroid disease.

They can be broadly sub classified according to whether they are euthyroid or have

clinical signs of thyroid dysfunction. In addition it needs to be established whether

they have a mass or not.

Assessment

History

Examination including USS

If a nodule is identified then it should be sampled ideally via an image guided

fine needle aspiration

Radionucleotide scanning is of limited use

Thyroid Tumours

Papillary carcinoma

Follicular carcinoma

Anaplastic carcinoma

Medullary carcinoma

Lymphoma's

Multinodular goitre

One of the most common reasons for presentation

Provided the patient is euthyroid and asymptomatic and no discrete nodules

are seen, they can be reassured.

In those with compressive symptoms surgery is required and the best

operation is a total thyroidectomy.

Sub total resections were practised in the past and simply result in recurrent

disease that requires a difficult revisional resection.

Endocrine dysfunction

In general these patients are managed by physicians initially.

Surgery may be offered alongside radio iodine for patients with Graves

disease that fails with medical management or in patients who would prefer

not to be irradiated (e.g. pregnant women).

Patients with hypothyroidism do not generally get offered a thyroidectomy.

Sometimes people inadvertently get offered resections during the early phase

of Hashimotos thyroiditis, however, with time the toxic phase passes and

patients can simply be managed with thyroxine.

Complications following surgery

Anatomical such as recurrent laryngeal nerve damage.

Bleeding. Owing to the confined space haematoma's may rapidly lead to

respiratory compromise owing to laryngeal oedema.

Damage to the parathyroid glands resulting in hypocalcaemia.

Further sources of information 1. http://www.acb.org.uk/docs/TFTguidelinefinal.pdf- Association of Clinical

Biochemistry guidelines for thyroid function tests.

2. British association of endocrine surgeons website- http://www.baets.org.uk

Which of the following is not a risk factor for developing tuberculosis?

A. Gastrectomy

B. Solid organ transplantation with immunosupression

C. Intravenous drug use

D. Haematological malignancy

E. Amiodarone

Risk factors for developing active tuberculosis include:

silicosis

chronic renal failure

HIV positive

solid organ transplantation with immunosuppression

intravenous drug use

haematological malignancy

anti-TNF treatment

previous gastrectomy

Tuberculosis

Tuberculosis (TB) is an infection caused by Mycobacterium tuberculosis that most

commonly affects the lungs. Understanding the pathophysiology of TB can be

difficult - the key is to differentiate between primary and secondary disease.

Primary tuberculosis A non-immune host who is exposed to M. tuberculosis may develop primary infection

of the lungs. A small lung lesion known as a Ghon focus develops. The Ghon focus is

composed of tubercle-laden macrophages. The combination of a Ghon focus and hilar

lymph nodes is known as a Ghon complex

In immunocompotent people the intially lesion usually heals by fibrosis. Those who

are immunocompromised may develop disseminated disease (miliary tuberculosis).

Secondary (post-primary) tuberculosis If the host becomes immunocompromised the initial infection may become

reactivated. Reactivation generally occurs in the apex of the lungs and may spread

locally or to more distant sites. Possible causes of immunocomprise include:

immunosuppressive drugs including steroids

HIV

malnutrition

The lungs remain the most common site for secondary tuberculosis. Extra-pulmonary

infection may occur in the following areas:

central nervous system (tuberculous meningitis - the most serious

complication)

vertebral bodies (Pott's disease)

cervical lymph nodes (scrofuloderma)

renal

gastrointestinal tract

What is the most common presentation of a parotid gland tumour?

A. Parapharyngeal mass

B. Mass at anterior border of masseter

C. Mass inferior to the angle of the mandible

D. Mass behind the angle of the mandible

E. Mass anterior to the ear

Parotid tumours may present at any region in the gland. However, most lesions will be

located behind the angle of the mandible, inferior to the ear lobe. Tumours of the deep

lobe of the parotid may present as a parapharyngeal mass and large lesions may

displace the tonsil.

Parotid gland malignancy

Most parotid neoplasms (80%) are benign lesions

Most commonly present with painless mass in cheek region

Up to 30% may present with pain, when this is associated with a discrete mass

lesion in the parotid it usually indicates perineural invasion.

Perineural invasion is very unlikely to occur in association with benign lesions

80% of patients with facial nerve weakness caused by parotid malignancies

will have nodal metastasis and a 5 year survival of 25%

Types of malignancy

Mucoepidermoid

carcinoma

30% of all parotid malignancies

Usually low potential for local invasiveness and metastasis

(depends mainly on grade)

Adenoid cystic

carcinoma

Unpredictable growth patter

Tendency for perineural spread

Nerve growth may display skip lesions resulting in incomplete

excision

Distant metastasis more common (visceral rather than nodal

spread)

5 year survival 35%

Mixed tumours Often a malignancy occurring in a previously benign parotid

lesion

Acinic cell carcinoma Intermediate grade malignancy

May show perineural invasion

Low potential for distant metastasis

5 year survival 80%

Adenocarcinoma Develops from secretory portion of gland

Risk of regional nodal and distant metastasis

5 year survival depends upon stage at presentation, may be up

to 75% with small lesions with no nodal involvement

Lymphoma Large rubbery lesion, may occur in association with Warthins

tumours

Diagnosis should be based on regional nodal biopsy rather

than parotid resection Treatment is with chemotherapy (and

radiotherapy)

A 52 year old male is referred to urology clinic with impotence. He is known to have

hypertension. He does not have any morning erections. On further questioning the

patient reports pain in his buttocks, this worsens on mobilising. On examination there

is some muscle atrophy. The penis and scrotum are normal. What is the most likely

diagnosis?

A. Leriche syndrome

B. S4-S5 cord lesion

C. Pudendal nerve lesion

D. Psychological impotence

E. Beta blocker induced impotence

Leriche syndrome

Classically, it is described in male

patients as a triad of symptoms:

1. Claudication of the buttocks and

thighs

2. Atrophy of the musculature of the

legs

3. Impotence (due to paralysis of the

L1 nerve)

Leriche syndrome, is atherosclerotic occlusive disease involving the abdominal aorta

and/or both of the iliac arteries. Management involves correcting underlying risk

factors such as hypercholesterolaemia and stopping smoking. Investigation is usually

with angiography.

Leriche syndrome

Atheromatous disease involving the iliac vessels. Blood flow to the pelvic viscera is

compromised. Patients may present with buttock claudication and impotence (in this

particular syndrome). Diagnostic work up will include angiography, where feasible

iliac occlusions are usually treated with endovascular angioplasty and stent insertion.

A 23 year old man presents with diarrhoea and passage of mucous. He is suspected of

having ulcerative colitis. Which of the following is least likely to be associated with

this condition?

A. Superficial mucosal inflammation in the colon

B. Significant risk of dysplasia in long standing disease

C. Epsiodes of large bowel obstruction during acute attacks

D. Haemorrhage

E. Disease sparing the anal canal

Large bowel obstruction is not a feature of UC, patients may develop megacolon.

However, this is a different entity both diagnostically and clinically. Ulcerative colitis

does not affect the anal canal and the anal transitional zone. Inflammation is

superficial. Dysplasia can occur in 2% overall, but increases significantly if disease

has been present over 20 years duration. Granulomas are features of crohn's disease.

Other features:

Disease maximal in the rectum and may spread proximally

Contact bleeding

Longstanding UC crypt atrophy and metaplasia/dysplasia

Ulcerative colitis

Ulcerative colitis is a form of inflammatory bowel disease. Inflammation always starts

at rectum, never spreads beyond ileocaecal valve and is continuous. The peak

incidence of ulcerative colitis is in people aged 15-25 years and in those aged 55-65

years. It is less common in smokers.

The initial presentation is usually following insidious and intermittent symptoms.

Features include:

bloody diarrhoea

urgency

tenesmus

abdominal pain, particularly in the left lower quadrant

extra-intestinal features (see below)

Questions regarding the 'extra-intestinal' features of inflammatory bowel disease are

common. Extra-intestinal features include sclerosing cholangitis, iritis and ankylosing

spondylitis.

{Common to both Crohn's

disease (CD) and Ulcerative

colitis (UC)}

{Notes}

{Related to

disease activity}

Arthritis: pauciarticular,

asymmetric

Erythema nodosum

Episcleritis

Osteoporosis

Arthritis is the most common

extra-intestinal feature in both

CD and UC

Episcleritis is more common in

CD

{Unrelated to

disease activity}

Arthritis: polyarticular,

symmetric

Uveitis

Pyoderma gangrenosum

Clubbing

Primary sclerosing cholangitis

Primary sclerosing cholangitis is

much more common in UC

Uveitis is more common in UC

Pathology

red, raw mucosa, bleeds easily

no inflammation beyond submucosa (unless fulminant disease)

widespread superficial ulceration with preservation of adjacent mucosa which

has the appearance of polyps ('pseudopolyps')

inflammatory cell infiltrate in lamina propria

neutrophils migrate through the walls of glands to form crypt abscesses

depletion of goblet cells and mucin from gland epithelium

granulomas are infrequent

Barium enema

loss of haustrations

superficial ulceration, 'pseudopolyps'

long standing disease: colon is narrow and short -'drainpipe colon'

Endoscopy

Superficial inflammation of the colonic and rectal mucosa

Continuous disease from rectum proximally

Superifical ulceration, mucosal islands, loss of vascular definition and

continuous ulceration pattern.

Management

Patients with long term disease are at increased risk of development of

malignancy

Acute exacerbations are generally managed with steroids, in chronic patients

agents such as azathioprine and infliximab may be used

Individuals with medically unresponsive disease usually require surgery- in

the acute phase a sub total colectomy and end ileostomy. In the longer term a

proctectomy will be required. An ileoanal pouch is an option for selected

patients

A 52 year old woman attends clinic for investigation of abdominal pain and

constipation. On examination you note blue lines on the gum margin. She mentions

that her legs have become weak in the past few days. What is the most likely

diagnosis?

A. Acute intermittent porphyria

B. Lead poisoning

C. Constipation

D. Guillan Barre syndrome

E. Rectal carcinoma

This would be an impressive diagnosis to make in the surgical out patient department!

The combination of abdominal pain and a motor periperal neuropathy, should indicate

this diagnosis. The blue line along the gum margin can occur in up to 20% patients

with lead poisoning.

Lead poisoning

Along with acute intermittent porphyria, lead poisoning should be considered in

questions giving a combination of abdominal pain and neurological signs

Features

abdominal pain

peripheral neuropathy (mainly motor)

fatigue

constipation

blue lines on gum margin (only 20% of adult patients, very rare in children)

Investigations

The blood lead level is usually used for diagnosis. Levels greater than 10

mcg/dl are considered significant

Full blood count: microcytic anaemia. Blood film shows red cell abnormalities

including basophilic stippling and clover-leaf morphology

Raised serum and urine levels of delta aminolaevulinic acid may be seen

making it sometimes difficult to differentiate from acute intermittent porphyria

Urinary coproporphyrin is also increased (urinary porphobilinogen and

uroporphyrin levels are normal to slightly increased)

Management - various chelating agents are currently used:

Dimercaptosuccinic acid (DMSA)

D-penicillamine

EDTA

Dimercaprol

Theme: Vasculitis

A. Wegeners granulomatosis

B. Polyarteritis nodosa

C. Giant cell arteritis

D. Takayasu's arteritis

E. Buergers disease

For each of the scenarios provided please select the most likely underlying diagnosis

from the list below. Each option may be used once, more than once or not at all.

5. A 20 year old lady is referred to the vascular clinic. She has been feeling

generally unwell for the past six weeks. She works as a typist and has noticed

increasing pain in her forearms whilst working. On examination she has absent

upper limb pulses. Her ESR is measured and mildly elevated.

Takayasu's arteritis

Takayasus arteritis may be divided into acute systemic phases and the chronic

pulseless phase. In the latter part of the disease process the patient may

complain of symptoms such as upper limb claudication. In the later stages of

the condition the vessels will typically show changes of intimal proliferation,

together with band fibrosis of the intima and media.

6. A 32 year old man presents to the vascular clinic with symptoms of foot pain

during exertion. He is a heavy smoker and has recently tried to stop smoking.

On examination he has normal pulses to the level of the popliteal. However,

foot pulses are absent. A diagnostic angiogram is performed which shows an

abrupt cut off at the level of the anterior tibial artery, together with the

formation of corkscrew shaped collateral vessels distally.

Buergers disease

Buergers disease is most common in young male smokers. This demographic

is changing in those areas where young female smokers are more common. In

the acute lesion the internal elastic lamina of the vessels is usually intact. As

the disease progresses the changes progress to hypercellular occlusive

thrombus. Tortuous corkscrew collaterals may reconstitute patent segments of

the distal tibial or pedal vessels.

7. A 78 year old man presents with symptoms of headaches and deteriorating

vision. He notices that there is marked pain on the right hand side of his face

when he combs his hair.

Giant cell arteritis

Temporal arteritis may present acutely with symptoms of headache and visual

loss, or with a less acute clinical picture. Sight may be threatened and

treatment with immunosupressants should be started promptly. The often

requested temporal artery biopsy (which can be the bane of many surgeons) is

often non diagnostic and unhelpful.

Vasculitis

The vasculitides are a group of conditions characterised by inflammation of the blood

vessel walls. This may, in turn, compromise vessel integrity. Constitutional symptoms

may be present. Whilst certain disease subtypes are reported to affect specific vessels,

there is often a degree of overlap clinically.

Vessel diameter and vasculitis classification

Aorta and branches Takayasu's arteritis

Buergers disease

Giant cell arteritis

Large and medium sized arteries Buergers disease

Giant cell arteritis

Polyarteritis nodosa

Medium sized muscular arteries Polyarteritis nodosa

Wegeners granulomatosis

Small muscular arteries Wegeners granulomatosis

Rheumatoid vasculitis

Specific conditions

Takyasu's arteritis Inflammatory, obliterative arteritis affecting aorta and

branches

Females> Males

Symptoms may include upper limb claudication

Clinical findings include diminished or absent pulses

ESR often affected during the acute phase

Buergers disease Segmental thrombotic occlusions of the small and

medium sized lower limb vessels

Commonest in young male smokers

Proximal pulses usually present, but pedal pulses are

lost

An acuter hypercellular occlusive thrombus is often

present

Tortuous corkscrew shaped collateral vessels may be

seen on angiography

Giant cell arteritis Systemic granulomatous arteritis that usually affects

large and medium sized vessels

Females > Males

Temporal arteritis is commonest type

Granulomatous lesions may be seen on biopsy

(although up to 50% are normal)

Polyarteritis nodosa Systemic necrotising vasculitis affecting small and

medium sized muscular arteries

Most common in populations with high prevalence of

hepatitis B

Renal disease is seen in 70% cases

Angiography may show saccular or fusiform

aneurysms and arterial stenoses

Wegeners

granulomatosis

Predominantly affects small and medium sized arteries

Systemic necrotising granulomatous vasculitis

Cutaneous vascular lesions may be seen (ulceration,

nodules and purpura)

Sinus imaging may show mucosal thickening and air

fluid levels

Treatment Conditions such as Buergers disease are markedly helped by smoking cessation.

Immunosupression is the main treatment for vasculitides.

A 28 year old man presents with right upper quadrant pain and hydatid disease is

suspected. Which of the following statements relating to the disease is untrue?

A. First line treatment is with pentavalent antimony.

B. Peritoneal contamination with active daughter cysts may complicate

surgery.

C. CT scanning of the liver may show a floating membrane.

D. Biliary communication with the cysts may occur.

E. It is caused by Echinococcus granulosus.

Drug treatment is with albendazole or mebendazole. Praziquantzel may be used in the

pre operative stages.

Hydatid cysts

Hydatid cysts are endemic in Mediterranean and Middle Eastern countries. They are

caused by the tapeworm parasite Echinococcus granulosus. An outer fibrous capsule

is formed containing multiple small daughter cysts. These cysts are allergens which

precipitate a type 1 hypersensitivity reaction.

Clinical features are as follows:

Up to 90% cysts occur in the liver and lungs

Can be asymtomatic, or symptomatic if cysts > 5cm in diameter

Morbidity caused by cyst bursting, infection and organ dysfunction (biliary,

bronchial, renal and cerebrospinal fluid outflow obstruction)

In biliary ruputure there may be the classical triad of; biliary colic, jaundice,

and urticaria

CT is the best investigation to differentiate hydatid cysts from amoebic and pyogenic

cysts.

Surgery is the mainstay of treatment (the cyst walls must not be ruptured during

removal and the contents sterilised first).

Which of the following statements relating to neutrophil polymorphs is true?

A. Produce nitrogen peroxide as a microbicidal agent

B. Not involved in opsonisation

C. Deficiency leads to AIDS

D. Neutrophil disorders cause chronic granulomatous diseases

E. Have a lifespan of 9 hours

Neutrophils are the main cells of acute inflammation, important action against

gram -ve and +ve bacteria

Appearance of segmented nucleus and granulated cytoplasm

Have a lifespan of 1-3 days (shorter when consumed during septic process,

though 9 hours is unusual)

Actions include: movement, opsonise microorganisms, phagocytosis &

intracellular killing of microorganisms via aerobic (produce HYDROGEN

PEROXIDE) & anaerobic mechanisms.

Neutrophil disorders include chronic granulomatous diseases: rare

AIDS associated with T cell deficiency

Acute inflammation

Inflammation is the reaction of the tissue elements to injury. Vascular changes occur,

resulting in the generation of a protein rich exudate. So long as the injury does not

totally destroy the existing tissue architecture, the episode may resolve with

restoration of original tissue architecture.

Vascular changes

Vasodilation occurs and persists throughout the inflammatory phase.

Inflammatory cells exit the circulation at the site of injury.

The equilibrium that balances Starlings forces within capillary beds is

disrupted and a protein rich exudate will form as the vessel walls also become

more permeable to proteins.

The high fibrinogen content of the fluid may form a fibrin clot. This has

several important immunomodulatory functions.

Sequelae

Resolution Typically occurs with minimal initial injury

Stimulus removed and normal tissue architecture

results

Organisation Delayed removed of exudate

Tissues undergo organisation and usually fibrosis

Suppuration Typically formation of an abscess or an empyema

Sequestration of large quantities of dead

neutrophils

Progression to chronic

inflammation

Coupled inflammatory and reparative activities

Usually occurs when initial infection or

suppuration has been inadequately managed

Causes

Microbacterial infections e.g. Viruses, exotoxins or endotoxins released by

bacteria

Chemical agents

Physical agents e.g. Trauma

Hypersensitivity reactions

Tissue necrosis

Presence of neutrophil polymorphs is a histological diagnostic feature of acute

inflammation A 33 year old man is involved in a road traffic accident. He is initially stable and

transferred to the accident and emergency department. On arrival he is catheterised.

One minute later he becomes hypotensive, with evidence of angioedema surrounding

his penis. What is the most likely explanation for this event?

A. Type V latex hypersensitivity reaction

B. Type IV latex hypersensitivity reaction

C. Type III latex hypersensitivity reaction

D. Type I latex hypersensitivity reaction

E. Type II latex hypersensitivity reaction

Theme from April 2012 Exam

Sudden collapse and angioedema following exposure to latex (of which most urinary

catheters are manufactured) suggests a type I hypersensitivity reaction.

Hypersensitivity reactions

The Gell and Coombs classification divides hypersensitivity reactions into 4 types

Type I Type II Type III Type IV

Description Anaphylactic Cytotoxic Immune

complex

Delayed type

Mediator IgE IgG, IgM IgG, IgM T-cells

Antigen Exogenous Cell surface Soluble Tissues

Response

time

Minutes Hours Hours 2-3 days

Examples Asthma

Hay fever

Autoimmune

haemolytic anaemia

Pemphigus

Goodpasture's

Serum

sickness

SLE

Aspergillosis

Graft versus host

disease

Contact

dermatitis

A 43 year old female develops severe chest wall cellulitis following a mastectomy.

On examination the skin is markedly erythematous. Which of the acute inflammatory

mediators listed below is least likely to produce vasodilation?

A. Complement component C5a

B. Lysosomal compounds

C. Histamine

D. Serotonin

E. Prostaglandins

Erythema is a classical feature of acute inflammation. Potent mediators of vascular

dilatation include; histamine, prostaglandins, nitric oxide, platelet activating factor,

complement C5a (and C3a) and lysosomal compounds. Although serotonin is

associated with acute inflammation it is a vasoconstrictor. The effects of serotonin are

dependant upon the state of the vessels in the tissues. Intact and healthy tissues and

vessels will respond to a serotonin infusion with vasodilation (hence the flushing seen

in carcinoid syndrome). In contrast it worsens cardiac ischaemia in myocardial

infarcts when released from damaged platelets.

Acute inflammation

Inflammation is the reaction of the tissue elements to injury. Vascular changes occur,

resulting in the generation of a protein rich exudate. So long as the injury does not

totally destroy the existing tissue architecture, the episode may resolve with

restoration of original tissue architecture.

Vascular changes

Vasodilation occurs and persists throughout the inflammatory phase.

Inflammatory cells exit the circulation at the site of injury.

The equilibrium that balances Starlings forces within capillary beds is

disrupted and a protein rich exudate will form as the vessel walls also become

more permeable to proteins.

The high fibrinogen content of the fluid may form a fibrin clot. This has

several important immunomodulatory functions.

Sequelae

Resolution Typically occurs with minimal initial injury

Stimulus removed and normal tissue architecture

results

Organisation Delayed removed of exudate

Tissues undergo organisation and usually fibrosis

Suppuration Typically formation of an abscess or an empyema

Sequestration of large quantities of dead

neutrophils

Progression to chronic

inflammation

Coupled inflammatory and reparative activities

Usually occurs when initial infection or

suppuration has been inadequately managed

Causes

Microbacterial infections e.g. Viruses, exotoxins or endotoxins released by

bacteria

Chemical agents

Physical agents e.g. Trauma

Hypersensitivity reactions

Tissue necrosis

Presence of neutrophil polymorphs is a histological diagnostic feature of acute

inflammation A 43 year old man presents with a 3 week history of malaise, sore throat,

odynophagia and dysphagia. On examination he is found to have patchy white spots

in his oropharynx. An upper GI endoscopy is performed and similar lesions are

identified in the oesophagus. Which investigation is most likely to identify the

underlying pathology in this case?

A. Serum urea and electrolytes

B. Oesophageal biopsy for culture

C. Oesophageal biopsy for histology

D. Glucose tolerance testing

E. Viral serology

Oesophageal candidiasis is associated with immunosupression; mainly in patients on

chemotherapy, with haematological malignancy, HIV or inhaled steroids. In patients

with HIV, oesophageal candidiasis is part of the spectrum of AIDS defining illnesses

and usually occurs when the CD4 count is less than 200. Others include PCP

pneumonia and CMV infections.

Oesophageal candidiasis

Characterised by white spots in the oropharynx with extension into the oesophagus.

Associated with broad spectrum antibiotic usage, immunosupression and

immunological disorders.

Patients may present with oropharyngeal symptoms, odynophagia and dysphagia.

Treatment is directed both at the underlying cause (which should be investigated for)

and with oral antifungal agents.

Theme: Colonic obstruction

A. Malignant obstruction

B. Olgilvies syndrome

C. Volvulus

D. Diverticular stricture

E. Ischaemic stricture

Please select the most likely cause of obstruction for the situation described. Each

option may be used once, more than once or not at all.

13. A 78 year old man has undergone a hemi-arthroplasty for a intracapsular hip

fracture. Post operatively he develops electrolyte derangement and receives

intravenous fluids. Over the previous 24 hours he develops marked abdominal

distension. On examination he has a tense, tympanic abdomen which is not

painful. A contrast enema shows flow of contrast through to the caecum and

through the ileocaecal valve.

Olgilvies syndrome

Patients with electrolyte disturbance and previous surgery may develop

colonic pseudo-obstruction (olgilvies syndrome). The diagnosis is made using

a contrast enema and treatment is usually directed at the underlying cause

with colonic decompression if indicated.

14. A 67 year old man has had multiple episodes with fever and left iliac fossa

pain. These have usually resolved with courses of intravenous antibiotics. He

is admitted with a history of increasing constipation and abdominal

distension. A contrast x-ray is performed which shows flow of contrast to the

sigmoid colon, here the contrast flows through a long narrow segment of

colon into dilated proximal bowel.

Diverticular stricture

The long history of left iliac fossa pain and development of bowel obstruction

suggests a diverticular stricture. These may contain a malignancy and most

will require resection.

15. A 78 year old lady from a nursing home is admitted with a 24 hour history of

absolute constipation and abdominal pain. On examination she has a

distended soft mass in her left iliac fossa. An x-ray is performed which shows

a large dilated loop of bowel in the left iliac fossa which contains a fluid

level.

Volvulus

Sigmoid volvulus may present with an asymmetrical mass in an elderly

patient. It may contain a fluid level, visible on plain films.

Colonic obstruction

Cause Features Treatment

Cancer Usually insidious onset

History of progressive constipation

Systemic features (e.g. anaemia)

Abdominal distension

Absence of bowel gas distal to site

of obstruction

Establish diagnosis (e.g.

contrast enema/

endoscopy)

Laparotomy and

resection, stenting,

defunctioning colostomy

or bypass

Diverticular

stricture

Usually history of previous acute

diverticulitis

Once diagnosis

established, usually

Long history of altered bowel habit

Evidence of diverticulosis on

imaging or endoscopy

surgical resection

Colonic stenting should

not be performed for

benign disease

Volvulus Twisting of bowel around its

mesentery

Sigmoid colon affected in 76%

cases

Patients usually present with

abdominal pain, bloating and

constipation

Examination usually shows

asymmetrical distension

Plain X-rays usually show

massively dilated sigmoid colon,

loss of haustra and "U" shape are

typical, the loop may contain fluid

levels

Initial treatment is to

untwist the loop, a

flexible sigmoidoscopy

may be needed

Those with clinical

evidence of ischaemia

should undergo surgery

Patient with recurrent

volvulus should undergo

resection

Acute colonic

pseudo-

obstruction

Symptoms and signs of large bowel

obstruction with no lesion

Usually associated with metabolic

disorders

Usually a cut off in the left colon

(82% cases)

Although abdomen tense and

distended, it is usually not painful

All patients should undergo contrast

enema (may be therapeutic!)

Colonoscopic

decompression

Correct metabolic

disorders

IV neostigmine

Surgery

A 52 year old man with dyspepsia is found to have a duodenal ulcer. A CLO test is

taken and is positive. Which statement relating to the likely causative organism is

false?

A. It is a gram negative organism

B. It lives only on gastric type mucosa

C. It may occupy areas of ectopic gastric metaplasia

D. In patients who are colonised there is commonly evidence of fundal

gastritis on endoscopy

E. It produces a powerful urease that forms the basis of the Clo test

Helicobacter pylori accounts for >75% cases of duodenal ulceration. It may be

diagnosed with either serology, microbiology, histology or CLO testing.

Theme from January 2011 Exam

Helicobacter pylori rarely produces any typical features on endoscopy. Where

infection is suspected the easiest course of action is to take an antral biopsy for Clo

testing in the endoscopy suite.

Helicobacter Pylori

Infection with Helicobacter Pylori is implicated in many cases of duodenal ulceration

and up to 60% of patients with gastric ulceration.

Gram negative, helix shaped rod, microaerophillic

Produces hydrogenase that can derive energy from hydrogen released by

intestinal bacteria

Flagellated and mobile

Those carrying the cag A gene may cause ulcers

It secretes urease that breaks down gastric urea> Carbon dioxide and

ammonia> ammonium>bicarbonate (simplified!) The bicarbonate can

neutralise the gastric acid.

Usually colonises the gastric antrum and irritates resulting in increased gastrin

release and higher levels of gastric acid. These patients will develop duodenal

ulcers. In those with more diffuse H-Pylori infection gastric acid levels are

lower and ulcers develop by local tissue damage from H-Pylori- these patients

get gastric ulcers.

Diagnosis may be made by serology (approx. 75% sensitive). Biopsy urease

test during endoscopy probably the most sensitive.

In patients who are colonised 10-20% risk of peptic ulcer, 1-2% risk gastric

cancer, <1% risk MALT lymphoma.

A 34 year old male presents with painful rectal bleeding and a fissure in ano is

suspected. On examination he has an epithelial defect at the mucocutaenous defect

that is located anteriorly. Approximately what proportion of patients with fissure in

ano will present with this pattern of disease?

A. 90%

B. 10%

C. 50%

D. 25%

E. 100%

Only a minority of patients with fissure in ano will have an anteriorly sited fissure.

They are particularly rare in males and an anterior fissure in a man should prompt a

search for an underlying cause.

Anal fissure

Anal fissures are a common cause of painful, bright red, rectal bleeding.

Most fissures are idiopathic and present as a painful mucocutaneous defect in the

posterior midline (90% cases). Fissures are more likely to be anteriorly located in

females, particularly if they are multiparous. Multiple fissures and those which are

located at other sites are more likely to be due to an underlying cause.

Diseases associated with fissure in ano include:

Crohns disease

Tuberculosis

Internal rectal prolapse

Diagnosis In most cases the defect can be visualised as a posterior midline epithelial defect.

Where symptoms are highly suggestive of the condition and examination findings are

unclear an examination under anaesthesia may be helpful. Atypical disease

presentation should be investigated with colonoscopy and EUA with biopsies of the

area.

Treatment Stool softeners are important as the hard stools may tear the epithelium and result in

recurrent symptoms. The most effective first line agents are topically applied GTN

(0.2%) or Diltiazem (2%) paste. Side effects of diltiazem are better tolerated.

Resistant cases may benefit from injection of botulinum toxin or lateral internal

sphincterotomy (beware in females). Advancement flaps may be used to treat resistant

cases.

Sphincterotomy produces the best healing rates. It is associated with incontinence to

flatus in up to 10% of patients in the long term.

The pathogenesis of osteopetrosis is best explained by a defect in which of the

following?

A. Osteoclast function

B. PTH receptors

C. Osteoblast function

D. Calcium resorption in proximal tubule

E. Calcium absorption

Osteopetrosis

Overview

aka marble bone disease

rare disorder of defective osteoclast function resulting in failure of normal

bone resorption

stem cell transplant and interferon-gamma have been used for treatment

A 13 month old boy is brought to the surgical clinic by his mother because his left

testicle is not located in the scrotum. At which of the following sites would the testicle

be located if it were an ectopic testis?

A. Canalicular

B. Inguinal

C. External inguinal ring

D. Superficial inguinal pouch

E. High scrotal

Theme from September 2011 Exam

Theme from January 2012 Exam

Ectopic testes are those that come to lie outside the normal range of embryological

descent (i.e. in the superficial inguinal pouch). Other sites of ectopic testes include;

base of penis, femoral and perineal.

Testicular disorders-paediatric

Testicular disorders Testicular disorders are some of the commonest conditions present in paediatric

urological practice.

Cryptorchidism

The embryological descent of the testicle from within the abdominal cavity

may be subject to a number of variations. Distinctions need to be made

clinically from a non descended testis and a testis that is retractile.

Testis that lie outside the normal path of embryological descent are termed

ectopic testis. Undescended testis occurs in 1% of male infants. Where the

testis does not lie in an intra scrotal location, its location should be

ascertained. Where both testes are absent the infant may be intersex.

MRI scanning may reveal intra-abdominal testes; however a GA is often

needed to perform this investigation in this age group.

Testes that are undescended should be placed in the scrotum after 1 year of

age as the testosterone surge that may facilitate descent occurs at 6 months of

age.

Where the testes lie distally e.g. Superficial inguinal pouch an open

orchidopexy is the procedure of choice.

With abdominal testes a laparoscopy should be performed. The risk of

seminoma is increased in individuals with a non descended testes and this risk

is not reduced by orchidopexy.

Testicular torsion

Typically the patient has severe sudden onset of scrotal pain. The difficulty in

paediatric practice is the lack of clear history.

On examination the testis is tender and enlarged.

Management is by surgical exploration.

Delay beyond 6 hours is associated with low salvage rates.

A torted hyatid produces pain that is far more localised and the testis itself

should feel normal. However, diagnostic doubt often exists and in such cases

surgical exploration is warranted.

A splenectomy increases the risk of infection from all the following organisms

except?

A. Pneumococcus

B. Klebsiella

C. Haemophilus influenzae

D. Staphylococcus aureus

E. Neisseria meningitidis

Theme from 2010 Exam

Staphylococcus aureus infection following splenectomy is no more common than in

non splenectomised individuals. The other organisms are encapsulated, which is why

they are more likely to cause overwhelming post splenectomy sepsis.

Post splenectomy sepsis

The loss of splenic function renders individuals at increased risk of fulminant sepsis.

Young children are at the highest risk, especially in the first 2 years following

surgery. Surgery for trauma is associated with a lower risk than when splenectomy is

performed as a treatment for haematological disorders.

Infection with encapsulated organisms poses the greatest risk, these organisms may be

opsonised, but this then goes undetected at an immunological level due to loss of the

spleen.

Prophylactic vaccinations are usually administered to reduce the risk of pneumococcal

septicaemia. Since the vaccine only covers up to 80% of pneumococcal infections,

patients will usually recieve long term, low dose penicillin prophylaxis in addition to

vaccination.

Which of the following is not an extraintestinal feature Crohns disease?

A. Iritis

B. Clubbing

C. Aphthous ulcers

D. Erythema multiforme

E. Pyoderma gangrenosum

Extraintestinal manifestation of

inflammatory bowel disease: A PIE

SAC

Aphthous ulcers

Pyoderma gangrenosum

Iritis

Erythema nodosum

Sclerosing cholangitis

Arthritis

Clubbing

Crohns disease

Crohns disease is a chronic transmural inflammation of a segment(s) of the

gastrointestinal tract and may be associated with extra intestinal manifestations.

Frequent disease patterns observed include ileal, ileocolic and colonic disease. Peri-

anal disease may occur in association with any of these. The disease is often

discontinuous in its distribution. Inflammation may cause ulceration, fissures, fistulas

and fibrosis with stricturing. Histology reveals a chronic inflammatory infiltrate that is

usually patchy and transmural.

Ulcerative colitis Vs Crohns

Crohn's disease Ulcerative colitis

Distribution Mouth to anus Rectum and colon

Macroscopic Cobblestone appearance, apthoid Contact bleeding

changes ulceration

Depth of

disease

Transmural inflammation Superficial inflammation

Distribution

pattern

Patchy Continuous

Histological

features

Granulomas (non caseating epithelioid

cell aggregates with Langhans' giant

cells)

Crypt abscesses,

Inflammatory cells in the

lamina propria

Extraintestinal manifestations of Crohns

Related to disease extent Unrelated to disease extent

Aphthous ulcers (10%) Sacroiliiitis (10-15%)

Erythema nodosum (5-10%) Ankylosing spondylitis (1-2%)

Pyoderma gangrenosum (0.5%) Primary sclerosing cholangitis (Rare)

Acute arthropathy (6-12%) Gallstones (up to 30%)

Ocular complications (up to 10%) Renal calculi (up to 10%)

Which of the following is not considered a risk factor for the development of

oesophageal malignancy?

A. Oesophageal metaplasia

B. Smoking

C. Excessive intake of alcoholic spirits

D. Achalasia

E. Blood group O

Blood group O is not a risk factor for oesophageal cancer. Achalasia is associated

with the risk of developing squamous cell carcinoma of the oesophagus.

Oesophageal cancer

Incidence is increasing

In most cases in the Western world this increase is accounted for by a rise in

the number of cases of adenocarcinoma. In the UK adenocarcinomas account

for 65% of cases.

Barretts oesophagus is a major risk factor for most cases of oesophageal

adenocarcinoma.

In other regions of the world squamous cancer is more common and is linked

to smoking, alcohol intake, diets rich in nitrosamines and achalasia.

Surveillance of Barretts is important as it imparts a 30 fold increase in cancer

risk and if invasive malignancy is diagnosed early then survival may approach

85% at 5 years.

Diagnosis

Upper GI endoscopy is the first line test

Contrast swallow may be of benefit in classifying benign motility disorders

but has no place in the assessment of tumours

Staging is initially undertaken with CT scanning of the chest, abdomen and

pelvis. If overt metastatic disease is identified using this modality then further

complex imaging is unnecessary

If CT does not show metastatic disease, then local stage may be more

accurately assessed by use of endoscopic ultrasound.

Staging laparoscopy is performed to detect occult peritoneal disease. PET CT

is performed in those with negative laparoscopy. Thoracoscopy is not

routinely performed.

Treatment Operable disease is best managed by surgical resection. The most standard procedure

is an Ivor- Lewis type oesophagectomy. This procedure involves the mobilisation of

the stomach and division of the oesophageal hiatus. The abdomen is closed and a right

sided thoracotomy performed. The stomach is brought into the chest and the

oesophagus mobilised further. An intrathoracic oesophagogastric anastomosis is

constructed. Alternative surgical strategies include a transhiatal resection (for distal

lesions), a left thoraco-abdominal resection (difficult access due to thoracic aorta) and

a total oesophagectomy (McKeown) with a cervical oesophagogastric anastomosis.

The biggest surgical challenge is that of anastomotic leak, with an intrathoracic

anastomosis this will result in mediastinitis. With high mortality. The McKeown

technique has an intrinsically lower systemic insult in the event of anastmotic leakage.

In addition to surgical resection many patients will be treated with adjuvent

chemotherapy.

Theme: Thyroid blood testing

A. Measurement of antibodies to TSH receptor

B. Thyroid peroxidase antibodies

C. Thyroglobulin antibodies

D. Serum calcitonin

Please select the blood test most commonly performed for the diagnosis or assessment

of the thyroid disorder described. Each answer may be used once, more than once or

not at all.

23. A 32 year old lady is diagnosed with Medullary carcinoma of the thyroid and

has undergone resection of the tumour.

Serum calcitonin

Measurement of basal or stimulated calcitonin concentrations is used to

assess the completeness of surgical resection, and is of use in detecting

diseases recurrences during follow up.

24. A 20 year old lady has undergone a total thyroidectomy for a well

differentiated papillary carcinoma. She attends clinic and is well and the

surgeon wishes to screen for disease recurrence.

Thyroglobulin antibodies

Antibodies to thyroglobulin, the major constituent of colloid and precursor of

thyroid hormones may be elevated in those with metastatic or recurrent

thyroid cancer. Results may be erronoeous in those with other thyroid

disorders.

25. A 33 year old lady presents with a recently diagnosed goitre and a diagnosis

of Hashimotos thyroiditis is suspected.

Thyroid peroxidase antibodies

Antibodies to thyroid peroxidase are found in most patients with Graves

disease or Hashimotos thyroiditis.

Blood testing in thyroid disease

Assay Usage

Thyroid peroxidase

(microsomal)

antibodies

Found in autoimmune disease affecting the thyroid

(Hashimotos 100%) and Graves (70%)

Antibodies to TSH

receptor

Individuals with Graves disease (95%)

Thyroglobulin

antibodies

Not useful for clinically distinguishing between

different types of thyroid disease, may be used as part

of thyroid cancer follow up

Calcitonin Released from the parafollicular cells

Usually found in patient with Medullary carcinoma

of the thyroid

Which one of the following complications is least associated with ventricular septal

defects?

A. Right heart failure

B. Aortic regurgitation

C. Eisenmenger's complex

D. Infective endocarditis

E. Atrial fibrillation

Atrial fibrillation is associated more with atrial septal defects

Ventricular septal defect

Ventricular septal defects are the most common cause of congenital heart disease.

They close spontaneously in around 50% of cases. Non-congenital causes include post

myocardial infarction

Features

classically a pan-systolic murmur which is louder in smaller defects

Complications

aortic regurgitation*

infective endocarditis

Eisenmenger's complex

right heart failure

*aortic regurgitation is due to a poorly supported right coronary cusp resulting in cusp

prolapse A 24 year old man from Sudan presents with a lymphadenopathy and weight loss. A

diagnosis of tuberculosis is suspected and a lymph node biopsy is performed. Staining with

which of the agents below is most likely to facilitate identification of the causative

organism?

A. Gram stain

B. Ziehl-Neelsen stain

C. Von Kossa stain

D. Van Gieson stain

E. Masson Trichrome stain

Ziehl-Neelsen stain is typically used to identify mycobacteria. They are not stained in the

Gram staining process. Van Gieson and Masson trichrome are histological staining methods

for identification of connective tissues. The Von Kossa technique is useful for identifying

tissue mineralisation.

Tuberculosis pathology

Is a form of primary chronic inflammation, caused by the inability of macrophages to kill the Mycobacterium tuberculosis.

The macrophages often migrate to regional lymph nodes, the lung lesion plus affected lymph nodes is referred to as a Ghon complex.

This leads to the formation of a granuloma which is a collection of epithelioid histiocytes.

There is the presence of caseous necrosis in the centre. The inflammatory response is mediated by a type 4 hypersensitivity reaction. In healthy individuals the disease may be contained, in the immunocompromised

disseminated (miliary TB) may occur.

Diagnosis

Waxy membrane of mycobacteria prevents binding with normal stains. Ziehl - Neelsen staining is typically used.

Culture based methods take far longer.

Image showing acid- alcohol fast mycobacteria stained using the Ziehl- Neelsen method

Image sourced from Wikipedia

A 24 year old man from Sudan presents with a lymphadenopathy and weight loss. A

diagnosis of tuberculosis is suspected and a lymph node biopsy is performed. Staining with

which of the agents below is most likely to facilitate identification of the causative

organism?

A. Gram stain

B. Ziehl-Neelsen stain

C. Von Kossa stain

D. Van Gieson stain

E. Masson Trichrome stain

Ziehl-Neelsen stain is typically used to identify mycobacteria. They are not stained in the

Gram staining process. Van Gieson and Masson trichrome are histological staining methods

for identification of connective tissues. The Von Kossa technique is useful for identifying

tissue mineralisation.

Tuberculosis pathology

Is a form of primary chronic inflammation, caused by the inability of macrophages to kill the Mycobacterium tuberculosis.

The macrophages often migrate to regional lymph nodes, the lung lesion plus affected lymph nodes is referred to as a Ghon complex.

This leads to the formation of a granuloma which is a collection of epithelioid histiocytes.

There is the presence of caseous necrosis in the centre. The inflammatory response is mediated by a type 4 hypersensitivity reaction. In healthy individuals the disease may be contained, in the immunocompromised

disseminated (miliary TB) may occur.

Diagnosis

Waxy membrane of mycobacteria prevents binding with normal stains. Ziehl - Neelsen staining is typically used.

Culture based methods take far longer.

Image showing acid- alcohol fast mycobacteria stained using the Ziehl- Neelsen method

Image sourced from Wikipedia

Which of the following is not a pathological feature of breast cancer?

A. Resemblance to ductal epithelial cells

B. Angiogenesis

C. Nuclear pleomorphism

D. Metastatic calcification

E. Vascular invasion

Dystrophic calcification may be present in breast malignancy and is the basis for the

breast screening programme. Metastatic calcification is calcification which occurs in

otherwise normal tissues, usually as a result of hypercalcaemia. Invasive ductal

carcinoma is the most common type of breast cancer, unless the tumour is very poorly

differentiated there is usually some resemblance to ductal epithelial cells.

Breast cancer pathology

The histological features of breast cancer depend upon the underlying diagnosis. The

invasive component is usually comprised of ductal cells (unless it is an invasive

lobular cancer). In situ lesions may co-exist (such as DCIS).

Typical changes seen in conjunction with invasive breast cancer include:

1. Nuclear pleomorphism

2. Coarse chromatin

3. Angiogenesis

4. Invasion of the basement membrane

5. Dystrophic calcification (may be seen on mammography)

6. Abnormal mitoses

7. Vascular invasion

8. Lymph node metastasis

The primary tumour is graded on a scale of 1-3 where 1 is the most benign lesion and

3 the most poorly differentiated.

Immunohistochemistry for oestrogen receptor and herceptin status is routinely

performed.

The grade, lymph node stage and size are combined to provide the Nottingham

prognostic index.

Theme: Renal lesions

A. Renal cell carcinoma

B. Renal transitional cell carcinoma

C. Nephroblastoma

D. Neuroblastoma

E. Angiomyolipoma

F. Renal squamous cell carcinoma

G. Retroperitoneal fibrosis

For each scenario please select the most likely underlying diagnosis. Each option may

be used once, more than once or not at all.

29. A 69 year old male presents with haematuria. He worked in the textile

industry. He has a left flank mass. A CT IVU shows a lesion of the left renal

pelvis.

You answered Renal squamous cell carcinoma

The correct answer is Renal transitional cell carcinoma

TCC is a rare form of renal cancer, accounting for approximately 7% of all

renal tumours. Risk factors include exposure to chemicals in the textile,

plastic and rubber industry.

30. A 2 year old boy presents with a right renal mass. On examination he has an

irregular mass arisong from the right flank and is hypertensive. A CT scan

shows a non calcified irregular lesion affecting the apex of the right kidney

and the right adrenal gland.

Nephroblastoma

Wilm's tumour of the kidney is the most common renal tumour in children.

Both nephroblastoma and neuroblastoma may occupy the adrenal and apex of

the kidney. In the case of neuroblastoma the lesion will have arisen from the

adrenal, in the case of nephroblastoma the lesion will have arisen from the

kidney. Hypertension is more commonly associated with nephroblastoma.

Neuroblastomas are usually calcified, whereas nephroblastomas are not and

this may be of diagnostic usefulness pre operatively.

31. A 35 year old male presents with haematuria. He is found to have bilateral

masses in the flanks. He has a history of epilepsy and learning disability.

You answered Neuroblastoma

The correct answer is Angiomyolipoma

This patient has tuberous sclerosus. This is associated with angiomyolipoma,

which is present in 60-80% patients. It is a benign lesion.

Renal lesions

Lesion Disease specific features Treatment

Renal cell Most present with Usually radical or partial

carcinoma haematuria (50%)

Common renal tumour (85%

cases)

Paraneoplastic features

include hypertension and

polycythaemia

Most commonly has

haematogenous mestastasis

nephrectomy

Nephroblastoma Rare childhood tumour

It accounts for 80% of all

genitourinary malignancies

in those under the age of 15

years

Up to 90% will have a mass

50% will be hypertensive

Diagnostic work up includes

ultrasound and CT scanning

Surgical resection combined

with chemotherapy (usually

vincristine, actinomycin D

and doxorubicin

Neuroblastoma Most common extracranial

tumour of childhood

80% occur in those under 4

years of age

Tumour of neural crest

origin (up to 50% occur in

the adrenal gland)

The tumour is usually

calcified and may be

diagnosed using MIBG

scanning

Staging is with CT

Surgical resection,

radiotherapy and

chemotherapy

Transitional cell

carcinoma

Accounts for 90% of upper

urinary tract tumour, but

only 10% of renal tumours

Males affected 3x more than

females

Occupational exposure to

industrial dyes and rubber

chemicals may increase risk

Up to 80% present with

painless haematuria

Diagnosis and staging is with

CT IVU

Radical nephroureterectomy

Angiomyolipoma 80% of these hamartoma

type lesions occur

sporadically, the remainder

are seen in those with

50% of patients with lesions

>4cm will have symptoms

and will require surgical

resection

tuberous sclerosis

Tumour is composed of

blood vessels, smooth

muscle and fat

Massive bleeding may occur

in 10% of cases

An 18 month old boy presents with recurrent urinary tract infections. As part of the

diagnostic work-up he is noted to have abnormal renal function. An ultrasound scan is

performed and shows bilateral hydronephrosis. What is the most likely underlying

diagnosis?

A. Urethral valves

B. Meatal stenosis

C. Hydronephrosis

D. Pelvico-ureteric junction obstruction

E. Benign prostatic hyperplasia

Theme from April 2012 Exam

A posterior urethral valve is an obstructive, developmental uropathy that usually

affects male infants (incidence 1 in 8000). Diagnostic features include bladder wall

hypertrophy, hydronephrosis and bladder diverticula.

Urethral valves

Posterior urethral valves are the commonest cause of infravesical outflow obstruction

in males. They may be diagnosed on ante natal ultrasonography. Because the bladder

has to develop high emptying pressures in utero the child may develop renal

parenchymal damage. This translates to renal impairment noted in 70% of boys at

presentation. Treatment is with bladder catheterisation. Endoscopic valvotomy is the

definitive treatment of choice with cystoscopic and renal follow up. At which of the following anatomical sites does dormant tuberculosis most frequently

reactivate?

A. Apex of the lung

B. Base of the lung

C. Brain

D. Terminal ileum

E. Lumbar spine

TB reactivation most commonly occurs at the lung apex. This site is better oxygenated than

elsewhere allowing the mycobacteria to multiply more rapidly and then spread both locally

and distantly.

Tuberculosis pathology

Is a form of primary chronic inflammation, caused by the inability of macrophages to kill the Mycobacterium tuberculosis.

The macrophages often migrate to regional lymph nodes, the lung lesion plus affected lymph nodes is referred to as a Ghon complex.

This leads to the formation of a granuloma which is a collection of epithelioid histiocytes.

There is the presence of caseous necrosis in the centre. The inflammatory response is mediated by a type 4 hypersensitivity reaction. In healthy individuals the disease may be contained, in the immunocompromised

disseminated (miliary TB) may occur.

Diagnosis

Waxy membrane of mycobacteria prevents binding with normal stains. Ziehl - Neelsen staining is typically used.

Culture based methods take far longer.

Image showing acid- alcohol fast mycobacteria stained using the Ziehl- Neelsen method

Image sourced from Wikipedia

What is the commonest tumour type encountered in the colon?

A. Squamous cell carcinoma

B. Adenocarcinoma

C. Lymphoma

D. Anaplastic carcinoma

E. Sarcoma

Adenocarcinoma are the most common and typically arise as a result of the adenoma -

carcinoma sequence.

Colorectal cancer

Annually about 150,000 new cases are diagnosed and 50,000 deaths from the

disease

About 75% will have sporadic disease and 25% will have a family history

Colorectal tumours comprise a spectrum of disease ranging from adenomas

through to polyp cancers and frank malignancy.

Polyps may be categorised into: neoplastic polyps, adenomatous polyps and

non neoplastic polyps.

The majority of adenomas are polypoidal lesions, although flat lesions do

occur and may prove to be dysplastic.

Non-neoplastic polyps include hyperplastic, juvenile, hamartomatous,

inflammatory, and lymphoid polyps, which have not generally been thought of

as precursors of cancer.

Three characteristics of adenomas that correlate with malignant potential have

been characterised. These include increased size, villous architecture and

dysplasia. For this reason most polyps identified at colonoscopy should be

removed.

The transformation from polyp to cancer is described by the adenoma -

carcinoma sequence and its principles should be appreciated. Essentially

genetic changes accompany the transition from adenoma to carcinoma; key

changes include APC, c-myc, K RAS mutations and p53 deletions.

Which of the following changes are most likely to be identified in the aortic wall of a

38 year old lady with a Marfans syndrome and a dissecting aortic aneurysm?

A. Transmural aortitis

B. Cystic medial necrosis

C. Foamy macrophages

D. Dense dystrophic calcification

E. None of the above

Cystic medial necrosis ( or cystic medial degeneration) occurs when basophils and

mucoid material lie in between the intimal elastic fibres of the aorta. It is typically

found in the aortic degeneration of Marfans syndrome, but may also be seen in aortic

degeneration in older adults.

Aortic dissection

More common than rupture of the abdominal aorta

33% of patients die within the first 24 hours, and 50% die within 48 hours if

no treatment received

Associated with hypertension

Features of aortic dissection: tear in the intimal layer, followed by formation

and propagation of a subintimal hematoma. Cystic medial necrosis (Marfan's)

Most common site of dissection: 90% occurring within 10 centimetres of the

aortic valve

Stanford Classification

Type Location Treatment

A Ascending aorta/ aortic root Surgery- aortic root replacement

B Descending aorta Medical therapy with antihypertensives

DeBakey classification

Type Site affected

I Ascending aorta, aortic arch, descending aorta

II Ascending aorta only

III Descending aorta distal to left subclavian artery

Clinical features

Tearing, sudden onset chest pain (painless 10%)

Hypertension or Hypotension

A blood pressure difference greater than 20 mm Hg

Neurologic deficits (20%)

Investigations

CXR: widened mediastinum, abnormal aortic knob, ring sign, deviation

trachea/oesophagus

CT (spiral)

MRI

Angiography (95% of patients diagnosed)

Management

Beta-blockers: aim HR 60-80 bpm and systolic BP 100-120 mm Hg.

Urgent surgical intervention: type A dissections. This will usually involve

aortic root replacement.

A 58 year old man undergoes an upper GI endoscopy for the investigation of

odynophagia. At endoscopy a reddish area is seen to protrude up into the oesophagus

from the gastrooesophageal junction. Which of the following pathological processes

is most likely to explain this process?

A. Metaplasia

B. Anaplasia

C. Dysplasia

D. Hypoplasia

E. Hyperplasia

This is most likely to represent Barretts oesphagus and is thus metaplasia. Dysplasia is

less likely in this setting although biopsies are mandatory.

Barrett's oesophagus

Intestinal metaplasia

Squamous epithelium replaced by columnar epithelium in the lower

oesophagus

3 types of columnar epithelium:

1. Junctional

2. Atrophic fundal

3. Specialised

Presence of goblet cells important in identification

Premalignant change (progress to dysplasia)

Risk of adenocarcinoma

Risk factors: middle age, men, smoker, Caucasian, gastro-oesophageal reflux,

obesity

Treatment

Long term proton pump inhibitor

Consider pH and manometry studies in younger patients who may prefer to

consider an anti reflux procedure

Regular endoscopic monitoring (more frequently if moderate dysplasia). With

quadrantic biopsies every 2-3 cm

If severe dysplasia be very wary of small foci of cancer

A male infant is born by emergency cesarean section at 39 weeks gestation for foetal

distress. Soon after the birth the baby becomes progressively hypoxic and on

examination is found to have a scaphoid abdomen. What is the most likely underlying

diagnosis?

A. Intestinal malrotation

B. Hiatus hernia

C. Foramen of Bochdalek hernia

D. Foramen of Morgagni hernia

E. Tracheooesphageal fistula

The finding of a scaphoid abdomen and respiratory distress suggests extensive intra

thoracic herniation of the abdominal contents. This is seen most frequently with

Bochdalek hernias. Morgagni hernias seldom present in such a dramatic fashion. The

other options do not typically present with the symptoms and signs described.

Embryology of the diaphragm and diaphragmatic hernia

Embryology The diaphragm is formed between the 5th and 7th weeks of gestation through the

progressive fusion of the septum transversum, pleuroperitoneal folds and via lateral

muscular ingrowth. The pleuroperitoneal membranes from which the diaphragm

originates originate at somites located in cervical segments 3 to 5, which accounts for

the long path taken by the phrenic nerve. The components contribute to the following

diaphragmatic segments:

Septum transversum - Central tendon

Pleuroperitoneal membranes - Parietal membranes surrounding viscera

Cervical somites C5 to C7 - Muscular component of the diaphragm

Diaphragmatic hernia

Type of hernia Features

Morgagni Anteriorly located

Minimal compromise on lung development

Minimal signs on antenatal ultrasound

Usually present later

Usually good prognosis

Bochdalek hernia Posteriorly located

Larger defect

Often diagnosed antenatally

Associated with pulmonary hypoplasia

Poor prognosis

The posterior hernias of Bochdalek are the most common type and if not diagnosed

antenatally will typically present soon after birth with respiratory distress. The

classical finding is that of a scaphoid abdomen on clinical examination because of

herniation of the abdominal contents into the chest. Bochdalek hernias are associated

with a number of chromosomal abnormalities such as Trisomy 21 and 18. Infants

have considerable respiratory distress due to hypoplasia of the developing lung.

Historically this was considered to be due to direct compression of the lung by

herniated viscera. This view over simplifies the situation and the pulmonary

hypoplasia occurs concomitantly with the hernial development, rather than as a direct

result of it. The pulmonary hypoplasia is associated with pulmonary hypertension and

abnormalities of pulmonary vasculature. The pulmonary hypertension renders infants

at risk of right to left shunting (resulting in progressive and worsening hypoxia).

Diagnostic work up of these infants includes chest x-rays/ abdominal ultrasound scans

and cardiac echo.

Surgery forms the mainstay of treatment and both thoracic and abdominal approaches

may be utilised. Following reduction of the hernial contents a careful search needs to

be made for a hernial sac as failure to recognise and correct this will result in a high

recurrence rate. Smaller defects may be primarily closed, larger defects may require a

patch to close the defect. Malrotation of the viscera is a recognised association and

may require surgical correct at the same procedure (favoring an abdominal approach).

The mortality rate is 50-75% and is related to the degree of lung compromise and age

at presentation (considerably better in infants >24 hours old).

A 72 year old lady falls and lands on her left hip. She attends the emergency

department and is given some paracetamol by the junior doctor and discharged.

Several months later she presents with ongoing pain and discomfort of the hip.

Avascular necrosis of the femoral head is suspected. Which of the following features

is least likely to be present?

A. Non union of the fracture

B. Angiogenesis at the fracture site

C. Increased numbers of fibroblasts at the fracture site

D. Osteochondritis dissecans

E. Apoptosis of osteoblasts

Apoptosis is not a feature of necrotic cell death. By this stage there would usually be

attempted repair so angiogenesis and proliferation of fibroblasts would be expected.

These cells may differentiate further to become osteoblasts which in turn will lay

down new matrix.

Avascular necrosis

Cellular death of bone components due to interruption of the blood supply,

causing bone destruction

Main joints affected are hip, scaphoid, lunate and the talus.

It is not the same as non union. The fracture has usually united.

Radiological evidence is slow to appear.

Vascular ingrowth into the affected bone may occur. However, many joints

will develop secondary osteoarthritis.

Causes P ancreatitis

L upus

A lcohol

S teroids

T rauma

I diopathic, infection

C aisson disease, collagen vascular disease

R adiation, rheumatoid arthritis

A myloid

G aucher disease

S ickle cell disease

Presentation Usually pain. Often despite apparent fracture union.

Investigation MRI scanning will show changes earlier than plain films.

Treatment In fractures at high risk sites anticipation is key. Early prompt and accurate reduction

is essential.

Non weight bearing may help to facilitate vascular regeneration.

Joint replacement may be necessary, or even the preferred option (e.g. Hip in the

elderly).

Which one of the following is least associated with the development of colorectal

cancer in patients with ulcerative colitis?

A. Unremitting disease

B. Disease duration > 10 years

C. Onset before 15 years old

D. Poor compliance to treatment

E. Disease confined to the rectum

Ulcerative colitis and colorectal cancer

Overview

risk of colorectal cancer is 10-20 times that of general population

the increased risk is mainly related to chronic inflammation

worse prognosis than patients without ulcerative colitis (partly due to delayed

diagnosis)

lesions may be multifocal

Factors increasing risk of cancer

disease duration > 10 years

patients with pancolitis

onset before 15 years old

unremitting disease

poor compliance to treatment

Which of the following statements about wound healing is false?

A. During the course of the first 2 weeks the fibrin plug is replaced by a

collagen rich mesh.

B. Scar contraction is a late feature and is mediated by myofibroblasts.

C. May be impaired when an infected foreign body is present.

D. Is more cosmetically acceptable when incisions are made parallel to

Langer's lines.

E. Has features of chronic inflammation from the beginning.

Although wounds often contain cell types (macrophages) and processes (angiogenesis) these

typically take time to occur. They are not present in the immature wound.

Wound healing

Surgical wounds are either incisional or excisional and either clean, clean contaminated or

dirty. Although the stages of wound healing are broadly similar their contributions will vary

according to the wound type.

The main stages of wound healing include:

Haemostasis

Vasospasm in adjacent vessels, platelet plug formation and generation of fibrin rich clot.

Inflammation

Neutrophils migrate into wound (function impaired in diabetes). Growth factors released, including basic fibroblast growth factor and vascular

endothelial growth factor. Fibroblasts replicate within the adjacent matrix and migrate into wound. Macrophages and fibroblasts couple matrix regeneration and clot substitution.

Regeneration

Platelet derived growth factor and transformation growth factors stimulate fibroblasts and epithelial cells.

Fibroblasts produce a collagen network. Angiogenesis occurs and wound resembles granulation tissue.

Remodeling

Longest phase of the healing process and may last up to one year (or longer). During this phase fibroblasts become differentiated (myofibroblasts) and these

facilitate wound contraction. Collagen fibres are remodeled. Microvessels regress leaving a pale scar.

The above description represents an idealised scenario. A number of diseases may distort

this process. It is obvious that one of the key events is the establishing well vascularised

tissue. At a local level angiogenesis occurs, but if arterial inflow and venous return are

compromised then healing may be impaired, or simply nor occur at all. The results of

vascular compromise are all too evidence in those with peripheral vascular disease or those

poorly constructed bowel anastomoses.

Conditions such as jaundice will impair fibroblast synthetic function and overall immunity

with a detrimental effect in most parts of healing.

Problems with scars:

Hypertrophic scars

Excessive amounts of collagen within a scar. Nodules may be present histologically

containing randomly arranged fibrils within and parallel fibres on the surface. The tissue

itself is confined to the extent of the wound itself and is usually the result of a full thickness

dermal injury. They may go on to develop contractures.

Image of hypertrophic scarring. Note that it remains confined to the boundaries of the

original wound:

Image sourced from Wikipedia

Keloid scars

Excessive amounts of collagen within a scar. Typically a keloid scar will pass beyond the

boundaries of the original injury. They do not contain nodules and may occur following even

trivial injury. They do not regress over time and may recur following removal.

Image of a keloid scar. Note the extension beyond the boundaries of the original incision:

Image sourced from Wikipedia

Drugs which impair wound healing:

Non steroidal anti inflammatory drugs Steroids Immunosupressive agents Anti neoplastic drugs

Closure

Delayed primary closure is the anatomically precise closure that is delayed for a few days but

before granulation tissue becomes macroscopically evident.

Secondary closure refers to either spontaneous closure or to surgical closure after

granulation tissue has formed.

heme: Causes of chest pain

A. Pulmonary embolism

B. Anterior myocardial infarction

C. Inferior myocardial infarction

D. Proximal aortic dissection

E. Distal aortic dissection

F. Boerhaave Syndrome

G. Mallory weiss tear

H. Perforated gastric ulcer

Please select the most likely cause of chest pain for the scenario given. Each option

may be used once, more than once or not at all.

8. A 52 year old male presents with tearing central chest pain. On examination

he has an aortic regurgitation murmur. An ECG shows ST elevation in leads

II, III and aVF.

You answered Inferior myocardial infarction

The correct answer is Proximal aortic dissection

An inferior myocardial infarction and AR murmur should raise suspicions of

an ascending aorta dissection rather than an inferior myocardial infarction

alone. Also the history is more suggestive of a dissection. Other features may

include pericardial effusion, carotid dissection and absent subclavian pulse.

9. A 52 year old male presents with central chest pain and vomiting. He has

drunk a bottle of vodka. On examination there is some mild crepitus in the

epigastric region.

Boerhaave Syndrome

The Mackler triad for Boerhaave syndrome: vomiting, thoracic pain,

subcutaneous emphysema. It commonly presents in middle aged men with a

background of alcohol abuse.

10. A 52 year old male presents with central chest pain. On examination he has

an mitral regurgitation murmur. An ECG shows ST elevation in leads V1 to

V6. There is no ST elevation in leads II, III and aVF.

Anterior myocardial infarction

The most likely diagnosis is an anterior MI. As there are no ST changes in the

inferior leads, aortic dissection is less likely.

Chest pain

Aortic dissection

This occurs when there is a flap or filling defect within the aortic intima.

Blood tracks into the medial layer and splits the tissues with the subsequent

creation of a false lumen. It most commonly occurs in the ascending aorta or

just distal to the left subclavian artery (less common). It is most common in

Afro-carribean males aged 50-70 years.

Patients usually present with a tearing intrascapular pain, which may be

similar to the pain of a myocardial infarct.

The dissection may spread either proximally or distally with subsequent

disruption to the arterial branches that are encountered.

In the Stanford classification system the disease is classified into lesions with

a proximal origin (Type A) and those that commence distal to the left

subclavian (Type B).

Diagnosis may be suggested by a chest x-ray showing a widened mediastinum.

Confirmation of the diagnosis is usually made by use of CT angiography

Proximal (Type A) lesions are usually treated surgically, type B lesions are

usually managed non operatively.

Pulmonary embolism

Typically sudden onset of chest pain, haemoptysis, hypoxia and small pleural

effusions may be present.

Most patients will have an underlying deep vein thrombosis

Diagnosis may be suggested by various ECG findings including S waves in

lead I, Q waves in lead III and inverted T waves in lead III. Confirmation of

the diagnosis is usually made through use of CT pulmonary angiography.

Treatment is with anticoagulation, in those patients who develop a cardiac

arrest or severe compromise from their PE, consideration may be given to

thrombolysis.

Myocardial infarction

Traditionally described as sudden onset of central, crushing chest pain. It may

radiate into the neck and down the left arm. Signs of autonomic dysfunction

may be present. The presenting features may be atypical in the elderly and

those with diabetes.

Diangosis is made through identification of new and usually dynamic ECG

changes (and cardiac enzyme changes). Inferior and anterior infarcts may be

distinguished by the presence of specific ECG changes (usually II, III and aVF

for inferior, leads V1-V5 for anterior).

Treatment is with oral antiplatelet agents, primary coronary angioplasty and/

or thrombolysis.

Perforated peptic ulcer

Patients usually develop sudden onset of epigastric abdominal pain, it may be

soon followed by generalised abdominal pain.

There may be features of antecendant abdominal discomfort, the pain of

gastric ulcer is typically worse immediately after eating.

Diagnosis may be made by erect chest x-ray which may show a small amount

of free intra-abdominal air (very large amounts of air are more typically

associated with colonic perforation).

Treatment is usually with a laparotomy, small defects may be excised and

overlaid with an omental patch, larger defects are best managed with a partial

gastrectomy.

Boerhaaves syndrome

Spontaneous rupture of the oesophagus that occurs as a result of repeated

episodes of vomiting.

The rupture is usually distally sited and on the left side.

Patients usually give a history of sudden onset of severe chest pain that may

complicate severe vomiting.

Severe sepsis occurs secondary to mediastinitis.

Diagnosis is CT contrast swallow.

Treatment is with thoracotomy and lavage, if less than 12 hours after onset

then primary repair is usually feasible, surgery delayed beyond 12 hours is

best managed by insertion of a T tube to create a controlled fistula between

oesophagus and skin.

Delays beyond 24 hours are associated with a very high mortality rate.

A 78 year old lady presents with a tender swelling in her right groin. On examination there is

a tender swelling that lies lateral to the pubic tubercle. It has a cough impulse. What is the

most likely underlying diagnosis?

A. Thrombophlebitis of the great saphenous vein

B. Femoral hernia

C. Thrombophlebitis of saphena varix

D. Inguinal hernia

E. Obturator hernia

Theme from April 2012 Exam

Whilst a thrombophlebitis of a saphena varix may cause a tender swelling at this site, it

would not usually be associated with a cough impulse.

Femoral canal

The femoral canal lies at the medial aspect of the femoral sheath. The femoral sheath is a

fascial tunnel containing both the femoral artery laterally and femoral vein medially. The

canal lies medial to the vein.

Borders of the femoral canal

Laterally Femoral vein

Medially Lacunar ligament

Anteriorly Inguinal ligament

Posteriorly Pectineal ligament

Image showing dissection of femoral canal

Image sourced from Wikipedia

Contents

Lymphatic vessels Cloquet's lymph node

Physiological significance

Allows the femoral vein to expand to allow for increased venous return to the lower limbs.

Pathological significance

As a potential space, it is the site of femoral hernias. The relatively tight neck places these at

high risk of strangulation.

A 3 month old boy is suspected of having hypospadias. At which of the following

locations is the urethral opening most frequently located in boys suffering from the

condition?

A. On the distal ventral surface of the penis

B. On the proximal ventral surface of the penis

C. On the distal dorsal surface of the penis

D. On the proximal dorsal surface of the penis

E. At the base of the scrotum

The defect is located ventrally and most often distally. Proximally located urethral

openings are well recognised. Circumcision may compromise reconstruction.

Hypospadias

The urethral meatus opens on the ventral surface of the penis. There is also a ventral

deficiency of the foreskin. The uretral meatus may open more proximally in the more

severe variants. However, 75% of the openings are distally located. The incidence is 1

in 300 male births.

Features include:

Absent frenular artery

Ventrally opened glans

Skin tethering to hypoplastic urethra

Splayed columns of spongiosum tissue distal to the meatus

Deficiency of the foreskin ventrally

Management:

No routine cultural circumcisions

Urethroplasty

Penile reconstruction

The foreskin is often utilised in the reconstructive process. In boys with very distal

disease no treatment may be needed.

A 52 year old male attends for a preoperative assessment for an inguinal hernia repair.

You notice that the chest x-ray shows a loculated left pleural effusion. On further

questioning the patient reports that he worked as a builder 30 years ago. What is the

most likely cause for the effusion?

A. Asbestosis

B. Pneumonia

C. Mesothelioma

D. Silicosis

E. Left ventricular failure

This patient has a risk of asbestos exposure through his occupation as a builder. As

there a is latent period of 30 years and a complicated effusion, the most likely cause is

mesothelioma.

Mesothelioma

Features

Dyspnoea, weight loss, chest wall pain

Clubbing

30% present as painless pleural effusion

Only 20% have pre-existing asbestosis

History of asbestos exposure in 85-90%, latent period of 30-40 years

Basics

Malignancy of mesothelial cells of pleura

Metastases to contralateral lung and peritoneum

Right lung affected more often than left

Management

Investigation: pleural biopsy, CT Scanning, (PET Scanning if surgery

considered)

Symptomatic

Industrial compensation

Chemotherapy, Surgery if operable

Prognosis poor, median survival 12 months

A 64-year-old woman who is reviewed due to multiple non-healing leg ulcers. She

reports feeling generally unwell for many months. Examination findings include a

blood pressure of 138/72 mmHg, pulse 90 bpm, pale conjunctivae and poor dentition

associated with bleeding gums. What is the most likely underlying diagnosis?

A. Thyrotoxicosis

B. Vitamin B12 deficiency

C. Vitamin C deficiency

D. Diabetes mellitus

E. Sarcoidosis

Bleeding gums and poor healing are suggestive of vitamin C deficiency.

Vitamin C deficiency

Vitamin C deficiency (scurvy) leads to defective synthesis of collagen resulting in

capillary fragility (bleeding tendency) and poor wound healing

Features

gingivitis, loose teeth

poor wound healing

bleeding from gums, haematuria, epistaxis

general malaise

Which of the following is not a typical feature of neuropraxia?

A. Transient delay in neuronal transmission

B. Axonal degeneration distal to the site of injury

C. Absence of neuroma formation

D. Preservation of autonomic function

E. Absence of axonal degeneration proximal to the site of injury

Full recovery may occur 6-8 weeks after nerve injury in neuropraxia.

Wallerian degeneration does not usually occur in simple neuropraxia.

Autonomic function is usually preserved.

Nerve injury

There are 3 types of nerve injury:

Neuropraxia Nerve intact but electrical conduction is affected

Full recovery

Autonomic function preserved

Wallerian degeneration does not occur

Axonotmesis Axon is damaged and the myelin sheath is preserved. The

connective tissue framework is not affected.

Wallerian degeneration occurs.

Neurotmesis Disruption of the axon, myelin sheath and surrounding

connective tissue.

Wallerian degeneration occurs.

Wallerian Degeneration

Axonal degeneration distal to the site of injury.

Typically begins 24-36 hours following injury.

Axons are excitable prior to degeneration occurring.

Myelin sheath degenerates and is phagocytosed by tissue macrophages.

Nerve repair

Neuronal repair may only occur physiologically where nerves are in direct

contact. Where a large defect is present the process of nerve regeneration is

hampered and may not occur at all or result in the formation of a neuroma.

Where nerve regrowth occurs it typically occurs at a rate of 1mm per day.

A 44 year old lady presents with a pathological fracture of the left femur. She has

previously undergone a renal transplant for end stage renal failure. Her blood test

results are as follows:

Serum Ca2+ 2.80

PTH 88pg/ml

Phosphate 0.30

A surgeon decides to perform a parathyroidectomy on the basis of these results. When

the glands are assessed histologically, which of the appearances is most likely to be

identified?

A. Metaplasia the gland

B. Hypertrophy of the gland

C. Hyperplasia of the gland

D. Parathyroid carcinoma

E. Necrosis of the parathyroid gland

This is likely to be a case of tertiary hyperparathyroidism (high Calcium, high PTH,

low phosphate). Therefore the glands will be hyperplastic. Hypertrophy is not correct

as this implies an increase in size without an increase in cellularity. This mistake has

cost many candidates marks in the MRCS exams over the years!

Parathyroid glands and disorders of calcium metabolism

Hyperparathyroidism

Disease type Hormone profile Clinical features Cause

Primary

hyperparathyroidism

PTH (Elevated)

Ca2+ (Elevated)

Phosphate

(Low)

Serum Calcium

: Creatinine

clearance ratio

> 0.01

May be

asymptomatic if

mild

Recurrent

abdominal pain

(pancreatitis, renal

colic)

Changes to

emotional or

cognitive state

Most cases due to

solitary adenoma

(80%), multifocal

disease occurs in

10-15% and

parathyroid

carcinoma in 1% or

less

Secondary

hyperparathyroidism

PTH (Elevated)

Ca2+ (Low or

normal)

Phosphate

(Elevated)

Vitamin D

levels (Low)

May have few

symptoms

Eventually may

develop bone

disease, osteitis

fibrosa cystica and

soft tissue

calcifications

Parathyroid gland

hyperplasia occurs

as a result of low

calcium, almost

always in a setting

of chronic renal

failure

Tertiary

hyperparathyroidism

Ca2+ (Normal

or high)

PTH (Elevated)

Phosphate

levels

(Decreased or

Normal)

Vitamin D

(Normal or

decreased)

Alkaline

phosphatase

(Elevated)

Metastatic

calcification

Bone pain and /

or fracture

Nephrolithiasis

Pancreatitis

Occurs as a result

of ongoing

hyperplasia of the

parathyroid glands

after correction of

underlying renal

disorder,

hyperplasia of all 4

glands is usually

the cause

Differential diagnoses It is important to consider the rare but relatively benign condition of benign familial

hypocalciuric hypercalcaemia, caused by an autosomal dominant genetic disorder.

Diagnosis is usually made by genetic testing and concordant biochemistry (Serum

Calcium : Creatinine clearance ratio <0.01-distinguished from primary

hyperparathyroidism).

Treatment

Primary hyperparathyroidism

Indications for surgery

Elevated serum Calcium > 1mg/dL above normal

Hypercalciuria > 400mg/day

Creatinine clearance < 30% compared with normal

Episode of life threatening hypercalcaemia

Nephrolithiasis

Age < 50 years

Neuromuscular symptoms

Reduction in bone mineral density of the femoral neck, lumbar spine, or distal

radius of more than 2.5 standard deviations below peak bone mass (T score

lower than -2.5)

Secondary hyperparathyroidism Usually managed with medical therapy.

Indications for surgery in secondary (renal) hyperparathyroidism:

Bone pain

Persistent pruritus

Soft tissue calcifications

Tertiary hyperparathyroidism Usually treatment is surgical

The presence of an autonomously functioning parathyroid gland may require surgery.

If the culprit gland can be identified then it should be excised. Otherwise total

parathyroidectomy and re-implantation of part of the gland may be required.

References 1. Pitt S et al. Secondary and Tertiary Hyperparathyroidism, State of the Art Surgical

Management. Surg Clin North Am 2009 Oct;89(5):1227-39.

2. MacKenzie-Feder J et al. Primary Hyperparathyroidism: An Overview. Int J

Endocrinol 2011; 2011: 251410.

A 56 year old man presents with lethargy, haematuria and haemoptysis. On

examination he is hypertensive and has a left loin mass. A CT scan shows a lesion

affecting the upper pole of the right kidney, it has a small cystic centre. Which of the

options below is the most likely diagnosis?

A. Squamous cell carcinoma of the kidney

B. Nephroblastoma

C. Renal adenocarcinoma

D. Transitional cell carcinoma of the kidney

E. Polycystic kidney disease

Renal adenocarcinoma are the most common renal tumours. These will typically

affect the renal parenchyma. Transitional cell carcinoma will usually affect urothelial

surfaces. Nephroblastoma would be very rare in this age group. Renal

adenocarcinoma may produce cannon ball metastasis in the lung which cause

haemoptysis, this is not a feature of PKD.

Renal tumours

Renal cell carcinoma Renal cell carcinoma is an adenocarcinoma of the renal cortex and is believed to arise

from the proximal convoluted tubule. They are usually solid lesions, up to 20% may

be multifocal, 20% may be calcified and 20% may have either a cystic component or

be wholly cystic. They are often circumscribed by a pseudocapsule of compressed

normal renal tissue. Spread may occur either by direct extension into the adrenal

gland, renal vein or surrounding fascia. More distant disease usually occurs via the

haematogenous route to lung, bone or brain.

Renal cell carcinoma comprise up to 85% of all renal malignancies. Males are more

commonly affected than females and sporadic tumours typically affect patients in

their sixth decade.

Patients may present with a variety of symptoms including; haematuria (50%), loin

pain (40%), mass (30%) and up to 25% may have symptoms of metastasis.Less than

10% have the classic triad of haematuria, pain and mass.

Investigation Many cases will present as haematuria and be discovered during diagnostic work up.

Benign renal tumours are rare, so renal masses should be investigated with multislice

CT scanning. Some units will add and arterial and venous phase to the scan to

demonstrate vascularity and evidence of caval ingrowth.

CT scanning of the chest and abdomen to detect distant disease should also be

undertaken.

Routine bone scanning is not indicated in the absence of symptoms.

Biopsy should not be performed when a nephrectomy is planned but is mandatory

before any ablative therapies are undertaken.

Assessment of the functioning of the contra lateral kidney.

Management T1 lesions may be managed by partial nephrectomy and this gives equivalent

oncological results to total radical nephrectomy. Partial nephrectomy may also be

performed when there is inadequate reserve in the remaining kidney.

For T2 lesions and above a radical nephrectomy is standard practice and this may be

performed via a laparoscopic or open approach. Preoperative embolisation is not

indicated nor is resection of uninvolved adrenal glands. During surgery early venous

control is mandatory to avoid shedding of tumour cells into the circulation.

Patients with completely resected disease do not benefit from adjuvant therapy

with either chemotherapy or biological agents. These should not be administered

outside the setting of clinical trials.

Patients with transitional cell cancer will require a nephroureterectomy with

disconnection of the ureter at the bladder.

References Lungberg B et al. EAU guidelines on renal cell carcinoma: The 2010 update.

European Urology 2010 (58): 398-406.

A 34-year-old man is taken immediately to theatre with aortic dissection. You note he

is tall with pectus excavatum and arachnodactyly. His condition is primarily due to a

defect in which one of the following proteins?

A. Polycystin-1

B. Fibrillin

C. Type IV collagen

D. Type I collagen

E. Elastin

Although fibrillin is the primary protein affected (due to a defect in the fibrillin-1

gene) it should be noted that fibrillin is used as a substrate of elastin.

Marfan's syndrome

Marfan's syndrome is an autosomal dominant connective tissue disorder. It is caused

by a defect in the fibrillin-1 gene on chromosome 15 and affects around 1 in 3,000

people.

Features

tall stature with arm span to height ratio > 1.05

high-arched palate

arachnodactyly

pectus excavatum

pes planus

scoliosis of > 20 degrees

heart: dilation of the aortic sinuses (seen in 90%) which may lead to aortic

aneurysm, aortic dissection, aortic regurgitation, mitral valve prolapse (75%),

lungs: repeated pneumothoraces

eyes: upwards lens dislocation (superotemporal ectopia lentis), blue sclera,

myopia

dural ectasia (ballooning of the dural sac at the lumbosacral level)

The life expectancy of patients used to be around 40-50 years. With the advent of

regular echocardiography monitoring and beta-blocker/ACE-inhibitor therapy this has

improved significantly over recent years. Aortic dissection and other cardiovascular

problems remain the leading cause of death however.

Which of the following are not typical of Lynch syndrome?

A. It is inherited in an autosomal recessive manner

B. Affected patients are more likely to develop right colon mucinous

tumours than the general population

C. Affected individuals have an 80% lifetime risk of colon cancer

D. Endometrial cancer is seen in 80%.

E. Gastric cancers are more common

Lynch syndrome is inherited in an autosomal dominant fashion. It is characterised by

microsatellite instability in the DNA mismatch repair genes. Colonic tumours in

patients with Lynch syndrome are more likely to be right sided tumours and to be

poorly differentiated.

Genetics and surgical disease

Some of the more commonly occurring genetic conditions occurring in surgical

patients are presented here.

Li-Fraumeni Syndrome

Autosomal dominant

Consists of germline mutations to p53 tumour suppressor gene

High incidence of malignancies particularly sarcomas and leukaemias

Diagnosed when:

*Individual develops sarcoma under 45 years

*First degree relative diagnosed with any cancer below age 45 years and another

family member develops malignancy under 45 years or sarcoma at any age

BRCA 1 and 2

Carried on chromosome 17

Linked to developing breast cancer (60%) risk.

Associated risk of developing ovarian cancer (55% with BRCA 1 and 25%

with BRCA2).

Lynch Syndrome

Autosomal dominant

Develop colonic cancer and endometrial cancer at young age

80% of affected individuals with get colonic and or endometrial cancer

High risk individuals may be identified using the Amsterdam criteria

Amsterdam criteria Three or more family members with a confirmed diagnosis of colorectal cancer, one

of whom is a first degree (parent, child, sibling) relative of the other two.

Two successive affected generations.

One or more colon cancers diagnosed under age 50 years.

Familial adenomatous polyposis (FAP) has been excluded.

Gardners syndrome

Autosomal dominant familial colorectal polyposis

Multiple colonic polyps

Extra colonic diseases include: skull osteoma, thyroid cancer and epidermoid

cysts

Desmoid tumours are seen in 15%

Mutation of APC gene located on chromosome 5

Due to colonic polyps most patients will undergo colectomy to reduce risk of

colorectal cancer

Now considered a variant of familial adenomatous polyposis coli

An enthusiastic medical student approaches you with a list of questions about blood

transfusion reactions. Which of her following points is incorrect?

A. Graft versus host disease involves neutrophil

proliferation

B. Thrombocytopaenia may occur in women with a prior

pregnancy

C. IgA antibodies may cause blood pressure compromise

during transfusion

D. Hypocalcaemia can occur

E. Iron overload can be avoided by chelation therapy

Mnemonic for transfusion

reactions:

Got a bad unit

G raft vs. Host disease

O verload

T hrombocytopaenia

A lloimmunization

B lood pressure unstable

A cute haemolytic reaction

D elayed haemolytic

reaction

U rticaria

N eutrophilia

I nfection

T ransfusion associated

lung injury

GVHD results from lymphocytic proliferation. The patient's own lymphocytes are

similar to the donor's lymphocytes, therefore don't perceive them as being foreign.

The donor lymphocytes, however, sees the recipient lymphocytes as being foreign.

Therefore they proliferate causing severe complications.

Thrombocytopaenia occurs a few days after transfusion and may resolve

spontaneously.

Patients with IGA antibodies need IgA deficient blood transfusions.

Blood transfusion reactions

Immune mediated Non immune mediated

Pyrexia Hypocalcaemia

Alloimmunization CCF

Thrombocytopaenia Infections

Transfusion associated lung injury Hyperkalaemia

Graft vs Host disease

Urticaria

Acute or delayed haemolysis

ABO incompatibility

Rhesus incompatibility

Notes:

GVHD: lymphocyte proliferation causing organ failure

Transfusion associated lung injury: neutrophil mediated allergic pulmonary oedema

ABO and Rhesus incompatibility: causes acute haemolytic transfusion reaction

leading to agglutination and haemolysis

An 82 year old lady presents with a carcinoma of the caecum. Approximately what

proportion of patients presenting with this diagnosis will have synchronous lesions?

A. <1%

B. 60%

C. 50%

D. 20%

E. 5%

Synchronous colonic tumours are seen in 5% cases and all patients having a flexible

sigmoidoscopy should have completion colonoscopy if tumours or polyps are found

Synchronous lesions may occur in up to 5% of patients with colorectal cancer. A full

and complete lumenal study with either colonoscopy, CT cologram or barium enema

is mandatory in all patients being considered for surgery.

Colorectal cancer

Annually about 150,000 new cases are diagnosed and 50,000 deaths from the

disease

About 75% will have sporadic disease and 25% will have a family history

Colorectal tumours comprise a spectrum of disease ranging from adenomas

through to polyp cancers and frank malignancy.

Polyps may be categorised into: neoplastic polyps, adenomatous polyps and

non neoplastic polyps.

The majority of adenomas are polypoidal lesions, although flat lesions do

occur and may prove to be dysplastic.

Non-neoplastic polyps include hyperplastic, juvenile, hamartomatous,

inflammatory, and lymphoid polyps, which have not generally been thought of

as precursors of cancer.

Three characteristics of adenomas that correlate with malignant potential have

been characterised. These include increased size, villous architecture and

dysplasia. For this reason most polyps identified at colonoscopy should be

removed.

The transformation from polyp to cancer is described by the adenoma -

carcinoma sequence and its principles should be appreciated. Essentially

genetic changes accompany the transition from adenoma to carcinoma; key

changes include APC, c-myc, K RAS mutations and p53 deletions. A 22 year old man undergoes a splenectomy for an iatrogenic splenic injury. On the

second post operative day a full blood count is performed. Which of the following

components of the full blood count is the first to be affected ?

A. Erythrocyte count

B. Reticulocyte count

C. Eosinophil count

D. Monocyte count

E. Lymphocyte count

Theme from January 2012 Exam

The granulocyte and platelet count are the first to be affected following

splenectomy. Then reticulocytes increase. Although a lymphocytosis and

monocytosis are reported, these take several weeks to develop.

Post splenectomy blood film changes

The loss of splenic tissue results in the inability to readily remove immature or

abnormal red blood cells from the circulation. The red cell count does not alter

significantly. However, cytoplasmic inclusions may be seen e.g. Howell-Jolly bodies.

In the first few days after splenectomy target cells, siderocytes and reticulocytes will

appear in the circulation. Immediately following splenectomy a granulocytosis

(mainly composed of neutrophils) is seen, this is replaced by a lymphocytosis and

monocytosis over the following weeks.

The platelet count is usually increased and this may be persistent, oral antiplatelet

agents may be needed in some patients.

Image showing Howell Jolly bodies (arrowed)

Image sourced from Wikipedia

A 28 year old lady presents with benign cyclical mastalgia. Which of the following is

not a recognised treatment for the condition?

A. Evening primrose oil

B. Bromocriptine

C. Methotrexate

D. Danazol

E. Tamoxifen

Surgical excision of tender breast

tissue is inappropriate

Methotrexate is used for the treatment of breast cancer. Whilst the use of tamoxifen is

of benefit other agents such as flaxseed oil or evening primrose oil should be tried

first. Danazol is effective, but many women dislike the side effects.

Benign cyclical mastalgia

Benign cyclical mastalgia is a common cause of breast pain in younger females. It

varies in intensity according to the phase of the menstrual cycle. It is not associated

with point tenderness of the chest wall (more likely to be Tietze's syndrome).

The underlying cause is difficult to pinpoint, examination should focus on identifying

focal lesions (such as cysts) that may be treated to provide symptomatic benefit.

Women should be advised to wear a supportive bra. Conservative treatments include

flax seed oil and evening primrose oil. There is slightly more evidence in favor of flax

seed oil, though neither has performed much better than placebo in RCT's.

Hormonal agents such as bromocriptine and danazol may be more effective.

However, many women discontinue these therapies due to adverse effects.

A 39 year old lady has undergone surgery for breast cancer. As part of the

histopathology report the pathologist provides the surgeon with a Nottingham

Prognostic Index score of 6.4. He also states that the tumour size is 2cm. Which of the

following inferences can be made in relation to this statement?

A. The tumour is likely to be grade 1

B. Vascular invasion is present

C. Lymph node metastasis are definitely present

D. The tumour is oestrogen receptor positive

E. None of the above

A score of this value is unlikely to be reached with a grade 1 tumour and a size of

2cm. Therefore lymph node metastasis are definitely present. In addition since the

maximal score for lymph node metastasis is 3 the tumour is likely be of a higher grade

(see below). The Nottingham Prognostic Index provides no information about

oestrogen receptor status or the presence or absence of vascular invasion.

Nottingham prognostic index

The Nottingham Prognostic Index can be used to give an indication of survival. In this

system the tumour size is weighted less heavily than other major prognostic

parameters.

Calculation of NPI

Tumour Size x 0.2 + Lymph node score(From table below)+Grade score(From

table below).

Score

Lymph nodes involved Grade

1 0 1

2 1-3 2

3 >3 3

Prognosis

Score Percentage 5 year survival

2.0 to 2.4 93%

2.5 to 3.4 85%

3.5 to 5.4 70%

>5.4 50%

This data was originally published in 1992. It should be emphasised that other factors

such as vascular invasion and receptor status also impact on survival and are not

included in this data and account for varying prognoses often cited in the literature.

Reference Galea, M.H., et al., The Nottingham Prognostic Index in primary breast cancer. Breast

Cancer Res Treat, 1992. 22(3): p. 207-19.

n patients with multiple endocrine neoplasia type IIb which of the following clinical

appearances is the patient most likely to display?

A. Acromegalic facies

B. Turners type features

C. Profound kyphoscoliosis

D. Multiple bony exostoses

E. Marfanoid features

Patients with MEN IIb may display Marfanoid features. It is unclear at the present

time whether they have discrete changes in the microfibrils of elastic fibres that are

present in Marfans.

Multiple Endocrine Neoplasia

Multiple endocrine neoplasia (MEN) is inherited as an autosomal dominant disorder.

The table below summarises the three main types of MEN:

MEN type I MEN type IIa MEN type IIb

Mnemonic 'three P's':

Parathyroid (95%): Parathyroid adenoma

Pituitary (70%):

Prolactinoma/ACTH/Growth Hormone

secreting adenoma

Pancreas (50%): Islet cell

tumours/Zollinger Ellison syndrome

also: Adrenal (adenoma) and thyroid

(adenoma)

Phaeochromocytoma

Medullary thyroid

cancer (70%)

Hyperparathyroidism

(60%)

Same as MEN IIa

with addition of:

Marfanoid body

habitus

Mucosal neuromas

MENIN gene (chromosome 11)

Most common presentation =

hypercalcaemia

RET oncogene

(chromosome 10)

RET oncogene

(chromosome 10)

32 year old man undergoes an appendicectomy. A large carcinoid tumour is identified

and a completion right hemicolectomy is performed. He is well for several months

and then develops symptoms of palpitations and facial flushing. Which of the

following diagnostic markers should be requested?

A. Alpha feto protein

B. 5-Hydroxyindoleacetic acid

C. Urinary catecholamines

D. Urinary VMA measurements

E. None of the above

5 HIAA is the most commonly used diagnostic marker for carcinoid syndrome.

Carcinoid syndrome

Carcinoid tumours secrete serotonin

Originate in neuroendocrine cells mainly in the intestine (midgut-distal

ileum/appendix)

Can occur in the rectum, bronchi

Hormonal symptoms mainly occur when disease spreads outside the bowel

Clinical features - Onset: years

- Flushing face

- Palpitations

- Tricuspid stenosis causing dyspnoea

- Asthma

- Severe diarrhoea (secretory, persists despite fasting)

Investigation - 5-HIAA in a 24-hour urine collection

- Scintigraphy

- CT scan

Treatment

Octreotide

Surgical removal

Which of the following breast tumours is most commonly associated with a

risk of metastasis to the contralateral breast?

A. Invasive ductal carcinoma

B. Invasive lobular carcinoma

C. Phyllodes tumour

D. Pagets disease of the breast

E. Atypical ductal hyperplasia

Risk of metastasis to the contralateral breast is a classical feature of invasive

lobular carcinoma.

Lobular carcinoma of the breast

Lobular breast cancers are less common than their ductal counterparts. They

typically present differently, the mass is usually more diffuse and less obvious

on the usual imaging modalities of ultrasound and mammography. This is

significant since the disease may be understaged resulting in inadequate

treatment when wide local excision is undertaken.

In women with invasive lobular carcinoma it is usually safest to perform an

MRI scan of the breast, if breast conserving surgery is planned.

Lobular carcinomas are also more likely to be multifocal and metastasise to

the contralateral breast.

Lobular carcinoma in situ is occasionally diagnosed incidentally on core

biopsies. Unlike DCIS, lobular carcinoma in situ is far less strongly associated

with foci of invasion and is usually managed by close monitoring.

With respect to oncogenes which statement is false?

A. Mutations in oncogenes lead to cell survival

B. Cells with oncogene mutations are resistant to cell necrosis

C. Oncogene mutations must typically involve two allelic mutations for

biological effects to become manifest

D. May prevent cellular apoptosis

E. Include MYC mutations in the development of Burkitts Lymphoma

Tumour suppressor gene mutations tend to be recessive and thus twin allelic

mutations are required as exemplified in the Knudson two hit hypothesis in the

development of retinoblastoma. Necrosis will occur at the centre of tumours that

outgrow their blood supply and induction of angiogenesis is a key feature of tumour

progression.

Oncogenes

Oncogenes are cancer promoting genes that are derived from normal genes (proto-

oncogenes). Proto-oncogenes play an important physiological role in cellular growth.

They are implicated in the development of up to 20% of human cancers.

Proto-oncogenes may become oncogenes via the following processes:

Mutation (point mutation)

Chromosomal translocation

Increased protein expression

Only one mutated copy of the gene is needed for cancer to occur - a dominant effect

Classification of oncogenes

Growth factors e.g. Sis

Transcription factors e.g. Myc

Receptor tyrosine kinase e.g. RET

Cytoplasmic tyrosine kinase e.g. Src

Regulatory GTPases e.g. Ras

Tumour supressor genes Tumour supressor genes restrict or repress cellular proliferation in normal cells. Their

inactivation through mutation or germ line incorporation is implicated in renal,

colonic, breast, bladder and many other cancers. One of the best known tumour

supressor genes is p53. p53 gene offers protection by causing apoptosis of damaged

cells. Other well known genes include BRCA 1 and 2.

Which is the characteristic finding on a blood film post splenectomy?

A. Stipple cell

B. Tear drop cell

C. Reticulocytes

D. Howell-Jolly bodies

E. Schistocyte

Blood film in hyposplenism:

Howell-Jolly bodies

Pappenheimer bodies

Poikilocytes (Target cells)

Erythrocyte containing siderotic granules

Heinz bodies

Splenectomy

Indications

Trauma: 1/4 are iatrogenic

Spontaneous rupture: EBV

Hypersplenism: hereditary spherocytosis or elliptocytosis etc

Malignancy: lymphoma or leukaemia

Splenic cysts, hydatid cysts, splenic abscesses

Post splenectomy changes

Platelets will rise first (therefore in ITP should be given after splenic artery

clamped)

Blood film will change over following weeks, Howell Jolly bodies will appear

Other blood film changes include target cells and Pappenheimer bodies

Increased risk of post splenectomy sepsis, therefore prophylactic antibiotics

and pneumococcal vaccine should be given.

Post splenectomy sepsis

Typically occurs with encapsulated organisms

Opsonisation occurs but then not recognised